Sunteți pe pagina 1din 258

Cuprins

II. Analiza matematica 0


7 Siruri si serii numerice 1
8 Calcul diferent ial pentru funct ii de o variabila reala 43
9 Calcul integral pentru funct ii de o variabila reala 62
10 Funct ii de mai multe variabile reale 93
11 Siruri si serii de funct ii: serii Taylor, serii Fourier 122
12 Funct ii complexe 170
III. Matematici discrete 195
13 Combinatorica si grafuri 196
14 Aritmetica si teoria numerelor 232
Lucrarea a fost elaborata dupa cum urmeaza:
Capitolul 7. Vasile Pop, Mircea Olteanu
Capitolul 8. Liliana Popa
Capitolul 9. Dorian Popa, Vasile Pop
Capitolul 10. Dorian Popa
Capitolul 11. Mircea Olteanu, Radu Strugariu
Capitolul 12. Liliana Popa
Capitolul 13. Monica Burlica, Mihai Ispas
Capitolul 14. Gabriel Mincu
Prefat a
Cartea de fat a a fost elaborata n cadrul proiectului POSDRU/56/1.2/S/32768, For-
marea cadrelor didactice universitare si a student ilor n domeniul utilizarii unor instru-
mente moderne de predare-nvat are-evaluare pentru disciplinele matematice, n vederea
crearii de competent e performante si practice pentru piat a muncii.
Finant at din Fondul Social European si implementat de catre Ministerul Educat iei,
Cercetarii, Tineretului si Sportului, n colaborare cu The Red Point, Oameni si Com-
panii, Universitatea din Bucuresti, Universitatea Tehnica de Construct ii din Bucuresti,
Universitatea Politehnica din Bucuresti, Universitatea din Pitesti, Universitatea Tehnica
Gheorghe Asachi din Iasi, Universitatea de Vest din Timisoara, Universitatea Dunarea
de Jos din Galat i, Universitatea Tehnica din Cluj-Napoca, Universitatea 1 Decembrie
1918 din Alba-Iulia, proiectul contribuie n mod direct la realizarea obiectivului general
al Programului Operat ional Sectorial de Dezvoltare a Resurselor Umane - POSDRU si se
nscrie n domeniul major de intervent ie 1.2 Calitate n nvat amantul superior.
Proiectul are ca obiectiv adaptarea programelor de studii ale disciplinelor matematice
la cerint ele piet ei muncii si crearea de mecanisme si instrumente de extindere a oportu-
nitat ilor de nvat are.
Evaluarea nevoilor educat ionale obiective ale cadrelor didactice si student ilor legate de uti-
lizarea matematicii n nvat amantul superior, masterate si doctorate precum si analizarea
ecacitat ii si relevant ei curriculelor actuale la nivel de performant a si ecient a, n vederea
dezvoltarii de cunostint e si competent e pentru student ii care nvat a discipline matematice
n universitat i, reprezinta obiective specice de interes n cadrul proiectului. Dezvoltarea
si armonizarea curriculelor universitare ale disciplinelor matematice conform exigent elor
de pe piat a muncii, elaborarea si implementarea unui program de formare a cadrelor
didactice si a student ilor interesat i din universitat ile partenere bazat pe dezvoltarea si
armonizarea de curriculum, crearea unei baze de resurse inovative, moderne si funct ionale
pentru predarea-nvat area-evaluarea n disciplinele matematice pentru nvat amantul uni-
versitar sunt obiectivele specice care au ca raspuns materialul de fat a.
Formarea de competent e cheie n matematica si informatica presupune crearea de abilitati
de care ecare individ are nevoie pentru dezvoltarea personala, incluziune sociala si insert ie
pe piat a muncii. Se poate constata nsa ca programele disciplinelor de matematica nu au
ntotdeauna n vedere identicarea si sprijinirea elevilor si student ilor potent ial talentat i la
matematica. Totusi, studiul matematicii a evoluat n exigent e pana la a ajunge sa accepte
provocarea de a folosi noile tehnologii n procesul de predare-nvat are-evaluare pentru a
face matematica mai atractiva.

In acest context, analiza exibilitat ii curriculei, nsot ita de
analiza metodelor si instrumentelor folosite pentru identicarea si motivarea student ilor
talentat i la matematica ar putea raspunde deopotriva cerint elor de masa, cat si celor de
elita.
Viziunea pe termen lung a acestui proiect preconizeaza determinarea unor schimbari n
abordarea fenomenului matematic pe mai multe planuri: informarea unui numar cat mai
Prefat a
mare de membri ai societat ii n legatura cu rolul si locul matematicii n educat ia de baza,
n instruct ie si n descoperirile stiint ice menite sa mbunatat easca calitatea viet ii, inclu-
siv popularizarea unor mari descoperiri tehnice, si nu numai, n care matematica cea mai
avansata a jucat un rol hotarator. De asemenea, se urmareste evident ierea a noi motivat ii
solide pentrunvat area si studiul matematicii la nivelele de baza si la nivel de performant a;
stimularea creativitat ii si formarea la viitorii cercetatori matematicieni a unei atitudini de-
schise fat a de nsusirea aspectelor specice din alte stiint e, n scopul participarii cu succes
n echipe mixte de cercetare sau a abordarii unei cercetari inter si multi disciplinare;
identicarea unor forme de pregatire adecvata de matematica pentru viitorii student i ai
disciplinelor matematicen scopul utilizarii la nivel de performant a a aparatului matematic
n construirea unei cariere profesionale.
Introducere
Concursurile de matematica, nat ionale si internat ionale pentru elevi au o tradit ie
ndelungata, primul concurs internat ional ind organizat la init iativa Romaniei, n
Romania n anul 1959 (Olimpiada Internat ionala de Matematica).

In tot i acesti ani, la
nivelul matematicii preuniversitare s-a ajuns la o programa de concurs comuna, unanim
acceptata de toate t arile participante la OIM (n prezent peste 120 de t ari) iar concur-
sul reprezinta pentru mult i dintre participant i cel mai important test de vericare al
nivelului pregatirii matematice si n acelasi timp un barometru pentru nivelul matematicii
competit ionale al t arii din care provin.
Este de dorit ca si la nivel universitar competit iile internat ionale sa urmeze modelul
OIM, n special ca forma de organizare si ca programa de concurs general acceptata si
cunoscuta.
La nivel universitar concursurile de matematica s-au desfasurat foarte mult timp doar
la nivel nat ional n diverse t ari si n multe cazuri sporadic. Cea mai veche competit ie
nat ionala cu desfasurare nentrerupta este concursul Putnam, organizat n Statele Unite
ale Americii ncepand cu anul 1938.

In Romania, Concursul Nat ional Student esc Traian
Lalescu s-a desfasurat la mai multe discipline, s-a ntrerupt n perioada 1992-2006 si a
fost reluat din 2007 la matematica.
Cea mai importanta competit ie internat ionala de matematica pentru student i este
IMC (International Mathematics Competition for University Students) care se organizeaza
itinerant din 1994 ind echivalentul Olimpiadei Internat ionale de Matematica la nivel
universitar.

In ultimii ani la aceasta competit ie participa peste 300 de student i din peste 70
de universitat i si peste 30 de t ari. Competit ia este individuala iar ecare echipa reprezinta o
universitate (nu o t ara). Dicultatea problemelor date n concurs este deosebit de ridicata,
iar rezultatul este edicator: concursul se desfasoara pe durata a doua zile si se dau 5 sau
6 probleme n ecare zi.

Incepand din 2007 se desfasoara Concursul Internat ional Student esc SEEMOUS (South
Eastern European Mathematical Olympiad for University Students), analogul Olimpiadei
Balcanice de Matematica pentru elevi, la care au participat n ecare an student i de la
universitat i din Romania (Bucuresti, Cluj-Napoca, Iasi, Timisoara).
Aceasta culegere de probleme a fost gandita pentru a pune la dispozit ia student ilor
din Romania un material necesar pentru o buna pregatire matematica n vederea ridicarii
nivelului pregatirii obisnuite la nivel competit ional (nat ional sau internat ional). La
elaborarea cart ii au fost implicat i profesori cu experient a la concursurile nat ionale si
internat ionale student esti.

In elaborarea programei care sta la baza culegerii am decis, dupa discut ii cu


reprezentant i ai majoritat ii universitat ilor din t ara, sa folosim curricula concursurilor
internat ionale de matematica la care student ii de la universitat ile din Romania participa
cel mai frecvent.
Problemele au fost mpart ite pe teme n 14 capitole:
Introducere
Algebra - capitolele 1 si 2,
Algebra liniara - capitolele 3, 4, 5,
Geometrie analitica - capitolul 6,
Analiza reala (funct ii de o variabila) - capitolele 7, 8, 9,
Analiza matematica (funct ii de mai multe variabile) - capitolul 10,
Siruri si serii de funct ii - capitolul 11,
Funct ii complexe - capitolul 12,
Matematici discrete - capitolele 13 si 14.
Fiecare capitol ncepe cu o prezentare a not iunilor si rezultatelor necesare rezolvarii
problemelor, urmata de un numar sucient de probleme rezolvate, unele clasice, dar sem-
nicative, altele pentru antrenament si altele selectate din concursurile internat ionale sau
nat ionale ale altor t ari ca: Rusia, Frant a, Iran, S.U.A., Ungaria, Cehia, Israel.
Culegerea cont ine peste 600 de probleme cu rezolvari complete, o lista de peste 50 de
titluri bibliograce (cart i editate n t ara sau n strainatate), precum si o lista de adrese
de Internet ale diverselor concursuri internat ionale student esti. Dupa cunostint a autorilor
aceasta culegere este prima n lume care trateaza o astfel de tematica la modul general,
neind dedicata doar unui anumit concurs.
Fiecare capitol al culegerii a fost elaborat de unul sau doi dintre cei 11 autori si ecare
a putut contribui cu probleme la orice alt capitol. De coordonarea ntregii culegeri si
nalizarea ei s-au ocupat conf. dr. Vasile Pop de la Universitatea Tehnica din Cluj-Napoca
si conf. dr. Cornel Baet ica de la Universitatea din Bucuresti.
Capitolul 7
Siruri si serii numerice
Denit ii si rezultate
Teorema Stolz-Cesaro 1. Fie (a
n
)
n0
, (b
n
)
n0
doua siruri de numere reale cu pro-
prietat ile urmatoare:
1) (b
n
)
n0
este strict monoton si nemarginit;
2) exista lim
n
a
n+1
a
n
b
n+1
b
n
= l, l R.
Atunci lim
n
a
n
b
n
= l.
Teorema Stolz-Cesaro 2. Fie (a
n
)
n0
, (b
n
)
n0
doua siruri de numere reale cu pro-
prietat ile urmatoare:
1) lim
n
a
n
= lim
n
b
n
= 0;
2) sirul (b
n
)
n0
este strict monoton;
3) exista lim
n
a
n+1
a
n
b
n+1
b
n
= l, l R.
Atunci lim
n
a
n
b
n
= l.
Corolar. Fie (a
n
)
n0
un sir de numere pozitive cu proprietatea ca exista
lim
n
a
n+1
a
n
= l, l R. Atunci lim
n
n

a
n
= l.
Teorema. Fie (a
n
)
n1
un sir de numere pozitive cu proprietatea ca exista lim
n
a
n+1
a
n
=
l. Atunci, daca l < 1 lim
n
a
n
= 0, iar dac a l > 1 lim
n
a
n
= .
Fie

n=1
a
n
o serie de numere reale. Sirul (s
n
)
n1
, unde s
n
=
n

k=1
a
k
, se numeste sirul
sumelor part iale ale seriei.
Daca exista limita sirului (s
n
)
n1
, atunci ea se numeste suma seriei.
Daca sirul sumelor part iale este convergent si lim
n
s
n
= s, atunci se spune ca seria

n=1
a
n
este convergenta si se scrie

n=1
a
n
= s.
Daca seria

n=1
[a
n
[ este convergenta se spune ca seria

n=1
a
n
este absolut conver-
1
2
genta.
O serie care este convergenta, dar nu este absolut convergenta se numeste serie
semiconvergenta.
Observat ii. a) Dintr-o serie data

n=1
a
n
se pot obt ine alte serii, prin schimbarea ordinei
termenilor (

n=1
a
(n)
, : N

bijectiva) sau prin asocierea unor termeni (

n=1
(a
f(n)+1
+
a
f(n)+2
+ + a
f(n+1)
), unde f : N

este o funct ie strict crescatoare).



In general,
aceste transformari pot schimba suma seriei si chiar natura seriilor.

In cazul seriilor absolut convergente avem:


Teorema. Daca ntr-o serie absolut convergenta schimbam ordinea termenilor sau asociem
secvent e de termeni, seria obt inuta are aceeasi suma cu seria init iala.

In cazul seriilor semiconvergente situat ia este complet diferita dupa cum arata
urmatoarea:
Teorema (Riemann).

Intr-o serie semiconvergenta se poate schimba ordinea termenilor
n asa fel ncat seria sa e divergenta sau sa e convergenta cu suma un numar real ar-
bitrar.
b) Pentru ecare numar natural m N

denim seria rest de ordin m prin R


m
=

n=m
a
n
.
Seria

n=1
a
n
are aceeasi natura cu orice serie rest a ei.
c) Daca seria

n=1
a
n
este convergenta, atunci sirul (a
n
)
n
este convergent la zero.
Un criteriu de divergent a este urmatorul:
C0. Daca sirul (a
n
)
n
nu converge la zero, atunci seria

n=1
a
n
este divergenta.
Seria geometrica
Daca q este un numar real, atunci seria

n=0
q
n
se numeste seria geometrica de rat ie q.
Pentru q (1, 1) seria geometrica este convergenta si suma ei este

n=0
q
n
=
1
1 q
.
Pentru q 1 seria este divergenta si are suma .
Pentru q 1 seria este divergenta si nu are suma.
Seria armonica generalizata
Daca este un numar real, atunci seria

n=1
1
n

se numeste serie armonica generalizata


de exponent .
Pentru > 1 seria armonica

n=1
1
n

este convergenta si suma ei se noteaza

n=1
1
n

=
(). Funct ia : (1, ) R se numeste funct ia zeta a lui Riemann. Pentru 1 seria
Siruri si serii numerice 3
armonica

n=1
1
n

este divergenta si are suma .


Criterii generale de convergent a
C1. (Criteriul general al lui Cauchy) Seria

n=1
a
n
este convergenta daca si numai
daca, pentru orice > 0 exista un rang N() N astfel ca pentru orice n N() si orice
p 1 sa avem:
[a
n+1
+a
n+2
+ +a
n+p
[ < .
C2. (Criteriul lui Abel-Dirichlet) Daca seria

n=1
a
n
are sirul sumelor part iale
marginit, iar sirul (b
n
)
n
este descrescator la zero, atunci seria

n=1
a
n
b
n
este convergenta.
C3. (Criteriul lui Abel) Daca seria

n=1
a
n
este convergenta iar sirul (b
n
)
n
este
monoton si marginit, atunci seria

n=1
a
n
b
n
este convergenta.
C4. (Criteriul lui Leibniz) Daca sirul (b
n
)
n1
este monoton si convergent la zero,
atunci seria

n=1
(1)
n
b
n
este convergenta.
Criterii de convergent a pentru serii cu termeni pozitivi

In urmatoarele criterii (C4-C10) termenii seriilor care apar sunt strict pozitivi.
A. Criterii intrinseci
C4. Criteriul raportului (dAlembert)
a) Daca exista q (0, 1) si N N

astfel ca
a
n+1
a
n
q pentru orice n > N, atunci seria

n=1
a
n
este convergenta.
b) Daca exista N N

astfel ca
a
n+1
a
n
1 pentru orice n > N, atunci seria

n=1
a
n
este divergenta.
C4. Daca exista limita lim
n
a
n+1
a
n
= l atunci:
a) pentru l [0, 1) seria

n=1
a
n
este convergenta;
b) pentru l (1, ) seria

n1
a
n
este divergenta;
c) pentru l = 1 criteriul este inecient.
C5. Criteriul radicalului (Cauchy)
4
a) Daca exista q (0, 1) si N N

astfel ca
n

a
n
q pentru orice n > N, atunci seria

n=1
a
n
este convergenta.
b) Daca exista o innitate de termeni pentru care
n

a
n
1 atunci seria este divergenta.
C5. Daca exista lim
n
n

a
n
= l atunci:
a) pentru l [0, 1) seria

n=1
a
n
este convergenta;
b) pentru l (1, ) seria

n1
a
n
este divergenta;
c) pentru l = 1 criteriul este inecient.
C6. Criteriul Raabe-Duhamel
a) Daca exista un numar real c > 1 si un numar natural N N

astfel ca
n
_
a
n
a
n+1
1
_
c, pentru orice n N,
atunci seria

n=1
a
n
este convergenta.
b) Daca exista un numar natural N pentru care
n
_
a
n
a
n+1
1
_
1, pentru orice n N,
atunci seria

n1
a
n
este divergenta.
C6. Daca exista limita lim
n
n
_
a
n
a
n+1
1
_
= l atunci:
a) pentru l > 1 seria

n=1
a
n
este convergenta;
b) pentru l < 1 seria

n1
a
n
este divergenta;
c) pentru l = 1 criteriul este inecient.
Observat ie.

In general criteriul Raabe-Duhamel se aplica la serii la care criteriul
raportului sau radicalului este inecient.
C7. Criteriul condensarii (Cauchy)
Daca sirul (a
n
)
n
este descrescator, atunci seriile

n=1
a
n
si

n=1
2
n
a
2
n au aceeasi natura
(sunt simultan convergente sau divergente).
B. Criterii de comparat ie
C8. Daca exista N N

astfel ca 0 < a
n
b
n
pentru orice n > N, atunci:
a) Daca seria

n=1
a
n
este divergenta, atunci seria

n=1
b
n
este divergenta.
Siruri si serii numerice 5
b) Daca seria

n=1
b
n
este convergenta, atunci seria

n=1
a
n
este convergenta.
C9. Daca exista N N

astfel ca
a
n+1
a
n

b
n+1
b
n
pentru orice n > N, atunci:
a) Daca seria

n=1
a
n
este divergenta, atunci seria

n=1
b
n
este divergenta.
b) Daca seria

n=1
b
n
este convergenta, atunci seria

n=1
a
n
este convergenta.
C10. Daca exista lim
n
a
n
b
n
= l atunci:
a) pentru l (0, ) seriile

n=1
a
n
si

n=1
b
n
au aceeasi natura;
b) pentru l = 0 avem implicat iile:

n=1
a
n
divergenta

n=1
b
n
divergenta;

n=1
b
n
convergenta

n=1
a
n
convergenta;
c) pentru l = avem implicat iile:

n=1
b
n
divergenta

n=1
a
n
divergenta;

n=1
a
n
convergenta

n=1
b
n
convergenta.
Observat ie.

In general pentru a decide natura unei serii

n=1
a
n
prin criteriul C10 se
folosesc pentru comparat ie serii armonice generalizate. Se obt ine criteriul 10.
C10. Daca exista R astfel ca
lim
n
n

a
n
= l (0, )
atunci:
a) pentru > 1 seria

n=1
a
n
este convergenta;
b) pentru 1 seria

n=1
a
n
este divergenta.
Produsul Cauchy a doua serii
Denit ie. Daca

n=1
a
n
si

n=1
b
n
sunt doua serii, atunci seria

n=1
c
n
cu termenul general
c
n
= a
1
b
n
+a
2
b
n1
+a
3
b
n2
+ +a
n
b
1
, n 1, se numeste produsul Cauchy al celor doua
serii.
Observat ie.

In general produsul Cauchy a doua serii convergente nu este neaparat o
serie convergenta (a
n
= b
n
=
(1)
n1

n
).
Teorema (Mertens). Daca seriile

n=1
a
n
si

n=1
b
n
sunt convergente, iar una din ele
6
este absolut convergenta, atunci produsul lor Cauchy

n=1
c
n
este o serie convergenta si
daca

n=1
a
n
= A,

n=1
b
n
= B, atunci

n=1
c
n
= AB.
Siruri. Probleme
Problema 7.1 Fie I R si f : I I. Denim sirul (a
n
)
n0
prin relat ia a
n+1
= f(a
n
),
n 0, a
0
I. Sa se arate ca:
1) Daca f este crescatoare, atunci (a
n
)
n0
este monoton;
2) Daca f este descrescatoare, atunci sirurile (a
2n
)
n0
, (a
2n+1
)
n0
sunt monotone si
au monotonii diferite.
Solut ie. 1) Daca a
0
a
1
rezulta ca f(a
0
) f(a
1
), adica a
1
a
2
si apoi prin induct ie
se arata ca a
n
a
n+1
pentru orice n 0. Daca a
0
a
1
rezulta analog ca sirul este
descrescator.
2) Avem
a
2n+1
= f(a
2n+1
) = (f f)(a
2n
), n 0
si
a
2n+2
= f(a
2n+1
) = (f f)(a
2n
), n 0.
Cum g = f f este crescatoare, din punctul 1) rezulta ca (a
2n
)
n0
si (a
2n+1
)
n0
sunt
siruri monotone. Daca presupunem ca (a
2n
)
n0
este crescator, din relat ia a
2n
a
2n+2
obt inem f(a
2n
) f(a
2n+1
) echivalent cu a
2n+1
a
2n+3
, n 0, ceea ce arata ca (a
2n+1
)
n0
este descrescator. Presupunerea ca (a
2n
)
n0
este descrescator conduce n mod analog la
faptul ca (a
2n+1
)
n0
este crescator. Deci sirurile (a
2n
)
n0
si (a
2n+1
)
n0
au monotonii
diferite.
Problema 7.2 a) Sa se arate ca lim
n
_
1
n + 1
+
1
n + 2
+ +
1
2n
_
= ln 2;
b) Sa se calculeze lim
n
n
_
1
n + 1
+
1
n + 2
+ +
1
2n
ln 2
_
.
Solut ie. a) Fie c
n
= 1 +
1
2
+ +
1
n
ln n, n 0. Avem
x
n
=
1
n + 1
+
1
n + 2
+ +
1
2n
ln 2 = (c
2n
c
n
) + ln 2n ln n =
= c
2n
c
n
+ ln 2,
de unde obt inem lim
n
x
n
= ln 2.
b) Fie y
n
=
1
n + 1
+
1
n + 2
+ +
1
2n
ln 2
1
n
, n 1,
a
n
=
1
n + 1
+
1
n + 2
+ +
1
2n
ln 2, b
n
=
1
n
.
Siruri si serii numerice 7
Condit iile celei de-a doua teoreme a lui Stolz-Cesaro sunt ndeplinite si avem
lim
n
a
n+1
a
n
b
n+1
b
n
lim
n

1
n + 1
+
1
2n + 1
+
1
2n + 2
1
n + 1

1
n
=
1
4
de unde rezulta ca lim
n
y
n
=
1
4
.
Problema 7.3 Fie f : [1, ) R o funct ie descrescatoare si marginita inferior. Sa se
arate ca sirul (a
n
)
n1
de termen general
a
n
= f(1) +f(2) + +f(n)
_
n
1
f(x)dx
este convergent.
Solut ie. Studiem monotonia lui (a
n
)
n1
. Avem
a
n+1
a
n
= f(n + 1)
_
n+1
1
f(x)dx +
_
n
1
f(x)dx =
= f(n + 1)
_
n+1
n
f(x)dx =
_
n+1
n
(f(n + 1) f(x))dx 0,
t inand seama ca f este descrescatoare. Rezulta ca sirul (a
n
)
n1
este descrescator. Demon-
stram ca sirul este marginit inferior. Avem
a
n
=
_
f(1)
_
2
1
f(x)dx
_
+
_
f(2)
_
3
2
f(x)dx
_
+ +
+
_
f(n 1)
_
n
n1
f(x)dx
_
+f(n) =
=
_
2
1
(f(1) f(x))dx +
_
3
2
(f(2) f(x))dx + +
+
_
n
n1
(f(n 1) f(x))dx +f(n),
de unde rezulta ca (a
n
)
n1
este marginit inferior, t inand seama de monotonia lui f si
de faptul ca f este marginita inferior. Prin urmare sirul (a
n
)
n1
este convergent, ind
monoton si marginit.
Observat ie. Pentru funct ia f : [1, ) R, f(x) =
1
x
, rezulta imediat ca sirul (c
n
)
n1
,
c
n
= 1 +
1
2
+ +
1
n
ln n
este convergent.
Problema 7.4 Sa se calculeze lim
n
[(n + 1)
n+1

n + 1 n
n

n].
8
Solut ie. Consideram funct ia f : [n, n+1] R, n N

, f(x) = x
1+
1
x
, careia i aplicam
teorema lui Lagrange. Rezulta ca exista c
n
(n, n + 1) astfel ca
f(n + 1) f(n) = c
1
cn
n
_
1
c
n
+ 1
ln c
n
c
n
_
.
Din c
n
> n rezulta ca lim
n
c
n
= si n continuare
lim
n
[f(n + 1) f(n)] = 1.
Problema 7.5 Demonstrat i ca daca sin x ,= 0, atunci sirul (sin nx)
n0
nu are limita.
Solut ie. Sa presupunem ca sirul (sin nx)
n0
este convergent. Din
cos nx =
sin(n + 1)x sin(n 1)x
2 sin x
rezulta ca lim
n
cos nx = 0.
T inand seama de relat ia
sin nx =
cos(n + 1)x cos(n 1)x
2 sin x
deducem ca lim
n
sin nx = 0, prin urmare lim
n
(sin
2
nx+cos
2
nx) = 0, contradict ie. Rezulta
ca sirul (sin nx)
n0
este divergent.
Problema 7.6 Sa se determine cel mai mic numar real pozitiv x pentru care sirul (a
n
)
n1
,
a
n
=
_
1 +
1
n
_
n+x
este descrescator.
Solut ie. Consideram funct ia f : [1, ) R, f(t) = (t +x) ln
_
1 +
1
t
_
, t 1. Evident
a
n
= e
f(n)
, n 1. Avem
f

(t) = ln
_
1 +
1
t
_

t +x
t(1 +t)
,
f

(t) =
t(2x 1) +x
t
2
(1 +t)
2
.
Daca x
1
2
rezulta f

(t) 0 pentru orice t 1, deci f

este strict crescatoare


pe [1, ). Cum lim
t
f

(t) = 0 rezulta f

(t) < 0, t 1, deci f este descrescatoare pe


[1, ). Rezulta ca (a
n
)
n1
este un sir descrescator pentru x
1
2
. Daca x <
1
2
, atunci
ecuat ia f

(t) = 0 are radacina t


0
=
x
1 2x
si f

(t) 0 pentru t t
0
. Rezulta ca f

este descrescatoare pe [t
0
, ) si cum lim
t
f

(t) = 0 avem f

(t) > 0 pentru t t


0
. Prin
urmare sirul (a
n
) este crescator pentru n > t
0
. Cel mai mic numar pentru care (a
n
)
n1
este descrescator este x =
1
2
.
Problema 7.7 Sa se arate ca dac a lim
n
a
n
n
= a, lim
n
b
n
n
= b, a, b > 0, atunci pentru orice
p 0, q 0 cu p +q = 1, are loc relat ia
lim
n
(pa
n
+qb
n
)
n
= a
p
b
q
.
Siruri si serii numerice 9
Solut ie. Aratam mai ntai ca lim
n
a
n
= 1 si lim
n
b
n
= 1. De aici deducem ca
lim
n
(pa
n
+qb
n
) = 1.
Apoi avem
lim
n
n(a
n
1) = ln a, lim
n
n(b
n
1) = ln b
si n continuare
lim
n
(pa
n
+qb
n
)
n
= e
lim
n
n(pan+qbn1)
=
= e
lim
n
[pn(an1)+qn(bn1)]
= e
p lna+q lnb
= a
p
b
q
.
Problema 7.8 Sa se calculeze lim
n
_
e
1+
1
2
++
1
n+1
e
1+
1
2
++
1
n
_
.
Solut ie. Fie c
n
= 1 +
1
2
+ +
1
n
ln n. Avem
x
n
= e
1+
1
2
++
1
n+1
e
1+
1
2
++
1
n
= e
1+
1
2
++
1
n
_
e
1
n+1
1
_
=
= e
cn+ln n
_
e
1
n+1
1
_
= e
cn

n
n + 1

e
1
n+1
1
1
n + 1
.
Rezulta ca lim
n
x
n
= e
c
, unde c este constanta lui Euler.
Problema 7.9 Sa se arate ca urmatoarele siruri sunt convergente, folosind problema 7.3.
a) a
n
= 1 +
1
2
+ +
1
n
ln n;
b) a
n
=
1
2 ln 2
+
1
3 ln 3
+ +
1
nln n
ln(ln n);
c) a
n
= 1 +
1
2

+ +
1
n


1
1
n
1
, (0, 1);
d) a
n
= 1 +
1
2

+ +
1
n

, > 1.
Solut ie. a) Se ia f(x) =
1
x
;
b) f(x) =
1
xln x
;
c) f(x) =
1
x

;
d) f(x) =
1
x

.
Problema 7.10 Sa se calculeze limitele urmatoarelor siruri:
a) a
n
=
1
ln n
_
1 +
1
2
+ +
1
n
_
, n 2;
b) a
n
=
1
ln(ln n)
_
1
2 ln 2
+
1
3 ln 3
+ +
1
nln n
_
, n 3;
c) a
n
=
1
n
1
_
1 +
1
2

+ +
1
n

_
, (0, 1).
Solut ie. Se utilizeaza prima teorema a lui Stolz-Cesaro obt inandu-se:
a) lim
n
a
n
= 1;
b) lim
n
a
n
= 1;
c) lim
n
a
n
=
1
1
.
10
Problema 7.11 Daca notam cu a limitele sirurilor de la exercit iul 7.9 sa se calculeze
limitele urmatoare:
a) lim
n
n
_
1 +
1
2
+ +
1
n
ln n a
_
;
b) lim
n
nln n
_
1
2 ln 2
+
1
3 ln 3
+ +
1
nln n
ln(ln n) a
_
;
c) lim
n
n

_
1 +
1
2

+ +
1
n


1
1
n
1
a
_
, (0, 1);
d) lim
n
n
1
_
1 +
1
2

+ +
1
n

a
_
, > 1.
Solut ie. Se aplica a doua teorema a lui Stolz-Cesaro.
a) x
n
= 1 +
1
2
+ +
1
n
ln n a, y
n
=
1
n
, n 1. Avem
lim
n
x
n
y
n
= lim
n
x
n+1
x
n
y
n+1
y
n
= lim
n
1
n + 1
ln(n + 1) + ln n
1
n + 1

1
n
=
= lim
x
xR
1
x + 1
ln(x + 1) + ln x
1
x + 1

1
x
= lim
x
xR

1
(x + 1)
2

1
x + 1
+
1
x

1
(x + 1)
2
+
1
x
2
=
1
2
;
b) Se obt ine limita 1;
c) Aplicand teorema a doua a lui Cesaro-Stolz obt inem
lim
n
n

_
1 +
1
2

+ +
1
n


1
1
n
1
a
_
=
= lim
n
1
(n + 1)


1
1
[(n + 1)
1
n
1
]
1
(n + 1)


1
n

=
= lim
n
1
_
(n + 1) n
_
n + 1
n
_

_
(1 )
n

(n + 1)

=
= lim
x0
xR
(1 )x (1 +x) + (1 +x)

x[1 (1 +x)

](1 )
=
1
2
aplicand regula lui lHospital de doua ori;
d) Se obt ine limita
1
1
.
Problema 7.12 Sa se arate ca daca p, q N

, p < q, au loc relat iile:


a) lim
n
qn

k=pn
1
k
= ln
q
p
;
b) lim
n
1
n
q
n

k=p
n
1
k
= ln
q
p
;
Siruri si serii numerice 11
c) lim
n
1
ln n
n
q

k=n
p
1
k
= q p;
d) lim
n
q
n

k=p
n
1
k ln k
= ln
_
ln q
ln p
_
;
e) lim
n
n
q

k=n
p
1
k ln k
= ln
q
p
;
Solut ie. Fie (a
n
)
n1
un sir de numere reale, s
n
= a
1
+a
2
+ +a
n
, n 1 si (b
n
)
n1
un sir cu proprietatea ca sirul (s
n
b
n
)
n1
este convergent. Daca (p
n
)
n1
, (q
n
)
n1
sunt
doua siruri de numere naturale, p
n
q
n
pentru n 1, atunci
qn

k=pn
a
k
= s
qn
s
pn
+a
pn
= (s
qn
b
qn
) (s
pn
b
pn
) + (b
qn
b
pn
) +a
pn
.
De aici obt inem
lim
n
qn

k=pn
a
k
= lim
n
[(b
qn
b
pn
) +a
pn
]
n ipoteza ca limita din dreapta exista.
a) p
n
= pn, q
n
= qn, b
n
= ln n,
lim
n
_
ln qn ln pn +
1
pn
_
= ln
q
p
.
b) p
n
= p
n
, q
n
= q
n
, a
k
=
1
k
, b
n
= ln n.
Pentru c), d), e) procedam analog.
Problema 7.13 Fie (a
n
)
n1
si (b
n
)
n1
doua siruri de numere ntregi cu proprietatea
0 < a
n
b
n
, n 1. Sa se arate ca
lim
n
a
n
b
n
bn

k=an
e
1
k
= 1.
Solut ie. ln
_
_
a
n
b
n
bn

k=an
e
1
k
_
_
=
bn

k=an
1
k
+ ln a
n
ln b
n
=
=
_
bn

k=1
1
k
ln b
n
_

_
an

k=1
1
k
ln a
n
_
+
1
a
n
c c + 0 = 0.
Problema 7.14 Demonstrat i ca
lim
n
_
1000
_
1 +
_
2 + +

n
_
= 1757.
Solut ie. Consideram sirurile (a
n
)
n5
, (b
n
)
n5
,
a
n
=
_
1 +
_
2 + +

n, b
n
=

1 +
_
2 + +
_
n +

2n.
12
Se arata usor ca (a
n
) este crescator, iar (b
n
) este descrescator si a
n
< b
n
, n 5, prin
urmare
1, 7575 < a
6
< lim
n
a
n
< b
6
= 1, 7579,
deci
lim
n
_
1000
_
1 +
_
2 + +

n
_
= 1757.
Problema 7.15 Fie a, b > 0 si (x
n
)
n1
, (y
n
)
n1
doua siruri de numere reale cu pro-
prietat ile:
lim
n
x
n
n
a
= A, lim
n
y
n
n
b
= B, A, B R.
Sa se calculeze
lim
n
(x
1
+x
2
+ +x
n
)(y
1
+y
2
+ +y
n
)
n(x
1
y
1
+x
2
y
2
+ +x
n
y
n
)
.
Solut ie.
(x
1
+ +x
n
)(y
1
+ +y
n
)
n(x
1
y
1
+ +x
n
y
n
)
=
x
1
+ +x
n
n
a+1

y
1
+ +y
n
n
b+1
x
1
y
1
+ +x
n
y
n
n
a+b+1
si
lim
n
x
1
+ +x
n
n
a+1
= lim
n
x
n+1
(n + 1)
a+1
n
a+1
=
A
a + 1
,
lim
n
y
1
+ +y
n
n
b+1
=
B
b + 1
,
lim
n
x
1
y
1
+ +x
n
y
n
n
a+b+1
= lim
n
x
n+1
y
n+1
(n + 1)
a+b+1
n
a+b+1
=
= lim
n
x
n+1
(n + 1)
a

y
n+1
(n + 1)
b
(n + 1)
a+b+1
n
a+b+1
(n + 1)
a+b
=
AB
a +b + 1
.
Limita ceruta este egala cu
a +b + 1
(a + 1)(b + 1)
.
Problema 7.16 (Transformarea Toeplitz) Fie c
n,k
: 1 k n, n 1 un sir dublu
de numere reale cu proprietat ile:
i) lim
n
c
n,k
= 0 pentru orice k N

;
ii) lim
n
n

k=1
c
n,k
= 1;
iii) exista c > 0 astfel ca
n

k=1
[c
n,k
[ c pentru orice n 1.
Atunci pentru orice sir convergent de numere reale (a
n
)
n1
, sirul (b
n
)
n1
denit prin
b
n
=
n

k=1
c
n,k
a
k
, n 1, este convergent si lim
n
b
n
= lim
n
a
n
.
Siruri si serii numerice 13
Solut ie. Daca a
n
= a pentru orice n 1, atunci din ii) avem
lim
n
b
n
= a lim
n
n

k=1
c
n,k
= a.
Astfel este sucient sa consideram cazul cand sirul (a
n
)
n1
converge la zero. Pentru
m > 1 si n m avem
(1) [b
n
0[ =

k=1
c
n,k
a
k

m1

k=1
[c
n,k
[ [a
k
[ +
n

k=m
[c
n,k
[ [a
k
[
Fie > 0. Din lim
n
a
n
= 0 rezulta ca exista n
1
N astfel ca [a
n
[ <

2c
pentru n n
1
.
Sirul (a
n
)
n1
este marginit si presupunem ca [a
n
[ D, pentru orice n 1. Din i) rezulta
ca exista n
2
N astfel ca pentru n n
2
n
1
1

k=1
[c
n,k
[ <

2D
.
Punand m = n
1
n (1), obt inem
[b
n
[ D
n
1
1

k=1
[c
n,k
[ +

2c
n

k=n
1
[c
n,k
[ <

2
+

2
=
pentru n maxn
1
, n
2
. Prin urmare lim
n
b
n
= 0.
Problema 7.17 Sa se demonstreze ca daca n exercit iul precedent c
nk
> 0, 1 k
n, n 1, atunci pentru orice sir (x
n
) cu limita , rezulta ca si transformata sa Toeplitz,
(y
n
), are limita .
Solut ie. Fie (x
n
) cu x
n
; se poate presupune ca tot i termenii termenii sirului (x
n
)
sunt strict pozitivi. Fie C > 0; din condit ia lim
n
n

k=1
c
nk
= 1, rezulta ca exista N
1
N
astfel ncat:
n

k=1
c
nk
>
1
2
, n N
1
.
Sirul (x
n
) ind nemarginit, exista N
2
N astfel ncat x
n
2C, n N
2
. Fie N
3
=
maxN
1
, N
2
; atunci, pentru orice n N
3
, avem:
n

k=1
c
nk
x
k
=
N
3

k=1
c
nk
x
k
+
n

k=N
3
c
nk
x
k

N
3

k=1
c
nk
x
k
+C > C,
ceea ce ncheie demonstrat ia.
Problema 7.18 Demonstrat i ca daca lim
n
a
n
= a, a R, atunci
lim
n
na
1
+ (n 1)a
2
+ +a
n
n
2
=
a
2
.
14
Solut ie. Se aplica teorema lui Toeplitz cu c
n,k
=
2(n k + 1)
n
2
sau se aplica teorema
Stolz-Cesaro de doua ori.
Problema 7.19 Daca lim
n
a
n
= a, lim
n
b
n
= b, a, b R, atunci
lim
n
a
1
b
n
+a
2
b
n1
+ +a
n
b
1
n
= ab.
Solut ie. Daca b ,= 0, luam c
n,k
=
b
nk+1
nb
n teorema lui Toeplitz.
Daca b = 0, punand c
n,k
=
1 +b
nk+1
n
, avem
lim
n
a
1
(1 +b
n
) +a
2
(1 +b
n1
) + +a
n
(1 +b
1
)
n
= a
si t inand seama ca lim
n
a
1
+ +a
n
n
= a rezulta concluzia.
Problema 7.20 Presupunem ca lim
n
a
n
= a, a R. Sa se calculeze:
a) lim
n
_
a
n
1
+
a
n1
2
+ +
a
1
2
n1
_
;
b) lim
n
_
a
1
1 2
+
a
2
2 3
+ +
a
m
n(n + 1)
_
;
c) lim
n
_
a
n
1

a
n1
2
+ + (1)
n1
a
1
2
n1
_
.
Solut ie. Se obt in, aplicand teorema lui Toeplitz, rezultatele:
a) 2a; b) a; c)
2
3
a.
Problema 7.21 Determinat i mult imea punctelor limita ale sirului (a
n
)
n1
, unde:
a) a
n
=
[1 (1)
n
] 2
n
+ 1
2
n
+ 3
;
b) a
n
=
_
cos
n
3
_
n
;
c) a
n
=
2n
2
7

_
2n
2
7
_
.
Solut ie. a) a
2n
=
1
2
n
+ 3
, a
2n+1
=
2
n+1
+ 1
2
n
+ 3
. Avem lim
n
a
2n
= 0 si lim
n
a
2n+1
= 2,
deci L(a
n
) = 0, 2;
b) L(a
n
) = 1, 0, 1;
c) a
7k
= 0, a
7k+1
=
2
7
, . . . , a
7k+6
=
2
7
. Se obt ine
L(a
n
) =
_
0,
1
7
,
2
7
,
4
7
_
.
Problema 7.22 Fie (a
n
)
n1
un sir de numere reale cu proprietatea ca lim
n
(a
n+1
a
n
) =
0. Aratat i ca mult imea punctelor limita ale lui (a
n
)
n1
este un interval nchis.
Solut ie. Fie a < b puncte limita ale sirului (a
n
)
n1
si c (a, b). Vom construi prin
recurent a un subsir (a
n
k
)
k1
avand limita c. Presupunand (a
n
k
)
k1
ales, e n
0
N astfel
Siruri si serii numerice 15
ca [a
n+1
a
n
[ <
1
k
, pentru n n
0
. Din faptul ca a, b sunt puncte limita ale lui (a
n
)
n1
,
rezulta ca exista p, q N, p, q > maxn
0
, n
k
cu proprietatea ca a
p
< c < a
q
. Notam cu
n
k+1
cel mai mare indice cuprins ntre p si q astfel ca c < a
n
k+1
+ 1. Rezulta ca
[a
n
k+1
c[ [a
n
k+1
a
n
k+1
+1
[
1
k
.
Aceasta construct ie arata ca mult imea punctelor limita ale lui (a
n
)
n1
este un interval.
Fie a o extremitate a acestui interval. Exista deci un sir (x
n
)
n1
format din puncte limita
pentru sirul (a
n
)
n1
astfel ca lim
n
x
n
= a. Este sucient sa alegem un subsir (a
n
k
)
k1
astfel ca [a
n
k
x
k
[
1
k
. Avem lim
n
a
n
k
= a, ceea ce ncheie demonstrat ia.
Problema 7.23 Fie f : R R o funct ie periodica cu perioada T > 0, continuan punctul
x R. Fie (S
n
)
n1
un sir satisfacand condit iile:
(i) lim
n
S
n
= ;
(ii) lim
n
(S
n+1
S
n
) = 0.
Atunci f(x) este un punct limita al sirului (f(S
n
))
n1
.
Solut ie. Deoarece f este continua n x, exista
1
> 0 astfel ncat [t x[ <
1
implica
[f(t)f(x)[ < 1. Cum lim
n
(S
n+1
S
n
) = 0, exista N
1
N astfel ncat pentru orice n N
1
sa avem [S
n+1
S
n
[ <
1
. Fie k
1
N cu proprietatea ca x+k
1
T S
N
1
. Din (i) rezulta ca
exista n
1
N, n
1
N
1
, astfel ncat S
n
1
x+k
1
T < S
n
1
+1
. Avem ca [x+k
1
T S
n
1
[ <
1
si atunci [(S
n
1
k
1
T) x[ <
1
, de unde [f(S
n
1
) f(x)[ = [f(S
n
1
k
1
T) f(x)[ < 1.
Deoarece f este continua n x, exista
2
> 0 astfel ncat [t x[ <
2
implica [f(t)
f(x)[ <
1
2
. Cum lim
n
(S
n+1
S
n
) = 0, exista N
2
N astfel ncat pentru orice n N
2
sa avem [S
n+1
S
n
[ <
2
. Fie k
2
N cu proprietatea ca x + k
2
T S
max(N
2
,n
1
+1)
. Din
(i) rezulta ca exista n
2
N, n
2
max(N
2
, n
1
+ 1), astfel ncat S
n
2
x + k
2
T < S
n
2
+1
.
Avem ca [x +k
2
T S
n
2
[ <
1
si atunci [(S
n
2
k
2
T) x[ <
2
, de unde [f(S
n
2
) f(x)[ =
[f(S
n
2
k
2
T) f(x)[ <
1
2
.
Continuand procedeul de mai sus vom obt ine un sir strict crescator (n
p
)
p1
care are
proprietatea ca [f(S
np
) f(x)[ <
1
p
si trecand la limita obt inem lim
p
[f(S
np
) f(x)[ = 0,
deci sirul (f(S
np
))
p1
converge la f(x).
Problema 7.24 Fie E
n
= 1 +
1
1!
+
1
2!
+ +
1
n!
, n 1.
Demonstrat i ca:
a) 0 < e E
n
<
1
n n!
, n 1;
b) e , Q;
c) lim
n
(n!e [n!e]) = 0.
Solut ie. a) E
m+n
E
n
=
1
(n + 1)!
+
1
(n +n)!
+ +
1
(n +m)!
<
<
1
(n + 1)!
_
1 +
1
n + 2
+
1
(n + 2)
2
+ +
1
(n + 2)
m1
_
<
1
(n + 1)!

n + 2
n + 1
Fixand n si facand m obt inem
e E
n

1
(n + 1)!

n + 2
n + 1
<
1
n n!
.
16
b) Sa presupunem ca e =
p
q
Q, p, q N, q ,= 0. Avem 0 < eE
q
<
1
q q!
si nmult ind
cu q! obt inem 0 < p(q 1)! q!E
q
<
1
q
, contradict ie, pentru ca (p(q 1)! q!E
q
) Z.
c) Din punctul a) rezulta ca pentru orice n 1 exista
n
]0, 1[ astfel ca
e = E
n
+

n
n n!
,
deci
[n!e] =
_
n!E
n
+

n
n
_
= n!E
n
,
deci
lim
n
(n!e [n!e]) = 0.
Problema 7.25 Sa se arate ca lim
n
nsin(2en!) = 2.
Solut ie. Din problema 7.24 a) rezulta ca pentru orice n N exista
n+1
(0, 1) astfel
ca
e = E
n+1
+

n+1
(n + 1)(n + 1)!
.
Avem
x
n
= nsin(2en!) = nsin
_
2
_
E
n+1
+

n+1
(n + 1)(n + 1)!
_
n!
_
=
nsin
_
2
_
E
n
+
1
(n + 1)!
+

n+1
(n + 1)(n + 1)!
_
n!
_
= nsin
_
2E
n
n! +
1
n + 1
+

n+1
(n + 1)
2
_
si cum n!E
n
N obt inem
x
n
= nsin
_
2
_
1
(n + 1)!
+

n+1
(n + 1)
2
__
=
sin
_
2(
1
(n+1)!
+

n+1
(n+1)
2
)
_
2
_
1
(n+1)!
+

n+1
(n+1)
2
_
_
2
n
n + 1
+
n
n+1
(n + 1)
2
_
,
deci lim
n
= 2.
Problema 7.26 Fie (a
n
)
n1
un sir de numere reale cu proprietat ile: 0 < a
n
1 pentru
orice n 1 si lim
n
(a
1
+a
2
+ +a
n
) = .
a) Sa se arate ca pentru orice l [1, ) exista o funct ie strict crescatoare
L : N

astfel ca
lim
n
a
1
+a
2
+ +a
L(n+1)
a
1
+a
2
+ +a
L(n)
= l.
b) Sa se determine funct ia L pentru a
n
=
1

n
, n 1.
Solut ie. a) Fie s
k
= a
1
+ + a
k
, k 1. Intervalele [s
k
, s
k+1
), k 1, determina o
partit ie a intervalului [a
1
, ).
1. Daca l > 1, atunci pentru orice n 1 exista un unic k N

astfel ca l
n
[s
k
, s
k+1
)
si denim funct ia L(n) = k, deci l
n
[s
L(n)
, s
L(n)+1
). Cum l
n+1
l
n
> 1 > a
L(n)+1
rezulta
s
L(n+1)
s
L(n)+1
si atunci L(n + 1) > L(n), deci L este funct ie strict crescatoare.
Avem:
s
L(n)
l
n
< s
L(n)+1
= s
L(n)
+a
L(n)+1
< s
L(n)
+ 1
Siruri si serii numerice 17
s
L(n+1)
l
n+1
< s
L(n+1)+1
din care deducem
l
n+1
l
n
<
s
L(n+1)
s
L(n)
<
l
n+1
l
n
1
,
de unde obt inem lim
n
s
L(n+1)
s
L(n)
= 1.
2. Daca l = 1, alegem L(n) = n si obt inem
lim
n
s
n+1
s
n
= 1 + lim
n
a
n+1
s
n
= 1.
3. Daca l = alegem L(n) astfel ca n
n
[s
L(n)
, s
L(n)+1
) si avem
s
L(n+1)
s
L(n)

(n + 1)
n+1
1
n
n
.
b) Sirul (a
n
)
n1
, a
n
= 1 +
1

2
+ +
1

n
2

n este convergent. Avem


s
L(n+1)
s
L(n)
=
a
L(n+1)
+ 2

a
L(n+1)
a
L(n)
+ 2

a
L(n)
,
lim
n
s
L(n+1)
s
L(n)
= lim
n

a
L(n+1)

a
L(n)
.
Pentru l = 1 alegem L(n) = n.
Pentru l > 1 alegem L(n) = [l
2n
].
Pentru l > 1 alegem L(n) = n
n
.
Problema 7.27 Fie a si b doua numere reale astfel ncat 0 < a < b. Denim sirurile:
a
1
=

ab, b
1
=
1
2
(a +b)
a
2
=
_
a
1
b
1
, b
2
=
1
2
(a
1
+b
1
)
. . . . . . . . .
a
n
=
_
a
n1
b
n1
, b
n
=
1
2
(a
n1
+b
n1
).
Sa se arate ca sirurile a
n
si b
n
sunt convergente si au aceeasi limita (numita media
aritmetico-geometrica a numerelor a si b).
Solut ie. Evident, din inegalitatea mediilor rezulta a
n
b
n
, n N si a < a
1
< b
1
< b.
Vom arata ca sirul (a
n
) este crescator, iar sirul b
n
este descrescator. Avem:
a
n+1
a
n
=
_
a
n
b
n
a
n
=
a
n
(b
n
a
n
)

a
n
b
n
+a
n
> 0, n N,
b
n+1
b
n
=
a
n
+b
n
2
b
n
=
a
n
b
n
2
< 0, n N.
Rezulta ca sirurile sunt convergente; daca notam L
1
= lim
n
a
n
si L
2
= lim
n
b
n
, atunci,
trecand la limita n relat ia a
n+1
=
1
2
(a
n
+b
n
), rezulta L
1
= L
2
.
18
Problema 7.28 Fie (x
n
) un sir de numere reale astfel ncat exista L R cu proprietatea:
lim
n
(2x
n+1
x
n
) = L
Sa se demonstreze ca lim
n
x
n
= L.
Solut ia 1. Fie > 0; din ipoteza, exista N() astfel ncat:
L < 2x
n+1
x
n
< L +, n N().
Fie n N() xat si e k N; nsumand inegalitat ile:
L < 2x
n+1
x
n
< L +, n N().
2(L ) < 4x
n+2
2x
n+1
< 2(L +)
......................
2
k1
(L ) < 2
k
x
n+k
2
k1
x
n+k1
< 2
k1
(L +),
Obt inem:
(1 + 2 +... + 2
k1
)(L ) < 2
k
x
n+k
x
n
< (1 + 2 +... + 2
k1
)(L +),
sau, echivalent (mpuart ind la 2
k
):
_
1 2
k
_
(L ) < x
n+k
2
k
x
n
<
_
1 2
k
_
(L +).
Alegem acum k astfel ncat:
[2
k
x
n
[ < si [2
k
(L )[ < .
Atunci, pentru orice p n +k (alesi ca mai sus), rezulta:
L 3 < x
m
< L + 3,
ceea ce ncheie demonstrat ia.
Solut ia 2. Scriem
L = lim
n
(2x
n+1
x
n
) = lim
n
2
n+1
x
n+1
2
n
x
n
2
n+1
2
n
.
Din teorema Cesaro-Stolz
lim
n
2
n+1
x
n+1
2
n
x
n
2
n+1
2
n
= lim
n
2
n
x
n
2
n
= lim
n
x
n
,
deci lim
n
x
n
= L.
Problema 7.29 Fie a si b doua numere pozitive. Sa se calculeze limita sirului (x
n
) denit
de relat ia:
x
n+1
=
_
a +bx
n
, n 1, x
1
=

a.

In particular, sa se calculeze:
lim
n

1 +
_
1 +
_
1 + +

1, (n radicali).
Siruri si serii numerice 19
Solut ie. Demonstram prin induct ie faptul ca (x
n
) este marginit, mai precis:
0 < x
n
<
b +

b
2
+ 4a
2
, n 1,
numarul
b+

b
2
+4a
2
ind solut ia pozitiva a ecuat iei x
2
bx a = 0. Evident, x
1
= a <
b+

b
2
+4a
2
; presupunand ca x
n
<
b+

b
2
+4a
2
, rezulta
x
n+1
=
_
a +bx
n
<

a +b
b +

b
2
+ 4a
2
=
b +

b
2
+ 4a
2
.
Demonstram ca x
n
este strict crescator; este evident ca:
x
2
=
_
a +b

a >

a = x
1
.
Relat ia x
n+1
> x
n
este echivalenta cu x
2
n
bx
n
a < 0. Ultima inegalitate este adevarata
deorece x
n
(0,
b+

b
2
+4a
2
).
Sirul (x
n
) este deci convergent si prin trecere la limita n relat ia de recurent a, rezulta
lim
n
x
n
=
b +

b
2
+ 4a
2
.
Problema 7.30 Sa se demonstreze formula lui Ramanujan:

1 + 2
_
1 + 3
_
1 + 4

1 +. . . = 3
Solut ie. Fie sirul de funct ii
f
1
(x) =

1 +x, f
2
(x) =
_
1 +x
_
1 + (x + 1), . . . ,
f
n
(x) =

1 +x
_
1 + (x + 1)
_
1 + + (x +n 2)
_
1 + (x +n 1) (n radicali)
Vom demonstra ca sirul (f
n
(x)) converge pentru orice x 1. Fie x 1, xat; evident,
(f
n
(x)) este crescator. Aratam n continuare ca este marginit. Evident:
f
n
(x)

x
_
x
_
. . .

x x
Pentru orice n N

si x 1, avem:
f
n
(x)

(x + 1)
_
(x + 2)
_
(x + 3) . . .
_
(x +n)

2x
_
3x
_
4x. . .
_
(n + 1)x

2x
_
4x
_
8x. . .

2
n
x =
= 2

n
k=1
k
2
k
x

n
k=1
1
2
k
4x.
20
Fie f(x) = lim
n
f
n
(x); din inegalitatea f(x) x, rezulta f(x) 2
1
(x + 1) si deci:
1
2
(x + 1) f(x) 4x, x 1.

Inlocuind x cu x + 1, rezulta:
1
2
(x + 2) f(x + 1) 4(x + 1), x 1.
Trecand la limita n relat ia de recurent a si apoi ridicand la patrat, obt inem:
(f(x))
2
= 1 +xf(x + 1)
Din dubla inegalitate de mai sus rezulta
x
1
2
(x + 2) + 1 (f(x))
2
4x(x + 1) + 1
Dupa calcule simple, obt inem:
2

1
2
(x + 1) f(x) 2(x + 1)
Repetam procedeul anterior, i.e. scriem inegalitatea anterioara pentru x+1, apoi nmult im
cu x si adunam 1:
2

1
2
x(x + 2) + 1 (f(x))
2
2x(x + 2) + 1
si dupa calcule rezulta:
2

1
2
2
(x + 1) f(x) 2
1
2
(x + 1)
Iterand de n ori, rezulta:
2

1
2
n
(x + 1) f(x) 2
1
2
n1
(x + 1), n = 1, 2, 3 . . .
Trecand la limita (n ) obt inem f(x) = x + 1.

In particular, pentru x = 2, se obt ine
formula lui Ramanujan:
_
1 + 2
_
1 + 3
_
1 + 4

1 +. . . = 3.
Problema 7.31 Sa se calculeze limita sirului:
n

k=1
_
k ln
_
2k + 1
2k 1
_
1
_
.
Solut ie. Termenul general se scrie:
n

k=1
_
k ln
_
2k + 1
2k 1
_
1
_
= ln
(2n + 1)
n
1 3 5 (2n 1) e
n
=
= ln
_
2n + 1
2n
_
n
+ ln
(2n)
n
1 3 5 (2n 1) e
n
=
= ln
_
2n + 1
2n
_
n
+ ln
4
n
n
n
n!
(2n)! e
n
Primul termen tinde la
1
2
; n al doilea termen nlocuim n! si (2n)! cu expresiile core-
spunzatoare din formula lui Stirling.

In nal obt inem limita
1
2
ln

2.
Siruri si serii numerice 21
Serii. Probleme
Sa se determine sumele seriilor:
Problema 7.32

n=1
(a + 1)(a + 2) . . . (a +n)
(b + 1)(b + 2) . . . (b +n)
, a > 0, b > a + 1.
Solut ie. Avem
a
n
=
(a + 1) . . . (a +n)
(b + 1) . . . (b +n)
= a
n1
a +n
b +n
,
din care rezulta
a
n1
(a +n) = a
n
(b +n)
sau
a
n1
(a +n) = a
n
[(a +n + 1) + (b a 1)],
deci
a
n1
(a +n) a
n
(a +n + 1) = (b a 1)a
n
.
Suma primilor termeni ai seriei este
S
n
=
n

k=1
a
k
=
1
b a 1
n

k=1
(f(n 1) f(n)) =
=
1
b a 1
(f(0) f(n)) =
1
b a 1
(a
0
a a
n
(a +n + 1)) =
=
1
b a 1
_
a
2
b
(a + 1)
(a + 2) . . . (a +n + 1)
(b + 1) . . . (b +n)
_
.
Deci
lim
n
S
n
=
a
2
b(b a 1)
(a + 1) lim
n
(a + 2) . . . (a +n + 1)
(b + 1) . . . (b +n)
.
Ultima limita o determinam astfel:
(a + 2) . . . (a +n + 1)
(b + 1) . . . (b +n)
=
=
1
_
1 +
b a 1
a + 2
__
1 +
b a 1
a + 2
_
. . .
_
1 +
b a 1
a +n + 1
_ <
<
1
b a 1
a + 2
+
b a 1
a + 3
+ +
b a 1
a +n + 1
=
=
1
b a 1

1
1
a + 2
+
1
a + 3
+ +
1
a +n + 1
care are limita zero caci seria

n=2
1
a +n
este divergenta (comparand-o cu seria armonica).
Deci

n=1
a
n
=
a
2
b(b a 1)
.
22
Problema 7.33

n=1
1
n

k=1
k
3
.
Solut ie.
n

k=1
k
3
=
n
2
(n + 1)
2
4
. Avem S
n
=
n

p=1
4
p
2
(p + 1)
2
=
= 4
n

p=1
_
2
_
1
p

1
p 1
_
+
1
p
2
+
1
(p + 1)
2
_
=
= 8
_
1
1
n + 1
_
+ 4
_
1 +
1
(n + 1)
2
+ 2
_
1 +
1
2
2
+ +
1
n
2
__
lim
n
S
n
= 4 + 8 lim
n
_
1 +
1
2
2
+ +
1
n
2
_
= 4 +
4
3

2
(suma seriei

n=1
1
n
2
este

2
6
).
Problema 7.34

n=1
_
a + 2
n
2
n+1
_
, a R.
Solut ie. Este cunoscuta identitatea:
_
a +
1
2
_
= [2a] [a], a R.
Avem
a
n
=
_
a + 2
n
2
n+1
_
=
_
a
2
n
_

_
a
2
n+1
_
,
S
n
=
n

k=1
a
k
= [a]
_
a
2
n+1
_
si
S
n
=
_
[a], daca a 0
[a] + 1, daca a < 0.
Problema 7.35

n=1
1
2
n
tg
a
2
n
, a R
_
2
n
_

2
+k
_
[ k, n Z
_
.
Solut ie. Avem identitatea tg x = ctg x 2ctg 2x si
a
n
=
1
2
n
tg
a
2
n
=
1
2
n
_
ctg
a
2
n
2ctg
a
2
n1
_
=
=
1
2
n
ctg
a
2
n

1
2
n1
ctg
a
2
n1
.
S
n
=
n

k=1
a
k
=
1
2
n
ctg
a
2
n
ctg a,
lim
n
S
n
= ctg a + lim
n
1
2
n
tg
a
2
n
= ctg a +
1
a
.
Siruri si serii numerice 23
Problema 7.36

n=0
(1)
n
cos
3
3
n
a
3
n
, a R.
Solut ie. Avem identitatea 4 cos
3
x = cos 3x + 3 cos x din care:
cos
3
3
n
a
3
n
=
1
4
_
cos 3
n+1
a
3
n
+
cos 3
n
a
3
n1
_
Suma primilor n termeni este
S
n
=
1
4
_
3 cos a + (1)
n
cos 3
n+1
a
3
n
_
si
lim
n
S
n
=
3
4
cos a,
care este suma seriei.
Problema 7.37

n=0
arctg
2
n
1 + 2
2n+1
.
Solut ie.
arctg 2x = arctg x + arctg
x
1 + 2x
2
,
din care
arctg
2
n
1 + 2
2n+1
= arctg 2
n+1
arctg 2
n
.
S
n
=
n

k=0
(arctg 2
k+1
arctg 2
k
) = arctg 2
n+1
arctg 1 = arctg 2
n+1
lim
n
S
n
=

2


4
=

4
.
Problema 7.38

n=3
arctg
3
n
2
n 1
.
Solut ie. Avem identitatea:
arctg a + arctg b =
_

_
arctg
a +b
1 ab
, daca ab < 1
+ arctg
a +b
1 ab
, daca ab > 1
a
n
= arctg
3
n
2
n 1
= arctg
3
1 +n
2
n 2
=
= arctg
(n + 1) (n 2)
1 + (n + 1)(n 2)
= arctg (n + 1) arctg (n 2).
S
n
=
n

k=3
(arctg (k + 1) arctg (k 2)) =
= arctg (n + 1) + arctg n + arctg (n 1) arctg 1 arctg 2 arctg 3.
lim
n
S
n
= 3

2


2
(arctg 2 + arctg 3) =
= 3

2


4

_
+ arctg
2 + 3
1 2 3
_
= 3

2


4
+

4
=

2
24
Problema 7.39

n=1
(1)
n+1
n
.
Solut ie. S
2n
=
_
1 +
1
2
+ +
1
2n
_
2
_
1
2
+
1
4
+ +
1
2n
_
=
=
_
1 +
1
2
+ +
1
2n
ln 2n
_

_
1 +
1
2
+ +
1
n
ln n
_
+ ln 2n ln n =
= c
2n
c
n
+ ln 2,
unde c
n
= 1 +
1
2
+ +
1
n
. Sirul (c
n
)
n
este convergent la constanta lui Euler c si atunci
lim
n
S
2n
= c c + ln 2 = ln 2
Analog
S
2n+1
= S
2n
+
1
2n + 1
ln 2
deci

n=1
(1)
n+1
n
= ln 2.
Problema 7.40 1 +
1
3
+ +
1
2p 1

1
2

1
4

2
2q
+
1
2p + 1
+
+
1
2p + 3
+ +
1
4p 1

1
2q + 2

1
2q + 4

1
4q
+. . . ,
unde p, q N.
Solut ie. Notam cu S(p, q) suma seriei,
a
n
= 1 +
1
2
+ +
1
n
si c
n
= 1 +
1
2
+ +
1
n
ln n,
sirul (c
n
)
n
ind convergent la constanta lui Euler c. Suma primilor n(p + q) termeni ai
seriei este
S
n
(p +q) = 1 +
1
3
+ +
1
2p 1
+
1
2p + 1
+ +
2
2np 1

_
1
2
+
1
4
+ +
1
2q
+
1
2q + 2
+ +
1
2nq
_
=
= a
2np

_
1
2
+
1
4
+ +
1
2p
+ +
1
2np
_

1
2
_
1 +
1
2
+ +
1
q
+
1
q + 1
+ +
1
nq
_
=
= a
2np

1
2
a
np

1
2
a
nq
= c
2np
+ ln(2np)
1
2
(c
np
+ ln(np))
1
2
(c
nq
+ ln(nq)) =
= c
2np

1
2
c
np

1
2
c
nq
+
1
2
ln
4n
2
p
2
npnq
.
Siruri si serii numerice 25
Trecand la limita obt inem:
S(p, q) = c
1
2
c
1
2
c +
1
2
ln
4p
q
=
1
2
ln
4p
q
Observat ie. 1) Daca q = 4p, atunci S(p, q) = 0, de exemplu
1
1
2

1
4

1
6

1
8
+
1
3

1
10

1
12

1
14

1
16
+ = 0.
Cum
4p
q
[ p, q N

= Q

+
, mult imea ln
4p
q
[ p, q N

este densa n R, deci pentru


orice l R si pentru orice > 0 se poate alege p, q N

astfel ca l < S(p, q) < l +.


2)

In seria semiconvergenta

n1
(1)
n+1
n
s-a permutat ordinea termenilor astfel ncat s-a obt inut o serie convergenta, dar cu o alta
suma. Astfel s-a exemplicat teorema lui Riemann referitoare la serii semiconvergente.
Problema 7.41

n=1
(1)
n
ln n
n
.
Solut ie. Sirul cu termenul general
x
n
=
ln 2
2
+
ln 3
3
+ +
ln n
n

ln
2
n
2
este convergent si notam limita sa cu l. Avem:
S
2n
=
2n

k=1
(1)
k
ln k
k
=
=
ln 1
1
+
ln 2
2

ln 3
3
+
ln 4
4

ln(2n 1)
2n 1
+
ln 2n
2n
=
=
_
ln 1
1
+
ln 2
2
+
ln 3
3
+
ln 4
4
+ +
ln(2n 1)
2n 1
+
ln(2n)
2n
_
+
+2
_
ln 2
2
+
ln 4
4
+ +
ln(2n)
2n
_
=
= x
2n
+x
n
+ ln 2
_
1 +
1
2
+ +
1
n
ln n
_

(ln 2)
2
2
lim
n
S
2n
= l +l + ln 2 c
(ln 2)
2
2
= ln 2
_
c
ln 2
2
_
unde c = lim
n
_
1 +
1
2
+ +
1
n
ln n
_
este constanta lui Euler.
Problema 7.42

n=1
(1)
n1
1
n + 1
_
1 +
1
2
+ +
1
n
_
.
26
Solut ie. Aratam ca seria

n=1
(1)
n1
2
n + 1
_
1 +
1
2
+ +
1
n
_
este produsul Cauchy
al seriei

n=1
(1)
n1
1
n
cu ea nsasi. Termenul general al produsului este
c
n
= (1)
n1
_
1
1 n
+
1
2(n 1)
+ +
1
n 1
_
dar
1
k(n + 1 k)
=
1
n + 1
_
1
k
+
1
n k + 1
_
,
deci
c
n
= (1)
n
2
n + 1
_
1 +
1
2
+ +
1
n
_
.
Deoarece seria produs este o serie alternanta iar sirul
1 +
1
2
+ +
1
n
n + 1
este descrescator
spre zero, conform criteriului lui Leibniz, seria produs este convergenta si atunci suma ei
este
S =
_

n=1
(1)
n1
1
n
_
2
= (ln 2)
2
.
Problema 7.43

n=0
(1)
n
F
n
F
n+1
, unde F
0
= F
1
= 1, F
n+1
= F
n
+ F
n1
, n 1 (sirul lui
Fibonacci).
Solut ie. Pentru matricea A =
_
1 1
1 0
_
,
A
n+1
= A
n
+A
n1
si A
n+1
=
_
F
n+1
F
n
F
n
F
n1
_
,
det(A
n+1
) = (det A)
n+1
,
deci
F
n1
F
n+1
F
2
n
= (1)
n+1
, n 1.
Suma primilor n termeni ai seriei este
S
n
=
n

k=0
(1)
k
F
k
F
k+1
= 1
n

k=1
F
k1
F
k+1
F
2
k
F
k
F
k+1
=
= 1
n

k=1
_
F
k1
F
k

F
k
F
k+1
_
= 1
F
0
F
1
+
F
n
F
n+1
=
F
n
F
n+1
din expresia lui F
n
=
1

5
_
_
_
1 +

5
2
_
n+1

_
1

5
2
_
n+1
_
_
rezulta
lim
n
S
n
=
2
1 +

5
=

5 1
2
.
Siruri si serii numerice 27
Problema 7.44 Fie F
n
sirul lui Fibonacci: F
0
= F
1
= 1, F
n+1
= F
n
+F
n1
, n 1 si e

n
=

n
k=0
F
2
k
. Sa se calculeze suma seriei:

n0
(1)
n

n
.
Solut ie. Vom presupune cunoscute relat iile (se pot demonstra prin induct ie):
F
n
=
1

5
_
_
_
1 +

5
2
_
n+1

_
1

5
2
_
n+1
_
_
, n 0 (1)
F
n1
F
n+1
F
2
n
= (1)
n+1
, n 1. (2)
Din denit ia lui F
k
rezulta:
F
k+1
F
k
= F
2
k
+F
k1
F
k
, k 1.

Insumand egalitat ile de mai sus pentru k = 1, 2, . . . , n, obt inem

n
= F
n+1
F
n
, n 0. (3)
Din relat iile (2) si (3) obt inem:
S
n
=
n

k=0
(1)
k

k
=

k=0
n
(1)
k
F
k
F
k+1
= 1
n

k=1
F
k1
F
k+1
F
2
k
F
k
F
k+1
=
= 1
n

k=1
_
F
k1
F
k

F
k
F
k+1
_
=
F
n
F
n+1
.
Aplicand acum (1), obt inem suma seriei:

n0
(1)
n

n
=
2
1 +

5
.
Problema 7.45

n=1
arctg
1
F
2n
, unde (F
n
)
n
este sirul lui Fibonacci.
Solut ie. Din problema anterioara avem relat ia
F
n1
F
n+1
F
2
n
= (1)
n+1
n care nlocuim unul din F
n
cu F
n+1
F
n1
si obt inem:
F
n1
F
n+1
F
n
(F
n+1
F
n1
) = (1)
n+1
sau
F
n1
(F
n+1
+F
n
) F
n
F
n+1
= (1)
n+1
sau
F
n1
F
n+2
F
n
F
n+1
= (1)
n+1
Avem
arctg
1
F
2n+1
arctg
1
F
2n+2
= arctg
F
2n+2
F
2n+1
F
2n+1
F
2n+2
+ 1
=
28
= arctg
F
2n
F
2n
F
2n+3
= arctg
1
F
2n+3
,
deci
arctg
1
F
2n+1
arctg
1
F
2n+3
= arctg
1
F
2n+2
Adunand relat iile de la n = 1 obt inem:
n+1

k=1
arctg
1
F
2k
= arctg
1
F
1
arctg
1
F
2n+3
Trecand la limita rezulta

n=1
arctg
1
F
2n
= arctg
1
F
1
=

4
.
Problema 7.46 Fie (x
n
)
n
un sir de numere reale astfel ncat exista P (0, ) cu
proprietatea:
lim
n
((x
1
+ 1)(x
2
+ 1) (x
n
+ 1)) = P.
Sa se calculeze suma seriei

n1
x
n
(x
1
+ 1)(x
2
+ 1) (x
n
+ 1)
.
Solut ie. Descompunem termenul general al seriei:
x
n
(x
1
+ 1)(x
2
+ 1) (x
n
+ 1)
=
x
n
+ 1 1
(x
1
+ 1)(x
2
+ 1) (x
n
+ 1)
=
=
1
(x
1
+ 1)(x
2
+ 1) (x
n1
+ 1)

1
(x
1
+ 1)(x
2
+ 1) (x
n
+ 1)
.
Rezulta pentru sirul sumelor part iale al seriei date formula:
S
n
= 1
1
(x
1
+ 1)(x
2
+ 1) (x
n
+ 1)
,
deci suma seriei este 1 P
1
(cu convent ia
1
= 0).
Problema 7.47 Sa se studieze convergent a seriei

n1
n!
_
a
n
_
n
, a > 0.
Solut ie. Se aplica criteriul raportului:
lim
n
x
n+1
x
n
= lim
n
a
_
n
n + 1
_
n
=
a
e
Daca a < e, atunci seria este convergenta; daca a > e, atunci seria este divergenta. Pentru
a = e, aplicam criteriul lui Raabe-Duhamel:
lim
n
n
_
x
n
x
n+1
1
_
= lim
n
n
__
n + 1
n
_
n
1
e
1
_
=
= n
__
1 +
1
n
_
n
1
e
1
_
=
1
e
lim
n
_
1 +
1
n
_
n
e
1
n
.
Ultima limita se calculeaza aplicand regula lui LHopital:
lim
x0
(1 +x)
1
x
e
x
= lim
x0
(1 +x)
1
x
1
[x (1 +x) ln(1 +x)]
x
2
=
e
2
;
rezulta ca seria este divergenta.
Siruri si serii numerice 29
Problema 7.48 Sa se studieze convergent a seriei

n2
1
n
p
ln
q
n
, p > 0, q > 0.
Solut ie. Daca p > 1, se aplica criteriul comparat iei: seria converge pentru orice q > 0
deoarece
1
n
p
ln
q
n

1
n
p
.
Daca p = 1, se aplica criteriul integral: seria converge daca si numai daca q > 1.
Daca p < 1 se aplica criteriul de condensare: seria are aceeasi natura cu seria cu termenul
general
1
n
q
2
n(p1)
ln
q
2
, care este divergenta pentru orice q > 0 (se poate aplica criteriul
raportului).
Problema 7.49 Fie (a
n
)
n
un sir de numere reale si e, pentru orice x R, seria

n1
an
n
x
.
Sa se demonstreze ca daca seria data converge pentru x = x
0
, atunci ea converge pentru
orice x x
0
.
Solut ie. Vom aplica criteriul lui Abel; seria data se scrie:

n1
a
n
n
x
=

n1
a
n
n
x
0

1
n
xx
0
Sirul
1
n
xx
0
este monoton (descresc ator) si marginit, iar seria

n1
an
n
x
0
este convergenta.
Problema 7.50

In seria convergenta:

n1
(1)
n+1
n
= 1
1
2
+
1
3

1
4
+
sa se permute ordinea termenilor astfel ncat sa se obt ina o serie convergenta, dar cu o
alta suma.
Solut ie. Seria

n1
(1)
n+1
n
este convergenta si suma sa este ln 2. Fie deci:
1
1
2
+
1
3

1
4
+ = ln 2

Inmult ind egalitatea de mai sus cu


1
2
, rezulta:
1
2

1
4
+
1
6

1
8
+ =
1
2
ln 2

Insumam acum cele doua egalitat i grupand termenii astfel:


1 +
_

1
2
+
1
2
_
+
1
3
+
_

1
4

1
4
_
+
1
5
+
_

1
6
+
1
6
_
+
1
7
+
+
_

1
8

1
8
_
+
1
9
+
_
1
10

1
10
_
+
1
11
+ =
3
2
ln 2.
Seria de mai sus este (dupa efectuarea calculelor din paranteze):
1 +
1
3

1
2
+
1
5
+
1
7

1
4
+
1
9
+
1
11
=
3
2
ln 2,
si este o permutare a seriei init iale.
Observat ie. Solut ia problemei se poate obt ine folosind cazul particular al problemei
7.40 pentru p = 2, q = 1.
30
Problema 7.51 Sa se precizeze natura seriilor:
a)

n=1
n
n2
e
n
n!
b)

n=1
n
n
e
n
n!
.
Solut ie. a)
a
n+1
a
n
=
_
1 +
1
n
_
n2
e
<
e
e
_
1 +
1
n
_
2
=
1
(n + 1)
2
1
n
2
=
b
n+1
b
n
Folosind criteriul de comparat ie C9 pentru seria convergenta

n=1
1
n
2
, rezulta ca seria
este convergenta.
b)
a
n+1
a
n
=
_
1 +
1
n
_
n
e
>
_
1 +
1
n
_
n
_
1 +
1
n
_
n+1
=
1
n + 1
1
n
.
Folosind criteriul de comparat ie C9 pentru seria divergenta

n=1
1
n
rezulta ca seria este
divergenta.
Problema 7.52 Fie

n=1
a
n
o serie convergenta cu termeni pozitivi. Sa se arate ca seria

n=1
n

a
1
a
2
. . . a
n
este convergenta si are loc inegalitatea:

n=1
n

a
1
a
2
. . . a
n
< e

n=1
a
n
(T. Carleman)
Solut ie. (G. Polya) Denim numerele c
1
, c
2
, . . . , c
n
, . . . prin relat iile c
1
c
2
. . . c
n
= (n+
1)
n
pentru orice n N

. Avem

n=1
n

a
1
a
2
. . . a
n
=

n=1
n

a
1
c
1
a
2
c
2
a
n
c
n
n + 1
()

n=1
a
1
c
1
+a
2
c
2
+ +a
n
c
n
n(n + 1)
=

n=1
1
n(n + 1)
_
n

k=1
(a
k
c
k
)
_
()

k=1
(a
k
c
k
)

n=k
1
n(n + 1)
=

k=1
(a
k
c
k
)

n=k
_
1
n

1
n + 1
_
=
=

k=1
a
k
c
k
1
k
=

k=1
a
k
(k + 1)
k
k
k1

1
k
=

k=1
a
k
_
1 +
1
k
_
k
()
<
<

k=1
a
k
e = e

k=1
a
k
.
Siruri si serii numerice 31

In () s-a folosit inegalitatea mediilor.

In () s-a folosit egalitatea:

n=1
a
n
_
n

k=1
b
k
_
=

k=1
b
k
_

n=k
a
n
_

In ( ) s-a folosit faptul ca sirul e


k
=
_
1 +
1
k
_
k
este crescator cu limita e, deci
e
k
< e, k N.
Problema 7.53 Fie (
n
)
n
un sir astfel ncat
n
1, 0, 1, n = 1, 2, . . . si e sirul
x
n
=
1
_
2 +
2
_
2 + +
n

2.
(a) Sa se demonstreze egalitatea:
x
n
= 2 sin
_

4
n

k=1

2

k
2
k1
_
, n = 1, 2, . . .
(b) Sa se demonstreze ca sirul (x
n
) este convergent.
G. Polya, G. Szego
Solut ie. (a) Daca
1
= 0, atunci relat ia este evident adevarata. Presupunem de aici
inainte ca
1
,= 0. Demonstram egalitatea prin induct ie; daca n = 1, egalitatea este
vericata. Presupunem acum adev arata relat ia:
x
n
= 2 sin
_

4
n

k=1

2

k
2
k1
_
.
Calculam, aplicand ipoteza de induct ie:
x
2
n+1
2 =
2
_
2 +
3
_
2 + +
n+1

2 = 2 sin
_

4
n+1

k=2

3

k
2
k2
_
=
= 2 cos
_

2
+

2
n+1

k=2

3

k
2
k1
_
= 2 cos
_

2
n+1

k=1

2

k
2
k1
_
,
ultima egalitate ind evidenta pentru
1
= 1; daca
1
= 1, atunci egalitatea rezulta din
paritatea funct iei cosinus. Evident, ipoteza de induct ie a fost aplicata n ipoteza
2
,= 0,
altfel egalitatea ceruta se verica imediat: x
n
=

2. Rezulta deci:
x
2
n+1
2 = 4 sin
2
_

4
n+1

k=1

2

k
2
k1
_
2,
si n concluzie
x
n+1
= 2 sin
_

4
n+1

k=1

2

k
2
k1
_
.
(b) Din relat ia demonstrata la punctul (a), notand cu S suma seriei (convergente)
n

k=1

2

k
2
k1
, rezulta lim
n
x
n
= 2 sin
_

4
S
_
.
32
Problema 7.54 Se considera sirul (a
n
)
n
denit prin relat ia de recurent a a
n+1
= ln(1 +
a
n
), n 1 si a
1
= 1.
a) Sa se arate ca lim
n
a
n
= 0.
b) Sa se arate ca seria

n=1
a
n
este divergenta.
c) Sa se arate ca seria

n=1
a
2
n
este convergenta.
Solut ie. a) Prin induct ie se arata ca a
n
> 0, n N

si din inegalitatea ln(1 + x)


x rezulta ca sirul (a
n
)
n
este descrescator (si marginit de zero) deci convergent. Daca
lim
n
a
n
= l atunci din relat ia de recurent a rezulta l = ln(1 +l) cu singura solut ie l = 0.
b) Comparam seria

n=1
a
n
cu seria

n=1
1
n
. Avem
lim
n
a
n
1
n
= lim
n
n
1
a
n
= lim
n
n + 1 n
1
a
n+1

1
a
n
=
= lim
n
a
n
a
n+1
a
n
a
n+1
= lim
n
a
n
ln(1 +a
n
)
a
n
ln(1 +a
n
)
= lim
x0
xln(1 +x)
x ln(1 +x)
=
= lim
x0
x
2
ln(1 +x)
x
x ln(1 +x)
= lim
x0
x
2
x ln(1 +x)
= lim
x0
2x
1
1
1 +x
=
= lim
x0
2(1 +x) = 2 (0, ),
deci seriile au aceeasi natura (divergente).
c) Aplicam criteriul comparat iei comparand cu seria

n=1
1
n
2
. Avem:
lim
n
a
2
n
1
n
2
= lim
n
(na
n
)
2
= 4 (0, ),
deci ambele serii sunt convergente.
Problema 7.55 Fie seria convergenta cu termeni pozitivi

n=1
a
n
. Sa se arate ca daca
exista limita lim
n
na
n
, atunci ea este egala cu zero.
Solut ie. Fie l = lim
n
na
n
, l 0. Daca presupunem l > 0, atunci avem
lim
n
a
n
1
n
= l > 0
deci seriile

a
n
si

1
n
au aceeasi natura, deci ambele divergente, contradict ie.
Siruri si serii numerice 33
Problema 7.56 Sa se arate ca daca sirul (a
n
)
n
este descrescator la zero si seria

n=1
a
n
este convergenta, atunci
lim
n
na
n
= 0.
Solut ie. Fie
x
n
= a
1
+a
2
+ +a
n
na
n
, n 1.
Sirul (x
n
)
n
este majorat de (S
n
)
n
, sirul sumelor part iale ale seriei date, deci este marginit.
Avem
x
n+1
x
n
= n(a
n
a
n+1
) 0,
deci sirul (x
n
)
n
este crecator.

In concluzie sirul x
n
= S
n
na
n
este convergent. Rezulta ca sirul (na
n
)
n
este convergent
si conform problemei 7.55 obt inem ca lim
n
na
n
= 0.
Problema 7.57 Sa se arate ca daca seriile

n=1
a
2
n
si

n=1
b
2
n
sunt convergente, atunci seriile

n=1
a
n
b
n
si

n=1
(a
n
+b
n
)
2
sunt convergente.
Solut ie. Avem
_
n

k=1
[a
k
b
k
[
_
2

k=1
a
2
k
n

k=1
b
2
k
sau
n

k=1
[a
k
b
k
[

_
n

k=1
a
2
k
n

k=1
b
2
k
din care rezulta

k=1
[a
k
b
k
[

k=1
a
2
k

k=1
b
2
k
Avem:
_
n

k=1
(a
k
+b
k
)
2
_
1/2

_
n

k=1
a
2
k
_
1/2
_
n

k=1
b
2
k
_
1/2
din care

k=1
(a
k
+b
k
)
2

k=1
a
2
k

k=1
b
2
k
.
(S-au folosit inegalitat ile Cauchy-Schwartz si Minkowski.)
Problema 7.58 Sa se arate ca daca seria

n=1
a
2
n
este convergenta, atunci seria

n=1
a
n
n
este convergenta.
Solut ie. Luam n exercit iul anterior b
n
=
1
n
.
34
Problema 7.59 Sa se arate ca seria

n=1
cos n
n
este convergenta, dar nu este absolut con-
vergenta.
Solut ie. Daca luam a
n
= cos n si b
n
=
1
n
, sirul sumelor part iale ale seriei

n=1
a
n
este
marginit, iar sirul (b
n
)
n
este descrescator la zero, deci conform criteriului lui Abel seria
este convergenta.
Pentru seria valorilor absolute

n=1
[ cos n[
n
, consideram funct ia
f(x) = [ cos x[ +[ cos(x + 1)[, f : R [0, ),
care este continua si are un minim diferit de zero, deci f(x) m > 0, x R (si atinge
minimul pe intervalul [0, 2]). Avem:
[ cos 1[
1
+
[ cos 2[
2
+
[ cos 3[
3
+
[ cos 4[
4
+ +
[ cos(2n 1)[
2n 1
+
[ cos 2n[
2n

[ cos 1[ +[ cos 2[
2
+
[ cos 3[ +[ cos 4[
4
+ +
[ cos(2n 1)[ +[ cos 2n[
2n

m
2
+
m
4
+ +
m
2n
=
m
2
_
1 +
1
2
+ +
1
n
_
,
deci sirul sumelor part iale are limita .
Problema 7.60 Fie

n=1
a
n
o serie divergenta cu termeni pozitivi si (S
n
)
n
sirul sumelor
part iale. Sa se arate ca:
a) Seria

n=1
a
n
S
n
este divergenta.
b) Seria

n=1
a
n
S
1+
n
este convergenta pentru > 0.
Solut ie. a) Avem:
a
n+1
S
n+1
+
a
n+2
S
n+2
+ +
a
n+p
S
n+p

a
n+1
+ +a
n+p
S
n+p
=
S
n+p
S
n
S
n+p
.
Dar
lim
p
S
n+p
S
n
S
n+p
= 1 ,= 0,
deci sirul sumelor part iale ale seriei

a
n
S
n
este divergent conform criteriului general al
lui Cauchy (C1).
b) Consideram diferent a:
ln S
n1
S

n1

ln S
n
S

n
pentru care aplicam teorema lui Lagrange si
avem:
ln S
n
S

ln S
n1
S

n1
= (S
n
S
n1
)f

(
n
),
n
(S
n1
, S
n
)
Siruri si serii numerice 35
si f(x) =
ln x
x
+1
.
Avem:
ln S
n1
S

n1

ln S
n
S

n
= (S
n
S
n1
)
ln S
n1
1
S
+1
n
>
>
S
n
S
n1
S
+1
n
=
a
n
S
+1
n
pentru n sucient de mare deci
a
n
S
+1
n
<
ln S
n1
S

n1

ln S
n
S

n
, n N.

nN
a
n
S
+1
n
<

nN
_
ln S
n1
S

n1

ln S
n
S

n
_
=
ln S
N1
S

N1
deci restul de ordin N al seriei

a
n
S
+1
n
este marginit, deci convergent.
Problema 7.61 Fie seria convergenta

n=0
a
n
si S
n
=
n

k=0
a
k
. Sa se arate ca pentru orice
a (1, 1) seria

n=0
S
n
a
n
este convergenta si

n=0
S
n
a
n
=
1
1 a

n=0
a
n
a
n
.
Solut ie. Seria

n=0
S
n
a
n
este produsul Cauchy al seriilor

n=0
a
n
si

n=0
a
n
a
n
, ambele
convergente, iar suma primei serii este
1
1 a
.
Problema 7.62 Fie (a
n
)
n
un sir cu termeni reali pozitivi. Sa se arate ca produsul

n=1
(1+
a
n
) este convergent daca si numai daca seria

n=1
a
n
este convergenta.
Solut ie. Avem inegalitat ile
a
1
+a
2
+ +a
n
(1 +a
1
)(1 +a
2
) (1 +a
n
)
si
(1 +a
1
)(1 +a
2
) (1 +a
n
) e
a
1
+a
2
++an
,
(e
x
1 +x, x 0).
Problema 7.63 Fie (
n
)
n
un sir cu termenii
n
1, 1, n N. Sa se arate ca suma
seriei

n=0

n
n!
este un numar irat ional.
36
Solut ie. Prin absurd presupunem ca

n=0

n
n!
=
p
q
Q.

Inmult im cu q! si obt inem


(q 1)! p =
q

n=0
q!
n
n!
+

n=q+1
q!
n
n!
Cum prima suma este numar ntreg rezulta ca

n=q+1
q!
n
n!
ar numar ntreg.
Avem:

n=q+1
q!
n
n!

n=q+1
q!
n!

1
q + 1
+
1
(q + 1)(q + 2)
+
1
(q + 1)(a + 2)
2
+ =
=
1
q + 1

1
1
1
q + 2
=
q + 2
(q + 1)
2

3
4
.
Ramane de aratat doar ca suma nu poate egala cu zero.
Dar:

n=q+1

n
q!
n!

1
q + 1

n=q+2
q!
n!

>
1
q + 1

1
q(q + 1)
0.
Observat ie.

In particular rezulta ca e , Q.
Problema 7.64 Se considera sirul (a
n
)
n
denit prin relat ia de recurent a
a
n+1
= arctg a
n
, n 1 si a
1
= 1.
a) Sa se arate ca lim
n
a
n
= 0.
b) Sa se arate ca lim
n

na
n
=
_
2
3
.
c) Sa se studieze convergent a seriei

n=1
a

n
, R.
Solut ie. a) Se arata imediat ca sirul (a
n
)
n
este strict descrescator si marginit inferior
de 0. Fie a = lim
n
a
n
. Trecand la limita relat ia de recurent a obt inem a = arctg a, deci
a = 0.
b) Fie x
n
=

na
n
=
_
n
1
a
2
n
. Cum sirul (
1
a
2
n
) este strict crescator si lim
n
1
a
2
n
= putem
aplica Stolz-Cesaro si obt inem:
lim
n
n
1
a
2
n
= lim
n
(n+1)n
1
a
2
n+1

1
a
2
n
= lim
n
1
1
a
2
n+1

1
a
2
n
= lim
n
(n+1)n
1
arctg
2
an

1
a
2
n
= lim
n
a
2
n
arctg
2
an
a
2
n
arctg
2
an
=
lim
n
a
2
n
arctg
2
an
a
4
n
lim
n
(
an
arctgan
)
2
= lim
n
an+arctgan
an
lim
n
anarctgan
a
3
n
= 2 lim
x0
xarctgx
x
3
=
2
3
si
de aici avem lim
n

na
n
=
_
2
3
.
Siruri si serii numerice 37
c) Din lim
n
a

n
1
n

2
= (
2
3
)

rezulta, pe baza criteriului comparat iei C10, ca seria

n=1
a

n
are aceeasi natura cu seria

n=1
1
n

2
, deci este convergenta daca si numai daca > 2.
Problema 7.65 Fie

n=1
a
n
o serie divergenta cu termeni pozitivi si a
1
> 1.
Sa se arate ca:
a) Seria

n=1
a
n+1
S
n
ln S
n
este divergenta.
b) Seria

n=1
a
n
S
n
ln
2
S
n
este convergenta, unde S
n
= a
1
+a
2
+ +a
n
.
Solut ie. a)

n=1
a
n+1
S
n
ln S
n
=

n=1
S
n+1
S
n
S
n
ln S
n
Se aplica teorema lui Lagrange funct iei f(x) = ln ln x pe intervalele [S
n
, S
n+1
] si avem
n

k=1
a
k+1
S
k
ln S
k
= ln ln S
n
ln ln S
1

b) Se aplica teorema lui Lagrange funct iei f(x) =
1
ln x
.
Problema 7.66 Fie

n=1
a
n
o serie divergenta cu termeni pozitivi astfel ca lim
n
a
n
= 0.
Sa se arate ca sirul (S
n
)
nN
este dens n [0, 1], unde S
n
= a
1
+a
2
+ +a
n
si x este
partea fract ionara a lui x.
Solut ie. Daca 0 < a < b < 1 si b a = , exista N

N astfel ca a
n
< , n > N().
Fie S
N
= m + b
N
, m Z, b
N
[0, 1). Daca b
N
(a, b) am terminat. Daca b
N
< a
mai adaugam un numar minim de termeni din serie pana intram n intervalul (a, b). Daca
b
N
> b, mai adaugam un numar minim de termeni pana ajungem la m+1+ cu (a, b).
Problema 7.67 Fie (p
n
)
nN
sirul numerelor prime. Sa se arate ca seria

n=1
1
p
n
este
divergenta.
Solut ie. Daca seria ar convergenta, atunci ar exista N N astfel ca

n=N+1
1
p
n
<
1
2
.
Orice numar de forma 1 + kp
1
p
2
. . . p
N
= 1 + kP este un produs de numere prime
(eventual cu exponent i) care nu fac parte din numerele p
1
, p
2
, . . . , p
N
. Mult imea numerelor
de forma
1
M
n care M este format doar cu numerele prime p
N+1
, p
N+2
, . . . este
_
1
p
n
1
,
1
p
n
1
p
n
2
,
1
p
n
1
p
n
2
. . . p
n
3
, . . .
_
38
suma lor ind

m=1
_

n=N+1
1
p
n
_
m
<

m=1
_
1
2
_
m
= 1
si atunci

k=1
1
1 +kP
< 1
deci seria

k=1
1
1 +kP
ar convergenta, contradict ie.
Problema 7.68 Sa se studieze convergent a seriei

n1
sin nx
n
, x R.
Generalizare la seria

n1
sin nx
n

, x R, R.
Solut ie. Fie x
n
=
sin nx
n
. Daca x = k, k Z, atunci x
n
= 0, n N; presupunem n
continuare ca x ,= k, k Z. Aratam mai ntai ca seria nu este absolut convergenta:
[x
n
[ =
[ sin(nx)[
n

sin
2
(nx)
n
=
1 cos(2nx)
2n
, n N

.
Deci, presupunand prin absurd c a seria data ar absolut convergenta, ar rezulta (cu
criteriul de comparat ie) ca si seria

n1
1cos(2nx)
2n
ar convergenta. Seria

n1
cos(2nx)
2n
este convergenta (criteriul lui Dirichlet): e a
n
=
1
2n
si u
n
= cos(2nx). Atunci (a
n
) este
descrescator la 0, iar (u
n
) are sirul sumelor part iale marginit:

k=1
cos(2nx)

sin(nx) cos(n + 1)x


sin x

1
[ sin x[
.
Rezulta ca si seria

n1
1
2n
ar trebui sa e convergenta, ind suma a doua serii conver-
gente, contradict ie.
Seria

n1
sin nx
n
este convergenta (ca mai sus, cu criteriul lui Dirichlet).
Problema 7.69 Sa se studieze convergent a seriei

n1
(1)
n n

nsin
1
n
.
Solut ie. Seria nu este absolut convergenta (se compara la limita cu seria armonica).
Seria este alternata; vom demonstra ca sirul a
n
=
n

nsin
1
n
este descrescator la 0, deci
seria converge.
Evident a
n
0; pentru a arata ca a
n
este descrescator (ncepand de la un rang), e
funct ia f(x) = x
1
x
sin
1
x
. Calculam
f

(x) = x
1
x
2
_
(1 ln x) sin
1
x
cos
1
x
_
.
Pentru a studia semnul derivatei (pentru x mare), calculam:
lim
x
(1 ln x) sin
1
x
cos
1
x
= 1 + lim
x
sin
1
x
1
x

1 ln x
x
= 1,
deci f

(x) < 0 pentru x sucient de mare, deci sirul a


n
este descrescator.
Siruri si serii numerice 39
Problema 7.70 Fie , , trei numere strict pozitive. Folosind criteriul lui Gauss, sa se
studieze convergent a seriei:

n1
( + 1) ( +n 1)
n!

( + 1) ( +n 1)
( + 1) ( +n 1)
.
Solut ie. Criteriul lui Gauss: daca (a
n
)
n
este un sir de numere strict pozitive, astfel
ncat exista > 1, R si un sir (
n
)
n
astfel ncat
a
n+1
a
n
= 1

n


n
n

,
atunci seria

n1
a
n
converge daca > 1 si diverge daca 1.
Aplicand criteriul lui Gauss seriei date, obt inem:
a
n+1
a
n
=
n
2
+ ( +)n +
n
2
+ (1 +)n +
= 1
1 +
n


n
2
Rezulta ca daca > + atunci seria data converge, iar daca + atunci seria
diverge.
Problema 7.71 Sa se dea un exemplu de doua siruri (a
n
)
n
si (b
n
)
n
astfel ncat:
lim
n
an
bn
= 1, dar seriile

n
a
n
si

n
b
n
sa aiba naturi diferite.
Solut ie. Fie, de exemplu, a
n
=
(1)
n
n
si b
n
=
(1)
n
n
+
1
nln n
. A doua serie este divergenta
(criteriul integral).
Problema 7.72 Fie (a
n
)
n
si (b
n
)
n
doua siruri reale astfel ncat seria

n1
(b
n
b
n+1
)
este absolut convergenta si seria

n1
a
n
este convergenta.
Sa se demonstreze ca seria

n1
a
n
b
n
este convergenta.
Solut ie. Facem mai ntai urmatoarea observat ie generala. Pentru orice siruri de numere
(x
n
)
n
si (y
n
)
n
are loc urmatoarea identitate (sumare prin part i):
n

k=1
x
k
y
k
=
n1

k=1
(x
1
+x
2
+ +x
k
)(y
k
y
k+1
) + (x
1
+x
2
+ +x
n
)y
n
Fie A
n
=

n
k=1
a
k
si S
n
=

n
k=1
a
k
b
k
sumele part iale asociate seriilor

n
a
n
si respectiv

n
a
n
b
n
.

Insumand prin part i, obt inem:
S
n
=
n

k=1
a
k
b
k
=
n1

k=1
A
k
(b
k
b
k+1
) +A
n
b
n
Demonstrat ia se ncheie daca aratam ca seria

n
A
n
(b
n
b
n+1
) si sirul (A
n
b
n
)
n
sunt
convergente. Din convergent a seriei

n
(b
n
b
n+1
) rezulta convergent a sirului (b
n
)
n
si deci
(A
n
b
n
)
n
este convergent. Sirul A
n
este marginit si din faptul ca seria

n1
(b
n
b
n+1
)
este absolut convergenta rezulta ca seria

n
A
n
(b
n
b
n+1
) este absolut convergenta, ceea
ce ncheie demonstrat ia.
Observat ie.

In ultima implicat ie ar fost sucient ca seria

n1
(b
n
b
n+1
) sa e
(doar) convergenta?
40
Problema 7.73 Sa se calcuze sumele urmatoarelor serii:
(i)

n0
c
n
, unde c
n
=
n

k=0
x
k
y
nk
, [x[ < 1, [y[ < 1.
(ii)

n1
c
n
, unde c
n
=
n

k=1
1
(n k + 1)!k(k + 1)
.
Solut ie. Vom aplica teorema lui Mertens pentru produsul Cauchy a doua serii.
(i) Evident,

n0
c
n
=
_
_

n0
x
n
_
_
_
_

n0
y
n
_
_
= (1 x)
1
(1 y)
1
.
(ii) Seria data este produsul seriilor

n1
1
n(n + 1)
si

n1
1
n!
. Prima serie are suma 1, iar a
doua e 1, deci raspunsul este e 1.
Produse innite
Fie (a
n
)
n
un sir de numere reale strict pozitive.
Produsul innit

n1
a
n
se numeste convergent daca lim
n
n

k=1
a
k
exista si este nenula, sau,
echivalent, daca seria

n1
ln a
n
este convergenta. Este usor de observat ca o conditi e nece-
sara (dar nu si sucienta) pentru convergent a produsului innit

n1
a
n
este a
n
1.
Dam n continuare o lista cu cateva formule-produs uzuale:
sin x = x

n1
_
1
x
2

2
n
2
_
= x

n1
cos
x
2
n
sinh x = x

n1
_
1 +
x
2

2
n
2
_
cos x =

n1
_
1
x
2

2
(n
1
2
)
2
_
cos x =

n1
_
1
x
2

2
(n +
1
2
)
2
_
Produse. Probleme
Problema 7.74 Sa se calculeze produsele:
(i)

n1
a
(1)
n
n
, a > 0.
(ii)

n1
e
1
n
1 +
1
n
.
Solut ie. (i) lim
n
n

k=1
a
(1)
k
k
= lim
n
a

n
k=1
(1)
k
k
= a
ln 2
.
(ii) lim
n
n

k=1
e
1
k
1 +
1
k
= lim
n
e

n
k=1
1
k
n + 1
= lim
n
e

n
k=1
1
k
n

n
n + 1
=
Siruri si serii numerice 41
= lim
n
n
n + 1
e

n
k=1
1
k
ln n
= e
c
, unde c este constanta lui Euler.
Problema 7.75 Fie (a
n
)
n
un sir cu termeni strict pozitivi. Atunci:
(i) produsul innit

n1
(1 +a
n
) converge daca si numai daca seria

n1
a
n
converge.
(ii) produsul innit

n1
(1 a
n
) converge daca si numai daca seria

n1
a
n
converge (cu
ipoteza suplimentara a
n
,= 1, n).
Solut ie. (i) Deoarece a
n
> 0 si din inegalitatea 1 +x e
x
, x 0, rezulta:
n

k=1
a
k

n

k=1
(1 +a
k
) e

n
k=1
a
k
,
ceea ce demonstreaza prima armat ie.
(ii) Presupunem mai ntai ca seria

n1
a
n
converge. Rezulta ca restul seriei tinde la 0, deci
exista m N sucient de mare (xat de aici nainte) astfel ncat

km
a
k
< 2
1
. Rezulta
(pentru n > m) inegalitatea:
n

k=m
(1 a
k
) 1
n

k=m
a
k
>
1
2
.
Daca notam P
n
=
n

k=1
(1 a
k
), atunci din relat ia P
n
= P
m1
n

k=m
(1 a
k
), rezulta ca sirul
_
P
n
P
m1
_
n
este descrescator si minorat de 2
1
, deci are o limita P [2
1
, 1]. Rezulta ca
P
n
este convergent la o limita nenula, deci produsul innit

n1
(1 a
n
) converge.
Pentru a demonstra reciproca, presupunem ca seria

n1
a
n
este divergenta (are suma ).
Putem presupune ca a
n
0 (altfel 1 a
n
, 1) si deci exista m N astfel ncat a
n

[0, 1), n m.
Din inegalitatea 1 x e
x
, x [0, 1), rezulta:
0
n

k=m
(1 a
k
) e

n
k=m
a
k
0 (pentru n ),
ceea ce ncheie demonstrat ia.
Problema 7.76 Notam cu (p) =

n=1
1
n
p
, p > 1 funct ia lui Euler si cu
c = lim
n
_
n

k=1
1
k
ln n
_
constanta lui Euler.
42
Sa se arate ca

p=2
(1)
p
p + 1
(p) = 1 +
c
2
ln

2.
Concurs Ukraina
Solut ie. Pentru x (0, 1) consideram funct ia
f(x) =

p=2
(1)
p
p + 1
x
p
.
Avem:
f(x) =
1
x

p=2
(1)
p
p + 1
x
p+1
=
1
x

p=2
(1)
p
_
x
0
t
p
dt
=
1
x
_
x
0
_
_

p=0
(t)
p
1 +t
_
_
dt =
1
x
_
x
0
_
1
1 +t
1 t
_
dt
=
1
x
_
ln(1 +t) t +
t
2
2
_

x
0
=
1
x
_
ln(1 +x) x +
x
2
2
_
= 1 +
x
2
+
ln(1 +x)
x
.
Acum suma ceruta o exprima cu funct ia f:
S =

p=2
(1)
p
p + 1
(p) =

p=2
(1)
p
p + 1

n=1
1
n
p
= lim
m
m

p=2
(1)
p
p + 1

n=1
1
n
p
= lim
m

n=1
m

p=2
(1)
p
(p + 1)n
p
=

n=1

p=2
(1)
p
(p + 1)n
p
=

n=1
f
_
1
n
_
=

n=1
_
1 +
1
2n
+nln
_
1 +
1
n
__
=
1
2

n=1
_
1
n
ln
_
1 +
1
n
__
+

n=1
__
n +
1
2
_
ln
_
1 +
1
n
_
1
_
=
1
2
lim
m
m

n=1
_
1
n
ln(n + 1) + ln n
_
+

n=1
ln
_
_
n + 1
n
_
n+
1
2

1
e
_
=
1
2
lim
m
_
1 +
1
2
+. . . +
1
m
ln(m+ 1)
_
+ lim
m
ln
n

k=1
_
_
k + 1
k
_
n+
1
2

1
e
_
=
1
2
e + ln lim
m
(n + 1)
m

n + 1
(n + 1)!e
n
=
1
2
e + ln lim
n
(n + 1)
n

n + 1
n
n

2n
=
1
2
e + ln
e

2
= 1 +
1
2
e ln

2
Capitolul 8
Calcul diferent ial pentru funct ii de
o variabila reala
Denit ii si rezultate
Denit ie. Funct ia f : D R se numeste uniform continua pe D daca pentru orice
> 0 exista > 0 astfel ncat oricare ar x

, x

D cu [x

[ < , avem[f(x

)f(x

)[ <
.
Teorema lui Fermat. Daca x
0
este punct de extrem (local) pentru funct ia f : I IR,
I IR interval, daca x
0
este punct interior pentru I si daca funct ia f este derivabila n
x
0
, atunci f

(x
0
) = 0.
Teorema. Fie f : I R o funct ie derivabila pe intervalul I. Atunci funct ia f

: I R
are proprietatea Darboux.
Teorema lui Rolle. Fie f : [a, b] R continua pe [a, b] si derivabila pe (a, b) astfel
ncat f(a) = f(b). Atunci exista c (a, b) astfel ncat f

(c) = 0.
Teorema lui Lagrange. Fie f : [a, b] R. Daca f este continua pe [a, b] si derivabila
pe (a, b), atunci exista c (a, b) astfel ncat
f(b) f(a) = f

(c)(b a).
Teorema lui Cauchy. Fie f : [a, b] R, g : [a, b] R doua funct ii care ndeplinesc
condit iile: f, g sunt continue pe [a, b], f, g sunt derivabile pe (a, b), g

(x) ,= 0 pentru orice


x (a, b). Atunci exista c (a, b) astfel ncat
f(b) f(a)
g(b) g(a)
=
f

(c)
g

(c)
.
Formula lui Taylor cu rest Peano. Fie I un interval deschis si f : I R o funct ie
de n ori derivabila n x
0
I. Atunci exista o funct ie : I R astfel ncat
f(x) = f(x
0
) +
f

(x
0
)
1!
(x x
0
) +
f

(x
0
)
2!
(x x
0
)
2
+. . .
+
f
n
(x
0
)
n!
(x x
0
)
n
+(x)(x x
0
)
n
, x I.
si
lim
xx
0
(x) = 0
Notat ie.

In cele ce urmeaza vom folosi notat ia o(f) pentru a desemna o funct ie g
(denita ntro vecinatate a lui 0), care are proprietatea ca lim
x0
g(x)
f(x)
= 0.
43
44

In cazul particular x
0
= 0 se obt ine formula lui MacLaurin:
() f(x) = f(0) +
f

(0)
1!
x +
f

(0)
2!
x
2
+. . . +
f
(n)
(0)
n!
x
n
+o(x
n
)
Formula lui Taylor cu restul lui Lagrange. Fie I interval deschis si f : I R de
n + 1 ori derivabila pe I. Atunci pentru x
0
si x I exista c ntre x si x
0
astfel ncat
f(x) = f(x
0
) +
f

(x
0
)
1!
(x x
0
) +. . . +
f
(n)
(x
0
)
n!
(x x
0
)
n
+
+
f
(n+1)
(c)
(n + 1)!
(x x
0
)
n+1
.

In cazul particular x
0
= 0, se obt ine formula lui MacLaurin :
f(x) = f(0) +
f

(0)
1!
x +
f

(0)
2!
x
2
+. . . +
f
(n)
(0)
n!
x
n
+
f
(n+1)
(c)
(n + 1)!
x
n+1
.
Probleme
Problema 8.1 Se considera funct ia f : R 0 R f(x) := x
_
1
x
_
. Precizat i disconti-
nuitat ile funct iei f si calculat i lim
x0
f(x).
Solut ie. Se expliciteaza: pe ecare interval
_
1
n+1
,
1
n
_
avem f(x) = nx. Limitele laterale
n
1
n
sunt 1 = f(
1
n
) resp.
n1
n
. Analog, pe ecare interval
_

1
n
,
1
n+1
_
avem f(x) =
(n +1)x. Pe (, 1) funct ia este x, iar pe (1, ) este 0. Din cele de mai sus rezulta
ca limita n 0 exista si este 1. Punctele de discontinuitate sunt de forma
1
k
, k Z
Problema 8.2 f : R R o funct ie continua, periodica de perioada 1, adica f(x + 1) =
f(x), x R.
(i) Sa se arate ca f este marginita si si atinge marginile.
(ii) Sa se arate ca exista x
0
R pentru care f(x
0
+) = f(x
0
).
Solut ie. (i) Se restrange la [0, 1], valorile n rest ind aceleasi.
(ii) Fie g(x) := f(x + ) f(x). Daca x
m
resp. x
M
denota puncte n care f si
atinge minimul, resp, maximul, avem g(x
m
) = f(x
m
+ ) f(x
m
) 0 iar g(x
M
) =
f(x
M
+)f(x
M
) 0, de unde existent a unui punct x
0
, ntre x
m
si x
M
, n care g(x
0
) = 0.
Problema 8.3 Funct ia F : (0, ) R este continua si lim
n
F(x +n) = 0 pentru ecare
x > 0. Rezulta ca lim
x
F(x) = 0?
Solut ie. Nu. Se construieste un contraexemplu astfel: e f
0
(x) := 1 2[x
1
2
[; apoi
f
k
este denita astfel: f
k
(x) := f
0
(kx) pentru x (0,
1
k
) si f
k
(x) := 0 pentru x
1
k
.

In
sfarsit: g(x) = f
k
(x k) pentru x [k, k + 1]. Funct ia g este bine denita si continua.
Deoarece g(k +
1
2k
) = 1, pentru ecare k, urmeaza ca lim
x
g(x) nu este 0. Pe de alta parte,
pentru orice x N avem g(x + n) = 0. Daca x = [x] + x, atunci, pentru k sucient de
mare astfel ncat x >
1
k
avem g(x +k) = 0.
Calcul diferent ial pentru funct ii de o variabila reala 45
Problema 8.4 Fie f : R R o funct ie reala. Analizat i daca urmatoarele armat ii sunt
adevarate si justicat i.
(a) Daca f este continua si Im(f) = y R[ x R, f(x) = y, atunci f este
monotona.
(b) Daca f este monotona si Im(f) = R, atunci f este continua.
(c) Daca f este monotona si f este continua, atunci Im(f) = R.
Solut ie. (a) Nu este adevarata. Consideram funct ia f(x) = x
3
x, care este continua,
Im(f) = R, dar de exemplu f(0) = 0, f(
1
2
) =
3
8
si f(1) = 0; de aceea f(0) > f(
1
2
), f(
1
2
) <
f(1) si f nu este monotona.
(b) Este adevarata. Presupunem pentru nceput ca f este nedescrescatoare. Pentru un
numar arbitrar a exista limitele laterale b := lim
xa
f(x) si c := lim
xa
f(x) si b c. Daca cele
doua limite sunt egale, atunci funct ia este continua n a.

In caz contrar, daca b < c, avem
f(x) b, x < a si and f(x) c, x > a si de aceea Im(f) (, b) (c, ) f(a) nu
reprezinta toata mult imea R. Pentru cazul funct iei necrescatoare se aplica rat ionamentul
funct iei g(x) = f(x).
(c) Fals. Funct ia f(x) = arctg x este monotona si continua, dar Im(f) = (

2
,

2
) ,= R.
Problema 8.5 Sa se determine toate funct iile continue f : R R cu proprietatea ca
oricare ar x, y R cu x y Q, are loc f(x) f(y) Q.
Solut ie. Pentru ecare r Q, funct ia x f(x + r) f(x) este continua si are
numai valori rat ionale. Deci este constanta. Sa denim atunci g(r) := f(x + r) f(x).
Prin recurent a f(nr) = f(0) + ng(r), n N. Deoarece f(0) f(r) = g(r), formula
se extinde la numere ntregi: f(kr) = f(0) + kg(r), k Z. Luand r =
1
n
, se gaseste
f(1) = f(0) + ng(
1
n
). De fapt: g(r) = rg(1), r Q. Adica f(r) = f(0) + rg(1), r Q.
Deoarece f este continua, urmeaza ca f(x) = ax + b, x R, cu a Q. Reciproc, orice
asemenea funct ie convine.
Problema 8.6 Se da funct ia f : R R. Sa se arate ca f este monotona daca si numai
daca pentru orice interval I R, f
1
(I) este interval.
( f
1
(A) = x R [f(x) A.)
Solut ie. Presupunem ca f este crescatoare. Fie I un interval x, x

f
1
(I) cu x x

;
exista deci y, y

I cu y y

astfel ca f(x) = y, f(x

) = y

. Pentru a arata ca f
1
(I)
este un interval, e x

[x, x

], din x x

rezulta f(x) f(x

) f(x

) si deoarece
I este interval, avem f(x

) I, deci x

f
1
(I).
Reciproc. Presupunem ca f nu este monotona. Exista deci x
1
, x
2
, cu x
1
< x
2
astfel
ca f(x
1
) f(x
2
) si x
3
< x
4
astfel ca f(x
3
) f(x
4
). sau f(x
1
) f(x
2
), f(x
3
) f(x
2
).
Comparand cele 4 valori se poate obt ine urmatoarea condit ie echivalenta. Exista x
1
< x
2
<
x
3
astfel ca f(x
1
) f(x
2
) si f(x
3
) f(x
2
). Fie a = min(f(x
1
), f(x
3
)) si b = f(x
2
). Atunci
rezulta ca f
1
[a, b) nu este interval, deoarece x
1
, x
3
f
1
[a, b), dar x
2
/ f
1
[a, b).
Problema 8.7 Fie C R o mult ime nevida, nchisa, marginita si f : C C o funct ie
nedescrescatoare. Aratat i ca exista un punct p C astfel ca f(p) = p.
Solut ie. Presupunem ca f(x) ,= x, x C. Fie [a, b] cel mai mic interval nchis
care cont ine C. Deoarece C este nchisa a, b C. Din ipoteza f(a) > a si f(b) < b. Fie
p = supx C : f(x) > x. Deoarece C este nchisa si f este continua f(p) p si astfel
f(p) > p. Pentru orice x > p, x C avem f(x) < x. De aceea f(f(p)) < f(p), ceea ce
contrazice faptul ca f este nedescrescatoare.
46
Problema 8.8 Fie f : [0, 1] [0, 1] cu proprietatea ca exista c [0, 1) astfel ca
[f(x) f(y)[ c[x y[, x, y [0, 1] (8.1)
Aratat i ca sirul denit
x
0
[0, 1], x
n+1
= f(x
n
) (8.2)
este convergent la x [0, 1] si acesta este unicul punct din interval cu proprietatea f(x) = x
(punct x). Aratat i ca are loc
[x x
n
[
c
n
1 c
[x
0
x
1
[.
Solut ie. Mai ntai din condit ia (8.1) deducem ca funct ia f este continua. Fixam x
0

[0, 1]. Pentru orice > 0, exista =
1
c
astfel ca daca [x x
0
[ < sa rezulte
[f(x) f(x
0
)[ c[x x
0
[ < .
Unicitatea. Presupunem ca ar exista doua puncte x
1
, x
2
, x
1
,= x
2
. Atunci are loc
[x
1
x
2
[ = [f(x
1
) f(x
2
)[ c[x
1
x
2
[ < [x
1
x
2
[.
Pentru convergent a se arata ca x
n
este sir Cauchy. Evaluam pentrunceput, folosind (8.1)
[x
n+1
x
n
[ c
n
[x
0
x
1
[
Apoi
[x
n+p
x
n
[ [x
n+p
x
n+p1
[ +[x
n+p1
x
n+p2
[ + +[x
n+1
x
n
[
(c
n+p1
+ +c
n
)[x
0
x
1
[ =
= c
n
1 c
p
1 c
[x
0
x
1
[
c
n
1 c
[x
0
x
1
[.
Folosind ultima relat ie, rezulta ca este sir Cauchy. Trecand la limita pentru p de-
ducem
[x
n
x
0
[
c
n
1 c
[x
0
x
1
[.
Problema 8.9 Fie f : R R o functie continua, cu proprietatea ca exista C > 0 astfel
incat
[f(x) f(y)[ C [x y[ , x, y R
Sa se arate ca funct ia f este surjectiva.
Solut ie. Evident functia este injectiva. Fiind si continua, este strict monotona. Astfel,
imaginea este un interval. Daca ar majorat (minorat se discuta la fel) cu marginea
superioara M < , atunci ar exista un sir x
n
cu f(x
n
) M. Din ipoteza, ar urma ca
sirul (x
n
) este sir Cauchy, deci convergent x
n
x, adica M = f(x)
Problema 8.10 Fie f : [a, b] R o funct ie continua, atunci f este uniform continua.
Calcul diferent ial pentru funct ii de o variabila reala 47
Solut ie. Prin reducere la absurd: faptul ca f nu ar uniform continua pe [a, b] se
traduce prin

0
> 0 astfel ncat n N, x

n
, x

n
[a, b] astfel ncat
() [x

n
x

n
[ <
1
n
dar [f(x

n
) f(x

n
)[
0
.
Pe baza lemei lui Cesaro, putem extrage un subsir (x

n
k
)
k
, care sa e convergent. Notam
limita cu x
0
[a, b]. Datorita (), subsirul (x

n
k
)
k
va avea aceeasi limita. Dar
0 <
0
[f(x

n
k
) f(x

n
k
)[ 0
ceea ce constituie contradict ia cautata.
Problema 8.11 Fie f : [0, 1] R o funct ie continua, cu f(0) = f(1) = 0. Pentru ce
valori (0, 1) exista x [0, 1] astfel ncat f(x +) = f(x)?
Solut ie. Se arata ca orice =
1
n
, n 2 convine. Se considera funct ia g : [0, 1] R,
g(x) := f(x +) f(x). Deoarece
g(0) +g() +g(2) +. . . +g((n 1)) = 0
Rezulta ca se produce cel put in o schimbare de semn n sirul g(0), g(), g(2), . . ., g((n
1)). Deci exista x pentru care g(x) = 0.
Daca nu este de forma
1
n
, atunci se construieste funct ia h(x) := 1 cos
_
2x

_
.
Aceasta funct ie este continua iar h(0) = 0. Este evident periodica, de perioada . Mai mult
h(x) = 0 x = k., cu k ntreg. Deci H := h(1) ,= 0. Fie atunci f(x) := h(x) Hx =
1 cos
_
2x

_
1 cos
_
2

__
x. Funct ia f are proprietat ile cerute, dar:
f(x +) f(x) = (h(x +) H.(x +)) (h(x) H.x) = H. ,= 0
Problema 8.12 Daca f este derivabila, atunci f este convexa daca si numai daca f

este
crescatoare.
Solut ie. Fie x
1
< x
2
I. Daca f

este crescatoare, sa denim funct ia


g(x) := f(x) f(x
1
) (x x
1
)
f(x
2
) f(x
1
)
x
2
x
1
Deoarece g(x
1
) = g(x
2
) = 0 si g

este crescatoare, urmeaza ca exista un punct c (x


1
, x
2
)
astfel ncait g

(x) 0, x (x
1
, c) si g

(x) 0, x (c, x
2
). Rezulta g(x) 0,
x (x
1
, x
2
).
Reciproc, daca f este convexa, iar x
1
< x
2
I, atunci pentru orice x (x
1
, x
2
) avem:
f(x) f(x
1
)
x x
1

f(x
2
) f(x
1
)
x
2
x
1

f(x) f(x
2
)
x x
2
de unde, prin trecere la limita, se obt ine f

(x
1
) f

(x
2
).
Problema 8.13 Gasit i numarul de extreme locale ale funct iei g(x) = f(f(x)), daca
f(x) = x
3
3x + 1.
48
Solut ie. Derivata funct iei este
g

= f

(f(x))f

(x) = 3(f
2
(x) 1)3(x
2
1) =
= 9(x
3
3x)(x
3
3x + 2)(x
2
1).
Radacinile derivatei, care verica condit ia de extrem sunt 0, 2, 1,

3. Exista deci 6
puncte de extrem.
Problema 8.14 Fie f : R R o funct ie de doua ori derivabila care verica
f

(x) +f

(x) = f(x), x [0, 1]


si astfel ca f(0) = f(1) = 0. Aratat i ca f(x) = 0, x [0, 1]
Solut ie. Din teorema lui Rolle, exista c (0, 1) astfel ca f

(c) = 0. Daca presupunem


ca c este un punct de maxim ar urma ca f(c) > 0, iar din ipoteza f

(c) = f(c) > 0 ar


imposibil, deoarece funct ia ar rezulta convexa n jurul punctului de maxim. Daca f(c) < 0
ar urma f

(c) < 0 situat ie iar imposibila. Analog daca c ar punct de minim.


Problema 8.15 Calculand n doua moduri derivatele de ordin p, 1 p n ale funct iei
(1 e
x
)
n
, sa se deduca valoarea sumei
n

k=0
(1)
k
C
k
n
k
p
.
Solut ie. Pe de o parte, deoarece (1 e
x
)
n
=
n

k=0
(1)
k
C
k
n
e
kx
, rezulta ca derivata de
ordin p este
n

k=0
(1)
k
C
k
n
k
p
e
kx
. Sumele cerute se obt in deci pentru x = 0.
Pe de alta parte, se arata prin recurent a ca derivata de ordin p ( pentru 1 p n)
este:
p

k=1
a
p
k
(e
x
)
k
(1 e
x
)
nk
, unde a
p
p
= (1)
p
A
p
n
(aranjamente).

In adevar, cazul p = 1 este imediat; iar prin derivare nca odata, se obt ine:
p

k=1
a
p
k
k(e
x
)
k1
(1 e
x
)
nk
+
p

k=1
a
p
k
(e
x
)
k
(1)
k
e
x
(n k)(1 e
x
)
nk1
Regrupand termenii, gasim:
a
p
1
(e
x
)(1 e
x
)
n1
+
p

k=2
_
a
p
k
k +a
p
k1
(k 1 n)

(e
x
)
k
(1 e
x
)
nk
+
+a
p
p
(p n)(e
x
)
p+1
(1 e
x
)
np1
ceea ce arata ca a
p+1
p+1
= a
p
p
(p n). Vericarea este astfel ncheiata.
Pe baza acestei formule, valoarea derivatei de ordin p n x = 0 este: 0, daca 1 p < n;
iar pentru p = n ramane a
n
n
= (1)
n
A
n
n
= (1)
n
n!.
Problema 8.16 Un segment de lungime 1 se misca astfel ncat capetele raman pe axele
de coordonate.

In timp ce se misc a n plan, segmentul schimba culoarea part ii marginite
de el la stanga si axe. Gasit i ecuat ia curbei care separa partea din plan n care culoarea
s-a schimbat, de partea din plan n care culoarea ra mane aceeasi.
Calcul diferent ial pentru funct ii de o variabila reala 49
Solut ie. Raspunsul este o parte a curbei a x
2/3
+y
2/3
= 1 (astroida). Evident pentru
x [0, 1] ordonata punctului corespunzator de pe curba va egala cu cea mai mare
valoare a ordonatelor punctelor de pe segment, pentru x xat. Sa examinam pozit ia n
care unul dintre capete este A(t, 0) si celalalt B(0,

1 t
2
), t [0, 1]. Fie y(x, t) ordonata
punctelor de pe segmentul de extremitat i aA si B, corespunzator abscisei x. y = y(x) =
maxy(x, t), t [x, 1]. (Pozit ia segmentului n care t < x nu trebuie examinata). Astfel,
y(x; t) = (1 x/t)
_
1 t
2
) si
y

=
x
t
2
_
1 t
2
(1
x
t
)
t

1 t
2
=
x t
3
t
2

1 t
2
.
Derivata se anuleaza n t =
3

x care este punct de maxim, de unde urmeaza


y(x,
3

x) =
_
1 x
2/3
_
3/2
.
Problema 8.17 Data funct ia
f(x) =
_
(1 + tg(2x))(1 + tg(4x))(1 + tg(6x)) (1 + tg(32x))
calculat i f

(0).
Solut ie. Notam u(x) = (1 + tg(2x))(1 + tg(4x))(1 + tg(6x)) (1 + tg(32x)) si avem
u(0) = 1. Observam ca f

(x) =
u

(x)
2
_
u(x)
, iar u

este o suma de 16 termeni, ecare ind


produs de 15 factori care apar n scrierea lui u si un factor de forma (1+tg2kx)

=
2k
cos
2
2kx
.
Fiecare termen are n x = 0 valoarea 2k, k = 1, , 16, iar
f

(0) =
u

(0)
2
_
u(0)
=
1
2
16

k=1
2k = 136.
Problema 8.18 Fie f : R R, f(x) =
1
x
2
+1
. Gasit i f
(6)
(0) .
Solut ie. Raspunsul este -720. Din formula lui MacLaurin:
f(x) = f(0) +
f

(0)
1!
x +
f

(0)
2!
x
2
+. . . +
f
(6)
(0)
6!
x
6
+o(x
6
).
Pe de alta parte seria geometrica cu rat ia x
2
este
1
x
2
+ 1
= 1 x
2
+x
4
x
6
+ .
Din unicitate rezulta armat ia.
Problema 8.19 O particula se misca de-a lungul unei drepte. Direct ia miscarii se poate
schimba, dar accelerat ia n orice moment nu depaseste 1 m/sec n valoare absoluta. Dupa
o secunda de la nceputul miscarii particula se ntoarce n punctul de pornire. Aratat i ca
viteza la 0,5 sec de la nceputul miscarii nu este mai mare ca 0,25 m/sec.
50
Solut ie.

In calculele urmatoare se subnt elege ca timpul se masoara n secunde, iar
distant ele n metri. Fie v(t) viteza, iar a(t) accelerat ia particulei la momentul t. Din enunt
avem [a(t)[ = [v

(t)[ 1 si
_
1
0
v(t)dt = 0. Sa estimam v(0, 5).
[v(0, 5)[ = [v(0, 5) 0[ =

v(0, 5)
_
1
0
v(t)dt

v(0, 5)
_
1
0
dt
_
1
0
v(t)dt

_
1
0
(v(0, 5) v(t))dt

.
Folosind teorema lui Lagrange v(0, 5) v(t) = v

(c)(0, 5 t), c (0, 5, t). Astfel


[v(0, 5)[ =

v(0, 5)
_
1
0
v(t)dt

_
1
0
[v

(c)(0, 5 t)[dt =
_
1
0
[v

(c)[[(0, 5 t)[dt
_
1
0
[(0, 5 t)[dt = 1/4.
Problema 8.20 Fie P
n
(x) un polinom de grad par n (n > 1), care are coecientul domi-
nant pozitiv si e P
n
(x) > P

n
(x), x R. Aratat i ca P
n
(x) > 0, x R.
Solut ie. Sa observam ca daca P
n
are grad par, lim
x
P
n
(x) = . De aceea exista
x
0
punct de minim absolut pentru polinom, n care P

n
(x
0
) = 0 si P

n
(x
0
) 0. Folosind
ipoteza avem
P
n
(x) P
n
(x
0
) > P

n
(x
0
) 0, x.
Problema 8.21 Aratat i ca pentru orice polinom p(x) de grad n > 1 si orice punct Q,
numarul tangentelor la gracul lui p(x) care trec prin Q nu depaseste n.
Solut ie. Derivata funct iei p(x) = a
0
+ a
1
x + + a
n
x
n
este p

(x) = a
1
+ 2a
2
x +
+ na
n
x
n1
. Ecuat ia tangentei n punctul (x
0
, p(x
0
)) este y y(x
0
) = p

(x)(x x
0
).
Presupunem ca Q are coordonatele (a, b) si nlocuind n ecuat ia tangentei obt inem
b (a
0
+a
1
x
0
+ +a
n
x
n
0
) = (a
1
+ 2a
2
x
0
+ +na
n
x
n1
0
)(a x
0
).
Daca n > 1, coecientii lui x
n
0
din cei doi membrii ai relat iei nu pot egali si egalitatea
conduce la o ecuat ie algebrica de ordin n; urmeaza ca este imposibil sa gasim un numar
de solut ii mai mare ca n.
Problema 8.22 Aratat i ca are loc identitatea:
2 arccos x = arccos(2x
2
1), 0 x 1.
Solut ie. Fie f(x) = 2 arccos x arccos(2x
2
1) 0 x 1. Funct ia este continua pe
[0, 1], derivabila pe (0, 1) si se verica imediat ca
f

(x) =
2

1 x
2
+
2x
[x[

1 x
2
.
Deci f

(x) = 0, x (0, 1). Deci f este o constanta, iar din faptul ca f(0) = 0 si continuitate
rezulta armat ia.
Calcul diferent ial pentru funct ii de o variabila reala 51
Problema 8.23 Fie f o funct ie derivabila pe (a, b) cu derivata continua, cu proprietat ile
lim
xa
f(x) = , lim
xb
f(x) = si f

(x) +f
2
(x) 1, x (a, b). Aratat i ca b a
si dat i un exemplu pentru care b a = .
Solut ie. Din inegalitate deducem
d
dx
(arctg f(x) +x) =
f

(x)
1 +f
2
(x)
+ 1 0 x (a, b)
urmeaza ca arctg f(x) +x este nedescrescatoare pe interval si folosind limitele obt inem
/2+a /2+b, de unde ba . Egalitatea are loc pentru f(x) = ctg x, a = 0, b = .
Problema 8.24 Fie f o funct ie de doua ori diferent iabila , cu derivata a doua continua
pe (0, ), astfel ca lim
x0
f

(x) = si lim
x0
f

(x) = . Aratat i ca lim


x0
f(x)
f

(x)
= 0.
Solut ie. Deoarece f

tinde la si f

tinde la , cand x tinde la 0 prin valori


mai mari, exista un interval (0, r) astfel ca f

(x) < 0 si f

(x) > 0, x (0, r). Deci


f este descrescatoare si f

este crescatoare pe (0, r). Din teorema de medie, pentru orice


0 < x < x
0
< r avem
f(x) f(x
0
) = f

()(x x
0
) > 0
pentru un (x, x
0
). Folosind faptul ca f

este crescatoare, f

(x) < f

() < 0, obt inem


x x
0
<
f

()
f

(x)
(x x
0
) =
f(x) f(x
0
)
f

(x)
< 0.
Luand limita cand x 0 obt inem
x
0
liminf
x0
f(x)
f

(x)
limsup
x0
f(x)
f

(x)
0.
Deoarece acest lucru are loc pentru orice x
0
(0, r) deducem ca exsita lim
s0
f(x)
f

(x)
si
valoarea ei este 0.
Problema 8.25 Fie f : [0, 1] R o funct ie derivabila, cu f(0) = 0. Daca exista M 0
astfel ncat [f

(x)[ M[f(x)[, pentru orice x [0, 1], atunci f este identic nula.
Solut ie. Denind g : [0, 1] R prin g(x) := f
2
(x), obt inem o funct ie pozitiva,
derivabila pe [0, 1], care satisface g(0) = 0 si [g

(x)[
M
2
[g(x)[, pentru orice x [0, 1]. Fie
mai departe h : [0, 1] R, h(x) := e

M
2
x
g(x). Deoarece h

(x) = e

M
2
x
[g

(x)
M
2
g(x)],
ipoteza arata ca h

(x) 0. Deci funct ia h rezulta descrescatoare, pozitiva si h(0) = 0. De


aici concluzia.
Problema 8.26 Exista funct ii continuu diferent iabile f : R R astfel ca pentru orice
x R sa avem f(x) > 0 si f

(x) = f(f(x))?
Solut ie. Presupunem ca ar exista o astfel de funct ie. Deoarece f

(x) = f(f(x)) > 0,


funct ia este strict crescatoare. Din f(x) > 0 deducem f(f(x)) > f(0), x. Deci f

(x) >
f(0) si pentru orice x < 0 avem f(x) < f(0) + xf(0) = (1 + x)f(0). Deci daca x 1
rezulta f(x) 0, contrazicand proprietatea f(x) > 0. Concluzia este ca astfel de funct ii
nu exista.
52
Problema 8.27 Funct ia f : R R este de doua ori diferent iabila si satisface f(0) =
2, f

(0) = 2 si f(1) = 1. Aratat i ca exista un numar real (0, 1), astfel ca


f()f

() +f

() = 0.
Solut ie. Denim funct ia g(x) :=
1
2
f
2
(x) +f

(x). Deoarece g(0) = 0 si and f(x)f

(x) +
f

(x) = g

(x) este sucient sa arat am ca exista un numar real 0 < 1 astfel ca g() = 0.
a) Daca f nu se anuleaza niciodata, e h(x) :=
x
2

1
f(x)
. Deoarece h(0) = h(1) =
1
2
,
exista un numar real 0 < < 1 pentru care h

() = 0. Dar g = f
2
h

si armat ia rezulta.
b) Daca f are cel put in un zero, e z
1
cel mai mic si z
2
cel mai mare, care exista
deoarece mult imea zerourilor este nchisa. Are loc 0 < z
1
z
2
< 1. Funct ia f este
pozitva pe intervalele [0, z
1
) si (z
2
, 1], ceea ce implica f

(z
1
) 0 si f

(z
2
) 0. Atunci
g(z
1
) = f

(z
1
) 0 si g(z
2
) = f

(z
2
) 0, si exista numarul real [z
1
, z
2
] pentru care
g() = 0.
Observam ca pentru funct ia f(x) =
2
x + 1
condit iile au loc si ff

+ f

este constant
egala cu 0.
Problema 8.28 Daca x, y > 0 aratat i ca x
y
+y
x
> 1.
Solut ie. Daca x sau y sunt 1 armat ia este evidenta. Presupunem 0 < x 1, 0 <
y 1. Putem nota y = kx, 0 < k < 1 si consideram funct ia
f(x) = x
kx
+ (kx)
x
.
Se poate arata ca x
x
are valoarea minima a = e

1
e
si deoarece pentru x (0, 1] are loc
k
x
k funct ia de mai sus poate minorata
f(x) a
k
+ka, k (0, 1].
Derivata funct iei h(k) = a
k
+ ak este pozitiva, deci h este cresca toare si urmeaza ca
h(k) > 1, de unde rezulta armat ia.
Problema 8.29 Fie f : (0, 1) R o funct ie de doua ori derivabila cu derivata a doua
continua, astfel ca [f

(x) + 2xf

(x) + (x
2
+ 1)f(x)[ 1, x. Aratat i ca lim
x
f(x) = 0.
Solut ie. Fie g(x) = f

(x)+xf(x); atunci f

(x)+2xf

(x)+(x
2
+1)f(x) = g

(x)+xg(x).
Aratam mai ntai ca daca h este continuu diferent iabila si satisface faptul ca h

(x) +xh(x)
este marginita atunci lim
x
h(x) = 0. Fie M marginea superioara pentru [h

(x) +xh(x)[ si
e p(x) = h(x)e
x
2
/2
. Atunci
[p

(x)[ = [h

(x) +xh(x)[e
x
2
/2
Me
x
2
/2
si
[h(x)[ =

p(x)
e
x
2
/2

=
=

p(0) +
_
x
0
p

e
x
2
/2

[p(0)[ +M
_
x
0
e
x
2
/2
dx
e
x
2
/2
Calcul diferent ial pentru funct ii de o variabila reala 53
Deoarece lim
x
e
x
2
/2
= rezulta, daca folosim regula lui lHopital, ca lim
x
_
x
0
e
t
2
/2
dt
e
x
2
/2
= 0;
aceasta implica lim
x
h(x) = 0.
Aplicand acest rezultat pentru h = g, apoi pentru h = f, rezulta armat ia problemei.
Problema 8.30 Presupunem ca funct iile diferent iabile a, b, f, g : R R satisfac
f(x) 0, f

(x) 0, g(x) > 0, g

(x) > 0, x R,
lim
x
a(x) = A > 0, lim
x
b(x) = B > 0, lim
x
f(x) = lim
x
g(x) = .
si
f

(x)
g

(x)
+a(x)
f(x)
g(x)
= b(x).
Aratat i ca lim
x
f(x)
g(x)
=
B
A+ 1
.
Solut ie. Fie 0 < < A un numar real oarecare. Pentru x sucient de mare f(x) >
0, g(x) > 0, [a(x) A[ < , [b(x) B[ <
B < b(x) =
f

(x)
g

(x)
+a(x)
f(x)
g(x)
<
f

(x)
g

(x)
+ (A+)
f(x)
g(x)
<
<
(A+)(A+ 1)
A
f

(x)(g(x))
A
+Af(x)(g(x))
A1
g

(x)
(A+ 1)(g(x))
A
g

(x)
=
=
(A+)(A+ 1)
A
(f(x)(g(x))
A
)

((g(x))
A+1
)

.
Din inegalitatea de mai sus deducem
(f(x)(g(x))
A
)

((g(x))
A+1
)

>
A(B )
(A+)(A+ 1)
.

In mod similar se obt ine, pentru x sucient de mare


(f(x)(g(x))
A
)

((g(x))
A+1
)

<
A(B +)
(A)(A+ 1)
.
Daca 0, avem
lim
x
(f(x)(g(x))
A
)

((g(x))
A+1
)

=
B
A+ 1
.
Aplicand regula lui lHopital deducem
lim
x
f(x)
g(x)
= lim
x
f(x)(g(x))
A
(g(x))
A+1
=
B
A+ 1
.
Problema 8.31 Presupunem ca f : R R este o funct ie de doua ori diferent iabila care
satisface f(0) = 1, f

(0) = 0 si care pentru orice x [0, ) are proprietatea


f

(x) 5f

(x) + 6f(x) 0.
Aratat i ca pentru orice x [0, ), are loc
f(x) 3e
2x
2e
3x
.
IMC, 2009
54
Solut ie. Avem f

(x) 2f

(x) 3(f

(x) 2f(x)) 0, x [0, ).


Fie g(x) = f

(x) 2f(x), x [0, ). Rezulta ca


g

(x) 3g(x) 0, x [0, ),


de unde urmeaza ca
(g(x)e
3x
)

0 x [0, ),
de unde
g(x)e
3x
g(0) = 2, x [0, ),
sau echivalent
f

(x) 2f(x) 2e
3x
, x [0, ).
Analog obt inem
(f(x)e
2x
)

2e
x
, x [0, ),
sau echivalent
(f(x)e
2x
+ 2e
x
)

0, x [0, ).
Rezulta ca
f(x)e
2x
+ 2e
x
f(0) + 2 = 3, x [0, )
sau echivalent
f(x) 3e
2x
2e
3x
, x [0, ).
Problema 8.32 Comparat i tg (sin x) si sin(tg x) pentru x (0, /2).
Solut ie. Fie f(x) = tg (sin x) sin(tg x). Atunci
f

(x) =
cos x
cos
2
(sin x)

cos(tg x)
cos
2
x
=
cos
3
x cos(tg x) cos
2
(sin x)
cos
x
cos
2
(tg x)
.
Fie 0 < x < arctg /2. Din concavitatea funct iei cosinus pe (0, /2), rezulta ca
3
_
cos(tg x) cos
2
(sin x) <
cos(tg x) + 2 cos(sin x)
3
cos
tg x + 2 sin x
3
< cos x.
Ultima inegalitate rezulta din
_
tg x + 2 sin x
3
_

=
1
3
_
1
cos
2
x
+ 2 cos x
_

3
_
1
cos
2
x
cos xcos x = 1.
De aici deducem ca
cos
3
x cos(tg x) cos
2
(sin x) > 0.
Rezulta ca f

(x) > 0 si deci f creste pe intervalul [0, arctg/2].


Urmeaza sa mai observam ca folosind si faptul ca 4 +
2
< 16 avem
tg[sin(arctg /2)] = tg
/2
1 +
2
/4
> tg /4 = 1.
Aceasta implica faptul ca daca x [arctg /2, /2] atunci tg(sin x) > 1 de unde
obt inem ca f(x) > 0.
Problema 8.33 Fie f : (1, 1) [0, ) o funct ie de trei ori derivabila, cu f(0) = 0. Sa
se discute derivabilitatea n 0 a funct iilor
_
f(x) si
3
_
f(x).
Calcul diferent ial pentru funct ii de o variabila reala 55
Solut ie. Funct ia f ind pozitiva si f(0) = 0, rezulta ca 0 este punct de minim, deci
f

(0) = 0. Astfel: f(x) =


f

(0)
2
x
2
+o(x
2
). Obt inem ca exista derivatele laterale n 0
pentru
_
f(x):
lim
x 0
x > 0
_
f(x)
x
= lim
x 0
x > 0
_
f

(0)
2
+o(1) =
_
f

(0)
2
Analog, derivata la stanga este
_
f

(0)
2
.

In concluzie, daca f

(0) = 0, atunci
_
f(x)
este derivabila n 0, cu derivata egala cu 0.
Pentru funct ia
3
_
f(x) se foloseste scrierea f(x) =
f

(0)
2
x
2
+
f

(0)
3!
x
3
+o(x
3
) si se
cauta eventuala limita n 0 pentru
3
_
f

(0)
2
x
1
+
f

(0)
6
+o(1)
Pentru existent a derivatei, condit ia f

(0) = 0 este necesara. Reciproc, daca f

(0) = 0,
atunci funct ia
3
_
f(x) este derivabila n 0, iar valoarea derivatei este
3
_
f

(0)
6
.
Problema 8.34 Sa se calculeze:
lim
x0,x>0
x
x

_
x
2
_
2x
x
sin x

_
x
2
_
sin 2x
.
Solut ie. Folosind scrierea u
v
= e
v lnu
se obt in dezvoltarile cu rest Peano:
x
x
= 1 +xln x +o(xln x)
_
x
2
_
2x
= 1 + 2xln
x
2
+o(xln x)
x
sin x
= 1 +xln x +o(xln x)
_
x
2
_
sin 2x
= 1 + 2xln
x
2
+o(xln x)
ceea ce arata ca limita cautata este 1.
Problema 8.35 Se considera funct ia f : (0, ) R
f(x) =
C
x
e
ax
, a > 0.
Aratat i ca nu exista nici un C R astfel ca
(1)
k
f
(k)
(x) 0, k N, x (0, ).
56
Solut ie. Derivata de ordin k a funct iei este
f(x)
(k)
= (1)
k
_
Ck!
x
k+1
a
k
e
ax
_
.
Daca nmult im cu (1)
k
avem
(1)
k
f(x)
(k)
=
Ck!
x
k+1
a
k
e
ax
iar aceasta este pozitiva. Deducem
C
a
k
k!
x
k+1
e
ax
, x > 0. (8.3)
Studiem variat ia funct iei g(x) = x
k+1
e
ax
. Din tabloul de variat ie deducem ca funct ia
g are un punct de maxim n x =
k+1
a
. Avem atunci
C
a
k
k!
g(
k + 1
a
) =
a
k
k!
_
k + 1
a
_
k+1
e
a
k+1
a
=
=
(k + 1)
k+1
k!
e
(k+1)
a
.
Pe baza formulei lui Stirling membrul din dreapta relat iei de mai sus poate minorat cu
(k + 1)
k+1

2k(
k
e
)
k
x
k
ae
k+1
=
1
ae

2
(1 +
1
k
)
k
k + 1

k
x
k
1, daca k
expresie care tinde la , pentru k . Deci inegalitatea (8.3) nu mai este posibila.
Problema 8.36 Fie f : [0, ) R o funct ie de doua ori derivabila pe [0, ). Daca
[f(x)[ A si [f

(x)[ B, pentru orice x [0, ), atunci [f

(x)[ 2

AB, pentru orice


x [0, ).
Solut ie. Pentru x (0, ) xat, pentru ecare h > 0, exista (0, 1) astfel ncat
f(x +h) = f(x) +hf

(x) +
h
2
2
f

(x +h)
De aici:
f

(x) =
f(x +h) f(x)
h

h
2
f

(x +h)
deci:
[f

(x)[
2A
h
+
hB
2
Scriind aceasta inegalitate pentru h := 2
_
A
B
, se obt ine rezultatul.
Problema 8.37 Fie f : (a, b) R o funct ie de doua ori continuu derivabila. Aratat i ca
pentru orice x (a, b) are loc
lim
h0
f(x +h) 2f(x) +f(x h)
h
2
= f

(x).
Calcul diferent ial pentru funct ii de o variabila reala 57
Solut ie. Folosim formula lui Taylor pentru h > 0. Exista z (x, x+h) si u (xh, h)
astfel ca
f(x +h) f(x) = f

(x)h +
f

(z)
2
h
2
f(x h) f(x) = f

(x)h +
f

(u)
2
h
2
.
Daca adunam cele doua relat ii si mpart im la h
2
trecem la limita si obt inem
lim
h0
f(x +h) 2f(x) +f(x h)
h
2
= lim
h0
f

(z) +f

(u)
2
= f

(x).
Problema 8.38 Fie f : [a, b] R de doua ori continuu derivabila pe (a, b). Aratat i ca
pentru orice x [a, b] exista (a, b) astfel ca
f(x) f(a)
f(b) f(a)
b a
(x a) =
1
2
(x a)(x b)f

().
Solut ie. Daca x = a sau x = b relat ia este evidenta.
Denim funct ia h : [a, b] R
h(x) =
_
f(x)f(a)
(xa)
x ,= a
f

(a) x = a
.
Funct ia h este continuu pe [a, b], derivabila pentru x (a, b) iar
h

(x) =
(x a)f

(x) (f(x) f(a))


(x a)
2
, x (a, b).
Deci pentru orice x (a, b) exista
1
(x, b) astfel ca
h(x) h(b) = h

(
1
)(x b). (8.4)
Aplicam dezvoltarea funct iei f n y (a, b) cu rest Lgrange. Exista
2
(a, y) astfel
ca
f(a) = f(y) + (a y)f

(y) +
(a y)
2
2
f

(
2
). (8.5)
Fie x (a, b) si
1
(x, b) care verica (8.4). Fie
2
(a,
1
) ales n (8.5) pentru
y =
1
. Astfel avem
h

(
1
) =
h(x) h(b)
x b
=
f(x)f(a)
xa

f(b)f(a)
ba
x b
=
(f(x) f(a))
f(b)f(a)
ba
(x a)
(x a)(x b)
.
Dar, daca folosim (8.5)
h

(
1
) =
(
1
a)f

(
1
) (f(
1
) f(a))
(
1
a)
2
=
f

(
2
)
2
,
de unde deducem armat ia problemei.
58
Problema 8.39 Fie I IR un interval deschis cu 0 I si f : I IR o funct ie continua,
cu f(0) = 0, derivabila (la dreapta) n 0. Atunci:
lim
n
n

k=1
f
_
k
n
2
_
=
1
2
f

d
(0)
Caz particular: Sa se calculeze
lim
n
n

k=1
_
_
1 +
k
n
2
1
_
Solut ie. Din denit ia derivatei la dreapta deducem ca: pentru orice > 0 exista n

astfel ncat pentru orice n n

si 1 k n:
f

d
(0) <
f
_
k
n
2
_
k
n
2
< f

d
(0) +
Prin sumare rezulta:
f

d
(0)
n
2
n(n + 1)
2
<
n

k=1
f
_
k
n
2
_
<
f

d
(0) +
n
2
n(n + 1)
2
deci limita exista si este egala cu
f

d
(0)
2
.
Observat ie. Sirul dat nu este reductibil la o suma integrala.
Problema 8.40 Fie f : (0, ) R o funct ie derivabila. Presupunem ca exista
lim
x
_
f(x) +f

(x)
_
= l R
Sa se arate ca lim
x
f(x) = l si lim
x
f

(x) = 0.
Solut ie. Considerand funct ia g(x) := f(x)e
x
, scriem f(x) =
g(x)
e
x
si aplicam regula lui
lHopital (generalizata!). Deoarece
g

(x)
e
x
= f(x) +f

(x) l, deducem ca si f(x) l.


Problema 8.41 Fie f : R R o funct ie cu derivata de ordin doi continua. Daca f(x +
y)f(x y) f
2
(x), oricare ar x, y R, sa se arate ca f(x)f

(x) f

2
(x) pentru orice
x R
Solut ie. Pentru x R denim g : R R prin g(y) := f(x + y)f(x y) f
2
(x).
Urmeaza ca
g

(y) = f

(x +y)f(x y) f(x +y)f

(x y)
g

(y) = f

(x +y)f(x y) 2f

(x +y)f

(x y) +f(x +y)f

(x y)

In particular: g(0) = g

(0) = 0 si g

(0) = 2[f(x)f

(x) f

2
(x)]. Ipoteza arata acum ca
g(y) g(0) = 0, pentru orice y R. Deci y = 0 este un punct de maxim, n care g

(0) 0
n mod necesar, deci concluzia.
Calcul diferent ial pentru funct ii de o variabila reala 59
Problema 8.42 Fie I R un interval. Fie f : I R o funct ie oarecare. Sa se arate ca
f are o mult ime cel mult numarabila de extreme stricte.
Solut ie. Pentru ecare subinterval [a, b] (0, 1) exista c (a, b) n care f si atinge
maximul absolut. Fiind si punct de minim local, urmeaza ca f este constanta pe o
vecinatate a punctului c. Sa notam
:= infx [a, b] [ f(x) = f(c)
Avem < c; daca am presupune a < prin acelasi rat ionament sar gasi puncte x <
cu f(x) = f(c). Considerand analog supx [a, b] [ f(x) = f(c), rezulta f constanta pe
ecare subinterval.
Problema 8.43 Fie I R un interval. Fie f : I R o funct ie cu derivate laterale n
ecare punct. Sa se arate ca f are derivata pe I, cu except ia eventuala a unei mult imi cel
mult numarabile.
Solut ie. Sa notam, pentru ecare r Q:
A
r
:= x I [ f

s
(x) < r < f

d
(x)
Este sucient sa aratam ca ecare A
r
este o mult ime cel mult numarabila. Consideram
funct ia g(x) := f(x) rx; deci g

s
(x) < 0 < g

d
(x) n ecare x A
r
. Dar aceasta relat ie
implica faptul ca x este punct de minim strict pentru g, de unde concluzia.
Problema 8.44 Fie f : [a, b] R o funct ie continua pe [a, b], derivabila n punctele a
si b; f

(a) < 0, f

(b) > 0. Sa se arate ca exista c (a, b) punct de minim pentru f.


Solut ie. f ind funct ie continu a pe [a, b], exista c [a, b] astfel ncat f(c) = inf
x[a,b]
f(x).
Avem de aratat ca c ,= a si c ,= b. Ipoteza ca f

(a) < 0 arata ca exista > 0 astfel ncat


f(x) < f(a), pentru orice x (a, a + ). De aici urmeaza ca c ,= a. Analog se arata si
c ,= b.
Problema 8.45 Fie f : R R o funct ie de doua ori derivabila. Daca f(x) 0 si
f

(x) 0, pentru orice x R, atunci f este constanta.


Solut ie. f

este descrescatoare, deci exista A := lim


x
f

(x) (, ] si
B := lim
x
f

(x) [, ) iar A B. Analizam pe rand cazurile:


I. A > 0; exista a R astfel ncat f

(x)
A
2
, pentru orice x (, a]. Astfel
f(a) f(x) = f

()(a x)
A
2
(a x), de unde f(x) f(a)
Aa
2
+
Ax
2
, ceea
ce contrazice f 0.
II. A 0. Daca B = 0, se obt ine concluzia: daca B < 0, se repeta rat ionamentul:
exista b R astfel ncat f

(x)
B
2
, pentru orice x [b, ), de unde:
f(x) f(b) = f

()(x b)
B
2
(x b)
deci f(x) f(b)
Bb
2
+
Bx
2
, din nou contradict ie.
60
Problema 8.46 Fie f : [0, 1] R o funct ie de doua ori derivabila, care verica f

(x) +
2f

(x) + f(x) 0, pentru orice x (0, 1) si f(0) = f(1) = 0. Atunci f(x) 0, pentru
orice x [0, 1].
Solut ie. Se considera funct ia g(x) := e
x
f(x). Atunci g

(x) 0 asigura concluzia.


Problema 8.47 Fie f : [a, b] R o funct ie cu derivata de ordin doi continua pe [a, b],
satisfacand f(a) = f(b) = 0. Notam M := sup
x[a,b]
[f

(x)[.
(i) Sa se arate ca, oricare ar x [a, b], au loc inegalitat ile:
() [f

(x)[
M(b a)
2
() [f(x)[
M(x a)(b x)
2
(ii) Daca exista x
0
[a, b] (resp. (a, b)) astfel ncat () (resp. ()) sa devina egalitate
pentru x = x
0
, atunci f este una din funct iile
M(x a)(b x)
2
.
Solut ie. (i) Pentru ecare x (a, b) xat, consideram funct ia g : [a, b] R,
g(t) := f(t)
A(t a)(t b)
2
n care A :=
2f(x)
(x a)(x b)
. Avem g(a) = g(x) = g(b) = 0, deci exista c

, c

(a, b) astfel
ncat g

(c

) = g

(c

) = 0. Exista deci c (a, b) astfel ncat g

(c) = 0. Aceasta revine la


f(x) =
(x a)(x b)
2
f

(c), de unde rezulta ().


Prin trecere la limita, se obt ine (), pentru x = a si x = b. Pentru a demon-
stra () n restul cazurilor, consideram x (a, b] xat si funct ia h : [a, b] R,
h(t) := f(t)
f(x)(t a)
x a
. Deoarece h(a) = h(x) = 0 si h

= f

, rezulta ca putem
aplica cele de mai sus funct iei h, pe intervalul [a, x]. Deducem ca [h

(x)[
M
2
(x a).

Insa h

(x) = f

(x)
f(x)
x a
, de unde
[f

(x)[ [h

(x)[ +
[f(x)[
x a

M
2
(x a) +
M(x a)(b x)
2(x a)
=
M(b a)
2
(ii) Observam ca, daca se realizeaza egalitate n (), pentru un x
0
(a, b), atunci
se realizeaza egalitate n acelasi punct si n (). Fie deci x (a, b), astfel ncat ()
devine egalitate. Denim funct ia u : [a, b] R, u(x) := f(x)
M(x a)(x b)
2
. Deci
u(a) = u(b) = 0; u(x) 0, pentru orice x [a, b]; u

(x) 0, pentru orice x (a, b).



In
plus, x
0
ind punct de minim pentru u, deducem ca u

(x
0
) = 0. u

ind descrescatoare,
rezulta ca u

(x) 0, pentru orice x [x


0
, b], deci u este descrescatoare pe acest interval.
Dar u(x
0
) = 0 arata ca u 0 pe acest interval. Analog se arata ca u 0 pe [a, x
0
].

In
sfarsit, daca () devine egalitate pentru x
0
= a sau b, atunci, cu aceleasi notat ii ca mai
sus, rezulta direct u

(a) = 0 si deci concluzia.


Calcul diferent ial pentru funct ii de o variabila reala 61
Problema 8.48 (i) Sa se arate c a exista o unica funct ie f : [0, ) R astfel ncat,
pentru ecare x [0, ) sa avem f
3
(x) + xf(x) = 1. Sa se arate ca f este derivabila, iar
f

(x) =
f(x)
x + 3f
2
(x)
, pentru orice x [0, ).
(ii) Sa se arate ca exista o unica funct ie f : R R, care are proprietat ile: x
3

f
3
(x) +xf(x) = 0, pentru orice x R si f(x) > 0, pentru orice x > 0. Sa se demonstreze
ca aceasta funct ie este derivabila pe R 0.
Solut ie. (i) Consideram funct ia g : (0, 1] [0, ), g(y) =
1 y
3
y
. Deoarece
lim
y0
g(y) = , g(1) = 0, g

(y) =
1
y
2
2y, rezulta ca g este biject ie. Inversa ei f(x) = y
verica exact relat ia propusa. Conform unor rezultate cunoscute, f este continua, deriv-
abila, iar derivata se calculeaza dupa formula:
f

(x) =
1
g

(f(x))
=
1

1
f
2
(x)
2f(x)
=
f(x)
x + 3f
2
(x)
(ii) Fie funct ia g : R R, g(y) = y
3
+ xy + x
3
. Pentru x < 0, g este strict de-
screscatoare pe R si lim
x
g(y) = , lim
x
g(y) = , deci exista y R unic astfel ncat
g(y) = 0.
Pentru x = 0, evident y = 0 este singura radacina a ecuat iei g(y) = 0.
Pentru x > 0 ecuat ia g(y) = 0 are o singura radacina y > 0 (si eventual altele negative).
Aceasta arata existent a si unicitatea funct iei f, cu proprietat ile din enunt .
Fie x
0
R. Scazand membru cu membru formulele pentru x si x
0
, gasim:
(x x
0
)(x
2
+xx
0
+x
2
0
+f(x
0
)) =
= (f(x) f(x
0
))(f
2
(x)f(x)f(x
0
) +f
2
(x
0
) x)
Pentru x
0
< 0, aceasta relat ie arata continuitatea funct iei f n x
0
. Pentru x
0
> 0, este
necesar sa observam, pe baza sirului lui Rolle, ca f
2
(x) > x; deci si n acest caz rezulta
continuitatea funct iei f n x
0
. Acum putem trece la limita n egalitatea:
f(x) f(x
0
)
x x
0
=
x
2
+xx
0
+x
2
0
+f(x
0
)
f
2
(x)f(x)f(x
0
) +f
2
(x
0
) x
de unde deducem ca f este derivabila n x
0
,= 0 si:
f

(x
0
) =
3x
2
0
+f(x
0
)
3f
2
(x
0
) x
0
Capitolul 9
Calcul integral pentru funct ii de o
variabila reala
Denit ii si rezultate
1. Integrala n sensul lui Riemann
Fie f : [a, b] R, a, b R, o funct ie si d = x
0
, x
1
, . . . , x
n
, a = x
0
< x
1
< x
2
< . . . <
x
n
= b o diviziune a intervalului [a, b].
Numarul real (d) := max
1in
(x
i
x
i1
) se numeste norma diviziunii d, iar numarul
real
d
(f) denit prin

d
(f) :=
n

i=1
f(
i
)(x
i
x
i1
),
i
[x
i1
, x
i
]
se numeste suma Riemann asociata funct iei f, diviziunii d si punctelor intermediare

1
,
2
, . . . ,
n
.
Funct ia f se numeste integrabila Riemann pe [a, b] daca exista un numar real I astfel
ca pentru orice > 0 exista un numar () > 0 cu proprietatea ca pentru orice diviziune
d a lui [a, b] cu (d) < () si orice alegere a punctelor intermediare
i
, 1 i n, avem
[
d
(f) I[ < .
Numarul I se noteaza
_
b
a
f(x)dx si se numeste integrala lui f pe [a, b].
Numim lungimea unui interval deschis si marginit I = (a, b) numarul real
m(I) := b a.
O mult ime A R se numeste de masura Lebesgue nula sau neglijabila daca pentru
orice > 0 exista un sir de intervale (I
n
)
n1
cu proprietatea ca
A

_
n=1
I
n
si

n=1
m(I
n
) < .
O mult ime cel mult numarabil a de numere reale este neglijabila.
Teorema. (Lebesgue) O funct ie f : [a, b] R este integrabila Riemann pe [a, b] daca
si numai daca este marginita si mult imea punctelor sale de discontinuitate este neglijabila.
62
Calcul integral pentru funct ii de o variabila reala 63
Din teorema lui Lebesgue rezulta imediat ca funct iile f : [a, b] R care au o mult ime
de discontinuitat i cel mult numarabila si sunt marginite sunt integrabile. De asemenea
funct iile monotone f : [a, b] R sunt integrabile. Are loc de asemenea urmatorul rezultat.
Daca f : [a, b] [c, d] este funct ie integrabila Riemann si g : [c, d] R este o funct ie
continua, atunci g f : [a, b] R este o funct ie integrabila Riemann.
Un rezultat util n rezolvarea problemelor de calcul integral este urmatorul.
Teorema. (Teorema de medie) Daca f : [a, b] R este o funct ie continua si
g : [a, b] [0, ) este o funct ie integrabila, atunci exista c [a, b] astfel ca
_
b
a
f(x)g(x)dx = f(c)
_
b
a
g(x)dx.
2. Integrale improprii
Fie I R un interval. O funct ie f : I R se numeste local integrabila (l.i.) pe I
daca este integrabila pe orice interval compact inclus n I.
Fie f : [a, b) R, a R, b R o funct ie l.i. pe [a, b). Denim F : [a, b) R prin
F(x) =
_
x
a
f(t)dt, x [a, b).
Funct ia f se numeste integrabila impropriu pe [a, b) daca lim
xb
x<b
F(x) exista si este
nita.

In cest caz se spune ca
_
b
a
f(x)dx este convergenta. Perechea de funct ii (f, F)
se numeste integrala improprie a lui f pe [a, b) si se noteaza
_
b
a
f(x)dx. Notam de
asemenea lim
xb
x<b
F(x) =:
_
b
a
f(x)dx daca limita din membrul stang exista.
Analog se deneste integrala improprie pentru funct ii denite pe intervale de forma
(a, b], a R, b R.
Ment ionam cateva criterii de convergent a pentru integrale improprii.
C1. Fie f, g : [a, b) R local integrabile pe [a, b), a R, b R astfel ca g(x) > 0 pe
[a, b) si lim
xb
f(x)
g(x)
= l, l R. Atunci:
i) Daca l ,= 0
_
b
a
f(x)dx si
_
b
a
g(x)dx au aceeasi natura.
ii) Daca l = 0 si
_
b
a
g(x)dx este convergenta
_
b
a
f(x)dx este absolut convergenta.
C2. Fie f : [a, ) R local integrabila pe [a, ) astfel ca
lim
x
x

f(x) = l, l R.
i) Daca > 1
_

a
f(x)dx este absolut convergenta.
ii) Daca 1 si l ,= 0
_

a
f(x)dx este divergenta.
C3. Fie f : [a, b) R, a, b R, local integrabila pe [a, b) astfel ca
lim
xb
(b x)

f(x) = l, l R.
64
i) Daca < 1
_
b
a
f(x)dx este absolut convergenta.
ii) Daca 1 si l ,= 0
_
b
a
f(x)dx este divergenta.
C4. Fie f : [0, +) R
+
o funct ie descrescatoare. Atunci
_

0
f(x)dx si

n=0
f(x) au
aceeasi natura.
C5. (Abel-Dirichlet) Fie f, g : [a, b) R, a R, b R, f continua si g monotona
pe [a, b). Daca:
i) lim
xb
g(x) = 0
ii) Funct ia F(x) =
_
x
a
f(t)dt, x [a, b) este marginita, atunci
_
b
a
f(x)g(x)dx
este convergenta.
Fie f : (a, b) R, a, b R local integrabila pe (a, b). Denim
F(x, y) =
_
y
x
f(t)dt
pentru orice x, y (a, b). Funct ia f se numeste integrabila impropriu pe (a, b) daca
lim
xa
yb
F(x, y) exista si este nita. Se arata ca
_
b
a
f(x)dx este convergenta daca si numai
daca exista c R, a < c < b, astfel ca integralele
_
c
a
f(x)dx si
_
b
c
f(x)dx sa e conver-
gente.

In acest caz avem
_
b
a
f(x)dx =
_
c
a
f(x)dx +
_
b
c
f(x)dx.
3. Integrale cu parametru
Fie I un interval cu capetele a, b R si f : I [c, d] R, c, d R, o funct ie cu
proprietatea ca este integrabila pe I pentru orice y [c, d].
Funct ia F : [c, d] R
F(y) =
_
b
a
f(x, y)dx
se numeste integrala cu parametru. Daca I = [a, b], a, b R, avem o integrala proprie
cu parametru, n caz contrar ea numindu-se integrala improprie cu parametru.
Integralele proprii cu parametru au proprietat ile urmatoare.
1) Daca f este continua pe [a, b] [c, d] F continua pe [c, d].
2) Daca f este continua pe [a, b] [c, d] si
f
y
exista si este continua pe [a, b] [c, d],
atunci F este derivabila pe [c, d] si are loc relat ia
F

(y) =
_
b
a
f
y
(x, y)dx.
Calcul integral pentru funct ii de o variabila reala 65
3) Daca f este continua pe [a, b] [c, d], atunci
_
b
a
__
d
c
f(x, y)dy
_
dx =
_
d
c
__
b
a
f(x, y)dx
_
dy.
Fie , : [c, d] [a, b] doua funct ii de clasa C
1
, f : [a, b] [c, d] R o funct ie continua
cu proprietatea ca
f
y
este continua pe [a, b][c, d]. Atunci integrala cu parametru cu limite
variabile I : [c, d] R, denita prin
I(y) =
_
(y)
(y)
f(x, y)dx
este derivabila pe [c, d] si are loc formula lui Leibniz:
I

(y) =
_
(y)
(y)
f
y
(x, y)dx +f((y), y)

(y) f((y), y)

(y), y [c, d].


4. Funct iile Beta si Gamma ale lui Euler
Funct iile B : (0, ) (0, ) R, : (0, ) R se denesc prin
B(a, b) =
_
1
0
x
a1
(1 x)
b1
dx, a, b (0, )
(a) =
_

0
x
a1
e
x
dx, a (0, )
Au loc relat iile:
1) B(a, b) = B(a, b), a, b > 0;
2) B(a, b) =
b 1
a +b 1
B(a, b 1), a > 0, b > 1;
B(a, b) =
_

0
x
a1
(1 +x)
a+b
dx, a, b > 0;
3) (a + 1) = a(a), a (0, );
4) (n + 1) = n!, n N;
5) B(a, b) =
(a)(b)
(a +b)
, a, b > 0;
6)
_
1
2
_
=

;
7) (a)(1 a) =

sin(a)
, a (0, 1).
8) (a) = lim
n
n
a

n!
a(a + 1) . . . (a +n)
, a > 0.
Probleme
Problema 9.1 Fie I
n
=
_
2
0
sin
n
xdx, n N.
a) Sa se arate ca I
n
=
n 1
n
I
n2
, n 2, si sa se calculeze I
n
.
b) Sa se demonstreze formula lui Wallis
= lim
n
1
n
_
2 4 . . . 2n
1 3 . . . (2n 1)
_
2
.
66
Solut ie. a) I
n
=
_
2
0
sin
n1
x(cos x)

dx
= sin
n1
xcos x

2
0
+ (n 1)
_
2
0
sin
n2
xcos
2
xdx
= (n 1)
_
2
0
sin
n2
x(1 sin
2
x)dx = (n 1)I
n2
(n 1)I
n
de unde rezulta relat ia din enunt . Din relat ia de recurent a se obt ine
I
n
=
_

_
1 3 . . . (2k 1)
2 4 . . . 2k


2
, n = 2k
2 4 . . . (2k 2)
1 3 . . . (2k 1)
, n = 2k 1.
b) Are loc relat ia evidenta
sin
2n+1
x sin
2n
x sin
2n1
x, x
_
0,

2
_
, n N

,
care integrata pe
_
0,

2
_
conduce la inegalitatea
I
2n+1
I
2n
I
2n1
de unde mpart ind cu I
2n+1
obt inem
1
_
1 3 . . . (2n 1)
2 4 . . . 2n
_
2

2n + 1
2

2n + 1
2n
sau

_
2 4 . . . 2n
1 3 . . . (2n 1)
_
2

1
n

2n + 1
2n
.
Trecand la limita n relat ia de mai sus obt inem formula lui Wallis.
Problema 9.2 (Formula lui Stirling) Sa se arate ca pentru orice n N

, exista
n

(0, 1) astfel ca
n! =

2n
_
n
e
_
n
e
n
12n
.
Solut ie. Consideram sirul (a
n
)
n1
, a
n
=
n!e
n
n
n+
1
2
, n 1. Demonstram ca sirul (a
n
)
n1
este descrescator. Avem
a
n
a
n+1
=
1
e
_
1 +
1
n
_
n+
1
2
, n 1.
Din dezvoltarile n serie de puteri
ln(1 +x) = x
x
2
2
+
x
3
3
. . .
ln(1 x) = x
x
2
2

x
3
3
. . .
, [x[ < 1
Calcul integral pentru funct ii de o variabila reala 67
obt inem
ln
1 +x
1 x
= 2x
_
1 +
1
3
x
2
+
1
5
x
4
+. . .
_
, [x[ < 1.
Punand x =
1
2n + 1
, n N

, n relat ia de mai sus avem


_
n +
1
2
_
ln
_
1 +
1
n
_
= 1 +
1
3

1
(2n + 1)
2
+
1
5

1
(2n + 1)
4
+. . .
de unde deducem
1 <
_
n +
1
2
_
ln
_
1 +
1
n
_
< 1 +
1
3
_
1
(2n + 1)
2
+
1
(2n + 1)
4
+. . .
_

1 <
_
n +
1
2
_
ln
_
n +
1
n
_
< 1 +
1
12n(n + 1)

e <
_
1 +
1
n
_
n+
1
2
< e
1+
1
12n(n+1)

1 <
a
n
a
n+1
< e
1
12n(n+1)
(1)
Rezulta ca (a
n
)
n1
este descrescator si ind marginit inferior este convergent. Fie
lim
n
a
n
= a. Din relat ia (1) obt inem
a
n
e

1
12n
< a
n+1
e

1
12(n+1)
, n 1,
deci sirul (a
n
e

1
12n
)
n1
este crescator si are limita a.
Prin urmare are loc relat ia
a < a
n
< a
1
12n
, n 1
deci exista
n
(0, 1) astfel ca
a
n
= ae
n
12n
, n 1. (2)
Demonstram n continuare ca a =

2.
Din formula lui Wallis rezulta ca sirul (b
n
)
n1
b
n
=
1

n

2 4 . . . 2n
1 3 . . . (2n 1)
=
1

n

2
2n
(n!)
2
(2n)!
(3)
este convergent si are limita

.
Din relat ia (2) obt inem
n! = a
_
n
e
_
n
n e
n
12n
,
n
(0, 1) (4)
(2n)! = a
_
2n
e
_
2n

2n e

2n
24n
,
2n
(0, 1)
care nlocuite n (3) conduc la
b
n
=
a

2
e
n
6n

2n
24n
.
Rezulta ca a = lim
n

2b
n
=

2.

Inlocuind n (4) obt inem
n! =

2n
_
n
e
_
n

n
12n
.
68
Problema 9.3 (Formula lui Taylor) Fie f : [a, b] R o funct ie de (n+1) ori derivabila
pe [a, b] cu f
(n+1)
integrabila pe [a, b]. Atunci pentru orice x [a, b] avem:
f(x) = f(a) +
f

(a)
1!
(x a) +
f

(a)
2!
(x a)
2
+ +
f
(n)
(a)
n!
(x a)
n
+R
n
(x)
unde
R
n
(x) =
_
x
a
f
(n+1)
(t)
(x t)
n
n!
dt.
Solut ie. f(x) f(a) =
_
x
a
f

(t)dt =
_
x
a
(t x)

(t)dt
= (t x)f

(t)

x
a

_
x
a
f

(t)(t x)dt
= f

(a)(x a)
_
x
a
f

(t)
_
(t x)
2
2
_

dt
= f

(a)(x a) f

(t)
(t x)
2
2

x
a
+
_
x
a
f

(t)
(t x)
2
2
dt
= f

(a)(x a) +
f

(a)
2!
(x a)
2
+
_
x
a
f

(t)
(t x)
2
2
dt
Continuand sa integram prin part i obt inem
f(x) = f(a) +
f

(a)
1!
(x a) + +
f
(n)
(a)
n!
(x a)
n
+
_
x
a
f
(n+1)
(t)
(x t)
n
n!
dt.
Problema 9.4 (A doua teorema de medie) Fie f : [a, b] R o funct ie de clasa C
1
,
pozitiva, descrescatoare, si g : [a, b] R o funct ie continua. Atunci exista c [a, b] astfel
ca
_
b
a
f(x)g(x)dx = f(a)
_
c
a
g(x)dx.
Solut ie. Fie G(x) =
_
x
a
g(t)dt, x [a, b]. Avem conform formulei de integrare prin
part i
_
b
a
f(x)g(x)dx = f(b)G(b)
_
b
a
f

(x)G(x)dx
Funct ia G ind continua pe [a, b] este marginita si si atinge marginile. Fie m =
min
axb
G(x), M = max
axb
G(x). Cum f este descrescatoare rezulta ca f

este negativa pe
[a, b] si avem
_
b
a
f(x)g(x)dx f(b)G(b) +M
_
b
a
(f

(x))dx
= f(b)G(b) +M(f(a) f(b))
Mf(a)
Analog se demonstreaza ca
_
b
a
f(x)g(x)dx mf(a)
Calcul integral pentru funct ii de o variabila reala 69

Intrucat G este continua pe [a, b], are proprietatea lui Darboux, prin urmare exista
c [a, b] astfel ca
_
b
a
f(x)g(x)dx = f(a)
_
c
a
g(x)dx.
Problema 9.5 (Formula lui Bonnet-Weierstrass) Fie f : [a, b] R o funct ie de clasa
C
1
, descrescatoare, si g : [a, b] R o funct ie continua. Atunci exista c [a, b] astfel ca
_
b
a
f(x)g(x)dx = f(a)
_
c
a
g(x)dx +f(b)
_
b
c
g(x)dx.
Solut ie. Fie h(x) = f(x) f(b), x [a, b]. Funct ia h este monoton descrescatoare si
h(x) 0, x [a, b]. Atunci, pe baza problemei 9.4 rezulta ca exista c [a, b] astfel ca
_
b
a
g(x)h(x)dx = h(a)
_
c
a
g(x)dx
sau
_
b
a
g(x)(f(x) f(b))dx = (f(a) f(b))
_
c
a
g(x)dx.
Prin urmare
_
b
a
f(x)g(x)dx = f(b)
_
b
a
g(x)dx + (f(a) f(b))
_
c
a
g(x)dx
= f(a)
_
c
a
g(x)dx +f(b)
_
b
c
g(x)dx.
Problema 9.6 (Lema lui Gronwall) Fie f, g : [a, b] R funct ii continue astfel ca
g(x) 0 pentru orice x [a, b]. Daca y : [a, b] R este o funct ie continua cu proprietatea
y(x) f(x) +
_
x
a
g(t)y(t)dt, x [a, b]
atunci
y(x) f(x) +
_
x
a
f(t)g(t)e

x
a
g(s)ds
dt.
Solut ie. Fie u(x) =
_
x
a
g(t)y(t)dt, x [a, b].

Inmult ind relat ia din ipoteza cu g(x) se
obt ine
u

(x) g(x)u(x) f(x)g(x), x [a, b]

Inmult ind ultima relat ie cu


x
a
g(t)dt
obt inem
_
u(x)e

x
a
g(t)dt
_

f(x)g(x)e

x
a
g(t)dt
care integrata pe intervalul [a, x] devine
u(x)e

x
a
g(t)dt

_
x
a
f(t)g(t)e

t
a
g(s)ds
dt
de unde obt inem
u(x)
_
x
a
f(t)g(t)e

t
a
g(s)ds
dt, x [a, b]
Rezulta
y(x) f(x) +u(x) f(x) +
_
x
a
f(t)g(t)e

t
x
g(s)ds
dt.
70
Problema 9.7 (Integrala Poisson) Utilizand relat ia
X
2n
1 = (X
2
1)
n1

k=1
_
X
2
2X cos
k
n
+ 1
_
sa se demonstreze ca valoarea integralei
I(a) =
_

0
ln(a
2
2a cos x + 1)dx, a R 1, 1
este 0 daca [a[ < 1 si 2 ln [a[ daca [a[ > 1.
Solut ie. Fie f(x) = ln(x
2
2a cos x + 1), x [0, ] si e
I
n
(a) =

n
n1

k=0
f
_
k
n
_
.
Avem evident lim
n
I
n
(a) = I(a).
Pe de alta parte avem
I
n
(a) =

n
n1

k=0
ln
_
a
2
2a cos
k
n
+ 1
_
=

n
ln(a 1)
2
+

n
ln
a
2n
1
a
2
1
Daca [a[ < 1 avem lima
2n
= 0, deci I(a) = 0. Daca [a[ > 1 avem
I
n
(a) =

n
ln(a 1)
2
+

n
ln
1 a
2n
1 a
2
+
2n 2
n
ln [a[
de unde obt inem lim
n
I
n
(a) = 2 ln [a[.
Problema 9.8 (Formula lui Gauss) Fie a > b > 0 si
G(a, b) =
_
/2
0
dx
_
a
2
cos
2
x +b
2
sin
2
x
Denim sirurile (a
n
)
n0
, (b
n
)
n0
prin relat iile de recurent a
a
n
=
a
n1
+b
n1
2
, b
n
=
_
a
n1
b
n1
, a
0
= a, b
0
= b.
a) Sa se demonstreze ca sirurile (a
n
)
n0
, (b
n
)
n0
sunt convergente si lim
n
a
n
=
lim
n
b
n
=: (a, b).
b) G(a, b) =
_
/2
0
dx
_
a
2
n
cos
2
x +b
2
n
sin
2
x
, n 0.
c) G(a, b) =

2(a, b)
.
Calcul integral pentru funct ii de o variabila reala 71
Solut ie. a) Avem a
1
=
a
0
+b
0
2
, b
1
=

a
0
b
0
de unde obt inem
b
0
< b
1
< a
1
< a
0
Prin induct ie se demonstreaza ca
b
0
< b
1
< < b
n
< a
n
< a
n1
< < a
0
de unde rezulta ca (a
n
)
n0
, (b
n
)
n0
sunt monotone si marginite.
Fie lim
n
a
n
= l
1
, lim
n
b
n
= l
2
, l
1
, l
2
R. Din relat ia a
n
=
a
n1
+b
n1
2
, prin trecere la
limita, obt inem l
1
= l
2
=: (a, b).
b) Facem schimbarea de variabila
sin x =
2a sin t
a +b + (a b) sin
2
t
, t
_
0,

2
_
(1)
obt inem
cos xdx = 2a
a +b (a b) sin
2
t
[a +b + (a b) sin
2
t]
2
cos tdt.
Din relat ia (1) se obt ine
cos x =
_
(a +b)
2
(a b)
2
sin
2
t
a +b + (a b) sin
2
t
cos t
de unde rezulta
dx = 2a
(a +b) (a b) sin
2
t
a +b + (a b) sin
2
t

dt
_
(a +b)
2
(a b)
2
sin
2
t
Avem de asemenea
_
a
2
cos
2
x +b
2
sin
2
x = a
a +b (a b) sin
2
t
a +b + (a b) sin
2
t
si n continuare
dx
_
a
2
cos
2
x +b
2
sin
2
x
==
dt

_
a +b
2
_
2
cos
2
t +ab sin
2
t
T inand seama ca a
1
=
a +b
2
si b
1
=

ab rezulta
G(a, b) =
_
/2
0
dt
_
a
2
1
cos
2
t +b
2
1
sin
2
t
Aplicand n mod repetat rat ionamentul anterior, obt inem:
G(a, b) =
_
/2
0
dt
_
a
2
n
cos
2
t +b
2
n
sin
2
t
, n 0.
c) Au loc relat iile

2a
n
G(a, b)

2b
n
, n 1
de unde facand n obt inem
G(a, b) =

2(a, b)
.
72
Problema 9.9 Fie f C
1
[a, b]. Fie sirul (u
n
)
n1
denit prin
u
n
=
_
b
a
f(x)dx
b a
n
n

k=1
f
_
a +k
b a
n
_
Demonstrat i ca lim
n
nu
n
=
a b
2
(f(b) f(a)).
Solut ie. Fie x
n
= a +k
b a
n
, 0 k n. Avem
u
n
=
k1

k=0
_
x
k+1
x
k
f(x)dx
n

k=1
(x
k+1
x
k
)f(x
k
)
Fie F o primitiva a funct iei f. Atunci
u
n
=
n1

k=0
(F(x
k+1
) F(x
k
))
n

k=1
(x
k+1
x
k
)f(x
k
)
= F(x
1
) F(x
0
) +
n1

k=1
(F(x
k+1
) F(x
k
) (x
k+1
x
k
)F

(x
k
))
b a
n
f(b).
Aplicand formula lui Taylor funct iei F pe intervalul [x
k
, x
k+1
] rezulta ca exista
k

(x
k
, x
k+1
) astfel ca
u
n
= F(x
1
) F(x
0
) +
n

k=1
(x
k+1
x
k
)
2
2
f

(
k
)
b a
n
f(b)
T inand seama ca n =
b a
x
k+1
x
k
avem
nu
n
= (b a)
F(x
1
) F(x
0
)
x
1
x
0
+
b a
2
n1

k=1
(x
k+1
x
k
)f

(
k
) (b a)f(b).
De asemenea
lim
n
F(x
1
) F(x
0
)
x
1
x
0
= F

(x
0
) = f(x
0
) = f(a)
si
lim
n
n1

k=1
(x
k+1
x
k
)f

(
k
) =
_
b
a
f

(t)dt = f(b) f(a).


Prin urmare
lim
n
nu
n
= (b a)f(a) +
b a
2
(f(b) f(a)) (b a)f(b)
=
a b
2
(f(b) f(a)).
Problema 9.10 Sa se determine a, b R astfel ca pentru orice n N

sa aiba loc
egalitatea
_

0
(ax +bx
2
) cos nxdx =
1
n
2
.
Sa se deduca de aici ca

n=1
1
n
2
=

2
6
.
Calcul integral pentru funct ii de o variabila reala 73
Solut ie. Integrand prin part i se gaseste a = 1, b =
1
2
. Rezulta ca
s
n
=
n

k=1
1
k
2
=
_

0
_
1
2
x
2
x
_
n

k=1
cos kxdx.
Avem
n

k=1
cos kx =
sin
(2n + 1)x
2
sin
x
2
2 sin
x
2
=
sin
_
nx +
x
2
_
sin
x
2
2 sin
x
2
=
1
2
_
sin nxctg
x
2
+ cos nx 1
_
Fie funct iile f, g : [0, ] R,
f(x) =
1
2
x
2
x
g(x) =
_
f(x)ctg
x
2
, x (0, ]
2, x = 0
Se arata usor ca g este de clasa C
1
[0, ]. Se obt ine
s
n
=
1
2
__

0
g(x) sin nxdx +
_

0
f(x) cos nxdx
_

0
_
1
2
x
2
x
_
dx
_
(1)
Pentru funct ia h C
1
[a, b] avem
_
b
a
h(x) sin nxdx = lim
n
_
b
a
h(x) cos nxdx = 0.

Intr-adevar, integrand prin part i obt inem


_
b
a
h(x) cos nxdx =
h(b) sin nb h(a) sin na
n

1
n
_
b
a
h

(x) sin nxdx


Cum h si h

sunt marginite pe [a, b] rezulta ca


lim
n
_
b
a
h(x) cos nxdx = 0
Trecand acum la limita n (1) obt inem lim
n
s
n
=

2
6
.
Problema 9.11 Fie f : [0, 1] R o funct ie continua cu proprietatea
_
1
0
f(x)dx =
_
1
0
xf(x)dx = 1.
Sa se arate ca
_
1
0
f
2
(x)dx 4.
74
Solut ie. Se cauta o funct ie de forma f(x) = ax +b, a, b R, care verica relat iile din
enunt . Prin identicare se obt ine f(x) = 6x 2. Avem
_
1
0
(f(x) (6x 2))
2
dx =
_
1
0
f
2
(x)dx 4 0.
Egalitatea se obt ine pentru f(x) = 6x 2.
Problema 9.12 Fie I
n
=
_
2
0
cos
n
xsin nx, J
n
=
_
2
0
cos
n
xcos nxdx, n N.
a) Sa se arate ca:
2I
n
=
1
n
+I
n1
,
I
n
=
1
2
I
n1
,
n 1.
b) Sa se deduca de aici ca
1
2
n+1
_
2
1
+
2
2
2
+ +
2
n
n
_
I
n
=

2
n+1
.
Solut ie. a) Integrand prin part i avem
I
n
=
1
n
_
2
0
cos
n
x(cos nx)

dx
=
1
n
cos
n
xcos nx

2
0

1
n
_
2
0
ncos
n1
xsin xcos nxdx
=
1
n

_
2
0
cos
n1
xsin xcos nxdx.
Adunand aceasta relat ie cu cea init iala obt inem
2I
n
=
1
n
+
_
/2
0
cos
n
xsin nxdx
_
/2
0
cos
n1
xsin xcos nxdx
=
1
n
+
_
/2
0
cos
n1
x(sin nxcos x cos nxsin x)dx
=
1
n
+
_
/2
0
cos
n1
xsin(n 1)xdx =
1
n
+I
n1
.
Analog se demonstreaza cealalt a recurent a.
b) Relat ia 2I
n
=
1
n
+I
n1
se scrie sub forma echivalenta
2
n
I
n
= 2
n1
I
n1
+
2
n1
n
, n 1.

Insumand relat iile anterioare de la 1 la n se obt ine relat ia ceruta. Din I


n
=
1
2
I
n1
rezulta ca (I
n
)
n1
este o progresie geometrica cu rat ia
1
2
, deci
I
n
=
1
2
n
I
0
=

2
n+1
.
Calcul integral pentru funct ii de o variabila reala 75
Problema 9.13 Sa se calculeze
_
1
0
arctg x
1 +x
dx.
Solut ie. Avem
I =
_
1
0
arctg x
1 +x
dx =
_
1
0
arctg x(ln(1 +x))

dx
= arctg xln(1 +x)

1
0

_
1
0
ln(1 +x)
1 +x
2
dx =

4
ln 2
_
1
0
ln(1 +x)
1 +x
2
dx.

In ultima integrala facem substitut ia x = tg t obt inand


J =
_
1
0
ln(1 +x)
1 +x
2
dx =
_
/4
0
ln(1 + tg t)dt
n care se face substitut ia

4
t = u. Se obt ine J =

8
ln 2 = I.
Problema 9.14 Fie f : R R o funct ie continua si periodica de perioada T > 0.
Sa se demonstreze relat iile:
a)
_
a+T
a
f(x)dx =
_
T
0
f(x)dx, a R;
b)
_
a+nT
a
f(x)dx = n
_
T
0
f(x)dx, a R, a N.
Aplicat ie. Sa se calculeze
_
2003
0
arcsin(sin x)dx.
Solut ie. a) Fie g : R R, g(a) =
_
a+T
a
f(x)dx. Avem g

(a) = 0 de unde rezulta


g(a) = g(0).
b)
_
a+nT
a
f(x)dx =
_
a+T
a
f(x)dx +
_
a+2T
a+T
f(x)dx +. . .
+
_
a+nT
a+(n1)T
f(x)dx = n
_
a+T
a
f(x)dx = n
_
T
0
f(x)dx.
Aplicat ie.
_
2003
0
arcsin(sin x)dx
=
_

0
arcsin(sin x)dx +
_
2003

arcsin(sin x)dx
=
_

0
arcsin(sin x)dx +
_
+10012

arcsin(sin x)dx
=
_

0
arcsin(sin x)dx + 1001
_
2
0
arcsin(sin x)dx
=
_

0
arcsin(sin x)dx + 1001
_

arcsin(sin x)dx
=
_

0
arcsin(sin x)dx =
_
/2
0
xdx +
_

/2
( x)dx =

2
4
.
76
Problema 9.15 Fie f : R R o funct ie continua si periodica de perioada T > 0.
Sa se arate ca
a) lim
x
1
x
_
x
0
f(t)dt =
1
T
_
T
0
f(t)dt.
b) lim
t
_
b
a
f(xt)dx =
b a
T
_
T
0
f(t)dt.
Solut ie. a) Fie x = k
n
T +a
n
, k
n
N, a
n
[0, T), x > 0. Avem
1
x
_
x
0
f(t)dt =
1
k
n
T +a
n
_
knT+an
0
f(t)dt
=
1
k
n
T +a
n
__
an
0
f(t)dt +
_
knT+an
an
f(t)dt
_
=
1
k
n
T +a
n
__
an
0
f(t)dt +k
n
_
T
0
f(t)dt
_
=
1
k
n
T +a
n
_
an
0
f(t)dt +
k
n
k
n
T +a
n
_
T
0
f(t)dt.
Facand n se obt ine relat ia ceruta.
b)

In
_
b
a
f(xt)dx se face substitut ia xt = u.
Problema 9.16 Fie f C
3
[1, 1] cu proprietatea f(1) = f(0) = f(1) = 0.
Sa se arate ca
_
1
1
[f(x)[dx
1
12
max
1x1
[f

(x)[.
Lema. Fie f C
2
[a, b] astfel ncat f(a) = f(b) = 0. Demonstram c a pentru orice
x [a, b], exista c
x
(a, b) astfel ncat
f(x) =
(x a)(x b)
2
f

(c
x
).
Generalizare. Fie f C
p
[a, b], p 2 astfel ncat exista
1
= a <
2
< . . . <
p
= b
cu f(
j
) = 0, j = 1, p. Atunci pentru orice x [a, b], exista c
x
(a, b) astfel ncat
f(x) =
p

j=1
(x
j
)
f
(p)
(c
x
)
p!
.
Demonstrat ie. e x (a, b) xat si A astfel ncat : [a, b] R,
(t) = f(t)
(t a)(t b)
2
A
sa se anuleze n x.
Avem (a) = (x) = (b) = 0. Din teorema lui Rolle rezulta ca exista x
1
, x
2
,
a < x
1
< x < x
2
< b astfel ncat

(x
1
) =

(x
2
) = 0 c
x
(x
1
, x
2
) astfel ncat

(c
x
) = 0 A = f

(c
x
). Daca x = a sau x = b atunci c
x
este arbitrar.
Calcul integral pentru funct ii de o variabila reala 77
Solut ie. Folosind generalizarea lemei anterioare se arata ca pentru orice x [a, b]
exista c
x
ntre a si b astfel ca
f(x) =
1
6
x(x 1)(x + 1)f

(c
x
).
_
1
1
[f(x)[dx
1
6
_
1
1
[x(1 x
2
)[dx sup
|x|1
[f

(x)[
=
1
3
_
1
0
x(1 x
3
)dx sup
|x|1
[f

(x)[ =
1
12
sup
|x|1
[f

(x)[.
Problema 9.17 Fie f C
1
[0, 1] astfel ncat
_
1
0
f(x)dx =
_
1
0
xf(x)dx = 1.
Sa se arate ca
_
1
0
(f

(x))
2
dx 30.
Solut ie. Integrand prin part i obt inem:
1 =
_
1
0
f(x)dx = xf(x)

1
0

_
1
0
xf

(x)dx = f(1)
_
1
0
xf

(x)dx
1=
_
1
0
_
x
2
2
_

f(x)dx =
x
2
2
f(x)

1
0

1
2
_
1
0
x
2
f

(x)dx=
1
2
f(1)
1
2
_
1
0
x
2
f

(x)dx.
Eliminand f(1) din relat iile anterioare rezulta
1 =
_
1
0
(x x
2
)f

(x)dx.
Inegalitatea Cauchy-Schwartz conduce la
1 =
__
1
0
(x x
2
)f

(x)dx
_
2

_
1
0
(x x
2
)
2
dx
_
1
0
(f

(x))
2
dx.
Cum
_
1
0
(x x
2
)
2
dx =
1
30
rezulta
_
1
0
(f

(x))
2
dx 30.
Egalitatea are loc pentru f

(x) = (x x
2
), de unde obt inem
f(x) =
_
x
2
2

x
3
3
_
+, , R.

Inlocuind n relat iile din enunt obt inem = 30, =


3
2
.
Problema 9.18 Fie f : [0, 1] R o funct ie avand derivata de ordinul doi integrabila. Sa
se demonstreze ca
lim
n
n
2
_
_
1
0
f(x)dx
1
n
n

k=1
f
_
2k 1
2n
_
_
=
f

(1) f

(0)
24
.
78
Solut ie. Notam
r
n
=
_
1
0
f(x)dx
1
n
n

k=1
f
_
2k 1
2n
_
.
Avem
r
n
=
n

k=1
_
k/n
(k1)/n
_
f(x) f
_
2k 1
2n
__
dx.
Fie
m
k
= inf
x[(k1)/n,k/n]
f

(x), M
k
= sup
x[(k1)/n,k/n]
f

(x),
k 0, 1, . . . , n 1. Conform formulei lui Taylor, avem:
f(x) f
_
2k 1
2n
_
=
_
x
2k 1
2n
_
f

_
2k 1
2n
_
+
1
2
_
x
2k 1
2n
_
2
f

(
k
),
x,
k
[(k 1)/n, k/n]. Folosind
_
k/n
(k1)/n
_
x
2k 1
2n
_
dx = 0,
_
k/n
(k1)/n
_
x
2k 1
2n
_
2
dx =
1
12n
3
obt inem
1
24n
1
n
n

k=1
m
k
nr
n

1
24n
1
n
n

k=1
M
k
,
de unde
lim
n
n
2
r
n
=
1
24
_
1
0
f

(x)dx =
f

(1) f

(0)
24
.
Problema 9.19 Fie g : R R de perioada T > 0 si f : [0, T] R funct ii integrabile. Sa
se demonstreze ca
lim
n
_
T
0
f(x)g(nx)dx =
1
T
_
T
0
f(x)dx
_
T
0
g(x)dx.
Solut ie. Pentru nceput presupunem g 0. Notam:
m
k
= inf
x[k
T
n
,(k+1)
T
n
]
f(x), M
k
= sup
x[k
T
n
,(k+1)
T
n
]
f(x),
k 0, 1, . . . , n 1. Avem:
_
T
0
f(x)g(nx)dx =
1
n
_
nT
0
f
_
t
n
_
g(t)dt
=
1
n
n1

k=0
_
(k+1)T
kT
f
_
t
n
_
g(t)dt =
1
n
n1

k=0
f
k
_
(k+1)T
kT
g(t)dt
=
1
n
n1

k=0
f
k
_
T
0
g(t)dt =
1
T
_
T
0
g(t)dt
T
n
n1

k=0
f
k
Calcul integral pentru funct ii de o variabila reala 79

1
T
_
T
0
g(t)dt
_
T
0
f(t)dt,
deoarece f
k
[m
k
, M
k
], k 0, 1, . . . , n 1.

In cazul n care funct ia g nu este pozitiva, ind integrabila, exista o constanta M > 0
astfel ncat g +M > 0.
Conform celor deja demonstrate, avem:
lim
n
_
T
0
f(t)(g(nt) +M)dx =
1
T
_
T
0
(g(t) +M)dt
_
T
0
f(t)dt,
de unde
lim
n
_
T
0
f(t)g(nt)dx =
1
T
_
T
0
g(t)dt
_
T
0
f(t)dt.
Problema 9.20 Fie f : [0, 1] R o funct ie cu derivata continua si pozitiva. Fie c
n

_
0,
1
n
_
, n N

, astfel ca
_
1/n
0
f(x)dx =
1
n
f(c
n
).
Sa se arate ca
lim
n
nc
n
=
1
2
.
Solut ie.
lim
n
nc
n
= lim
n
n
c
n
0
f(c
n
) f(0)
(f(c
n
) f(0))
=
1
f

(0)
lim
n
n
2
_
1/n
0
(f(x) f(0))dx =
1
f

(0)
lim
t0
1
t
2
_
t
0
(f(x) f(0))dx
=
1
f

(0)
lim
t0
f(t) f(0)
2t
=
f

(0)
2f

(0)
=
1
2
.
Problema 9.21 Daca f : [0, 1] [0, 1] este o funct ie continua atunci
__
1
0
f(x)dx
_
2
2
_
1
0
xf(x)dx.
Solut ie. Consideram funct ia
F(t) =
__
t
0
f(x)dx
_
2
2
_
t
0
xf(x)dx.
Avem
F

(t) = 2f(t)
_
t
0
f(x)dx 2tf(t) = 2f(t)
_
t
0
(f(x) 1)
. .
0
dx 0.
Rezulta ca F este descrescatoare, deci
F(1) F(0) = 0,
adica
__
1
0
f(x)dx
_
2
2
_
1
0
xf(x)dx.
80
Problema 9.22 Fie f : [a, b] R continua pe [a, b] si derivabila pe (a, b), astfel ca
f(a) = f(b) = 0. Sa se arate ca exista c (a, b) astfel ca
[f

(c)[
4
(b a)
2
_
b
a
f(x)dx.
Solut ie. Admitem ca
[f

(x)[ <
4
(b a)
2
_
b
a
f(x)dx, x (a, b).
Avem

f(x) f(a)
x a

= [f

()[ <
4
(b a)
2
_
b
a
f(x)dx, x (a, b),
de unde
[f(x)[ <
4(x a)
(b a)
2
_
b
a
f(x)dx, x (a, b).
Analog
[f(x)[ <
4(b x)
(b a)
2
_
b
a
f(x)dx, x (a, b).
Avem:
_
b
a
[f(x)[dx =
_
(a+b)/2
a
[f(x)[dx +
_
b
(a+b)/2
[f(x)[dx
<
4
(b a)
2
_
_
(a+b)/2
a
(x a)dx +
_
b
(a+b)/2
(b x)dx
_
_
b
a
f(x)dx,
deci
_
b
a
[f(x)[dx <
_
b
a
f(x)dx.
Rezulta ca exista c (a, b) astfel ca
[f

(c)[
4
(b a)
2
_
b
a
f(x)dx.
Problema 9.23 Fie f : (1, 2) R continua. Sa se arate ca
lim
x1+
_
x
2
x
f(t)dt = lim
x1+
(x 1)f(x) ln 2.
Solut ie. Aplicand teorema de medie pe intervalul [x, x
2
] putem scrie:
lim
x1+
_
x
2
x
f(t)dt = lim
x1+
_
x
2
x
f(t)(t 1)
1
t 1
dt
= lim
x1+
(c 1)f(c)
_
x
2
x
1
t 1
dt
(vezi x < c < x
2
),
= lim
x1+
(c 1)f(c) ln 2 = ln 2 lim
x1+
(x 1)f(x).
Calcul integral pentru funct ii de o variabila reala 81
Problema 9.24 Sa se demonstreze relat ia
_
1
0
x
x
dx =

n=1
n
n
.
Solut ie. Pentru x (0, 1] are loc relat ia
x
x
= e
xlnx
= 1 +

n=1
(1)
n
x
n
ln
n
x
n!
.
Seria converge uniform pe (0, 1] deoarece sup[xln x[ : x (0, 1] =
1
e
si seria

n=1
1
n!
_
1
e
_
n
este convergenta (criteriul de convergent a uniforma a lui Weierstrass). Avem:
_
1
0
x
x
dx = 1 +

n=1
(1)
n
n!
_
1
0
x
n
ln
n
xdx.
Fie I
n
=
_
1
0
x
n
ln
n
xdx. Substitut ia x = e
t
conduce la
I
n
= (1)
n
_

0
e
(n+1)t
t
n
dt.
Notand (n + 1)t = u obt inem
I
n
= (1)
n
_

0
e
u

u
n
(n + 1)
n

du
n + 1
=
(1)
n
(n + 1)
n+1
_

0
e
u
u
n
du
=
(1)
n
(n + 1)
n+1
(n + 1) =
(1)
n
(n + 1)
n+1
n!.
Rezulta
_
1
0
x
x
dx =

n=0
(n + 1)
(n+1)
=

n=1
n
n
.
Problema 9.25 Fie M = f C
1
[0, 1] [ f(1) = 1, f(0) = 0 si funct ia
J : M R, J(f) =
_
1
0
(1 +x
2
)(f

(x))
2
dx.
Sa se determine min
fM
J(f).
Vojtech Jarnik, 2009
Solut ie. 1 = [f(1) f(0)[ =

_
1
0
f

(x)dx

1 =
__
1
0
f

(x)dx
_
2
=
__
1
0
(
_
1 +x
2
f

(x))
1

1 +x
2
dx
_
2
CBS

_
1
0
(1 +x
2
)(f

(x))
2
dx
_
1
0
1
1 +x
2
dx =
_
1
0
(1 +x
2
)(f

(x))
2
dx

4
82

_
1
0
(1 +x
2
)(f

(x))
2
dx
4

Luand f
0
(x) =
4

arctg x avem
f
0
(0) = 0, f
0
(1) = 1 si f

0
(x) =
4


1
1 +x
2
,
J(f
0
) =
_
1
0
(1 +x
2
)
_
4

_
2

1
(1 +x
2
)
2
dx =
_
4

_
2
_
1
0
1
1 +x
2
dx =
_
4

_
2


4
=
4

,
deci
min
fM
J(f) = J(f
0
) =
4

.
Problema 9.26 Sa se determine funct iile f : [0, 1] (0, ) de clasa C
1
care verica
relat iile:
f(1) = ef(0) si
_
1
0
(f

(x))
2
dx +
_
1
0
1
(f(x))
2
dx 2.
Putnam
Solut ie. 0
_
1
0
_
f

(x)
1
f(x)
_
2
dx
=
_
1
0
(f

(x))
2
dx 2
_
1
0
f

(x)
f(x)
dx +
_
1
0
dx
(f(x))
2
2 2 ln f(x)

1
0
= 2 2 ln
f(1)
f(0)
= 0
Rezulta
f

(x)
1
f(x)
= 0, x [0, 1]
(f
2
(x))

= 2 f
2
(x) = 2x +c f(x) =

2x +c, c > 0
si revenind la f(1) = f(0) e rezulta c =
2
e
2
1
si
f(x) =
_
2x +
2
e
2
1
, x [0, 1].
Problema 9.27 Fie f : [0, 1] [0, 1] o funct ie continua. Sa se arate ca ecuat ia
2x
_
x
0
f(t)dt = 1
are o singura solut ie n [0, 1].
Putnam
Solut ie. Fie g(x) = 2x
_
x
0
f(t)dt1, funct ie care este derivabila si g

(x) = 2f(x)
2 1 = 1, deci g este strict crescatoare si atunci ecuat ia data are cel mult o solut ie. Avem
g(0) = 1 < 0 si
g(1) = 2
_
1
0
f(t)dt 1 = 1
_
1
0
f(t)dt 0
deci ecuat ia g(x) = 0 are o solut ie unica.
Calcul integral pentru funct ii de o variabila reala 83
Problema 9.28 Stiind ca
_
1
0
ln(1 +x)
x
dx =

2
12
sa se calculeze
_
1
0
ln(1 x
3
)
x
dx.
Putnam
Solut ie. Fie I
k
=
_
1
0
ln(1 x
k
)
x
dx =
1
k
_
1
0
ln(1 t)
t
dt =
1
k
I
1
.
Apoi
I =
_
1
0
ln(1 +x)
x
dx = I
2
I
1
=
1
2
I
1
I
3
=
1
3
I
1
=
2
3
I =

2
18
Problema 9.29 Fie f C[0, 1] astfel ca xf(y) +yf(x) 1, pentru orice x, y [0, 1].
Sa se arate ca
_
1
0
f(x)dx

4
.
Putnam
Solut ie. Avem
I =
_
1
0
f(x)dx =
_
2
0
f(sin t) cos tdt =
_
2
0
f(cos t) sin tdt
2I =
_
2
0
(f(sin t) cos t +f(cos t) sin t)dt
_
2
0
1 dt =

2
.
Problema 9.30 Fie M
_
f C[0, ] [
_

0
f(x) sin xdx =
_

0
f(x) cos xdx = 1
_
.
Sa se determine min
fM
_

0
(f(x))
2
dx.
Putnam
Solut ie. Cautam o funct ie f
0
(x) = a sin x +b cos x, x [0, ] care verica relat iile din
enunt . Se obt ine:
f
0
(x) =
2

(sin x + cos x) M.
Avem:
0
_

0
(f(x) f
0
(x))
2
dx
=
_

0
(f(x))
2
dx 2
_

0
f
0
(x)f(x)dx +
_

0
(f
0
(x))
2
dx
_

0
(f(x))
2
dx 2
_

0
f
0
(x)f(x)dx
_

0
(f
0
(x))
2
dx
=
8

=
4

=
_

0
(f
0
(x))
2
dx.
Minimul este
4

si se atinge pentru f = f
0
.
84
Problema 9.31 Fie f : [0, 1] R o funct ie de clasa C
1
. Sa se arate ca
f
_
1
2
_

_
1
0
[f(x)[dx +
1
2
_
1
0
[f

(x)[dx.
Putnam
Solut ie. Avem:
_
1
0
xf

(x)dx = xf(x)

1
0

_
1
0
f(x)dx = f(1)
_
1
0
f(x)dx
si
_
1
1
2
f

(x)dx = f(1) f
_
1
2
_
.
Astfel
f
_
1
2
_
=
_
1
0
xf

(x)dx
_
1
1
2
f

(x)dx +
_
1
0
f(x)dx
=
_ 1
2
0
xf

(x)dx +
_
1
1
2
(x 1)f

(x)dx +
_
1
0
f(x)dx
si

f
_
1
2
_

_ 1
2
0
[xf

(x)[dx +
_
1
1
2
[(x 1)f

(x)[dx +
_
1
0
[f(x)[dx

_
1
0
[f(x)[dx +
1
2
_ 1
2
0
[f

(x)[dx +
1
2
_
1
1
2
[f

(x)[dx
=
_
1
0
[f(x)[dx +
1
2
_
1
0
[f

(x)[dx.
Problema 9.32 Sa se arate ca nu exista funct ia derivabila f : R R astfel ca
[f(x)[ < 2x si f(x)f

(x) sin x, pentru orice x R.


Solut ie. Avem:
(f(x))
2
(f(0))
2
=
_
x
0
2f(t)f

(t)dt 2
_
x
0
sin tdt = 2(1 cos x).
Pentru x = rezulta (f())
2
4 n contradict ie cu [f()[ < 2.
Problema 9.33 Sa se arate ca lim
n
_
3
0
x
2
(1 x)x
n
1 +x
2n
dx = 0.
Solut ie.

_
3
0
x
2
(1 x)x
n
1 +x
2n
dx

_
3
0
x
2
[1 x[x
n
1 +x
2n
dx
=
_
1
0
x
2
(1 x)x
n
1 +x
2n
dx +
_
3
1
x
2
(x 1)x
n
1 +x
2n
dx
_
1
0
x
n
dx + 18
_
3
1
x
n
x
2n
dx
=
1
n + 1
+
18
n 1

18
3
n1
(n 1)
0.
Calcul integral pentru funct ii de o variabila reala 85
Problema 9.34 Fie f
1
, f
2
, f
3
, f
4
R[x]. Atunci
F(x) =
_
x
1
f
1
(t)f
3
(t)dt
_
x
1
f
2
(t)f
4
(t)dt
_
x
1
f
1
(t)f
4
(t)dt
_
x
1
f
2
(t)f
3
(t)dt
este un polinom divizibil cu (x 1)
4
.
Putnam, 1946
Solut ie. F(1) = 0,
F

(x) = f
1
(x)f
3
(x)
_
x
1
f
2
f
4
dt +f
2
(x)f
4
(x)
_
x
1
f
1
f
3
f
1
(x)f
4
(x)
_
x
1
f
2
f
3
f
2
(x)f
4
(x)
_
x
1
f
1
f
4
F

(1) = 0
F

(1) = (f
1
f
3
)

_
x
1
+f
1
f
2
f
3
f
4
+ (f
2
f
4
)

_
x
1
+f
2
f
4
f
1
f
3
. . .
= (f
1
f
3
)

_
x
1
f
2
f
4
+ (f
2
f
4
)

_
x
1
f
1
f
3
(f
1
f
4
)

_
x
1
f
2
f
3
(f
2
f
3
)

_
x
1
f
1
f
4
F

(x) = (f
1
f
2
)

_
x
1
f
2
f
4
+ (f
1
f
2
)

(f
2
f
2
) +. . . =
(
f
1
f
3
)

f
2
f
4
+ (f
2
f
4
)

(f
1
f
3
) (f
1
f
4
)

(f
2
f
3
) (f
1
f
4
)(f
2
f
3
)

[(f
1
f
2
)(f
3
f
4
)]

[(f
1
f
4
)(f
2
f
3
)]

= 0
Problema 9.35 Fie f : [0, ] R continua astfel ca:
_

0
f(x) cos xdx =
_

0
f(x) sin xdx = 0.
Sa se arate ca ecuat ia f(x) = 0 are cel put in doua radacini n intervalul (0, ).
Putnam, 1963
Solut ie. Din
_

0
f(x) sin xdx = 0 si sin x 0, x [0, ] rezulta ca funct ia f si
schimba semnul pe [0, ], deci exista x
1
[0, ] astfel ca f(x
1
) = 0. Daca n x
1
ar
singura schimbare de semn, la fel ca funct ia sin(x x
1
), atunci
_

0
f(x) sin(x x
1
)dx ,= 0
__

0
f(x) sin xdx
_
. .
=0
cos x
1

__

0
f(x) cos xdx
_
. .
=0
sin x
1
,= 0 fals
rezulta ca exista x
1
,= x
2
n care f schimba semnul, deci f(x
1
) = f(x
2
) = 0.
Problema 9.36 Fie f : [0, 1] [0, ) crescatoare. Atunci
_
1
0
xf
2
(x)dx
_
1
0
f(x)dx
_
1
0
f
2
(x)dx
_
1
0
xf(x)dx.
86
Putnam, 1957
Solut ie.
_
1
0
xf
2
(x)dx
_
1
0
f(y)dy
_
1
0
xf(x)dx
_
1
0
f
2
(y)dy 0
I =:
_
1
0
_
1
0
f(x)f(y)x(f(x) f(y))dxdy 0.
Schimband x cu y n relat ia anterioara obt inem
I =
_
1
0
_
1
0
f(x)f(y)y(f(y) f(x))dxdy,
deci
2I =
_
1
0
_
1
0
f(x)f(y)(x y)(f(x) f(y))dxdy 0.
Cum f este crescatoare rezulta ca (x y)[f(x) f(y)[ 0 pentru orice x, y [0, 1]. Prin
urmare I 0.
Problema 9.37 Fie f : [1, 1] R de doua ori derivabila astfel ncat
[f(x)[ 1, x [0, 1], [f

(x)[ 1, x [0, 1].


Sa se arate ca [f

(x)[ 2, x [0, 1].


Putnam, 1962
Solut ie. Conform formulei lui Taylor avem:
f(1) = f(x) + (1 x)f

(x) +
1
2
(1 x)
2
f

(), (x, 1)
f(1) = f(x) + (1 x)f

(x) +
1
2
(1 x)
2
f

(), (1, x)
f(1) f(1) = 2f

(x) +
1
2
(1 x)
2
f

()
1
2
(1 +x)
2
f

()
2(f

(x)) [f(1)[ +[f(1)[ +


1
2
(1 x)
2
[f

()[ +
1
2
(1 +x)
2
[f

()[
2 +
1
2
(1 x)
2
+
1
2
(1 +x)
2
= 3 +x
2
4 [f

(x)[ 2.
Problema 9.38 Sa se determine maximul expresiei
_
1
0
x
2
f(x)dx
_
1
0
x(f(x))
2
dx pentru f C[0, 1].
Putnam, 2006
Solut ie.
_
1
0
x
2
f(x)dx
_
1
0
x(f(x))
2
dx
=
_
1
0
_
x
3
4
x
_
f(x)
x
2
_
2
_
dx
_
1
0
x
3
4
dx =
1
16
cu egalitate pentru f(x) =
x
2
, x [0, 1].
Calcul integral pentru funct ii de o variabila reala 87
Problema 9.39
_
1
0
__
1
0
f(x, y)dx
_
2
dy +
_
1
0
__
1
0
f(x, y)dy
_
2
dx

__
1
0
_
1
0
f(x, y)dxdy
_
2
+
_
1
0
_
1
0
(f(x, y))
2
dxdy.
Putnam, 2004
Solut ie. Inegalitatea este echivalenta cu
_
1
0
_
1
0
_
1
0
_
1
0
(F(x, y, z, t))
2
dxdydzdt 0
unde
F(x, y, z, t) = f(x, y) +f(z, t) f(x, t) f(z, y), x, y, z, t [0, 1].
Problema 9.40 Sa se determine f : (0, ) (0, ) pentru care
f

_
1
x
_
=
x
f(x)
, x (0, ).
Putnam, 2005
Solut ie. Avem
f

(x) =
1
xf
_
1
x
_, x (0, )
f

(x) =
f
_
1
x
_

1
x
f

_
1
x
_
x
2
f
2
_
1
x
_ =
1
x
2
f
_
1
x
_ +
f

_
1
x
_
x
3
f
2
_
1
x
_
=
f

(x)
x
+
x
f(x)
x
(f

(x))
2
=
f

(x)
x
+
(f

(x))
2
f(x)

xf(x)f

(x) +f(x)f

(x) = x(f

(x))
2
[: (f(x))
2

(x)
f(x)
+
xf

(x)
f(x)

x(f

(x))
2
(f(x))
2

_
xf

(x)
f(x)
_

= 0
xf

(x)
f(x)
= c
f

(x)
f(x)
=
c
x
f(x) = dx
c
Revenind: d
2
c = 1, deci f(x) = dx
1
d
2
, d (0, ).
Problema 9.41 Fie f C
1
[0, 1] cu f(0) = 0, 0 f

(x) 1, x (0, 1). Sa se arate ca


__
1
0
f(x)dx
_
2

_
1
0
(f(x))
3
dx.
Cand are loc egalitatea?
88
Putnam, 1973
Solut ie. Fie G(t) = 2
_
t
0
f(x)dx (f(t))
2
G(0) = 0, G

(t) = 2f(t)(1 f

(t)) 0
G(t) 0 si f(t)G(t) 0.
Fie H(t) =
__
t
0
f(x)dx
_
2

_
t
0
(f(x))
3
dx, t [0, 1]. Avem:
H(0) = 0, H

(t) = f(t)G(t) 0.
Rezulta H(t) 0, deci H(1) 0, q.e.d.
Egalitatea are loc numai pentru f(t)G(t) = H

(t) = 0, t [0, 1].


Obt inem f(x) = x.
Problema 9.42 Fie f : [0, 1] R continua astfel ca
_
1
0
f(x)dx =
_
1
0
xf(x)dx = . . . =
_
1
0
x
n1
f(x)dx = 0
si
_
1
0
x
n
f(x)dx = 1.
Sa se arate ca exista a [0, 1] astfel ca [f(a)[ 2
n
(n + 1).
Putnam, 1972
Solut ie. Din condit iile date rezulta
_
1
0
_
x
1
2
_
n
f(x)dx = 1.
Daca prin absurd [f(x)[ < 2
n
(n + 1), x [0, 1] rezulta
1
_
1
0

x
1
2

n
[f(x)[dx < 2
n
(n + 1)
_
1
0

x
1
2

n
dx
= 2
n
(n + 1) 2
_
1
1
2
_
x
1
2
_
n
dx = 2
n
(n + 1) 2
_
x
1
2
_
n+1
n + 1

1
1
2
= 2
n+1

1
2
n+1
= 1, contradict ie.
Problema 9.43 Fie f : R R o funct ie aditiva (f(x + y) = f(x) + f(y), x, y R)
si integrabila pe orice interval compact din R. Sa se arate ca exista a R astfel ncat
f(x) = ax, x R.
Calcul integral pentru funct ii de o variabila reala 89
Solut ie.
_
x
0
f(t +y)dt =
_
x
0
f(t)dt +
_
x
0
f(y)dt
xf(y) =
_
x
0
f(t +y)dt
_
x
0
f(t)dt
=
_
x+y
y
f(t)dt
_
x
0
f(t)dt
=
_
x+y
0
f(t)dt
_
x
0
f(t)dt
_
y
0
f(t)dt
. .
(x,y)
(x, y) = (y, x) xf(y) = yf(x), x, y R

f(x)
x
=
f(1)
1
= f(1), x R f(x) = xf(1), x R.
Problema 9.44 Fie f : [0, 1] R continua. Sa se arate ca
_
1
0
__
1
x
f(t)dt
_
dx =
_
1
0
tf(t)dt.
Iran
Solut ie.

In prima integrala integram prin part i
_
1
0
__
1
x
f(t)dt
_
dx =
_
1
0
x

(F(1) f(x))dx
x =
_
1
x
f(t)dt

1
0

_
1
0
x(f(x))dx =
_
1
0
xf(x)dx.
Problema 9.45 Fie f : [0, ) R o funct ie continua cu lim
x
f(x) = 1.
Sa se calculeze lim
n
_
1
0
f(nx)dx.
Iran
Solut ie. Vom calcula
lim
t
_
1
0
f(tx)dx = lim
t
1
t
_
t
0
f(u)du
= lim
t
F(t) F(0)
t
= lim
t
f(t) = 1,
unde
F(t) =
_
t
0
f(u)du, t [0, 1].
Problema 9.46 Fie f : [0, 1] [0, ) astfel ncat
_
1
0
f(x)dx = 1.
Sa se arate ca
_
1
0
_
x
_
1
0
tf(t)dt
_
2
f(x)dx
1
4
.
90
Iran
Solut ie. Vom arata ca pentru orice R avem:
_
1
0
(x )
2
f(x)dx
_
1
0
(x )
2
f(x)dx
unde =
_
1
0
xf(x)dx.
Avem:
_
1
0
(x )
2
f(x)dx =
_
1
0
((x ) + ( ))
2
f(x)dx
=
_
1
0
(x )
2
f(x)dx + ( )
2
_
1
0
f(x)dx + 2( )
_
1
0
(x )f(x)dx
=
_
1
0
(x )
2
f(x)dx + ( )
2

_
1
0
(x )
2
f(x)dx.
Pentru =
1
2
obt inem:
_
1
0
(x )
2
f(x)dx
_
1
0
_
x
1
2
_
2
f(x)dx
_
1
0
1
4
f(x)dx =
1
4
.
Problema 9.47 Sa se calculeze
_

0
x
1 +e
x
dx.
Solut ie. Fie f(x) =
x
1 +e
x
, x [0, ). Cum lim
x
x
2
f(x) = 0 rezulta ca integrala
este convergenta.
I : =
_

0
x
1 +e
x
dx =
_

0
xe
x
1 +e
x
dx =
_

0
x(ln(1 +e
x
))

dx
= xln(1 +e
x
)

0
+
_

0
ln(1 +e
x
)dx
=
_

0
ln(1 +e
x
)dx.
Punand e
x
= t obt inem
I =
_
1
0
ln(1 +t)
t
dt =
_
1
0
_
1
t
2
+
t
2
3
. . .
_
dt
= 1
1
2
2
+
1
3
2
. . .
Din relat ia 1 +
1
2
2
+
1
3
2
+. . . =

2
6
obt inem
1
1
2
2
+
1
3
2
. . . =

2
12
.
Problema 9.48 Fie f : [0, ) R o funct ie continua cu proprietatea ca
_

0
f(x)dx
este convergenta. Sa se arate ca
lim
n
1
n
_
n
0
xf(x)dx = 0.
Calcul integral pentru funct ii de o variabila reala 91
Solut ie. Fie
_

0
f(x)dx = I, I R. Avem:
I = lim
x
_
x
0
f(t)dt = lim
x
x
_
x
0
f(t)dt
x
= lim
x
_
x
_
x
0
f(t)dt
_

(x)

= lim
x
__
x
0
f(t)dt +xf(x)
_
= I + lim
x
xf(x).
Rezulta ca lim
x
xf(x) = 0. Evident
lim
n
1
n
_
n
0
xf(x)dx = lim
n
nf(n) = 0.
Problema 9.49 Fie M = f [ f C
2
[0, 1], f(0) = f(1) = 0, f

(0) = 1.
Sa se determine
min
fM
_
1
0
(f

(x))
2
dx
si funct iile pentru care se atinge minimul.
Solut ie. Fie f M. Are loc relat ia
_
1
0
(1 x)f

(x)dx = (1 x)f

(x)

1
0
+
_
1
0
f

(x)dx = 1.
Din inegalitatea lui Cauchy-Schwartz obt inem:
__
1
0
(1 x)f

(x)dx
_
2

_
1
0
(1 x)
2
dx
_
1
0
(f

(x))
2
dx (1)
de unde rezulta ca
_
1
0
(f

(x))
2
dx 3.

In (1) egalitatea are loc pentru f

(x) = (1 x), R. Punand condit ia ca f M


rezulta
f(x) =
1
2
(x
3
3x
2
+ 2x), x [0, 1].
Problema 9.50 Fie f C[a, b] astfel ca
_
b
a
x
n
f(x)dx = 0
pentru orice n N. Sa se arate ca f este identic nula.
Solut ie. Din relat ia
_
b
a
x
n
f(x)dx = 0 rezulta ca pentru orice funct ie polinomiala P
are loc relat ia
_
b
a
f(x)P(x)dx = 0.
92
Din teorema lui Weierstrass rezulta ca exista un sir de funct ii polinomiale (P
n
)
n1
care este uniform convergent la f pe intervalul [a, b]. Cum f este marginita rezulta ca
P
n
f f
2
pe [a, b]. Avem:
_
b
a
f
2
(x)dx = lim
n
_
b
a
P
n
(x)f(x)dx = 0,
de unde rezulta ca f(x) = 0 pentru orice x [a, b].
Problema 9.51 Fie f C
1
[a, b], f

(a) ,= 0. Pentru orice x (a, b] e (x) [a, x] astfel


ca
_
x
a
f(t)dt = (x a)f((x)).
Sa se calculeze lim
xa
(x) a
x a
.
Solut ie. Existent a lui (x) rezulta din teorema de medie. Avem evident lim
xa
(x) = a.
lim
xa
(x) a
x a
= lim
xa
(x) a
f((x)) f(a)

f((x)) f(a)
x a
= lim
xa
(x) a
f((x)) f(a)
lim
xa
1
x a
_
x
a
f(t)dt f(a)
x a
=
1
f

(a)
lim
xa
_
x
a
f(t)dt f(a)(x a)
(x a)
2
=
1
f

(a)
lim
xa
f(x) f(a)
2(x a)
=
1
f

(a)

1
2
f

(a) =
1
2
.
Observat ie. Din f

(a) ,= 0 rezulta ca exista o vecinatate a lui a unde f este strict


monotona, prin urmare f((x)) f(a) ,= 0 pentru x sucient de apropiat de a.
Capitolul 10
Funct ii de mai multe variabile
reale
Denit ii si rezultate
Fie R
n
= R R R, x = (x
1
, . . . , x
n
) R
n
, y = (y
1
, . . . , y
n
) R
n
si R.
Operat iile + : R
n
R
n
R
n
, : R R
n
R
n
date de
x +y = (x
1
+y
1
, . . . , x
n
+y
n
)
x = (x
1
, . . . , x
n
)
(1)
determina pe R
n
o structura de spat iu vectorial peste R.
Aplicat ia , : R
n
R
n
R,
x, y =
n

i=1
x
i
y
i
(2)
este un produs scalar pe R
n
. Aceasta determina pe R
n
aplicat iile | | : R
n
R, d :
R
n
R
n
R denite prin
|x| =
_
x, x =

_
n

i=1
x
2
i
(3)
d(x, y) = |x y| =

_
n

i=1
(x
i
y
i
)
2
(4)
numite norma, respectiv distant a euclidiana pe R
n
.
Spat iul vectorial R
n
nzestrat cu produsul scalar denit prin relat ia (1) se numeste
spat iul euclidian R
n
.
Fie (x
p
)
p1
, x
p
= (x
1
p
, x
2
p
, . . . , x
n
p
), un sir din R
n
si a = (a
1
, a
2
, . . . , a
n
) R
n
. Sirul
(x
p
)
p1
se numeste convergent cu limita a daca pentru orice > 0 exista p

N astfel ca
pentru orice p p

sa avem
|x
p
a| < . (5)

In acest caz notam lim


p
x
p
= a sau x
p
a n R
n
. Are loc relat ia
x
p
a n R
n
x
1
p
a
1
, . . . , x
n
p
a
n
n R.
93
94
Sirul (x
p
)
p1
se numeste fundamental sau sir Cauchy daca pentru orice > 0 exista
p

N astfel ca pentru orice p, q p

sa aiba loc relat ia |x


p
x
q
| < .
Spat iul metric (R
p
, d) este complet, i.e. orice sir din R
p
este convergent daca si numai
daca este fundamental.
Mult imi remarcabile din R
n
Fie a R
n
, r R, r 0.
B(a, r) = x R
n
[ |x a| < r se numeste bila deschisa de centru a si raza r
B(a, r) = x R
n
[ |x a| r se numeste bila nchisa de centru a si raza r
S(a, r) = x R
n
[ |x a| = r se numeste sfera de centru a si raza r.
O mult ime V R
n
se numeste vecinatate a lui a daca exista B(a, r) V . Notam
cu 1(a) mult imea vecinatat ilor lui a. O mult ime G R
n
se numeste deschisa daca este
vecinatate pentru orice punct al sau. Mult imea vida se considera deschisa. O mult ime
F R
n
se numeste nchisa daca C
R
nF = R
n
F este mult ime deschisa.
Fie A R
n
. Punctul a R
n
se numeste:
- punct interior al lui A daca exista B(a, r) A
- punct aderent al lui A daca pentru orice V 1(a) V A ,=
- punct de acumulare al lui A daca pentru orice V 1(a)
V (A a) ,=
- punct izolat al lui A daca exista V 1(a) astfel ca V A = a
- punct frontiera al lui A dac a V A ,= si V (R
n
A) ,= .
Notam prin intA, A, A

, izA, frA mult imea punctelor interioare, aderente, de acumu-


lare, izolate respectiv frontiera ale lui A.
Mult imea A se numeste marginita daca exista M > 0 astfel ca |x| M pentru orice
x A.
Mult imea A se numeste densa n R
n
daca A = R
n
.
Au loc relat iile:
a A (x
p
)
p1
n A cu lim
p
x
p
= a;
a A

(x
p
)
p1
n A a cu lim
p
x
p
= a;
A este nchisa orice sir convergent din A are limita n A.
Mult imea A se numeste compacta daca din orice sir de puncte din A se poate extrage
un subsir convergent la un element din A. Are loc caracterizarea:
A este compacta A este nchisa si marginita.
Mult imea A se numeste conexa daca nu exista doua submult imi deschise si nevide ale
lui R
n
astfel ca A U V , A U ,= , A V ,= si A U V = .
Intuitiv o mult ime conexa e formata dintr-o singura bucata.
Mult imea A se numeste conexa prin arce daca pentru orice puncte a, b A exista
un arc continuu cu capetele a si b cont inut n A. Orice mult ime conexa prin arce e conexa.
Are loc urmatorul rezultat:
O mult ime deschisa A R
n
este conexa daca si numai daca pentru orice a, b A
exista o linie poligonala cu capetele n a si b cont inuta n A.
Mult imea A se numeste convexa daca pentru orice a, b A segmentul cu capetele a
si b este cont inut n A.
Funct ii de mai multe variabile reale 95
Funct ii continue
Fie f : A R
m
, A R
n
si a A.
Funct ia f se numeste continua n a daca pentru orice V 1(f(a)) exista U 1(a)
astfel ca f(x) V pentru orice x U A.
Teorema. Urmatoarele relat ii sunt echivalente:
1) f este continua n a.
2) Pentru orice > 0, exista

> 0 astfel ca |f(x) f(a)| < pentru orice x A cu


|x a| <

.
3) Pentru orice sir (x
p
)
p1
din A cu lim
p
x
p
= a lim
p
f(x) = f(a).
Funct ia f se numeste continua pe B A daca este continua n ecare punct al lui B.
Denit ie. Funct ia f se numeste uniform continua pe A daca pentru orice > 0
exista

> 0 astfel ca pentru orice x, y A cu |x y| <

sa avem |f(x) f(y)| < .


Daca f este funct ie Lipschitz, i.e. L 0 astfel ca
|f(x) f(y)| L|x y|, x, y A
atunci f este uniform continua pe A.
Teorema. (Weiertrass) Fie A R
n
o mult ime compacta si f : A R o funct ie
continua pe A. Atunci f este marginita si si atinge marginile.
Teorema. Fie A R
n
o mult ime compacta si f : A R
n
o funct ie continua. Atunci
f este uniform continua.
Teorema. (Darboux) Fie A R
n
o mult ime conexa si f : A R o funct ie continua
cu proprietatea ca exista a, b A astfel ca f(a) < 0 si f(b) > 0. Atunci exista c A astfel
ca f(c) = 0.
Derivate part iale
Fie A R
n
o mult ime nevida, a = (a
1
, . . . , a
n
) intA si f : A R o funct ie care
depinde de x = (x
1
, . . . , x
n
) A. Spunem ca f este derivabila part ial n raport cu
variabila x
k
, 1 k n, n punctul a daca
lim
x
k
a
k
f(a
1
, . . . , a
k1
, x
k
, a
k+1
, . . . , a
n
) f(a
1
, . . . , a
n
)
x
k
a
k
exista si este nita. Valoarea limitei anterioare se noteaza cu f

x
k
(a) sau
f
x
k
(a) si se
numeste derivata part iala a lui f n raport cu x
k
n punctul a. Funct ia f se numeste
derivabila n raport cu x
k
, 1 k n, pe o mult ime B A, daca f

x
k
(x) exista si este
nita pentru orice x B.
Derivatele de ordin superior se denesc prin
f

x
k
x
i
= (f

x
k
)

x
i
sau

2
f
x
k
x
i
=

x
k
_

x
i
_
1 i, k n. Se noteaza f

x
i
x
i
= f

x
2
i
sau

2
f
x
i
x
i
=

2
f
x
2
i
.

In general pentru = (
1
, . . . ,
n
) N
n
se denesc analog
f
||
x

1
1
...x
n
n
sau

||
f
x

1
1
. . . x
n
n
96
unde [[ =
1
+
2
+ +
n
.
Teorema. (Schwarz) Daca f : A R
n
R admite derivatele part iale f

x
k
x
i
si f

x
i
x
k
pe o vecinatate V a punctului a intA si acestea sunt continue n a, atunci f

x
k
x
i
(a) =
f

x
i
x
k
(a).
Teorema. (Derivata funct iilor compuse) Fie D R
m
si A R
n
mult imi deschise
si u = (u
1
, . . . , u
n
) : D A cu proprietatea ca u
1
, . . . , u
n
admit derivate part iale n raport
cu variabila x
k
, 1 k m pe D. Daca f C
1
(A), atunci funct ia compusa F : D R
F(x) = f(u
1
(x), . . . , u
n
(x))
admite derivate part iale n raport cu x
k
pe D si
F
x
k
=
f
u
1

u
1
x
k
+
f
u
2

u
2
x
k
+ +
f
u
m

u
m
x
k
.
Fie A R
n
, a intA, s R
n
, |s| = 1. Spunem ca f este derivabila pe direct ia s
n punctul a daca
lim
t0
f(a +ts) f(a)
t
exista si este nita. Ea se noteaza cu
df
ds
(a) si se numeste derivata lui f pe direct ia s
n punctul a.
Teorema. Daca f : A R, A R
n
are derivate part iale de ordinul unu continue pe o
vecinatate a lui a intA, atunci ea este derivabila pe orice direct ie s = (s
1
, . . . , s
n
) R
n
,
|s| = 1 si are loc relat ia
df
ds
(a) = f

x
1
(a)s
1
+f

x
2
(a)s
2
+ +f

xn
(a)s
n
.
Prin introducerea operatorului gradient notat prin grad sau si denit prin
: C
1
(A) C(A, R
n
), f =
_
f
x
1
,
f
x
2
, . . . ,
f
x
n
_
,
unde A R
n
este o mult ime deschisa, derivata pe direct ia s se poate exprima prin
df
ds
(a) = f(a), s.
Diferent iala unei funct ii
Fie f : I R, I R, o funct ie derivabila n punctul a intI. Relat ia
lim
xa
f(x) f(a)
x a
= f

(a)
se poate scrie sub forma echivalent a
lim
xa
f(x) f(a) f

(a)(x a)
x a
= 0. (1)
Fie T : R R funct ia liniara denita prin T(h) = f

(a)h. Relat ia (1) se scrie n mod


echivalent
lim
xa
f(x) f(a) T(x a)
x a
= 0 (2)
Funct ii de mai multe variabile reale 97
prin urmare derivabilitatea lui f n a implica existent a unei aplicat ii liniare T astfel ca (2)
sa aiba loc. De aici se poate deduce relat ia
f(x) f(a)

= T(x a)
pe o vecinatate a lui a, deci variat ia lui f n jurul lui a poate aproximata printr-o funct ie
liniara.
Funct ia f se numeste diferent iabila n a daca exista T : R R, liniara astfel ca
relat ia (2) sa aiba loc. Se arata ca aplicat ia T este unica. Ea se numeste diferent iala lui f
n punctul a si se noteaza cu df(a). Funct ia f este diferent iabila n a daca si numai daca
este derivabila n a. Avem
T(h) = df(a)(h) = f

(a)h.
Cum diferent iala aplicat iei identice 1
R
: R R, 1
R
(x) = x, pentru orice x R este
d1
R
(x)(h) = h, se noteaza n mod tradit ional h = dx. Deci:
df(x)(dx) = f

(x)dx.
Fie f : A R
m
, A R
n
, a intA. Funct ia f se numeste diferent iabila n a intA,
daca exista T : R
n
R
m
liniara astfel ca
lim
xa
f(x) f(a) T(x a)
|x a|
= 0. (3)
Aplicat ia T din (3) se numeste diferent iala lui f n a si se noteaza cu df(a). Daca
f = (f
1
, . . . , f
m
), atunci f este diferent iabila n a daca si numai daca f
1
, . . . , f
m
sunt
diferent iabile n a si n acest caz
df(a) = (df
1
(a), df
2
(a), . . . , df
m
(a)).
Teorema. Fie f : A R, A R
n
, diferent iabila n a intA. Atunci:
i) f este continua n a;
ii) f este derivabila pe orice direct ie s R
n
, |s| = 1 si are loc relat ia
df
ds
(a) = df(a)(s).
Teorema. Fie f : A R, A R
n
, diferent iabila n a intA. Atunci
df(a)(h) = f

x
1
(a)h
1
+f

x
2
(a)h
2
+ +f

xn
(a)h
n
.
Notand h
k
= dx
k
, 1 k n, avem
df(a)(dx) = f

x
1
(a)dx
1
+f

x
2
(a)dx
2
+ +f

xn
(a)dx
n
.
Teorema. Daca A R
n
este o mult ime deschisa si f C
1
(A), atunci f este
diferent iabila pe A.
Teorema. (Diferent iala funct iilor compuse) Fie f : A B, g : B R
p
, A R
n
,
B R
m
. Daca f este diferent iabila n a intA si g este diferent iabila n b = f(a) intB,
atunci g f : A R
p
este diferent iabila n a si are loc relat ia
d(g f)(a) = dg(b) df(a).
98
Teorema. (Teorema de medie) Fie A R
n
o mult ime deschisa si convex a si f :
A R o funct ie diferent iabila pe A si a, b A. Atunci exista c pe segmentul [a, b] astfel
ca
f(b) f(a) = df(c)(b a).
Teorema. Fie A R
n
o mult ime deschisa si conexa. Dac a f : A R este
diferent iabila pe A si df(x) = 0 pentru orice x A, atunci f este constanta pe A.
Calculul diferent ialei se poate face si prin utilizarea regulilor de diferent iere. Daca
f, g : A R sunt diferent iabile n a intA, atunci
d(f +g)(a) = df(a) +dg(a)
d(f)(a) = df(a), R
d(fg)(a) = f(a)dg(a) +g(a)df(a)
d
_
f
g
_
(a) =
g(a)df(a) f(a)dg(a)
g
2
(a)
, g(a) ,= 0.
Pentru o funct ie f C
p
(A), A R
m
ind o mult ime deschisa, se deneste diferent iala
de ordinul n, 1 n p, a lui f prin
d
n
f(x) =
_

x
1
dx
1
+ +

x
m
dx
m
_
n
f(x) =
=

k
1
++km=n
n!
k
1
!k
2
! . . . k
m
!

k
1
++km
f
x
k
1
1
. . . x
km
m
(x)dx
k
1
1
. . . dx
km
m
.
Pentru n = 2 se obt ine formula
d
2
f(x) =
n

k=1

2
f(x)
x
2
k
dx
2
k
+ 2

1j<kn

2
f(x)
x
k
x
j
dx
k
dx
j
.
Teorema. (Formula lui Taylor) Fie A R
n
o mult ime deschisa si convexa, f
C
m+1
(A) si a A. Atunci pentru orice x A exista pe segmentul [a, x] astfel ca
f(x) = f(a) +
df(a)(x a)
1!
+ +
d
m
f(a)(x a)
m!
+
d
m+1
f()(x a)
(m+ 1)!
.
Teorema. (Teorema funct iilor implicite) Fie A R
n
R
m
o mult ime deschisa si
f : A R
m
, f = (f
1
, f
2
, . . . , f
m
) si (x
0
, y
0
) A, x
0
R
n
, y
0
R
m
. Dac a sunt ndeplinite
condit iile:
1) f(x
0
, y
0
) = 0;
2) f C
1
(A);
3)
D(f
1
, . . . , f
m
)
D(y
1
, . . . , y
m
)
(x
0
, y
0
) ,= 0
atunci exist a o vecinatate deschisa U a lui x
0
n R
n
, o vecinatate deschisa V a lui y
0
n
R
m
si o funct ie : U V astfel ca
a) (x
0
) = y
0
;
b) f(x, (x)) = 0, x U;
c) este diferent iabila pe U.
Funct ii de mai multe variabile reale 99
Extremele funct iilor de mai multe variabile
Extreme locale
Fie f : D R, D R
n
. Punctul a D se numeste punct de minim (maxim) local al
funct iei f daca exista o vecinatate V a lui a astfel ca
f(a) f(x) (f(a) f(x)), pentru orice x V D.
Punctul a D se numeste punct de minim (maxim) global al lui f daca f(a) f(x)
(f(a) f(x)) pentru orice x D.
Punctele de minim sau maxim local (global) se numesc puncte de extrem local (global)
ale lui f.
Teorema. (Fermat) Daca a intD este punct de extrem local al lui f si f este
diferent iabila n a, atunci df(a) = 0.
Punctele a Dn care df(a) = 0 se numesc puncte stat ionare ale lui f. Rezulta ca un
punct a D este stat ionar daca si numai daca
f

x
1
(a) = 0, f

x
2
(a) = 0, . . . , f

xn
(a) = 0.
Pentru a decide care din punctele stat ionare sunt puncte de extrem local se poate folosi
urmatorul rezultat:
Teorema. Fie D R
n
o mult ime deschisa, f C
2
(D) si a D un punct stat ionar
al lui f. Au loc:
a) Daca diferent iala de ordinul al doilea d
2
f(a) este pozitiv denita, atunci a este punct
de minim local al lui f;
b) Daca d
2
f(a) este negativ denit a, atunci a este punct de maxim local al lui f;
c) Daca d
2
f(a) este nedenita, atunci a nu este punct de extrem local al lui f.
Prezentam mai n detaliu cazul funct iilor de doua variabile reale. Fie D R
2
o mult ime
deschisa, f C
2
(D) si (x
0
, y
0
) un punct stat ionar al lui f. Atunci
d
2
f(x
0
, y
0
) = pdx
2
+ 2qdxdy + dy
2
,
unde p = f

x
2
(x
0
, y
0
), q = f

xy
(x
0
, y
0
), r = f

y
2
(x
0
, y
0
). Notam := q
2
pr. Au loc:
1) Daca > 0 atunci (x
0
, y
0
) nu este punct de extrem local al lui f,
2) Daca < 0 si p > 0 atunci (x
0
, y
0
) este punct de minim local al funct iei f,
3) Daca < 0 si p < 0 atunci (x
0
, y
0
) este punct de maxim local al funct iei f.
Revenind la cazul general, reamintim un rezultat din algebra. Fie : R
n
R o
forma patratica si A matricea sa. Notam cu
1
,
2
, . . . ,
n
valorile proprii ale lui A si cu

1
,
2
, . . . ,
n
minorii sai principali.
Teorema. (Sylvester) Au loc echivalent ele:
1) este pozitiv denita
1
> 0,
2
> 0, . . . ,
n
> 0

1
> 0,
2
> 0, . . . ,
n
> 0
2) este negativ denita
1
< 0,
2
< 0, . . . ,
n
< 0

1
< 0,
2
> 0,
3
< 0, . . . , (1)
n

n
> 0
3) este nedenita A are dou a valori proprii de semne contrare.
Extreme condit ionate
Fie D R
n
o mult ime nevida, f : D R si g
1
, g
2
, . . . , g
m
: D R m funct ii date
(cu m < n). Fie
C = x D [ g
1
(x) = 0, . . . , g
m
(x) = 0, x = (x
1
, . . . , x
n
).
100
Un punct a C se numeste punct de extrem local condit ionat daca a este punct de extrem
local pentru funct ia f[
C
(restrict ia lui f la C). Relat iile
g
1
(x) = 0, g
2
(x) = 0, . . . , g
m
(x) = 0
se mai numesc legaturi, din aceasta cauza extremele condit ionate se mai numesc extreme
cu legaturi.
Fie G = (g
1
, . . . , g
n
) : D R
m
si L : D R
m
R, denita prin
L(x, ) = f(x) +
1
g
1
(x) + +
m
g
m
(x), = (
1
, . . . ,
m
) R
m
.
Teorema. Presupunem ca D este mult ime deschis a si f, g
1
, . . . , g
m
C
1
(D). Daca
a C este punct de extrem local condit ionat al lui f si matricea jacobiana J
G
verica
relat ia rang J
G
(a) = m, atunci exista
0
= (
0
1
,
0
2
, . . . ,
0
m
) R
m
astfel ca
_
L

x
k
(a,
0
) = 0, 1 k n
g
j
(a) = 0, 1 j m
Funct ia L se numeste Lagrangeanul lui f iar
0
1
,
0
2
, . . . ,
0
m
se numesc multiplicatorii
lui Lagrange.
Observat ie. Daca a este punct de extrem local condit ionat, atunci (a,
0
) este punct
stat ionar al lui L.
Are loc urmatorul rezultat:
Teorema. Presupunem ca D este mult ime deschisa, f, g
1
, . . . , g
m
C
1
(D) si (a,
0
)
C R
m
este un punct stat ionar al lui L. Daca pentru orice h R
n
0 cu proprietatea
dG(a)(h) = 0 avem
d
2
L(a,
0
)(h) > 0 (d
2
L(a,
0
)(h) < 0),
atunci a este punct de minim (maxim) local condit ionat al lui f.
Probleme
Problema 10.1 Fie m, n N

si f : R
2
R denita prin
f(x, y) =
_
_
_
x
m
y
n
x
2
xy +y
2
, (x, y) ,= (0, 0)
0, (x, y) = (0, 0)
Sa se studieze continuitatea, existent a derivatelor part iale de ordinul I, derivabilitatea
pe o direct ie si diferent iabilitatea lui f n punctul (0, 0).
Solut ie.
Continuitatea: daca m = n = 1, considerand dreapta y = x, R, avem
lim
x0
f(x, x) =

1 +
2
,
care depinde de , deci lim
(x,y)(0,0)
f(x, y) nu exista. Daca m+n > 2 avem
[f(x, y)[ =
[x
m
y
n
[
x
2
xy +y
2

[x
m
y
n
[
xy
= [x[
m1
[y[
n1
Funct ii de mai multe variabile reale 101
pentru (x, y) ,= (0, 0), de unde rezulta
lim
(x,y)(0,0)
f(x, y) = 0 = f(0, 0).
Deci f este continua n (0, 0) daca si numai daca m+n > 2.
Derivabilitatea n raport cu x si y:
lim
x0
f(x, 0) f(0, 0)
x 0
= lim
x0
0
x
= 0
si
lim
y0
f(0, y) f(0, 0)
y 0
= lim
y0
0
y
= 0,
deci f

x
(0, 0) = f

y
(0, 0) = 0.
Derivabilitatea pe o direct ie: e s = (u, v) R
2
cu u
2
+v
2
= 1, uv ,= 0. Avem
lim
t0
f((0, 0) +t(u, v)) f(0, 0)
t
= lim
t0
t
m+n
u
m
v
n
t
3
(u
2
uv +v
2
)
=
= lim
t0
t
m+n3
u
m
v
n
u
2
uv +v
2
= lim
t0
t
m+n3
f(u, v).
Daca m+n > 3 rezulta ca
df
ds
(0, 0) = 0 pentru orice s.
Daca m+n = 3 rezulta ca
df
ds
(0, 0) = f(u, v).
Daca m+n < 3, rezulta m = n = 1 si f nu este derivabila pe nici o direct ie s.
Diferent iabilitatea n (0, 0): daca m = n = 1, f nu este diferent iabila n (0, 0), deoarece
nu este continua n (0, 0). Pentru m + n 3, cum f

x
(0, 0) = f

y
(0, 0) = 0, daca f ar
diferent iabila n (0, 0) ar trebui ca
df(0, 0)(h, k) = f

x
(0, 0)h +f

y
(0, 0)k = 0.
Relat ia
lim
(x,y)(0,0)
f(x, y) f(0, 0) T(x 0, y 0)
_
x
2
+y
2
= 0
lim
(x,y)(0,0)
x
m
y
n
(x
2
xy +y
2
)
_
x
2
+y
2
= 0
este vericata daca si numai daca m + n > 3 (pentru m + n = 3 se considera limita pe
dreapta y = x). Deci f este diferent iabila n (0, 0) daca si numai daca m+n > 3.
Problema 10.2 Fie f : R
2
R denita prin
f(x, y) =
_
_
_
(x
2
+y
2
) sin
1
_
x
2
+y
2
, (x, y) ,= (0, 0)
0, (x, y) = (0, 0).
Demonstrat i ca f este diferent iabila n (0, 0), dar f

x
si f

y
nu sunt continue n (0, 0).
Solut ie. Avem
f

x
(0, 0) = lim
x0
f(x, 0) f(0, 0)
x
= lim
x0
[x[ sin
1
[x[
= 0
102
si analog
f

y
(0, 0) = lim
y0
f(0, y) f(0, 0)
y
= lim
y0
[y[ sin
1
[y[
= 0.
Vom arata ca df(0, 0) = 0.

Intr-adevar, avem
lim
(x,y)(0,0)
f(x, y) f(0, 0) df(0, 0)(x 0, y 0)
_
x
2
+y
2
=
= lim
(x,y)(0,0)
_
x
2
+y
2
sin
1
_
x
2
+y
2
= 0.
Pentru (x, y) ,= (0, 0) avem
f

x
(x, y) = 2xsin
1
_
x
2
+y
2

x
_
x
2
+y
2
cos
1
_
x
2
+y
2
f

y
(x, y) = 2y sin
1
_
x
2
+y
2

y
_
x
2
+y
2
cos
1
_
x
2
+y
2
.
Cum
lim
n
f

x
_
1
2n
, 0
_
= lim
n
(
1
n
sin 2n cos 2n) = 1
si
lim
n
f

x
_
0,
1
2n
_
= 0
rezulta ca lim
(x,y)(0,0)
f

x
(x, y) nu exista, deci f

x
este discontinua n (0, 0). Prin simetrie
rezulta ca si f

y
este discontinua n (0, 0).
Observat ie. Continuitatea derivatelor part iale ntr-un punct nu este o condit ie nece-
sara pentru diferent iabilitatea funct iei n acel punct.
Problema 10.3 Sa se arate ca funct ia f : R
2
R
f(x, y) =
_
[x[
a
sin
y
x
, x ,= 0
0, x = 0
este diferent iabila pentru orice a > 1.
Solut ie. Funct ia f este diferent iabila n (x
0
, y
0
) daca si numai daca exista derivatele
part iale
f
x
(x
0
, y
0
),
f
y
(x
0
, y
0
) si n plus
lim
(x,y)(x0,y0)
f(x, y) f(x
0
, y
0
)
f
x
(x
0
, y
0
)(x x
0
)
f
y
(x
0
, y
0
)(y y
0
)
_
(x x
0
)
2
+ (y y
0
)
2
= 0.
Funct ia f are derivate part iale continue pe mult imea R
2
(0, b)[ b R, deci este
diferent iabila n aceste puncte. Ramane de studiat diferent iabilitatea n punctele de forma
(0, b), b R.

In (0, 0) avem:
f
x
(0, 0) = lim
x0
f(x, 0) f(0, 0)
x
= 0,
Funct ii de mai multe variabile reale 103
f
y
(0, 0) = lim
y0
f(0, y) f(0, 0)
y
= 0
si [f(x, y)[ [x[
a
, deci
lim
(x,y)(0,0)
[f(x, y)[
_
x
2
+y
2
= lim
(x,y)(0,0)
[x[
_
x
2
+y
2
[x[
a1

lim
(x,y)(0,0)
[x[
a1
= 0,
deci exista diferent iala n (0, 0) si este egala cu zero.

In (0, b) cu b ,= 0,
f
x
(0, b) = lim
x0
f(x, b) f(0, b)
x
= lim
x0
x
a1
sin
b
x
= 0
f
y
(0, b) = 0,
iar

f(x, y) f(0, b)
f
x
(0, b)x
f
y
(0, b)(y b)

= [f(x, y)[ [x[


a
si
lim
(x,y)(0,0)
[f(x, y)[
_
x
2
+ (y b)
2
lim
(x,y)(0,0)
[x[
_
x
2
+ (y b)
2
[x[
a1

lim
(x,y)(0,0)
[x[
a1
= 0, deci df(0, b) = 0.
Problema 10.4 Fie f : D R, D R
2
si (x
0
, y
0
) intD. Sa se arate ca daca f are
derivate part iale ntr-o vecinatate V a lui (x
0
, y
0
) si daca una din ele este continua n
(x
0
, y
0
), atunci f este diferent iabila n (x
0
, y
0
).
Solut ie. Sa presupunem ca f

x
este continua n (x
0
, y
0
). Pentru orice (x, y) V D
avem:
f(x, y) f(x
0
, y
0
) = f(x, y) f(x
0
, y) +f(x
0
, y) f(x
0
, y
0
).
Conform teoremei lui Lagrange exista c
1
ntre x
0
si x astfel ca
f(x, y) f(x
0
, y) = (x x
0
)f

x
(c
1
, y).
Cum f

y
(x
0
, y
0
) exista rezulta ca
lim
yy
0
f(x
0
, y) f(x
0
, y
0
)
y y
0
f

y
(x
0
, y
0
) = 0
deci
f(x
0
, y) f(x
0
, y
0
) = (y y
0
)(f

y
(x
0
, y
0
) +
2
(x
0
, y))
cu lim
yy
0

2
(x
0
, y) = 0.
Din continuitatea lui f

x
n (x
0
, y
0
) rezulta ca
f

x
(c
1
, y) = f

x
(x
0
, y
0
) +
1
(x, y)
cu lim
(x,y)(x
0
,y
0
)

1
(x, y) = 0.
104
Rezulta ca are loc relat ia
f(x, y) f(x
0
, y
0
) = (x x
0
)f

x
(x
0
, y
0
) + (y y
0
)f

y
(x
0
, y
0
)+
+
1
(x, y)(x x
0
) +
2
(x
0
, y)(y y
0
)
pentru (x, y) V D, ceea ce arata ca f este diferent iabila n (x
0
, y
0
).
Problema 10.5 Fie g : R R o funct ie derivabila pe R, satisfacand condit iile g(0) = 0
si g

(0) ,= 0. Denim funct ia f : R


2
R prin relat ia
f(x, y) =
_
_
_
g(xy)
x
2
y
2
x
2
+y
2
, (x, y) ,= (0, 0)
0, (x, y) = (0, 0)
a) Sa se studieze existent a derivatelor part iale de ordinul I si diferent iabilitatea lui f
n (0, 0).
b) Sa se arate ca f

xy
(0, 0) ,= f

yx
(0, 0).
Solut ie. a) f

x
(0, 0) = lim
x0
f(x, 0) f(0, 0)
x
= lim
x0
0
x
= 0 si analog f

y
(0, 0) = 0. Demon-
stram ca f este diferent iabila n (0, 0) si T = df(0, 0) = 0. Avem
lim
(x,y)(0,0)
f(x, y) f(0, 0) T(x 0, y 0)
_
x
2
+y
2
=
= lim
(x,y)(0,0)
g(xy)
_
x
2
+y
2

x
2
y
2
x
2
+y
2
=
= lim
(x,y)(0,0)
g(xy) g(0)
xy

xy
_
x
2
+y
2

x
2
y
2
x
2
+y
2
=
= g

(0) lim
(x,y)(0,0)
xy
_
x
2
+y
2

x
2
y
2
x
2
+y
2
= 0,
deoarece

xy
_
x
2
+y
2

x
2
y
2
x
2
+y
2

=
[x[
_
x
2
+y
2
. .
1

x
2
y
2
x
2
+y
2

. .
1
[y[ [y[,
pentru (x, y) ,= (0, 0).
b) Avem
f

x
(x, y) = yg

(xy)
x
2
y
2
x
2
+y
2
+g(xy)
4xy
2
(x
2
+y
2
)
2
si
f

y
(x, y) = xg

(xy)
x
2
y
2
x
2
+y
2
g(xy)
4x
2
y
(x
2
+y
2
)
2
.
Obt inem
f

xy
(0, 0) = (f

x
)

y
(0, 0) = lim
y0
f

x
(0, y) f

(0, 0)
y
=
= lim
y0
yg

(0)
y
= g

(0)
si
f

yx
(0, 0) = (f

y
)

x
(0, 0) = lim
x0
f

y
(x, 0) f

y
(0, 0)
x
= lim
x0
xg

(0)
x
= g

(0).
Funct ii de mai multe variabile reale 105
Problema 10.6 Se da funct ia f : R
2
R denita prin
f(x, y) =
_
_
_
y
2
ln
_
1 +
x
2
y
2
_
, y ,= 0
0, y = 0
Sa se arate ca derivatele mixte de ordinul doi ale lui f nu sunt continue n origine si
totusi f

xy
(0, 0) = f

yx
(0, 0).
Solut ie.

In punctele de forma (x, y) cu y ,= 0 avem
f

x
(x, y) =
2xy
2
x
2
+y
2
si f

y
(x, y) = 2y ln
_
1 +
x
2
y
2
_

2x
2
y
x
2
+y
2
iar n punctele de forma (x
0
, 0) avem
f

x
(x
0
, 0) = lim
xx
0
f(x, 0) f(x
0
, 0)
x x
0
= lim
xx
0
0
x x
0
= 0
f

y
(x
0
, 0) = lim
y0
f(x
0
, y) f(x
0
, 0)
y
= lim
y0
y ln
_
1 +
x
2
0
y
2
_
= 0.
Se obt ine de asemenea
f

xy
(x, y) =
_
_
_
4x
3
y
(x
2
+y
2
)
2
, y ,= 0
0, y = 0
si
f

yx
(x, y) =
_
_
_
4x
3
y
(x
2
+y
2
)
2
, y ,= 0
0, y = 0
deci f

xy
= f

yx
. Calculand limitele pe dreapta y = x se obt in rezultatele
lim
x0
f

xy
(x, x) = lim
x0
f

yx
(x, x) =
4
(1 +
2
)
2
,
de unde rezulta ca funct iile f

xy
si f

yx
nu au limita n (0, 0), deci sunt discontinue n (0, 0).
Observat ie. Continuitatea derivatelor part iale mixte de ordinul II din teorema lui
Schwarz nu este o condit ie necesara pentru egalitatea lor.
Problema 10.7 Fie g : [0, +) R si f : R
n
R, f(x) = g(|x|) pentru orice x R
n
.
Sa se arate ca urmatoarele armat ii sunt echivalente:
i) f este o funct ie de clasa C
1
;
ii) g este o funct ie de clasa C
1
si g

(0) = 0.
Solut ie. i) ii) Presupunem ca f este de clasa C
1
si e u R
n
, |u| = 1. Pentru
orice t 0 avem
f(tu) = g(|tu|) = g(t)
si t inand seama ca f C
1
(R
n
) rezulta ca g C
1
([0, )). Avem de asemenea relat ia
f(t(u)) = g(| tu|) = g([ t[ |u|) = g(t)
106
pentru orice t 0. Deci pentru orice t 0 avem:
g(t) = f(tu) = f(tu).
Notand u = (u
1
, u
2
, . . . , u
n
) si derivand relat ia anterioara avem
g

(t) = u
1
f

x
1
(tu) + +u
n
f

xn
(tu) = u
1
f

x
1
(tu) u
n
f

xn
(tu)
de unde rezulta
g

(0) = u
1
f

x
1
(0) + +u
n
f

xn
(0) = u
1
f

x
1
(0) u
n
f

xn
(0)
ceea ce este echivalent cu g

(0) = 0.
ii) i) Cum norma euclidiana a lui R
n
este de clasa C
1
pe R
n
0 rezulta ca f
este de clasa C
1
pe R
n
0. Demonstram ca f

x
k
(0) = 0 pentru 1 k n. Avem:
lim
x
k
0

f(0, . . . , 0, x
k
, 0, . . . , 0) f(0, . . . , 0)
x
k

= lim
x
k
0
[g([x
k
[) g(0)[
[x
k
[
=
= [g

(0)[ = 0 pentru 1 k n.
De asemenea pentru x R
n
, x ,= 0, avem
f

x
k
(x) = g

(|x|)
x
k
|x|
, 1 k n.
Rezulta ca
[f

x
k
(x)[ = [g

(|x|)[
[x
k
[
|x|
[g

(|x|)[
si t inand seama de faptul ca lim
x0
g

(|x|) = g

(0) = 0 obt inem


lim
x0
f

x
k
(x) = 0 = f

x
k
(0), 1 k n.
Deci f C
1
(R
n
).
Problema 10.8 Fie f : R
3
R o funct ie denita prin relat ia
f(x) =
_
_
_
sin(a|x|)
|x|
, x ,= 0
a, x = 0
unde a R. Sa se arate ca f C
2
(R
3
) si f +a
2
f = 0.
Solut ie. Fie g : R R, g(t) =
sin(at)
t
pentru t ,= 0 si g(0) = a. Se verica imediat ca
g este o funct ie de clasa C

(R), dezvoltand funct ia sin(at) n serie de puteri ale lui t, si


ca g

(0) = 0. Cum f(x) = g(|x|) pentru orice x R


3
rezulta din problema 10.7 ca f este
de clasa C
1
(R
3
) si f

x
k
(0) = 0, 1 k 3.
Printr-un rat ionament analog celui din problema 10.7 se arata n continuare ca f
C
2
(R
3
). Avem
f

x
2
k
(x) = g

(|x|)
x
2
k
|x|
2
+g

(|x|)
|x|
2
x
2
k
|x|
3
pentru 1 k 3 de unde rezulta
f = g

(|x|) +g

(|x|)
2
|x|
= a
2
f.
Funct ii de mai multe variabile reale 107
Problema 10.9 Fie K R
n
o mult ime nevida cu proprietatea ca pentru orice x K
si orice t > 0 avem tx K. Fie de asemenea f : K R o funct ie omogena de gradul p,
p R, adica avand proprietatea
f(tx) = t
p
f(x)
pentru orice x K si orice t > 0.
a) Sa se arate ca daca f C
1
(K), atunci
x
1
f
x
1
(x) + +x
n
f
x
n
(x) = pf(x), x K.
b) Sa se arate ca daca f C
2
(K), atunci
_
x
1

x
1
+ +x
n

x
n
_
(2)
f(x) = p(p 1)f(x), x K.
(Identitat ile lui Euler pentru funct ii omogene.)
Solut ie. a) Derivand relat ia
f( tx
1
..
u
1
, tx
2
..
u
2
, . . . , tx
n
..
un
) = t
p
f(x), x = (x
1
, . . . , x
n
) K,
n raport cu t obt inem
f
u
1
(tx)
du
1
dt
+ +
f
u
n
(tx)
du
n
dt
= pt
p1
f(x)
sau
x
1
f
u
1
(tx) + +x
n
f
u
n
(tx) = pt
p1
f(x).
Punand n ultima relat ie t = 1 obt inem
x
1
f
x
1
(x) + +x
n
f
x
n
(x) = pf(x).
b) Derivam relat ia de la punctul a) n raport cu x
1
, x
2
, . . . , x
n
obt inem succesiv
f
x
1
+x
1

2
f
x
2
1
+x
2

2
f
x
1
x
2
+ +x
n

2
f
x
1
x
n
= p
f
x
1
x
1

2
f
x
2
x
1
+
f
x
2
+x
2

2
f
x
2
2
+ +x
n

2
f
x
2
x
n
= p
f
x
2
. . . . . .
x
1

2
f
x
n
x
1
+ +
f
x
n
+x
n

2
f
x
2
n
= p
f
x
n
.

Inmult ind relat iile anterioare cu x


1
, x
2
, . . . , x
n
si adunandu-le obt inem
n

k=1
x
2
k

2
f
x
2
k
+ 2

1k<jn

2
f
x
k
x
j
x
k
x
j
+
n

k=1
x
k
f
x
k
= p
n

k=1
x
k
f
x
k
si nlocuind
n

k=1
x
k
f
x
k
= pf obt inem
_
x
1

x
1
+ +x
n

x
n
_
(2)
f = p(p 1)f.
108
Problema 10.10 Sa se calculeze

m+n
f
x
m
y
n
si diferent iala de ordinul n pentru funct ia:
f(x, y) = (x
2
+y
2
)e
x+y
, (x, y) R
2
.
Solut ie. Aplicand formula lui Leibniz avem

m
f
x
m
= C
0
m
e
x+y
(x
2
+y
2
) +C
1
m
e
x+y
2x +C
2
m
e
x+y
2 =
= e
x+y
(x
2
+y
2
+ 2mx +m
2
m)

m+n
f
x
m
y
n
= C
0
n
e
x+y
(x
2
+y
2
+ 2mx +m
2
m) +C
1
n
e
x+y
2y +C
2
n
e
x+y
2 =
= e
x+y
(x
2
+y
2
+ 2mx + 2ny +m
2
+n
2
mn).

Inlocuind derivatele part iale n formula


d
n
f =
_

x
dx +

y
dy
_
n
f
se obt ine diferent iala de ordinul n a lui f.
Problema 10.11 a) Sa se arate ca derivata funct iei f : R

R R, f(x, y) =
y
2
x
n
orice punct al elipsei 2x
2
+y
2
= 1 pe direct ia normalei la elipsa este egala cu zero.
b) Sa se calculeze derivata funct iei f(x, y, z) =
1
_
x
2
+y
2
+z
2
, pe direct ia gradientului
sau.
Solut ie. a) Fie (x
0
, y
0
) un punct al elipsei 2x
2
+y
2
= 1. Tangenta la elipsa n (x
0
, y
0
)
are ecuat ia 2xx
0
+yy
0
= 1, deci versorul normalei la elipsa este s = (s
1
, s
2
) unde
s
1
=
2x
0
_
4x
2
0
+y
2
0
, s
2
=
y
0
_
4x
2
0
+y
2
0
.
Aplicand formula derivatei pe direct ie avem
df
ds
(x
0
, y
0
) = f

x
(x
0
, y
0
)s
1
+f

y
(x
0
, y
0
)s
2
=
=
y
2
0
x
2
0

2x
0
_
4x
2
0
+y
2
0
+
2y
0
x
0

y
0
_
4x
2
0
+y
2
0
= 0.
b) Fie punctul M(x, y, z), diferit de originea O.
Notam prin r =
_
x
2
+y
2
+z
2
. Avem
gradf =
_

x
r
3
,
y
r
3
,
z
r
3
_
iar versorul sau este s =
_

x
r
,
y
r
,
z
r
_
. Obt inem
df
ds
(M) = f

x
(M)s
1
+f

y
(M)s
2
+f

z
(M)s
3
=
1
r
2
.
Funct ii de mai multe variabile reale 109
Problema 10.12 Sa se arate ca ecuat ia xe
y
+ye
x
= 1 deneste o funct ie implicita
y = f(x)
ntr-o vecinatate a punctului (0, 1). Sa se determine primii trei termeni din dezvoltarea lui
f dupa formula lui Taylor n punctul 0.
Solut ie. Fie F : R
2
R, F(x, y) = xe
y
+ye
x
1. Cum F este de clasa C

, F(0, 1) = 0
si F

y
(0, 1) = 1 ,= 0 rezulta ca ecuat ia F(x, y) = 0 deneste o funct ie y = f(x) de clasa C

ntr-o vecinatate a lui (0, 1). Derivand ecuat ia F(x, y) = 0 n care y = f(x) obt inem:
e
y
+xy

e
y
+y

e
x
+ye
x
= 0
si punand x = 0, y = 1 rezulta y

(0) = 1 e. Derivand din nou obt inem


2y

e
y
+xy

e
y
+x(y

)
2
e
y
+y

e
x
+ 2y

e
x
+ye
x
= 0
si punand din nou x = 0, y = 1 rezulta
y

(0) = 2e
2
+ 4e + 1.

In sfarsit derivand din nou ultima relat ie si punand x = 0, y = 1 se obt ine


y

(0) = 9e
3
24e
2
15e 1.
Dezvoltarea ceruta este
f(x) = f(0) +
f

(0)
1!
x +
f

(0)
2!
x
2
+
f

(0)
3!
x
3
+R
3
(x)
f(x) = 1 (1 +e)x +
_
e
2
+ 2e +
1
2
_
x
2

_
3
2
e
3
+ 4e
2
+
5
2
e +
1
6
_
x
3
+R
3
(x).
Problema 10.13 Consideram sistemul de ecuat ii
_
_
_
3x +y z +u
4
= 0
x y + 2z +u = 0
2x + 2y 3z + 2u = 0
a) Demonstrat i ca sistemul dat deneste pe x, y, u ca funct ii de z satisfacand condit iile
x(0) = y(0) = u(0) = 0 pe un interval de forma ] , [ cu > 0.
b) Demonstrat i ca nu exista nici un interval de forma ] , [ cu > 0 pe care sistemul
dat sa deneasca pe x, y, z ca funct ii de u.
Solut ie. a) Fie F
1
, F
2
, F
3
: R
4
R,
F
1
(x, y, z, u) = 3x +y z +u
4
,
F
2
(x, y, z, u) = x y + 2z +u,
F
3
(x, y, z, u) = 2x + 2y 3z + 2u.
Avem
D(F
1
, F
2
, F
3
)
D(x, y, u)
(0, 0, 0, 0) =

3 1 4u
3
1 1 1
2 2 2

(0, 0, 0, 0) =
110
=

3 1 0
1 1 1
2 2 2

= 12 ,= 0
de unde rezulta pe baza teoremei funct iilor implicite ca exista o vecinatate U a lui 0 si o
vecinatate V a punctului (0, 0, 0) si o unica funct ie vectoriala F = (f
1
, f
2
, f
3
) : U V
denita de sistemul dat si care satisface condit ia F(0) = (0, 0, 0). Din faptul ca U 1
R
(0)
rezulta ca exista > 0 astfel ca ] , [ U. Cum F
1
, F
2
, F
3
sunt de clasa C

pe ] , [
rezulta ca si funct ia F este de clas a C

pe ] , [.
b) Presupunem ca exista > 0 astfel ca sistemul dat sa deneasca pe x, y, z ca funct ii
de u pe intervalul ] , [. Scazand ecuat iile 2 si 3 ale sistemului din prima ecuat ie se
obt ine u
4
3u = 0, de unde rezult a ca u 0,
3

3, deci ] , [ 0,
3

3, contradict ie.
Problema 10.14 Sa se transforme ecuat ia (1 x
2
)y

xy

+
2
y = 0 prin schimbarea
de variabila x = cos t.
Solut ie. Fie z(t) = y(cos t). Avem z

(t) = y

(cos t) sin t de unde y

(cos t) =
z

(t)
sin t
.
Derivand din nou aceasta relat ie se obt ine
y

(cos t) sin t =
z

(t) sin t z

(t) cos t
sin
2
t
y

(cos t) =
z

(t) sin t z

(t) cos t
sin
3
t
.

Inlocuind n ecuat ie se obt ine z

(t) +
2
z(t) = 0.
Problema 10.15 Sa se transforme ecuat ia y

2
+ 2xy

3
= 0 schimband rolul vari-
abilelor.
Solut ie. Avem din formula de derivare a funct iei inverse
y

(x) =
1
x

(y)
, y = y(x).
Derivand din nou aceasta relat ie n raport cu x se obt ine
y

(x) =
x

(y)y

(x

(y))
2
=
x

(y)
(x

(y))
3
.

Inlocuind n ecuat ia data, obt inem ecuat ia:


x

+x

2x = 0.
Problema 10.16 Se da ecuat ia cu derivate part iale
az

x
2
+ 2bz

xy
+cz

y
2
= 0
unde a, b, c R si ac b
2
< 0. Sa se ae , R astfel ca prin schimbarea de variabile
_
u = x +y
v = x +z
ecuat ia sa devina w

uv
= 0. Sa se rezolve ecuat ia data.
Funct ii de mai multe variabile reale 111
Solut ie. Fie z(x, y) = w(x +y, x +y). Avem
z

x
= w

u
u

x
+w

v
v

x
= w

u
+w

v
z

y
= w

u
u

y
+w

v
v

y
= w

u
+w

v
z

x
2
= w

u
2
u

x
+w

uv
v

x
+w

uv
u

x
+w

v
2
v

x
= w

u
2
+ 2w

uv
+w

v
2
z

xy
= w

u
2
u

y
+w

uv
v

y
+w

uv
u

y
+w

v
2
v

y
= w

u
2
+ ( +)w

uv
+w

v
2
z

y
2
= (w

u
2
u

y
+w

uv
v

y
) +(w

uv
u

y
+w

v
2
v

y
) =
2
w

u
2
+ 2w

uv
+
2
w

v
2
.

Inlocuind n ecuat ia data se obt ine


(a + 2b +c
2
)w

u
2
+ (2a + 2b( +) +c
2
)w

uv
+ (a + 2b +c
2
)w

v
2
= 0.
Rezulta ca si trebuie sa e radacini ale ecuat iei
c
2
+ 2b +a = 0.
Pentru aceste valori ecuat ia devine w

uv
= 0 cu solut ia
w(u, v) = (u) +(v)
unde , sunt funct ii arbitrare de clasa C
2
. Solut ie ecuat iei date este
z(x, y) = (x +
1
y) +(x +
2
y).
Problema 10.17 Sa se transforme ecuat ia lui Laplace
z = z

x
2
+z

y
2
= 0
trecand la coordonate polare.
Solut ie. Fie
_
x = r cos t
y = r sin t
si e funct ia w denita prin
w(r, t) = z(r cos t, r sin t).
Din formula de derivare a funct iilor compuse avem
w

r
= z

x
cos t +z

y
sin t
w

t
= z

x
r sin t +z

y
r cos t.
Derivand din nou aceste relat ii obt inem:
w

r
2
= z

x
2
cos
2
t + 2z

xy
sin t cos t +z

y
2
sin
2
t
w

t
2
= z

x
2
cos
2
t 2z

xy
r
2
sin t cos t +z

y
2
r
2
cos
2
t z

x
r cos t z

y
r sin t
de unde se obt ine
w

r
2
+
1
r
2
w

t
2
+
1
r
w

r
= 0.
112
Problema 10.18 Sa se rezolve ecuat ia z

x
2
+2z

xy
+z

y
2
= 0 cu schimbarea de variabile si
de funct ie
_
_
_
u = x +y
v = x y
w = xy z.
Solut ie. Fie
w(x +y, x y) = xy z(x, y)
sau echivalent
w(u, v) =
u
2
v
2
2
z
_
u +v
2
,
u v
2
_
.
Avem:
z(x, y) = xy w(x +y, x y)
de unde obt inem
z

x
= y w

u
u

x
w

v
v

x
= y w

u
w

v
z

y
= x w

u
u

y
w

v
v

y
= x w

u
+w

v
si n continuare
z

x
2
= w

u
2
2w

uv
w

v
2
z

xy
= 1 w

u
2
w

v
2
z

y
2
= w

u
2
+ 2w

uv
w

v
2
.

Inlocuind n ecuat ia init iala obt inem


w

u
2
2w

uv
w

v
2
+ 2 2w

u
2
+ 2w

v
2
w

u
2
+ 2w

uv
w

v
2
= 0
ceea ce este echivalent cu
w

u
2
=
1
2
.
Integrand succesiv n raport cu u se obt ine
w

u
=
1
2
u +(v)
w =
1
4
u
2
+(v)u +(v).
Revenind la variabilele x si y se obt ine solut ia
z(x, y) =
(x y)
2
4
(x +y)(x y) (x y)
unde si sunt funct ii arbitrare de clasa C
2
.
Problema 10.19 Sa se arate ca funct ia f : R
2
R denita prin
f(x, y) =
_
_
_
xy sin
1
x
2
+y
2
, (x, y) ,= (0, 0)
0, (x, y) = (0, 0)
este diferent iabila pe R
2
, dar nu este de clasa C
1
.
Funct ii de mai multe variabile reale 113
Solut ie. Avem f

x
(0, 0) = f

y
(0, 0) = 0, iar pentru (x, y) ,= (0, 0)
f

x
(x, y) = y sin
1
x
2
+y
2

2x
2
y
(x
2
+y
2
)
2
cos
1
x
2
+y
2
f

y
(x, y) = xsin
1
x
2
+y
2

2xy
2
(x
2
+y
2
)
2
cos
1
x
2
+y
2
.
Se arata ca lim
(x,y)(0,0)
f

x
(x, y) nu exista si ca df(0, 0) = 0.
Problema 10.20 Sa se determine funct iile f de clasa C
2
de forma:
a) f(x, y) = (x
2
+y
2
);
b) f(x, y) = (y
2
x
2
);
c) f(x, y) =
_
y
x
_
unde C
2
(R), care verica ecuat ia lui Laplace.
Solut ie. a) Se obt ine f = 4u

(u) +4

(u), u(x, y) = x
2
+y
2
. Notand

(u) = (u)
se obt ine u

(u) +(u) = 0 ceea ce se scrie (u(u))

= 0 de unde rezulta ca u(u) = C


1
,
C
1
R. Deci

(u) =
C
1
u
cu solut ia
(u) = C
1
ln [u[ +C
2
, C
2
R.
Prin urmare
f(x, y) = C
1
ln(x
2
+y
2
) +C
2
, (x, y) R
2
(0, 0).
b) Procedeu analog. Se obt ine f(x, y) = C
1
(y
2
x
2
) +C
2
.
c) f =
x
2
+y
2
x
4

(u) + 2
y
x
3

(u) = 0, u(x, y) =
y
x
.
Se obt ine ecuat ia
(1 +u
2
)

(u) + 2u(u) = 0, (u) =

(u)
((1 +u
2
)(u))

= 0 (u) =
C
1
1 +u
2

(u) = C
1
arctg u +C
2
, C
1
, C
2
R.
Deci f(x, y) = C
1
arctg
y
x
+C
2
, x ,= 0.
Problema 10.21 Sa se determine funct ia f C
2
(R
2
), care satisface relat ia
a
2
f

x
2
b
2
f

y
2
= 0, ab ,= 0,
efectuand schimbarea de variabile
_
u = bx +ay
v = bx ay
Solut ie. Punand g(bx + ay, bx ay) = f(x, y) se obt ine g

uv
= 0, de unde rezulta
g(u, v) = (u) +(v), adica
f(x, y) = (bx +ay) +(bx ay),
cu , funct ii de clasa C
2
(R).
114
Problema 10.22 Sa se arate ca funct ia f : R
n
R, denita prin
f(x) =
_
_
_
|x|
2
sin
1
|x|
, x ,= 0
0, x = 0
unde | | este norma euclidiana din R
n
, este diferent iabila pe R
n
, dar nu este de clasa C
1
.
Solut ie. Se arata ca f

x
k
(0) = 0, 1 k n si df(0) = 0.
Problema 10.23 Fie D o submult ime nevida, deschisa si conexa a lui R
m
si f : D R
n
o funct ie cu proprietatea ca exista > 1 si L > 0 astfel ca
|f(x) f(y)| L|x y|

pentru orice x, y D. Sa se arate ca f este constanta.


Solut ie. Vom demonstra ca f este diferent iabila pe D si ca df(x) = 0 pentru orice
x D. Fie x
0
D. Avem
|f(x) f(x
0
)|
|x x
0
|
L|x x
0
|
1
, x D x
0
.
De aici rezulta ca lim
xx
0
f(x) f(x
0
)
|x x
0
|
= 0, deci df(x
0
) = 0. Cum D este conexa rezulta
ca f este constanta pe D.
Problema 10.24 Fie D R
m
o mult ime nevida, deschisa si convexa. Sa se arate ca daca
f : D R este diferent iabila pe D si are derivate part iale marginite pe D, atunci f este
uniform continua pe D.
Solut ie. Fie M > 0 astfel ca [f

x
k
(x)[ M pentru orice x D si orice k, 1 k m.
Fie x, y D. Din teorema de medie rezulta ca
f(x) f(y) = df(c)(x y), c = x +(y x), 0 < < 1.
Avem
[f(x) f(y)[ =

k=1
f

x
k
(c)(x
k
y
k
)

k=1
[f

x
k
(c)[ [x
k
y
k
[ M
n

k=1
[x
k
y
k
[
mM|x y|, x = (x
1
, . . . , x
m
), y = (y
1
, . . . , y
m
).
Rezulta ca f este lipschitziana, deci este uniform continua.
Problema 10.25 Fie f : D R, D R
2
si (x
0
, y
0
) intD. Presupunem ca exista
o vecinatate V D a lui (x
0
, y
0
) astfel ncat f sa e continua n (x
0
, y
0
) si sa admita
derivate part iale de ordinul unu pe V (x
0
, y
0
) cu proprietatea
lim
(x,y)(x
0
,y
0
)
f

x
(x, y) = a, lim
(x,y)(x
0
,y
0
)
f

y
(x, y) = b, a, b R.
Sa se arate ca f este diferent iabila n (x
0
, y
0
).
Funct ii de mai multe variabile reale 115
Solut ie. Exista o bila deschisa B cu centrul n (x
0
, y
0
) astfel ca B V D. Pentru
(x, y) B avem
[f(x, y) f(x
0
, y
0
) a(x x
0
) b(y y
0
)[ = [f(x, y) f(x, y
0
)+
+f(x, y
0
) f(x
0
, y
0
) a(x x
0
) b(y y
0
)[ = [(y y
0
)f

y
(x, c
2
)+
+(x x
0
)f

x
(c
1
, y
0
) a(x x
0
) b(y y
0
)[
[x x
0
[ [f

x
(c
1
, y
0
) a[ +[y y
0
[ [f

x
(x, c
2
) b[
unde c
1
, c
2
sunt punctele intermediare din teorema lui Lagrange aplicata funct iilor f(, y
0
)
si f(x, ).
Rezulta, notand =
_
(x x
0
)
2
+ (y y
0
)
2
, ca
[f(x, y) f(x
0
, y
0
) a(x x
0
) b(y y
0
)[


[x x
0
[

[f

x
(c
1
, y
0
) a[+
+
[y y
0
[

[f

x
(x, c
2
) b[, pentru (x, y) ,= (x
0
, y
0
).
Cum
[x x
0
[

1,
[y y
0
[

1 rezulta ca membrul drept tinde la 0 cand (x, y)


(x
0
, y
0
), deci f este diferent iabila n (x
0
, y
0
) si
df(x
0
, y
0
)(h, k) = ah +bk.
Problema 10.26 Sa se arate ca funct ia f : R
n
B(0, 1), unde B(0, 1) este bila deschisa
din R
n
denita prin relat ia
f(x) =
x
_
1 +|x|
2
,
pentru orice x R
n
este o biject ie de clasa C
1
cu inversa de clasa C
1
(difeomorsm sau
transformare regulata).
Solut ie. Evident f C
1
(R
n
, R
n
). Se gaseste f
1
: B(0, 1) R
n
,
f
1
(x) =
x
_
1 |x|
2
, x B(0, 1)
care este de clasa C
1
.
Problema 10.27 Fie f : R R o funct ie de clasa C
2
si F : R
2
R
F(x, y) =
_
_
_
f(x) f(y)
x y
, x ,= y
f

(x), x = y.
Sa se arate ca F este funct ie de clasa C
1
.
Solut ie. Pentru x ,= y avem
F

x
(x, y) =
f

(x)(x y) f(x) +f(y)


(x y)
2
.
Fie a R. Avem
F

x
(a, a) = lim
xa
F(x, a) F(a, a)
x a
116
= lim
xa
f(x) f(a) (x a)f

(a)
(x a)
2
H
= lim
xa
f

(x) f

(a)
2(x a)
=
1
2
f

(a).
Aratam ca F

x
este continua n (a, a) ceea ce revine la a arata ca
lim
(x,y)(a,a)
F

x
(x, y) =
1
2
f

(a).
Din formula lui Taylor aplicata funct iei f obt inem
f(y) f(x) = (y x)f

(x) +
(y x)
2
2
f

(x +(y x))
cu 0 < < 1, prin urmare
F

x
(x, y) =
(y x)
2
f

(x +(y x))
2(x y)
2
=
1
2
f

(x +(y x))
si de aici rezulta imediat ca
lim
(x,y)(a,a)
F

x
(x, y) =
1
2
f

(a).
Demonstrat ia este analoaga pentru existent a si continuitatea lui F

y
.
Problema 10.28 Se considera funct ia f : R
2
R de clasa C
1
care satisface relat ia
f
x
(x, y) +
f
y
(x, y) = f(x, y), (x, y) R.
1. Sa se arate ca funct ia g : R
2
R denita prin g(x, y) = f(x, x +y) satisface relat ia
g
x
(x, y) = g(x, y), (x, y) R
2
.
2. Sa se arate ca exista o funct ie : R R de clasa C
1
astfel ca
f(x, y) = (y x)e
x
, (x, y) R
2
.
Solut ie. 1. Fie x = u, x +y = v. Avem
g
x
(x, y) =
f
u
(u, v) +
f
v
(u, v) = f(u, v) = g(x, y).
2. Relat ia
g
x
(x, y) g(x, y) = 0, (x, y) R
2
este echivalenta cu

x
(g(x, y)e
x
) = 0
de unde rezulta ca
g(x, y)e
x
= (y) g(x, y) = (y)e
x
sau, t inand seama de denit ia lui g,
f(x, y) = (y x)e
x
, (x, y) R
2
unde C
1
(R).
Funct ii de mai multe variabile reale 117
Problema 10.29 Fie D R
n
o mult ime deschisa si f : D R
m
o funct ie diferent iabila
cu proprietatea ca exista M > 0 astfel ca
|f(x) f(y)| M|x y|, x, y D.
Sa se arate ca |df(x)| M, x A.
Solut ie. Fie a D arbitrar si T = df(a). Demonstram ca |T| M. Din
diferent iabilitatea lui f n a rezult a ca exista o funct ie : D R
n
astfel ca
f(x) f(a) = T(x a) +|x a|(x)
cu lim
xa
(x) = 0. Fie s R
m
, |s| = 1. Cum D este deschisa rezulta ca exista > 0 astfel
ca
a +ts D, t (0, ).
Avem
f(a +ts) f(a) = tT(s) +t(a +ts), t (0, ).
Obt inem:
t|T(s)| = |f(a +ts) f(a) t(a +ts)|
|f(a +ts) f(a)| +t|(a +ts)|
t(M +|(a +ts)|).
Facand acum t 0 n relat ia
|T(s)| M +|(a +ts)|
rezulta ca |T| M.
Problema 10.30 Fie B = x R
n
[ |x| < 1 bila unitate din R
n
si e f : B B o
funct ie continua cu proprietatea |f(x)| < |x| pentru orice x B, x ,= 0.
Sa se demonstreze ca sirul (x
m
)
m0
denit prin relat ia
x
m+1
= f(x
m
), m 0, x
0
B, x
0
,= 0
n
,
are limita 0
n
. (| | noteaza norma euclidiana din R
n
, iar 0
n
vectorul nul din R
n
).
Berkeley, 1991
Solut ie. Din continuitata lui f si din relat ia |f(x)| < |x|, x B, x ,= 0
n
, rezulta ca
f(0
n
) = 0
n
. Daca exista k N astfel ca x
k
= 0
n
rezulta ca x
m
= 0
n
pentru orice m k,
deci lim
m
x
m
= 0
n
. Sa presupunem ca x
k
,= 0
n
pentru orice k N.
Fie (t
m
)
m0
sirul de numere reale dat prin t
m
= |x
m
|, m 0. Din relat ia |f(x
m
)| <
|x
m
| rezulta ca (t
m
)
m0
este strict descrescator si ind marginit inferior de 0 este con-
vergent.
Fie lim
m
t
m
= t. Vom arata c a t = 0. Sa presupunem ca t > 0. Sirul (x
m
)
m0
ind
marginit are un subsir convergent (x
m
j
)
j0
,
lim
j
x
m
j
= x, x B.
Avem |x| = lim
j
|x
m
j
| = t, deci |f(x)| < t. Din continuitatea lui f obt inem
f(x) = lim
j
f(x
m
j
) = lim
j
x
m
j
+1
si |x
m
j
+1
| t pentru orice j N, contradict ie.
118
Problema 10.31 Sa se demonstreze inegalitatea
[(x +y)e
x
2
y
2
[
1

e
, (x, y) R
2
.
Solut ia 1. Trecand la coordonate polare
x = r cos t, y = r sin t, r 0, t [0, 2],
inegalitatea devine:
[r(cos t + sin t)e
r
2
[
1

e
sau echivalent

2 cos
_
t

4
_
e
r
2

e
.
Consideram funct ia f(r) = re
r
2
, r [0, ). Avem
f

(r) = e
r
2
(1 2r
2
).
Radacina ecuat iei f

(r) = 0 este r =

2
2
. Cum lim
r
f(r) = f(0) = 0 rezulta ca
0 f(r) f
_

2
2
_
=

2
2
e

1
2
.
T inand seama ca

2 cos
_
t

4
_

2, t [0, 2]
rezulta inegalitatea din enunt .
Egalitatea are loc pentru x = y =
1
2
si x = y =
1
2
.
Solut ia 2. Determinam extremele globale ale funct iei
f : R
2
R, f(x, y) = (x +y)e
x
2
y
2
.
Consideram compactul D(0, r) = (x, y) R
2
[ x
2
+ y
2
r
2
, r > 1 si determinam
extremele lui f pe D(0, r). Avem
f

x
(x, y) = e
x
2
y
2
(1 2x(x +y)), f

y
(x, y) = e
x
2
y
2
(1 2y(x +y)).
Punctele stat ionare ale lui f sunt
_
1
2
,
1
2
_
,
_

1
2
,
1
2
_
care sunt puncte de extrem
local
f
_
1
2
,
1
2
_
=
1

e
, f
_

1
2
,
1
2
_
=
1

e
.
Determinam extremele pe frontiera lui D(0, r) deci pentru x
2
+y
2
= r
2
. Avem
f(x, y) = (x +y)e
r
2
.
Fie
L(x, y; ) = (x +y)e
r
2
(x
2
+y
2
r
2
), R.
Funct ii de mai multe variabile reale 119
Punctele stat ionare condit ionate se obt in din sistemul
L

x
= 0, L

y
= 0, x
2
+y
2
= 0.
Rezulta
x = y =
r

2
2
.
Cum
f
_
r

2
2
,
r

2
2
_
= r

2e
r
2

e
rezulta ca
max
R
2
[f(x, y)[ =
1

e
.
Problema 10.32 Sa se arate ca pentru orice x, y [0, ) are loc inegalitatea
x
2
+y
2
4
e
x+y2
.
Berkeley, 1993
Solut ie. Fie x = r cos t, y = r sin t, r 0, t
_
0,

2
_
. Inegalitatea este echivalenta cu
r
2
4
e
r(cos t+sin t)2
.
Fie n continuare r xat. Avem
e
r(cos t+sin t)2
= e
r

2 sin(t+

4
)2
e
r2
.
Inegalitatea are loc daca aratam ca e
r2

r
2
4
.
Fie f : [0, ) R, f(r) = e
r2

r
2
4
. Avem
f

(r) = e
r2

r
2
si f

(r) = e
r2

1
2
.
Ecuat ia f

(r) = 0 are solut ia unica r


0
= 2 ln 2. Cum f

este strict descrescatoare pe


[0, r
0
] si strict crescatoare pe [r
0
, +),
f

(0) = e
2
> 0, f

(r
0
) =
ln 2 1
2
< 0, lim
r
f

(r) = +
rezulta ca ecuat ia f

(r) = 0 are doua radacini r


1
[0, r
0
) si r
2
= 2 (r
0
, +). T inand
seama de semnul lui f

rezulta ca min f(r) = 0 = f(2).



In inegalitatea init iala egalitatea
are loc n punctele (0, 2), (2, 0).
Problema 10.33 Fie f : R
2
R o funct ie de clasa C
2
cu proprietatea ca
f

x
2
(x, y) +f

y
2
(x, y) > 0, (x, y) R
2
.
Sa se arate ca f nu admite maxime locale.
Berkeley, 1998
120
Solut ie. Sa presupunem ca f admite un maxim local n punctul (x
0
, y
0
) R
2
. Atunci
d
2
f(x
0
, y
0
) este negativ semidenita, adica
f

x
2
(x
0
, y
0
)u
2
+ 2f

xy
(x
0
, y
0
)uv +f

y
2
(x
0
, y
0
)v
2
0
pentru orice (u, v) R
2
. Pentru (u, v) = (1, 0) obt inem f

x
2
(x
0
, y
0
) 0, iar pentru (u, v) =
(0, 1) obt inem f

y
2
(x
0
, y
0
) 0, deci
f

x
2
(x
0
, y
0
) +f

y
2
(x
0
, y
0
) 0
contradict ie.
Problema 10.34 Fie a, b, c, d, e, f, g R cu proprietatea b
2
4ac < 0 si e M mult imea
perechilor (x, y) R
2
cu proprietatea
ax
2
+bxy +cy
2
+dx
3
+ex
2
y +fxy
2
+gy
3
= 0.
Demonstrat i ca exista un num ar r > 0 cu proprietatea ca n mult imea
D = (x, y) R
2
[ 0 < x
2
+y
2
< r
2

nu exista nici un punct din M.


Putnam, 1970
Solut ie. Evident M ,= deoarece (0, 0) M.
Fie F(x, y) = ax
2
+bxy +cy
2
+dx
3
+ex
2
y +fxy
2
+gy
3
pentru (x, y) R
2
. Vom arata
ca F admite un extrem local n punctul (0, 0). Avem
F

x
(x, y) = 2ax +by + 3dx
2
+ 2exy +fy
2
F

y
(x, y) = bx + 2cy +ex
2
+ 2fxy + 3gy
2
.
Evident (0, 0) este punct stat ionar al lui F.
Pe de alta parte f

x
2
(0, 0) = 2a, f

xy
(0, 0) = b si f

y
2
(0, 0) = 2c, deci:
d
2
f(0, 0)(h, k) = 2ah
2
+ 2bhk + 2ck
2
, (h, k) R
2
.
Condit ia b
2
4ac < 0 implica si a ,= 0, deci (0, 0) este punct de extrem local al lui F.
Atunci exista o bila B(0, r) astfel ca
F(x, y) > 0 sau F(x, y) < 0 n B(a, r) (0, 0).

In concluzie D = B(a, r) (0, 0).


Problema 10.35 Fie B = (x, y) R
2
[ x
2
+ y
2
1 si f : B R o funct ie de clasa
C
1
cu proprietatea [f(x, y)[ 1 pentru orice (x, y) B. Sa se arate ca exista un punct
(x
0
, y
0
) interior lui B astfel ca
_
f
x
(x
0
, y
0
)
_
2
+
_
f
y
(x
0
, y
0
)
_
2
16.
Putnam, 1967
Funct ii de mai multe variabile reale 121
Solut ie. Fie g : B R, g(x, y) = f(x, y) + 2(x
2
+y
2
) pentru orice (x, y) B. Pentru
(x, y) cu proprietatea x
2
+ y
2
= 1 avem g(x, y) 1 iar g(0, 0) = f(0, 0) 1. Cum g
este continua pe compactul B ea este marginita si si atinge marginile pe B. Atunci g
este constanta sau si atinge minimul ntr-un punct interior (x
0
, y
0
) a lui B. Daca g este
constanta atunci evident
f(x, y) = 1 2(x
2
+y
2
)
si
_
f
x
(x, y)
_
2
+
_
f
y
(x, y)
_
2
= 16(x
2
+y
2
) 16.
Daca g si atinge minimul n (x
0
, y
0
) atunci
g
x
(x
0
, y
0
) =
g
y
(x
0
, y
0
) = 0
iar
f
x
(x
0
, y
0
) =
g
x
(x
0
, y
0
) 4x
0
= 4x
0
f
y
(x
0
, y
0
) =
g
y
(x
0
, y
0
) 4y
0
= 4y
0
.
Obt inem
_
f
x
(x
0
, y
0
)
_
2
+
_
f
y
(x
0
, y
0
)
_
2
= 4(x
2
0
+y
2
0
) < 16.
Capitolul 11
Siruri si serii de funct ii: serii
Taylor, serii Fourier
Denit ii si rezultate
Siruri de funct ii

In cele ce urmeaza (X, d) va desemna un spat iu metric.


Denit ie. Fie A (X, d) si T(A) := f [ f : A R. Se numeste sir de funct ii din
T(A) o aplicat ie g : N T(A), g(n) = f
n
.
Notat ie: (f
n
)
nN
sau (f
n
).
Daca (f
n
) este un sir de funct ii si x A, atunci (f
n
(x))
nN
formeaza un sir nu-
meric. Daca sirul (f
n
(x)) este convergent vom spune ca x este un punct de convergent a al
sirului (f
n
). Totalitatea punctelor de convergent a ale sirului (f
n
) formeaza mult imea de
convergent a a sirului (f
n
).
Denit ie. Fie A (X, d) si f
n
, f : A R. Vom spune ca sirul de funct ii (f
n
) converge
punctual sau simplu la f pe mult imea A daca sirul numeric (f
n
(x)) converge la f(x)
pentru orice x A. Vom nota acest lucru prin f
n
s

A
f sau f
n
s
f.
Folosind scrierea analitica a convergent ei unui sir numeric, putem da o formulare
echivalenta. Asadar, sirul de funct ii (f
n
) converge punctual la f pe mult imea A daca
x A, > 0, n
x,
N, n n
x,
: [f
n
(x) f(x)[ < .

In general, convergent a punctuala nu conserva proprietat ile sirului de funct ii. Acesta
este principalul motiv pentru introducerea conceptului de convergent a uniforma.
Denit ie. Fie A (X, d) si f
n
, f : A R. Vom spune ca sirul de funct ii (f
n
) converge
uniform la f pe mult imea A daca
> 0, n

N, n n

, x A : [f
n
(x) f(x)[ < .
Vom nota acest lucru prin f
n
u

A
f sau f
n
u
f.
Se observa cu usurint a faptul c a daca un sir de funct ii (f
n
) converge uniform la f pe
o mult ime A, atunci (f
n
) converge punctual la f pe A.
Prezentam n continuare criterii care asigura convergent a uniforma.
Teorema. (criteriul cu supremum) Fie A (X, d) si f
n
, f : A R. Sirul (f
n
) converge
uniform la f daca si numai daca
lim
n
_
sup
xA
[f
n
(x) f(x)[
_
= 0.
122
Siruri si serii de funct ii: serii Taylor, serii Fourier 123
Un alt criteriu, de tip Cauchy, este prezentat n urmatoarea teorema.
Teorema. (criteriul Cauchy) Fie A (X, d) si f
n
, f : A R. Sirul (f
n
) converge
uniform la f daca si numai daca
> 0, n

N, n, m n

, x A : [f
n
(x) f
m
(x)[ < . (11.1)
Daca un sir (f
n
) satisface condit ia (11.1), se va numi sir uniform fundamental sau
uniform Cauchy.
Un alt criteriu l reprezinta cel al majorarii.
Teorema. (criteriul majorarii) Fie A (X, d) si f
n
, f : A R. Daca exista un sir de
numere pozitive (a
n
) convergent la 0, astfel nc at
[f
n
(x) f(x)[ < a
n
, n N, x A,
atunci f
n
u

A
f.
Desi, dupa cum se poate observa din exemple simple, convergent a punctuala nu antre-
neaza convergent a uniforma, n anumite condit ii speciale acest lucru are loc.
Teorema. (prima teorema a lui Dini) Fie A (X, d) o mult ime compacta. Daca
(f
n
) este un sir de funct ii continue pe A astfel ncat, pentru orice x A, sirul numeric
(f
n
(x)) este (des)cresc ator, iar (f
n
) converge punctual pe A la o funct ie continua f, atunci
f
n
u

A
f.
Teorema. (a doua teorema a lui Dini) Fie f
n
: [a, b] R un sir de funct ii
(des)crescatoare pe [a, b] astfel ncat (f
n
) converge punctual pe [a, b] la o funct ie continu a
f, atunci f
n
u

[a,b]
f.

In continuare vom prezenta principalele proprietat i ale sirurilor uniform convergente,


legate, dupa cum spuneam mai sus, de transferul unor proprietat i ale termenilor sirului
catre funct ia limita.
Teorema. (transfer de marginire) Fie A (X, d) si f
n
: A R. Daca f
n
este
marginita pe A pentru orice n N si f
n
u

A
f, atunci f este marginita pe A.
Teorema. (transfer de existent a a limitei ntr-un punct) Fie A (X, d) si f
n
: A R
astfel ncat f
n
u

A
f. Daca x este punct de acumulare pentru A si lim
xx
f
n
(x) pentru
ecare n N, atunci lim
xx
f(x) si, n plus,
lim
xx
f(x) = lim
n
_
lim
xx
f
n
(x)
_
.
Teorema. (transfer de continuitate) Fie A (X, d) si f
n
: A R astfel ncat f
n
u

A
f.
Daca funct iile f
n
sunt continue n x A (respectiv pe A), atunci f este continua n x
(respectiv pe A).
Teorema. (transfer de integrabilitate) Fie f
n
: [a, b] R integrabile Riemann pe [a, b]
astfel nc at f
n
u

[a,b]
f. Atunci f este integrabila Riemann pe [a, b] si
lim
n
b
_
a
f
n
(x)dx =
b
_
a
f(x)dx.
124
Teorema. (transfer de derivabilitate) Fie I un interval marginit din R si f
n
: I R un
sir de funct ii derivabile pe I. Daca exista x I astfel ncat sirul (f
n
(x)) sa e convergent
si exista g : I R astfel nc at f

n
u

I
g, atunci:
(i) exista o funct ie f : I R astfel nc at f
n
u

I
f,
(ii) f este derivabil a pe I, iar derivata sa este g, adica
_
lim
n
f
n
_

= lim
n
f

n
.
Observat ie. Remarcam faptul ca uniforma convergent a ofera condit ii suciente
pentru transferul unor proprietat i, nu nsa si necesare. Exemplu: f
n
: [0, 1] R,
f
n
(x) =
nx
1 +n
2
x
2
. Atunci f
n
s

[0,1]
0, nsa f
n
u
,
[0,1]
0. Totusi, atat f
n
, cat si f, sunt
continue pe [0, 1] (deci marginite, integrabile). De asemenea, e sirul g
n
: [0, 1] R,
g
n
(x) =
ln(1 +n
4
x
2
)
2n
. Se arata ca g
n
s

[0,1]
0, g

n
s

[0,1]
0, adica (g
n
) se poate deriva termen cu
termen. Totusi, g

n
u
,
[0,1]
0.
Serii de funct ii
Denit ie. Fie A (X, d) si f
n
: A R un sir de funct ii. Spunem ca seria

n=1
f
n
este
convergenta punctual pe mult imea A daca sirul sumelor part iale (S
n
), cu S
n
:=
n

k=1
f
k
,
este convergent punctual pe mult imea A. Daca S
n
s

A
f, atunci f se numeste suma seriei
de funct ii

n=1
f
n
n sensul convergent ei punctuale si vom nota acest lucru prin

n=1
f
n
s
=
A
f sau

n=1
f
n
s
= f.
Daca seria

n=1
[f
n
[ este convergenta punctual pe mult imea A, spunem ca seria

n=1
f
n
este absolut convergenta pe A.
Mult imea tuturor punctelor din A n care seria

n=1
f
n
converge punctual se numeste
mult imea de convergent a a seriei

n=1
f
n
.
Denit ie. Fie A (X, d) s i f
n
: A R un sir de funct ii. Spunem ca seria

n=1
f
n
este
convergent a uniform pe mult imea A daca sirul sumelor part iale (S
n
) este convergent
uniform pe mult imea A. Daca S
n
u

A
f, atunci f se numeste suma seriei de funct ii

n=1
f
n
n sensul convergent ei uniforme si vom nota acest lucru prin

n=1
f
n
u
=
A
f sau

n=1
f
n
u
= f.
Prezentam n continuare criterii care asigura convergent a uniforma a unei serii de
funct ii.
Siruri si serii de funct ii: serii Taylor, serii Fourier 125
Teorema. (criteriul lui Cauchy) Fie A (X, d) si f
n
: A R. Seria de funct ii

n=1
f
n
converge uniform pe A daca si numai daca
> 0, n

N, n n

, p N, x A : [f
n+1
(x) + +f
n+p
(x)[ < .
Teorema. (criteriul lui Weierstrass) Fie A (X, d) si f
n
: A R. Daca exista o serie
numerica convergenta cu termeni pozitivi

n=1
a
n
astfel ncat
[f
n
(x)[ a
n
, n N

, x A,
atunci seria

n=1
f
n
este uniform si absolut convergenta pe A.
Vom da n continuare doua criterii de convergent a uniforma neabsoluta. Prezentam
mai ntai denit ia uniformei marginiri a unui sir de funct ii.
Denit ie. Fie A (X, d) si f
n
: A R. Sirul (f
n
) se numeste uniform marginit (pe
A) daca exista M R astfel ncat
[f
n
(x)[ M, n N

, x A.
Teorema. (criteriul lui Abel) Fie A (X, d) si f
n
, g
n
: A R. Dac a seria

n=1
f
n
este
uniform convergenta pe A iar sirul (g
n
) este uniform marginit si monoton pentru ecare
x A, atunci seria

n=1
f
n
g
n
este uniform convergenta pe A.
Teorema. (criteriul lui Dirichlet) Fie A (X, d) si f
n
, g
n
: A R. Daca seria

n=1
f
n
are sirul sumelor part iale uniform marginit iar sirul (g
n
) este uniform descrescator (pentru
orice x A) si convergent uniform la 0, atunci seria

n=1
f
n
g
n
este uniform convergenta
pe A.
O aplicat ie directa a criteriului lui Dirichlet o reprezinta criteriul lui Leibniz.
Teorema. (criteriul lui Leibniz) Fie A (X, d) si f
n
: A R. Daca sirul (f
n
)
este uniform descrescator (pentru orice x A) si convergent uniform la 0, atunci seria

n=1
(1)
n
f
n
este uniform convergenta pe A.
De asemenea, drept consecint a a primei teoreme a lui Dini de la siruri de funct ii,
obt inem urmatorul rezultat:
Teorema. (criteriul lui Dini) Fie A (X, d) o mult ime compacta si f
n
: A R un sir
de funct ii continue cu proprietatea ca f
n
0 pe A. Daca

n=1
f
n
converge punctual pe A
la funct ia continua f, atunci

n=1
f
n
u
=
A
f.

In continuare, vom prezenta cele mai importante proprietat i ale seriilor uniform con-
vergente.
Teorema. (transfer de marginire) Fie A (X, d) si f
n
: A R. Daca

n=1
f
n
u
=
A
f si
f
n
sunt funct ii marginite pe A, atunci f este marginit a pe A.
126
Teorema. (transfer de existent a a limitei ntr-un punct) Fie A (X, d) si f
n
, f : A R
astfel ncat

n=1
f
n
u
=
A
f. Daca x este un punct de acumulare al mult imii A si lim
xx
f
n
(x)
pentru orice n N

, atunci lim
xx
f(x) si, n plus,
lim
xx
f(x) =

n=1
_
lim
xx
f
n
(x)
_
.
Teorema. (transfer de continuitate) Fie A (X, d) si f
n
, f : A R astfel ncat

n=1
f
n
u
=
A
f. Daca funct iile f
n
sunt continue ntr-un punct x A (respectiv pe A), atunci
f este continua n x (respectiv pe A).
Teorema. (transfer de integrabilitate) Fie f
n
: [a, b] R integrabile Riemann pe [a, b]
astfel nc at

n=1
f
n
u
=
[a,b]
f. Atunci f este integrabila Riemann pe [a, b] si

n=1
_
_
b
_
a
f
n
(x)dx
_
_
=
b
_
a
f(x)dx.
Teorema. (transfer de derivabilitate) Fie I un interval marginit din R si f
n
: I R
un sir de funct ii derivabile pe I. Daca exista x I astfel nc at seria

n=1
f
n
(x) sa e
convergenta si exista g : I R astfel nc at

n=1
f

n
u
=
I
g, atunci:
(i) exista o funct ie f : I R astfel nc at

n=1
f
n
u
=
I
f,
(ii) f este derivabil a pe I, iar derivata sa este g, adica
_

n=1
f
n
_

n=1
f

n
.
Serii de puteri
Denit ie. Numim serie de puteri o serie de forma
a
0
+a
1
x +a
2
x
2
+ +a
n
x
n
+ ,
unde (a
n
) este un sir numeric. Numerele a
n
se numesc coecient ii seriei.
Seriile de puteri sunt de fapt serii de funct ii, funct iile avand o forma particulara:
f
n
(x) = a
n
x
n
. Este de asteptat ca seriile de puteri sa aiba proprietat i speciale, avand n
vedere proprietat ile funct iilor polinomiale. Observam mai ntai ca orice serie de puteri are
ca punct de convergent a originea.
Teorema. (prima teorema a lui Abel) Daca o serie de puteri converge ntr-un punct
x
0
,= 0, atunci seria este absolut convergenta n orice punct x cu [x[ < [x
0
[. Daca o serie
de puteri diverge ntr-un punct x
1
, atunci seria diverge n orice punct x cu [x[ [x
1
[.
Observat ie. Analizand teorema anterioara observam ca exista:
Serii convergente doar n origine:

n=0
n!x
n
;
Siruri si serii de funct ii: serii Taylor, serii Fourier 127
Serii convergente pentru orice x real:

n=0
1
n!
x
n
;
Serii de puteri pentru care exista r > 0 astfel ncat seria converge n orice x cu
[x[ < r si diverge pentru orice x cu [x[ > r. Acest r este de fapt
r = sup
_
[x[ :

n=0
a
n
x
n
<
_
si se numeste raza de convergent a a seriei.
Teorema. Fie seria

n=0
a
n
x
n
si = limsup
n
n
_
[a
n
[. Atunci raza de convergent a a seriei
este r =
1

(cu convent iile


1

= 0,
1
0
= ).
Corolar. Fie seria

n=0
a
n
x
n
. Daca exista lim
n
n
_
[a
n
[ = , atunci r =
1

este raza de
convergent a a seriei. Daca exista lim
n

a
n+1
a
n

= , atunci r =
1

este raza de convergent a


a seriei.
Prezentam n continuare rezultate privind convergent a uniforma a seriilor de puteri.
Teorema. Fie seria de puteri

n=0
a
n
x
n
avand raza de convergent a r > 0. Atunci seria
este uniform convergenta pe orice interval [, ] (r, r).
Teorema. Funct ia suma a unei serii de puteri avand raza de convergent a r > 0 este
continua pe (r, r).
Teorema. (a doua teorema a lui Abel) Fie seria de puteri

n=0
a
n
x
n
avand raza de
convergent a r > 0. Daca seria converge n r sau r, atunci suma sa este continua n r,
respectiv r.
Teorema. Seriile de puteri

n=0
a
n
x
n
,

n=0
(n+1)a
n+1
x
n
,

n=0
a
n
n + 1
x
n+1
au aceeasi raz a
de convergent a.
Teorema. Daca seria

n=0
a
n
x
n
are raza de convergent a r > 0 si suma f pe (r, r),
atunci
x
_
0
f(t)dt =

n=0
a
n
n + 1
x
n+1
, pentru [x[ < r,
adica orice serie de puteri poate integrata termen cu termen pe orice interval [0, x], unde
x (r, r).
Teorema. Daca seria de puteri

n=0
a
n
x
n
are raza de convergent a r > 0 si suma f pe
(r, r), atunci f este derivabila pe (r, r) si
f

(x) = a
1
+ 2a
2
x + 3a
3
x
2
+ +na
n
x
n1
+. . . pentru orice x (r, r),
adica seria de puteri poate derivata termen cu termen pe intervalul deschis de
convergent a.
128
Mai mult, f este de clasa C

pe (r, r),
f
(k)
(x) =

n=k
n(n 1) . . . (n k + 1)a
n
x
nk
, pentru orice x (r, r)
si f
(k)
(0) = k!a
k
, k = 0, 1, 2 . . .
Cu acest rezultat se poate demonstra unicitatea seriilor de puteri:
Teorema. Daca seriile de puteri

n=0
a
n
x
n
si

n=0
b
n
x
n
au aceeasi raza de convergent a
si aceeasi suma pe intervalul de convergent a, atunci a
n
= b
n
, n N.
Prezentam n continuare un rezultat legat de operat ii cu serii de puteri. Consideram
doua serii de puteri

n=0
a
n
x
n
si

n=0
b
n
x
n
si numarul real R. Se pot construi urmatoarele
serii:
seria suma: (a
0
+b
0
) + (a
1
+b
1
)x + + (a
n
+b
n
)x
n
+ ;
seria produs a unei serii cu un scalar: a
0
+a
1
x + +a
n
x
n
+ ;
produsul Cauchy a doua serii:

n=0
c
n
x
n
, unde c
n
=
n

k=0
a
k
b
nk
, n N.
Teorema. Fie doua serii de puteri

n=0
a
n
x
n
,

n=0
b
n
x
n
avand razele de convergent a r
1
,
respectiv r
2
si numarul real R. Atunci:
(i) Raza de convergent a a seriei suma este cel put in egala cu minr
1
, r
2
si

n=0
(a
n
+b
n
)x
n
=

n=0
a
n
x
n
+

n=0
b
n
x
n
.
(ii) Raza de convergent a a seriei

n=0
(a
n
)x
n
este egala cu cea a seriei

n=0
a
n
x
n
pentru
orice ,= 0 si

n=0
(a
n
)x
n
=

n=0
a
n
x
n
.
(iii) Seria produs Cauchy are raza de convergent a r minr
1
, r
2
.

In plus, suma
seriei produs este egala cu produsul sumelor celor doua serii pe intersect ia mult imilor lor
de convergent a.
S-a observat mai sus ca, avand data o serie de puteri, se pot obt ine unele informat ii
privind continuitatea, derivabilitatea sau integrabilitatea sumei sale.

In practica problema
se pune de multe ori invers: avand o funct ie pe un interval, n ce condit ii poate scrisa
aceasta ca suma unei serii de puteri pe acel interval, sau pe o submult ime a sa? Conform
celor aratate, suma f a unei serii de puteri poate scrisa sub forma
f(x) =

n=0
f
(n)
(0)
n!
x
n
.
Vom prezenta n continuare un exemplu de serie de puteri cu important a n practica.
Fie R. Seria de puteri
1 +

1!
x +
( 1)
2!
x
2
+ +
( 1) . . . ( n + 1)
n!
x
n
+
Siruri si serii de funct ii: serii Taylor, serii Fourier 129
se numeste serie binomiala.
Suma acestei serii este f(x) = (1 +x)

. Atribuind diferite valori particulare pentru


se obt in sumele unor importante serii de puteri.
1. Pentru = 1 se obt in sumele seriilor geometrice
1
1 +x
= 1 x +x
2
x
3
+ + (1)
n
x
n
+ x (1, 1)
1
1 x
= 1 +x +x
2
+x
3
+ +x
n
+ x (1, 1)
2. Pentru =
1
2
avem, x (1, 1)

1 +x = 1 +
1
2 1!
x
1
2
2
2!
x
2
+ + (1)
n1
(2n 3)!!
2
n
n!
x
n
+
3. Pentru =
1
2
obt inem, x (1, 1)
1

1 +x
= 1
1
2 1!
x +
1 3
2
2
2!
x
2
+ + (1)
n1
(2n 1)!!
2
n
n!
x
n
+
4. Trecand n prima serie pe x n x
2
obt inem, x (1, 1)
1
1 +x
2
= 1 x
2
+x
4
x
6
+ + (1)
n
x
2n
+
5. Prin integrare termen cu termen a seriei de mai sus avem, x (1, 1)
arctg x = x
x
3
3
+
x
5
5

x
7
7
+ + (1)
n
x
2n+1
2n + 1
+
6. Trecand pe x n x
2
n seria 3 si integrand obt inem, x (1, 1)
ln(x +
_
1 +x
2
) = x
x
3
3 2 1!
+ + (1)
n
(2n 1)!!
(2n)!!
x
2n+1
+
7.

In aceeasi serie 3, trecand pe x n x
2
si integrand apoi termen cu termen obt inem,
x (1, 1)
arcsin x = x +
x
3
3 2
+
1 3
5 4 2
x
5
+ +
(2n 1)!!
(2n + 1)(2n)!!
x
2n+1
+
Probleme
Problema 11.1 Sa se studieze convergent a simpla si uniforma a seriilor de funct ii pe
mult imile indicate:
(i)

n=1
_

2
arctg(n
2
(1 +x
2
))
_
, x R.
(ii)

n=1
ln(1 +nx)
nx
n
, x 2.
(iii)

n=1
n
2
x
2
e
n
2
|x|
, x R.
130
Solut ie. (i) Din identitatea arctg x + arctg
1
x
=

2
, rezulta:

2
arctg(n
2
(1 +x
2
))

= arctg
1
n
2
(1 +x
2
)

1
n
2
(1 +x
2
)

1
n
2
, x R.
Seria

n=1
1
n
2
este convergenta, deci n baza criteriului lui Weierstrass, seria data este uniform
convergenta pe R.
(ii) si (iii) Analog, cu acelasi criteriu:

ln(1 +nx)
nx
n

1
x
n1

1
2
n1
, x 2
sup
xR

n
2
x
2
e
n
2
|x|

=
4
n
2
e
2
.
Problema 11.2 Sa se studieze convergent a simpla si uniforma pe R a seriilor de funct ii:

n=1
(1)
n1
x
2
+n
2
si

n=1
(1)
n1
x
2
(1 +x
2
)
n
.
Ce se poate spune despre absolut convergent a lor?
Solut ie. Ambele serii converg simplu (pentru x R xat se aplica criteriul lui Leib-
niz). Pentru a demonstra convergent a uniforma, demonstram ca resturile celor doua serii
converg uniform la 0. Fie R
n
(x)si respectiv T
n
(x) resturile de ordin n ale celor doua serii.
Atunci:
[R
n
(x)[
1
x
2
+n + 1

1
n + 1
, x R,
[T
n
(x)[
x
2
(1 +x
2
)
n+1
=
x
2
1 +C
1
n+1
x
2
+C
2
n+1
x
4
+ +x
2n+2

1
n + 1
, x ,= 0.
Evident T
n
(0) = 0, deci seriile sunt uniform convergente.

In ceea ce priveste convergent a absoluta, prima serie nu converge nici simplu: n x = 0


se obt ine seria armonica. Seria

n=1
x
2
(1 +x
2
)
n
este punctual convergenta (serie geometrica
cu rat ie subunitara), dar nu este uniform convergenta; daca notam restul de ordinul n cu
P
n
, pentru x ,= 0, avem:
P
n
(x) =
x
2
(1 +x
2
)
n+1
+
x
2
(1 +x
2
)
n+2
+
=
x
2
(1 +x
2
)
n+1

1 +x
2
x
2
=
1
(1 +x
2
)
n
1 pentru x 0.
Problema 11.3 Fie sirul u
n
: [0, ) R, u
n
(x) =
ne
x
+xe
n
x +n
, n N si e A
n
=
1
_
0
u
n
(x)dx, n N

. Sa se studieze convergent a sirului u


n
si sa se calculeze lim
n
A
n
.
Siruri si serii de funct ii: serii Taylor, serii Fourier 131
Solut ie. Fie x 0, xat.
lim
n
u
n
(x) = lim
n
ne
x
+xe
n
x +n
= e
x
,
deci u
n
converge punctual la f(x) = e
x
, x 0.
Evaluam n continuare
g
n
(x) = [u
n
(x) f(x)[ =
_
1
n
n +x
_
[e
n
e
x
[, x > 0, n N

.
Daca x (n, ) atunci:
e
x
< e
n
[e
n
e
x
[ < e
n
g
n
(x) < e
n
0.
Daca x [0, n] atunci
e
x
e
n
[e
n
e
x
[ [e
n
[ +[e
x
[ =
= e
n
+e
x
2e
x
,
deci
g(x)
2xe
x
x +n
, x [0, n].
Studiem acum variat ia funct iei [0, 1] x
2xe
x
x +n
.
_
2xe
x
x +n
_

= 2
e
x
(x +n)
2
(x
2
nx +n),
deci funct ia
2xe
x
x +n
este crescatoare pe
_
0,

n
2
+4nn
2
_
si descrescatoare pe
_

n
2
+4nn
2
, n
_
,
deci
g
n
(x)
8n
(

n
2
+ 4n +n)
2
e

2n

n
2
+4n+n
0.
Deci u
n
converge uniform la f.
Sirul se poate integra termen cu termen:
lim
n
A
n
=
1
_
0
f(x)dx =
1
_
0
e
x
dx =
e 1
e
.
Problema 11.4 Fie f
n
(x) =
n1

i=0
1
n
f
_
x +
i
n
_
, unde f este o funct ie continua pe R. Sa se
arate ca (f
n
) converge uniform pe orice interval compact din R.
Solut ie. Fie [a, b] R si x [a, b]. Evaluam diferent a
f
n
(x)
x+1
_
x
f(t)dt =
n1

i=0
1
n
f
_
x +
i
n
_

n1

i=0
x+
i+1
n
_
x+
i
n
f(t)dt.
132
Cum funct ia f este continua pe R, aplicam o teorema de medie pe ecare subinterval
de forma
_
x +
i
n
, x +
i+1
n

, i = 0, n 1 pentru a gasi c = c
x,i,n

_
x +
i
n
, x +
i+1
n
_
astfel
ncat
x+
i+1
n
_
x+
i
n
f(t)dt =
1
n
f(c
x,i,n
). Atunci

f
n
(x)
x+1
_
x
f(t)dt

n1

i=0
1
n

f
_
x +
i
n
_
f(c
x,i,n
)

.
Cum f este continua pe [a, b], este uniform continua pe acest interval (n baza teoremei lui
Cantor). Atunci, pentru orice > 0 oarecare xat, =

> 0 astfel ncat [f(x) f(x

)[ <
pentru orice x, x

[a, b] cu proprietatea ca [x x

[ < . Fie acum n

N astfel ncat
1
n
< pentru tot i n n

. Atunci, deoarece x +
i
n
, c
x,i,n
[a, b] si

x +
i
n
c
x,i,n

<
1
n
< ,
obt inem ca

f
_
x +
i
n
_
f(c
x,i,n
)

< , de unde

f
n
(x)
x+1
_
x
f(t)dt

n1

i=0
1
n
= .

In concluzie, f
n
u

[a,b]
g, unde g(x) =
x+1
_
x
f(t)dt.
Problema 11.5 Fie sirul f
n
:
_
0,

2

R, denit prin relat ia de recurent a:


f
1
(x) = x, f
n+1
(x) = sin (f
n1
(x)) .
Sa se studieze convergent a punctuala si uniforma.
Solut ie. Fie x
_
0,

2

xat. Sirul f
n
(x) este descrescator:
f
n+1
(x) = sin (f
n
(x)) f
n
(x), n N
si marginit:
0 f
n
(x) x, n N.
Deci sirul f
n
este punctual convergent.
Fie f :
_
0,

2

R, f(x) = lim
n
f
n
(x). Trecand la limita n relat ia de recurent a, se obt ine:
f(x) = sin (f(x)) , x
_
0,

2

. Se demonstreaza simplu ca singura solut ie a ecuat iei


t = sin t este t = 0, deci funct ia f este constanta zero.
Studiem acum convergent a uniforma; pentru aceasta, trebuie calculat sup
x[0,

2
]
[f
n
(x)[.
Demonstram n continuare ca funt ia f
n
este crescatoare pentru orice n N si deci supre-
mumul de mai sus este f
n
(

2
). Sirul f
n
(

2
) converge la zero (deoarece s-a demonstrat mai
sus ca lim
n
f
n
(x) = 0, x
_
0,

2

) si deci f
n
este uniform convergent.
Demonstrat ia faptului ca f
n
sunt funct ii crescatoare se face prin induct ie: f
1
este
crescatoare; presupunem ca f
k
sunt crescatoare pentru orice 1 k n si demonstram ca
f
n+1
este crescatoare. Fie 0 x < y

2
; atunci:
f
n+1
(x) = sin (f
n
(x)) sin (f
n
(y)) = f
n+1
(y),
deci f
n+1
este funct ie crescatoare, ceea ce ncheie demonstrat ia.
Siruri si serii de funct ii: serii Taylor, serii Fourier 133
Problema 11.6 Consideram sirul de funct ii P
n
: [0, 1] R denit prin:
P
0
(x) = 0,
P
n+1
(x) = P
n
(x) +
1
2
(x P
2
n
(x)), daca n N

.
Studiat i convergent a uniforma a sirului (P
n
) pe [0, 1].
Solut ie. Se verica imediat, prin induct ie, ca aplicat iile P
n
sunt polinomiale, deci
continue.

In continuare, vom arata inductiv ca
(A
n
) : 0 P
n
(x)

x, n N, x [0, 1].
Armat ia este evident adevarata pentru n = 0. Presupunem acum ca (A
k
) este adevarata.
Atunci, deoarece x P
2
k
(x) si P
k
(x) 0, obt inem P
k+1
(x) 0, x [0, 1]. De asemenea,
observam ca

x P
k+1
(x) =
_
x P
k
(x)
_
_
1
1
2
(

x +P
k
(x))
_
.
Cum

x +P
k
(x) 2

x 2, deducem ca P
k+1
(x)

x, x [0, 1].
Folosind acum (A
n
), se observa ca pentru orice x [0, 1] xat, sirul (P
n
(x)) este
crescator. Cum, pentru orice x [0, 1], (P
n
(x)) este majorat de

x, obt inem ca (P
n
(x))
este convergent. Notam aceasta limita cu f(x). Atunci, f(x) va satisface pentru orice
x [0, 1] relat ia
f(x) = f(x) +
1
2
(x f
2
(x)). (11.2)
Cum 0 f(x)

x, avem din (11.2) ca f(x) =

x, x [0, 1]. Evident, f este continua.
Aplicand acum prima teorema a lui Dini se obt ine ca P
n
u

[0,1]
f.
Problema 11.7 1) Fie sirul de funct ii f
n
: R
+
R
+
dat prin:
f
n
(x) =
_ _
1
x
n
_
n
, daca x [0, n]
0, daca x > n.
Aratat i ca f
n
u

R
+
f, unde f este funct ia x e
x
.
2) Aratat i ca
I =

_
0
e
t
2
dt = lim
n

n
_
0
_
1
t
2
n
_
n
dt.
Deducet i de aici valoarea lui I.
Solut ie. 1) Se observa ca, pentru orice n N

, funct ia f
n
este continua, descrescatoare
si cu valori pozitive. Fixam acum x R
+
. Gasim atunci n
0
N

astfel ncat n
0
x. Se
observa ca, pentru orice n n
0
,
f
n
(x) =
_
1
x
n
_
n
,
deci sirul (f
n
(x))
nn
0
este convergent la e
x
. Cum sirurile (f
n
(x))
n1
si (f
n
(x))
nn
0
au
aceeasi natura, eventual aceeasi limita n caz de convergent a, deducem ca (f
n
) converge
134
punctual pe R
+
la f. Fie A R

+
xat. Folosind a doua teorema a lui Dini obt inem ca
f
n
u

[0,A]
f. Utilizand inegalitatea
ln(1 +x) x, x > 1, (11.3)
deducem ca
0 f
n
(x) f(x), n N

, x R
+
.
Fixam acum > 0. Exista atunci A > 0 astfel ncat e
x
< , x A. Asadar,
sup
x[A,)
[f(x) f
n
(x)[ = sup
x[A,)
(f(x) f
n
(x))
sup
x[A,)
f(x) . (11.4)
De asemenea, folosind convergent a uniforma a lui (f
n
) pe [0, A] (criteriul cu supremum),
avem ca
lim
n
_
sup
x[0,A]
[f(x) f
n
(x)[
_
= 0,
deci exista n
1
N astfel ncat, pentru orice n n
1
,
sup
x[0,A]
[f(x) f
n
(x)[ < . (11.5)

In concluzie, folosind (11.4) si (11.5), pentru orice > 0, exista n


1
N astfel ncat, pentru
orice n n
1
, sup
xR
+
[f(x) f
n
(x)[ < 2. Deci,
lim
n
_
sup
xR
+
[f(x) f
n
(x)[
_
= 0.
Folosind din nou criteriul cu supremum, f
n
u

R
+
f.
2) Folosind punctul 1), g(x) = e
x
2
este limita uniforma pe R
+
a sirului (g
n
)
nN
de
aplicat ii continue de la R
+
n R
+
denit prin:
g
n
(x) =
_ _
1
x
2
n
_
n
, daca x [0,

n]
0, daca x >

n.
De asemenea, stim ca 0 g
n
(x) g(x), n N

, x R
+
.
Se verica usor convergent a integralelor improprii
I =
+
_
0
g(x)dx si I
n
=
+
_
0
g
n
(x)dx =

n
_
0
_
1
x
2
n
_
n
dx.
Pentru orice A R

+
, putem scrie
0 I I
n

A
_
0
(g(x) g
n
(x)) dx +
+
_
A
g(x)dx. (11.6)
Siruri si serii de funct ii: serii Taylor, serii Fourier 135
Fixam acum > 0. Cum
+
_
0
g(x)dx este convergenta, exista A R

+
astfel ncat
+
_
A
g(x)dx < . De asemenea, folosind convergent a uniforma a sirului (g
n
) pe [0, A],
aplicand transferul de integrabilitate, avem ca
lim
n
A
_
0
(g(x) g
n
(x)) dx = 0.
Asadar, exista n
2
N astfel ncat, pentru orice n n
2
,
A
_
0
(g(x) g
n
(x)) dx .

In
concluzie, deducem din (11.6) ca I
n
I.
Facand schimbarea de variabila
t

n
u, observam ca I
n
=

n

2
_
0
sin
2n+1
udu. Folosind
faptul ca

2
_
0
sin
2n+1
udu =
2 4 (2n)
1 3 (2n + 1)
,
avem din formula lui Wallis ca I = lim
n
I
n
=

2
.
Problema 11.8 Studiat i existent a limitei sirului x
n
=
_
1
n
_
n
+
_
2
n
_
n
+ +
_
n 1
n
_
n
.
Solut ie. Fie u
k
: N

R
+
dat prin
u
k
(n) =
_ _
1
k
n
_
n
, daca k 1, n 1
0, daca k n.
Remarcam ca
x
n
=

k=1
u
k
(n).
De asemenea, pentru orice k N

xat, lim
n
u
k
(n) = e
k
. Folosind acum (11.3), avem si
ca nln
_
1
k
n
_
k, k 1, n 1, de unde
0 u
k
(n) e
n
, k N

, n N

.
Cum

n=1
e
n
este evident convergenta, aplicam criteriul lui Weierstrass si obt inem ca

k=1
u
k
converge uniform pe N

. Cum + este (singurul) punct de acumulare al mult imii


N

, aplicam acum transferul de existent a al limitei si deducem ca lim


n
x
n
si
lim
n
x
n
= lim
n

k=1
u
k
(n) =

k=1
lim
n
u
k
(n) =

k=1
e
k
=
1
e 1
.
136
Problema 11.9 Fie seria de funct ii

n=1
(1)
n
x
2
+n
n
2
, x R.
(a) Sa se studieze convergent a punctuala pentru orice x R si convergent a uniforma
pe orice interval [a, b]. Este seria uniform convergenta pe R?
(b) Sa se studieze absolut convergent a pentru orice x R.
(c) Sa se studieze continuitatea sumei seriei (acolo unde ea exista).
(d) Se poate deriva seria termen cu termen ?
Solut ie. (a) Fie x R; seriile numerice

n=1
(1)
n
x
2
n
2
si

n=1
(1)
n
1
n
sunt ambele con-
vergente, deci seria data este punctual convergenta pentru orice x R.
Studiem acum convergent a uniforma pe intervalul [a, b]. Seria

n=1
(1)
n
x
2
n
2
este uni-
form convergenta pe [a, b]; pentru aceasta, aplicam criteriul lui Weierstrass de convergent a
uniforma:

(1)
n
x
2
n
2

b
2
n
2
, x [a, b],
iar seria numerica

n=1
1
n
2
este convergenta.
Seria nu este uniform convergenta pe R; pentru aceasta, e A suma seriei

n=1
(1)
n
n
2
si B
suma seriei

n=1
(1)
n
n
. Evident, seria data converge punctual la funct ia f(x) = Ax
2
+ B.
Fie s
n
(x) =
n

k=1
(1)
k
x
2
+k
k
2
. Calculam:
lim
n
sup
xR
[f(x) s
n
(x)[ = lim
n
sup
xR
x
2

k=1
(1)
k
k
2
A

= ,
deci seria nu converge uniform pe R la f.
(b) Seria nu converge absolut pentru niciun x R deoarece seria

n=1
x
2
+n
n
2
este
divergenta (se poate compara cu seria armonica).
(c) Evident, funct ia f (suma seriei) este continua pe R (desi seria nu converge uniform
pe R).
(d) Seria nu verica ipotezele teoremei de derivare termen pe R; seria derivatelor este

n=1
(1)
n
2x
n
2
, serie care nu este uniform convergenta pe R:
lim
n
sup
xR

2x
n

k=1
(1)
k
k
2
2Ax

= .
Totusi, seria derivatelor converge uniform pe orice compact din R. Seria data se poate
deriva termen cu termen, egalitatea
_
_
x
2

n1
(1)
n
n
2
+

n1
(1)
n
n
_
_

= 2x

n1
(1)
n
n
2
, x R
ind adevarata.
Siruri si serii de funct ii: serii Taylor, serii Fourier 137
Problema 11.10 Fie seria

n=1
nx
n
+x
n
2
+ 1
.
(a) Pentru ce valori ale lui x R seria converge ?
(b) Sa se studieze convergent a uniforma.
(c) Se poate deriva seria termen cu termen ?
Solut ie. (a) Fie x (1, 1), xat. Descompunem seria:

n=1
nx
n
+x
n
2
+ 1
= x

n=1
1
n
2
+ 1
+

n=1
n
n
2
+ 1
x
n
.
Prima serie este convergenta pentru orice x R. A doua serie converge absolut daca
x (1, 1); pentru demonstrat ie se poate aplica criteriul raportului:
lim
n

n+1
(n+1)
2
+1
n
n
2
+1

x
n+1
x
n

= [x[ < 1.
Daca x = 1, seria converge (cele doua serii de mai sus sunt convergente) dar nu converge
absolut.
Daca x = 1, seria este divergenta (se poate compara cu seria armonica).
Daca [x[ > 1, seria diverge (se poate aplica criteriul necesar).

In concluzie, seria converge daca si numai daca x [1, 1).


(b) Aplicand criteriul lui Weierstrass, seria converge absolut si uniform pe orice com-
pact [r, r] (1, 1):

n=1

nx
n
+x
n
2
+ 1

n=1
nr
n
+r
n
2
+ 1
, [x[ r,
ultima serie (numerica) ind convergenta.
(c) Seria derivatelor este:

n=1
_
nx
n
+x
n
2
+ 1
_

n=1
_
n
2
n
2
+ 1
x
n1
+
1
n
2
+ 1
_
=
=

n=1
1
n
2
+ 1
+

n=1
n
2
n
2
+ 1
x
n1
.
Seria derivatelor converge uniform pe orice interval [r, r] (1, 1), deci seria se poate
deriva termen cu termen.
Problema 11.11 Fie seria

n=1
sin(2
n
x)
2
n
, x R.
(i) Sa se demostreze ca seria converge absolut si uniform.
(ii) Sa se studieze derivabilitatea sumei seriei n 0.
Solut ie. (i) Evident.
(ii) Fie f suma seriei si e x
m
=

2
m+1
; deoarece sin(2
k
x
m
) = 0, k m+ 1, aplicand
inegalitatea sin x
2

x, x
_
0,

2
_
, rezulta:
f(x
m
) =
m

k=1
sin(2
k
x
m
)
2
k

m

k=1
2


2
k
x
m
2
k
=
2

mx
m
.
138
Funct ia nu este derivabila n 0:
f(x
m
) f(0)
x
m

m , pentru m .
Problema 11.12 Sa se calculeze suma seriei

n=0
x
4n
(4n)!
si sa se generalizeze rezultatul la
seria

n=0
1
(pn)!
, p N

xat.
Solut ie. Seria de puteri

n=0
1
(4n)!
x
4n
are raza de convergent a . Notand cu f suma se-
riei, problema revine la a calcula f(1). Pentru aceasta, vom obt ine (prin derivari succesive)
o ecuat ie diferent iala satisfacuta de funct ia f. Avem:
f
(4)
(x) =

n=1
(4n)(4n 1)(4n 2)(4n 3)
1
(4n)!
x
4n4
=

n=1
1
(4n 4)!
x
4n4
= f(x).
Deci f este solut ia problemei Cauchy:
f
(4)
= f, f(0) = 1, f

(0) = f

(0) = f

(0) = 0.
Polinomul caracteristic este
4
1, iar radacinile sunt radacinile de ordinul 4 ale unitat ii,

k
= e
i
k
2
, k = 0, 1, 2, 3. Rezulta
f(x) = c
1
e

1
x
+c
2
e

2
x
+c
3
e

3
x
+c
4
e

4
x
,
constantele c
1
, c
2
, c
3
, c
4
satisfacand sistemul:
c
1

1
+c
2

2
+c
3

3
+c
4

4
= 1,
c
1

m
1
+c
2

m
2
+c
3

m
3
+c
4

m
4
= 0, m = 2, 3, 4.
Rezulta imediat c
1
= c
2
= c
3
= c
4
=
1
4
si deci
f(x) =
1
4
_
e

1
x
+e

2
x
+e

3
x
+e

4
x
_
=
1
4
_
e
x
+e
x
+ 2 cos x
_
.

In concluzie, suma seriei

n=0
x
4n
(4n)!
este
1
4
(e +e
1
+ 2 cos 1).
Printr-un rat ionament analog (cu
1
,
2
, . . .,
p
radacinile de ordinul p ale unitat ii), se
obt ine:

n=0
1
(pn)!
x
pn
=
1
p
p

k=1
e

k
x
si

n=0
1
(pn)!
=
1
p
p

k=1
e

k
.
Problema 11.13 Fie a si b doua numere reale astfel ncat a < b si e f
0
: [a, b] R o
funct ie continua. Fie sirul de funct ii f
n
: [a, b] R denit prin:
f
n+1
(x) =
_
x
a
f
n
(t)dt.
Sa se studieze convergent a si sa se calculeze suma seriei

n=1
f
n
.
Siruri si serii de funct ii: serii Taylor, serii Fourier 139
Solut ie. Vom demonstra mai ntai ca seria converge absolut si uniform pe [a, b].
Fie M 0 astfel ncat sup
x[a,b]
[f
0
(x)[ M. Se demonstreaza simplu prin induct ie inegali-
tatea:
[f(x)[ M
(x a)
n
n!
, x [a, b].
Rezulta deci:
sup
x[a,b]
[f(x)[ M
(b a)
n
n!
.
Seria

n=1
M
(ba)
n
n!
este convergenta, deci seria

n=1
f
n
converge absolut si uniform pe [a, b].
Rezulta ca funct ia suma S este continua.
Demonstram acum ca seria derivatelor

n=1
f

n
este uniform convergenta. Este sucient sa
observam ca f

n+1
= f
n
, deci se poate repeta rat ionamentul anterior. Rezulta ca seria

n=1
f
n
se poate deriva termen cu termen:
S

n=1
f

n
=

n=0
f
n
= f
0
+S.
Problema Cauchy S

= f
0
+S, S(a) = 0, are solut ia
S(x) = e
x
__
x
a
f
0
(t)e
t
dt
_
, x [a, b],
ceea ce ncheie demonstrat ia.
Problema 11.14 (Teorema lui Abel) Fie (a
n
)
n
un sir de numere reale astfel ncat seria
de puteri

n=0
a
n
x
n
are raza de convergent a R (0, ). Presupunem ca seria

n=0
a
n
R
n
este convergenta. Sa se demonstreze ca seria

n=0
a
n
x
n
este uniform convergenta pe [0, R]
si
lim
n

n=0
a
n
x
n
=

n=0
a
n
R
n
.
Solut ie. Vom demonstra ca restul seriei

n=0
a
n
x
n
tinde uniform la 0 pe [0, R].
Fie, pentru orice K natural, restul de ordinul K al seriei

n=0
a
n
R
n
:
Q
K
=

n=K
a
n
R
n
0 pentru K .
140
Pentru orice x [0, R) avem:
Q
K
=

n=K
a
n
R
n
_
x
R
_
n
=

n=K
(Q
n
Q
n+1
)
_
x
R
_
n
=
=

n=K
Q
n
_
x
R
_
n

n=K
Q
n+1
_
x
R
_
n
=
cele doua serii de mai sus converg
=

n=K
Q
n
_
x
R
_
n

n=K+1
Q
n
_
x
R
_
n1
=
= Q
K
_
x
R
_
K
+

n=K+1
Q
n
_
_
x
R
_
n

_
x
R
_
n1
_
.
Fie > 0; atunci exista un rang m
0
astfel ncat [Q
n
[ , n m
0
. Rezulta ca pentru
orice K m
0
si x [0, R), avem pentru restul de ordinul K al seriei

n=0
a
n
x
n
evaluarea:

nK
a
n
x
n

nK+1

_
x
R
_
n

_
x
R
_
n1

+
_
x
R
_
K
2.
Evaluarea de mai sus este adevarata si pentru x = R, (prin alegerea lui m
0
) si deci seria

n=0
a
n
x
n
converge uniform pe [0, R]. Fie f suma seriei; atunci f este funct ie continua pe
[0, R] si deci lim
xR
f(x) = f(R), ceea ce ncheie demonstrat ia.
Problema 11.15 a) Fie (a
n
)
nN
un sir crescator de numere reale strict pozitive, cu limita
. Aratat i ca funct ia f : R

+
R, data prin
f(x) =

n=0
(1)
n
e
anx
este bine denita si continua.
b) Aratat i ca

_
0
f(x)dx este convergenta, avand valoarea

n=0
(1)
n
a
n
. Cazuri particulare:
i) a
n
= n + 1; ii) a
n
= 2n + 1.
Solut ie. a) Fie u
n
: R

+
R, u
n
(x) = e
anx
. Consideram asadar seria

n=0
(1)
n
u
n
.
Fie x R

+
arbitrar. Sirul (u
n
(x)) este descrescator la 0, deci seria numerica

n=0
(1)
n
u
n
(x) este convergenta conform criteriului Leibniz pentru serii numerice. Asadar,

n=0
(1)
n
u
n
converge punctual pe R

+
. Notam suma sa cu f. De asemenea, folosind mono-
tonia sirului (u
n
(x)), se arata ca f(x) 0 si ca pentru orice n N

k=n
(1)
k
u
k
(x)

u
n
(x). (11.7)
Fie acum R

+
xat. Cum

e
anx
0

= e
anx
e
an
, x [, )
Siruri si serii de funct ii: serii Taylor, serii Fourier 141
si cum lim
n
e
an
= 0, avem aplicand criteriul majorarii ca (u
n
)
u

[,)
0. Remarcand si ca
(u
n
) este uniform descrescator la 0, putem folosi criteriul lui Leibniz pentru serii de funct ii
pentru a deduce ca

n=0
(1)
n
u
n
converge uniform pe [, ) la f. Cum funct iile (1)
n
u
n
sunt continue, avem ca f e continua pe [, ). Folosind acum ca este arbitrar, avem ca
f e continua pe R

+
.
b) Fie n N. Pentru orice x R
+
avem ca
x
_
0
e
ant
dt =
1
a
n
_
1 e
anx
_
.
Cum a
n
> 0, rezulta ca integrala

_
0
e
ant
dt este convergenta la
1
a
n
. Folosind (11.7) cu
n = 0, avem ca
0 f(x) = [f(x)[ e
a
0
x
, x R

+
.
Deducem de aici ca f, prelugita prin continuitate cu f(0) = 0, admite pe [0, ) o integrala
improprie convergenta. De asemenea, folosind iarasi (11.7), avem ca pentru orice n N

f(x)
n1

k=0
(1)
n
u
k
(x)

e
anx
.
De aici, cum toate integralele care apar sunt convergente,

_
0
f(x)dx
n1

k=0
(1)
k
a
k

_
0
f(x)dx
n1

k=0
(1)
k

_
0
u
k
(x)dx

_
0
_
f(x)
n1

k=0
(1)
k
u
k
(x)
_
dx

_
0

f(x)
n1

k=0
(1)
k
u
k
(x)

dx

_
0
e
anx
dx =
1
a
n
.
Trecand la limita pentru n n relat ia de mai sus, obt inem ca

_
0
f(x)dx =

n=0
(1)
n
a
n
.
Ne ocupam acum de cele doua cazuri particulare specicate.
i) Daca a
n
= n + 1, atunci
f(x) =

n=0
(1)
n
e
(n+1)x
=
e
x
1 +e
x
.
Cum

_
0
e
x
1 +e
x
dx = ln 2, regasim egalitatea cunoscuta: ln 2 =

n=0
(1)
n
n + 1
.
ii) Daca a
n
= 2n + 1,
f(x) =

n=0
(1)
n
e
(2n+1)x
=
e
x
1 +e
2x
.
142
Cum

_
0
e
x
1 +e
2x
dx =

4
, regasim o alta egalitate cunoscuta:

4
=

n=0
(1)
n
2n + 1
.
Problema 11.16 Fie sirul de funct ii f
n
: R
2
R dat prin f
n
(x, y) =
x
n
1 +y
2n
.
1) Determinat i =
_
(x, y) R
2
[

n=0
f
n
(x, y) este convergenta
_
.
2) Aratat i ca suma seriei

n=0
f
n
(x, y) =: f(x, y) este de clasa (
1
pe .
Solut ie. Se observa cu usurint a faptul ca funct iile f
n
sunt de clasa (

pe R
2
.
Fixam y R. Atunci

n=0
x
n
1 +y
2n
este o serie de puteri, avand raza de convergent a
r
y
= max1, y
2
. Cum pentru x = r
y
, termenul general al seriei numerice

n=0
f
n
(x, y) nu
converge la 0, rezulta ca
=
_
(x, y) R
2
[ [x[ < max1, y
2

_
.
Remarcam ca este o mult ime deschisa.
Consideram pe R
2
norma maxim |(x, y)|

= max[x[ , [y[ si bila nchisa asociata de


centru (x
0
, y
0
) si raza r > 0 : B
r
(x
0
, y
0
) = (x, y) R
2
: max[x x
0
[ , [y y
0
[ r.
Vom nota cu := sup
(x,y)Br(x
0
,y
0
)
[x[ si := inf
(x,y)Br(x
0
,y
0
)
[y[ . Atunci
= max[x
0
r[ , [x
0
+r[ si (11.8)
=
_
0, daca [y
0
[ r
min[y
0
r[ , [y
0
+r[, daca [y
0
[ > r.
Daca (x
0
, y
0
) , gasim r > 0 astfel ncat B
r
(x
0
, y
0
) , deoarece este deschisa.

In
acest caz, avem n plus ca [[ < 1 daca [y
0
[ 1.
a) Continuitatea lui f : R. Fie (x
0
, y
0
) . Exista atunci B
r
(x
0
, y
0
) . Folosind
(11.8), deducem ca pentru orice n N
sup
(x,y)Br(x
0
,y
0
)
[f
n
(x, y)[

n
1 +
2n
.
Cum (, ) B
r
(x
0
, y
0
) , avem ca seria

n=0

n
1 +
2n
converge, de unde aplicand cri-
teriul lui Weierstrass obt inem ca

n=0
f
n
(x, y) converge uniform pe B
r
(x
0
, y
0
) la f. Deducem
ca f este continua pe B
r
(x
0
, y
0
), deci n (x
0
, y
0
). De aici, f este continua pe .
b) Existent a si continuitatea lui
f
x
: R. Pentru y R xat, seria de puteri

n=0
x
n
1 +y
2n
poate derivata termen cu termen pe intervalul (r
y
, r
y
). Exista asadar
f
x
:
R,
f
x
(x, y) =

n=1
nx
n1
1 +y
2n
. Rat ionand ca la punctul a), se arata ca
f
x
este continua
pe .
c) Existent a si continuitatea lui
f
y
: R. Pentru orice n N, consideram aplicat iile
f
n
y
: R
2
R,
f
n
y
(x, y) =
2nx
n
y
2n1
(1 +y
2n
)
2
. Pentru y R xat,

n=1
f
n
y
(x, y) este o serie
Siruri si serii de funct ii: serii Taylor, serii Fourier 143
de puteri avand raza de convergent a r

y
= max
_
y
2
,
1
y
2
_
. Deducem ca

n=1
f
n
y
converge
punctual pe

=
_
(x, y) R
2
: [x[ < max
_
y
2
,
1
y
2
__
.
Vom studia comportarea acestei serii pe

.
Fie (x
0
, y
0
) si r > 0 astfel ncat B
r
(x
0
, y
0
) . Consideram urmatoarele cazuri:
(i) [y
0
[ < 1. Notam B
1
:= (x, y) B
r
(x
0
, y
0
) : [y[ 1. Atunci, pentru orice n N

,
sup
(x,y)B
1

f
n
y
(x, y)

2n
n
.
Cumn acest caz < 1, avem ca

n=1
2n
n
este convergenta, de unde avem folosind criteriul
lui Weierstrass ca

n=1
f
n
y
converge uniform pe B
1
.
(ii) [y
0
[ = 1. Notam B
2
:=
_
(x, y) B
r
(x
0
, y
0
) : y
_
1
2
,
3
2
_
. Atunci, folosind variat ia
funct iei u
u
(1 +u
2
)
de la R
+
la R, deducem ca pentru orice (x, y) B
2
[y[
2n1
(1 +y
2n
)
2

1
4 [y[

1
2
.
De aici, pentru orice n N

,
sup
(x,y)B
2

f
n
y
(x, y)

n
n
.
Din nou, < 1, deci

n=1
f
n
y
converge uniform pe B
2
.
(iii) [y
0
[ > 1. Alegem r (0, 1) astfel ncat B
r
(x
0
, y
0
) . Notam B
3
:=
(x, y) B
r
(x
0
, y
0
) : [y[ 1 . Atunci, cum [y
0
[ > 1 > r, avem ca > 0. De aseme-
nea, folosind faptul ca funct ia u
u
(1 +u
2
)
este descrescatoare pentru u 1, obt inem ca
pentru orice (x, y) B
3
,
[y[
2n1
(1 +y
2n
)
2
=
1
[y[

y
2n
(1 +y
2n
)
2

1



2n
(1 +
2n
)
2
.
De aici, pentru orice n N

,
sup
(x,y)B
3

f
n
y
(x, y)

2n
n

2n1
(1 +
2n
)
2
.
Cum (, )

, seria

n=1
f
n
y
converge uniform pe B
3
.
Revenind, pentru orice (x
0
, y
0
) , n toate cele trei cazuri considerate, exista s
(0, r) astfel ncat

n=1
f
n
y
(x
0
, ) converge uniform pe I
s
:= (y
0
s, y
0
+ s). Cum seria

n=0
f
n
(x
0
, ) converge punctual pe I
s
, deducem existent a lui
f
y
(x
0
, y
0
) =

n=1
f
n
y
(x
0
, y
0
).
144
De asemenea, analizand cazurile de mai sus, pentru orice (x
0
, y
0
) , exista r > 0 si
B B
1
, B
2
, B
3
astfel ncat (x
0
, y
0
) B B
r
(x
0
, y
0
) si

n=1
f
n
y
converge uniform
pe B. Cum toate aplicat iile
f
n
y
sunt continue n (x
0
, y
0
), deducem ca
f
y
este continua
n (x
0
, y
0
).
Serii Fourier
Serii Fourier n spat ii Hilbert
Fie X un spat iu liniar real si , : X X R un produs scalar. Atunci aplicat ia
|| : X R data prin |x| :=
_
x, x, x X deneste o norma pe X. De asemenea,
aplicat ia d : XX R data prin d(x, y) := |x y| , x, y X deneste o metrica pe X.
Spat iul cu produs scalar (X, , ) se numeste spat iu Hilbert daca este complet n raport
cu metrica indusa de norma, adica daca orice sir Cauchy de puncte din X este convergent
la un element din X. Spat iile Hilbert reprezinta generalizari (posibil innit dimensionale)
ale spat iilor euclidiene. Un exemplu de spat iu Hilbert innit dimensional l reprezinta

2
:=
_
(x
n
)
nN
R [

n=0
x
2
n
<
_
,
spat iul sirurilor de patrat sumabil, n raport cu produsul scalar , :
2

2
R dat prin
x, y :=

n=0
x
n
y
n
, x = (x
n
), y = (y
n
)
2
.
Fie (X, , ) un spat iu cu produs scalar. Doi vectori x, y X se numesc ortogonali
daca x, y = 0, iar acest lucru se mai noteaza si x y. De asemenea, pentru x X,
A X, vom nota x A daca x y, y A.
Denit ie. Un sistem ortonormat n X este un sir de vectori (e
n
)
nN
X cu propri-
etatea ca e
n
, e
m
=
nm
=
_
1, daca n = m
0, daca n ,= m
.
Cu alte cuvinte, sistemul (e
n
)
nN
este si ortogonal, adica e
n
e
m
, n ,= m.

In
particular, acesta este si liniar independent.

In continuare, vom nota cu (e
n
)
nN
un sistem
ortonormat n spat iul cu produs scalar (X, , ).
Denit ie. Pentru orice x X si n N, notam cu c
n
:= x, e
n
si numim numerele c
n
coecient ii Fourier asociat i lui x, iar seria

n=0
c
n
e
n
o numim seria Fourier asociata lui
x n raport cu sistemul ortonormat (e
n
)
nN
.
O prima problema care apare este aceea a convergent ei seriei Fourier asociate unui
vector x X. Are loc urmatoarea teorema.
Teorema. Fie (X, , ) un spat iu cu produs scalar, x X, (c
n
)
nN
sirul coecient ilor
Fourier asociat lui x n raport cu sistemul ortonormat (e
n
)
nN
si (
n
)
nN
R un sir
arbitrar de numere reale. Atunci:
(i)
_
_
_
_
x
n

k=0
c
k
e
k
_
_
_
_

_
_
_
_
x
n

k=0

k
e
k
_
_
_
_
, n N;
(ii)

n=0
[c
n
[
2
|x|
2
(inegalitatea lui Bessel);
Siruri si serii de funct ii: serii Taylor, serii Fourier 145
(iii) lim
n
c
n
= 0.
Denit ie. Sistemul ortonormat (e
n
)
nN
se numeste total daca x X0, n N astfel
ncat x, e
n
,= 0. Sistemele ortonormate totale se mai numesc si baze ortonormate.
Cu alte cuvinte, (e
n
) este total daca din x e
n
, n N rezulta x = 0. Urmatorul
rezultat face legatura ntre sistemele ortonormate totale si convergent a seriilor Fourier n
spat ii Hilbert.
Teorema. Fie X un spat iu Hilbert si (e
n
)
nN
un sistem ortonormat n X. Urmatoarele
armat ii sunt echivalente:
(i) Sistemul ortonormat (e
n
) este total;
(ii) x X, seria Fourier asociat a lui x converge la x;
(iii) x X, |x|
2
=

n=0
[c
n
[
2
(egalitatea lui Parseval-Liapunov);
(iv) Aplicat ia : X
2
, denita prin (x) := (c
n
), unde (c
n
) este sirul coecient ilor
Fourier asociat lui x, este un izomorsm de spat ii normate.
Un alt exemplu important de spat iu Hilbert este L
2
[a, b]. Deoarece nu putem intra n
toate detaliile de ordin tehnic ce implica not iuni legate de integrabilitatean sens Lebesgue,
precizam doar faptul ca orice funct ie integrabila Riemann pe intervalul compact [a, b]
este integrabila n sens Lebesgue pe [a, b], iar valorile celor doua integrale coincid. Mai
mult, L
2
[a, b] este format din clasele de echivalent a de funct ii de patrat integrabil n sens
Lebesgue, egale aproape peste tot (adica egale cu except ia unor mult imi de puncte de
masura Lebesgue nula; de exemplu, doua mult imi care difera printr-o mult ime cel mult
numarabila de puncte sunt egale aproape peste tot). Teoria seriilor Fourier clasice admite
o dezvoltare foarte naturala n L
2
[, ], spat iu Hilbert n care sistemul trigonometric cla-
sic
_
1

2
,
1

cos x,
1

sin x, . . . ,
1

cos nx,
1

sin nx, . . .
_
este un sistem ortonormat total,
deci toate rezultatele din teorema de mai sus sunt adevarate.
Serii Fourier clasice
O serie de forma
a
0
2
+

n=1
_
a
n
cos
nx
l
+b
n
sin
nx
l
_
, (11.9)
unde a
0
, a
n
, b
n
R (n = 1, 2, . . . ) si l > 0 se numeste serie trigonometrica de perioada
2l.
Teorema. Daca presupunem ca seria (11.9) converge uniform pe intervalul [l, l] catre
s, atunci s este continua pe [l, l], iar coecient ii seriei (11.9) sunt dat i de formulele
a
n
=
1
l
l
_
l
s(x) cos
nx
l
dx, pentru n N, (11.10)
b
n
=
1
l
l
_
l
s(x) sin
nx
l
dx, pentru n N

. (11.11)

In mod evident, n locul intervalului [l, l] putem considera orice alt interval de lungime
2l.
Fie acum f : R R o funct ie periodica de perioada 2l, integrabila n sens propriu
sau absolut integrabila n sens impropriu pe [l, l]. Numerele a
n
si b
n
date de formulele
146
(11.10) si (11.11), n care l nlocuim pe s cu f, se numesc coecient ii Fourier ai funct iei
f, iar seria trigonometrica de perioada 2l formata cu acesti coecient i se numeste seria
Fourier asociata funct iei f. Not am uneori seria Fourier asociata funct iei f prin
f(x)
a
0
2
+

n=1
_
a
n
cos
nx
l
+b
n
sin
nx
l
_
.
Funct ia este dezvoltabila n serie Fourier pe mult imea A R daca seria Fourier
asociata converge pe aceasta mult ime catre f. Datorita periodicitat ii lui f, coecient ii sai
Fourier nu depind de intervalul de lungime 2l pe care se calculeaza formulele ce ne dau
acesti coecient i. Mai remarcam faptul ca, pentru a studia posibilitatea dezvoltarii n serie
Fourier pe R, este sucient sa facem acest studiu pe [l, l].
Damn continuare cateva criterii utile pentru a studia posibilitatea dezvoltarii n serie
Fourier.
Teorema. (Criteriul lui Dirichlet) Daca funct ia f este monotona pe port iuni n in-
tervalul [l, l] si are n acest interval cel mult un num ar nit de puncte de discontinui-
tate de spet a I, atunci seria Fourier asociat a converge n ecare punct x
0
[l, l] catre
f(x
0
+ 0) +f(x
0
0)
2
.
Teorema Daca funct ia f este derivabila sau derivabila pe port iuni n intervalul
[l, l], atunci seria Fourier asociata converge n ecare punct x
0
[l, l] catre
f(x
0
+ 0) +f(x
0
0)
2
.

In cazul funct iilor neperiodice, denite de exemplu pe un interval [l, l], se considera
o funct ie ajutatoare

f : R R, data prin

f(x) :=
_
f(x), daca x (l, l]
f(l), daca x = l
pe intervalul
[l, l] si prelungita prin periodicitate. Seria Fourier atasata lui

f se va numi seria Fourier
atasata lui f pe [l, l]. Convergent a acestei serii revine la ndeplinirea de catre f a unui
criteriun acest sens pe [l, l].

In punctele l, suma acestei serii va


f(l + 0) +f(l 0)
2
.
Daca funct ia f : [l, l] R este para, atunci coecient ii Fourier au valorile:
a
n
=
2
l
l
_
0
f(x) cos
nx
l
dx, pentru n N, b
n
= 0, pentru n N

, (11.12)
adica seria Fourier asociata ei pe intervalul [l, l] cont ine numai cosinusuri. Daca funct ia
f : [l, l] R este impara, atunci coecient ii Fourier au valorile:
a
n
= 0, pentru n N, b
n
= b
n
=
2
l
l
_
0
f(x) sin
nx
l
dx, pentru n N

, (11.13)
adica seria Fourier asociata ei pe intervalul [l, l] cont ine numai sinusuri.
Daca o funct ie f este denita numai pe intervalul [0, l] si ndeplineste condit iile de
dezvoltare n serie Fourier n interiorul acestui interval, cerandu-se dezvoltarea ei numai
n serie de cosinusuri sau sinusuri, se folosesc urmatoarele funct ii ajutatoare:
f
1
(x) =
_
f(x), daca x [0, l]
f(x), daca x [l, 0)
si
f
2
(x) =
_
f(x), daca x [0, l]
f(x), daca x [l, 0)
.
Siruri si serii de funct ii: serii Taylor, serii Fourier 147
Egalitatea lui Parseval-Liapunov, valabila, dupa cum am vazut mai sus, n spat ii
Hilbert generale, are urmatoarea forma particulara.
Teorema. (Egalitatea Parseval-Liapunov) Daca funct ia f : [l, l] R este integrabila
n sens propriu sau de patrat integrabila n sens impropriu pe [l, l], atunci coecient ii
Fourier asociat i veric a egalitatea
a
2
0
2
+

n=1
(a
2
n
+b
2
n
) =
1
l
l
_
l
f
2
(x)dx. (11.14)
Are loc de asemenea urmatorul rezultat.
Teorema. Daca sirurile (a
n
) si (b
n
) formate cu coecient ii seriei (11.9) sunt monotone
si converg la 0, atunci seria este convergent a pentru orice x ,= 2kl, k Z si uniform
convergenta n orice interval compact care nu cont ine puncte de acesta forma.
Prezentam mai jos cateva teoreme legate de derivarea si integrarea termen cu termen
a seriilor Fourier.
Teorema. Fie f o funct ie continua de perioada 2l admit and o derivata absolut inte-
grabila (care poate sa nu existe ntr-un numar nit de puncte dintr-un interval de lungime
egala cu perioada). Atunci seria Fourier a lui f

poate obt inuta din seria Fourier a lui


f prin derivare termen cu termen.
Teorema. Fie f o funct ie continua denita pe [l, l] si admit and o derivata absolut
integrabila (care poate sa nu existe ntr-un numar nit de puncte dintr-un interval de
lungime 2l). Atunci
f

(x)
c
2
+

n=1
_
(nb
n
+ (1)
n
c) cos
nx
l
na
n
sin
nx
l
_
,
unde a
n
si b
n
sunt coecient ii Fourier ai funct iei f, iar constanta c este data de una din
egalitat ile:
c =
f(l) f(l)
l
sau
c = lim
n
_
(1)
n+1
nb
n

daca aceast a limita exista.


Teorema anterioara presupune continuitatea lui f si existent a unei derivate absolut
integrabile.

In aplicat ii se pot ntalni cazuri n care cunoastem numai seria Fourier a lui f.
Atunci problema devine mai dicila: trebuie sa deducem dupa seria Fourier daca funct ia
este derivabila si derivata integrabila si, daca raspunsul este armativ, sa formam seria
Fourier a acestei derivate. Teorema urmatoare contribuie la rezolvarea acestei probleme.
Teorema. Fie seria (11.9). Daca seria
c
2
+

n=1
_
(nb
n
+ (1)
n
c) cos
nx
l
na
n
sin
nx
l
_
(11.15)
unde c = lim
n
_
(1)
n+1
nb
n

este seria Fourier a unei anumite funct ii absolut integra-


bile , atunci seria (11.9) este seria Fourier a funct iei
f(x) =
x
_
0
(x)dx +
a
0
2
+

n=1
a
n
148
continua pentru x (l, l).

In plus, (11.15) este convergenta catre f si avem f

(x) = (x)
n orice punct de continuitate a lui .
Pentru funct iile pare, respectiv impare, posibilitatea derivarii termen cu termen ia
urmatoarele forme particulare.
Teorema. Daca funct ia f este continua pe [0, l], admite o derivata absolut integrabila si
este dezvoltabila n serie Fourier de cosinusuri sau de sinusuri, atunci seria cosinusurilor
poate derivata ntotdeauna termen cu termen, iar acest lucru este valabil si pentru seria
de sinusuri daca f(0) = f(l) = 0.
Teorema. Fie f o funct ie continua pe [0, l], cu derivata absolut integrabila (care poate
sa nu existe n anumite puncte) si dezvoltabila n serie Fourier de sinusuri
f(x) =

n=1
b
n
sin
nx
l
, x (0, l).
Atunci
f

(x)
c
2
+

n=1
[nb
n
d + (c +d)(1)
n
] cos
nx
l
,
unde:
c =
2(f(l) f(0))
l
, d =
2
l
f(0) sau
c = lim
n
[2nb
2n
] , d = lim
n
[(2n + 1)b
2n+1
c] daca aceste limite exista.
Teorema. (Integrarea termen cu termen) Daca funct ia f : [l, l] R este integrabila
n sens propriu sau absolut integrabila n sens impropriu pe intervalul [l, l] iar seria
trigonometrica (11.9) este seria Fourier asociata ei, atunci pentru orice interval [x

, x

]
[l, l] avem
x

_
x

f(t)dt =
x

_
x

a
0
2
+

n=1
x

_
x

_
a
n
cos
nt
l
+b
n
sin
nt
l
_
dt.
Pentru a studia convergent a uniforma a seriilor Fourier, se aplica de obicei urmatoarele
criterii.
Teorema. (Criteriul Dirichlet-Jordan) Daca funct ia f : [l, l] R este continua si
cu variat ie marginita pe [l, l] si satisface condit ia f(l) = f(l), atunci seria Fourier
asociata ei converge uniform catre f pe acest interval.
Corolar. Daca funct ia f : [l, l] R este derivabila cu derivata integrabila pe [l, l]
si satisface condit ia f(l) = f(l), atunci seria Fourier asociata ei converge absolut si
uniform catre f pe acest interval.
Probleme
Problema 11.17 Sa se determine:
(a) Seria Fourier asociata funct iei f : R R, f(x) = xsin x pe intervalul [0, ];
(b) Seria Fourier numai de cosinusuri si seria Fourier numai de sinusuri asociate funct iei
f pe acelasi interval;
(c) Mult imea pe care ecare din aceste trei serii converge catre f.
Siruri si serii de funct ii: serii Taylor, serii Fourier 149
Solut ie. Deoarece funct ia este continua pe R, ea este integrabila pe orice interval
compact, deci are sens problema determinarii seriei Fourier asociate ei pe un interval.
(a) Avem
a
n
=
2

_
0
xsin xcos 2nx dx =
=
1

_
0
x[sin(2n + 1)x sin(2n 1)x] dx =
2
4n
2
1
, n N
si
b
n
=
2

_
0
xsin xsin 2nx dx =
=
1

_
0
x[cos(2n 1)x cos(2n + 1)x] dx =
16n
(4n
2
1)
2
, n N

.
Atunci seria Fourier asociata funct iei f pe [0, ] este
1
2

n=1
_

4n
2
1
cos 2nx +
8n
(4n
2
1)
2
sin 2nx
_
. (11.16)
(b) Pentru a determina coecient ii Fourier ai seriei numai de cosinusuri asociate funct iei
f pe [0, ] vom aplica formulele coecient ilor Fourier, n care luam l = . Avem
a
n
=
2

_
0
xsin xcos nx dx =
1

_
0
x[sin(n + 1)x sin(n 1)x] dx.
Integrand prin part i, obt inem a
n
= (1)
n+1
2
n
2
1
, pentru n = 0, 2, 3, . . . , si a
1
=
1
2
.
Prin urmare, seria ceruta este
1
1
2
cos x + 2

n=2
(1)
n+1
n
2
1
cos nx. (11.17)
Asemanator, pentru seria numai de sinusuri corespunzatoare funct iei f pe [0, ] obt inem
b
n
=
16k
(4k
2
1)
, daca n = 2k si b
n
= 0, daca n = 2k + 1. Atunci seria ceruta este

2
sin x
16

n=1
k
(4k
2
1)
sin 2kx. (11.18)
(c) Funct ia considerata verica condit iile Dirichlet-Jordan pe [, ] si este para. Rezulta
atunci ca seria (11.17) converge uniform catre f pe acest interval. Dar funct ia f verica
condit iile Corolarului de mai sus pe [0, ], deci seriile (11.16) si (11.18) converg uniform
pe acest interval catre f.
Deoarece funct iile care apar n seria (11.16) sunt periodice de perioada , rezulta ca
aceasta serie deneste pe R o funct ie periodica de perioada , catre care seria converge
150
uniform pe R.

In virtutea paritat ii lui f deducem ca si seria (11.16) converge uniform catre


f pe intervalul [, ]. Aceste trei serii mai converg catre f si n punctele de forma k cu
k Z, deoarece n aceste puncte valoarea 0 a funct iei se repeta periodic. Asadar, seriile
(11.16) si (11.17) converg catre f pe mult imea [, ] k [ k Z, iar seria (11.18) pe
mult imea [0, ] k [ k Z.
Problema 11.18 1) Vericat i:
x
2
=

2
3
+ 4

n=1
(1)
n
cos nx
n
2
, x [, ], (11.19)
x = 2

n=1
(1)
n+1
sin nx
n
, x (, ). (11.20)
2) Fie a R

+
. Aratat i ca exista o aplicat ie f : [, ] R, de clasa (
2
, astfel
ncat: f(x) =

n=1
(1)
2
cos nx
n
2
+a
2
. Format i o ecuat ie diferent iala satisfacuta de f, si deducet i
expresia lui f(x).
3) Existent a si calculul lui

n=1
(1)
n
nsin nx
n
2
+a
2
, x (, ).
Solut ie. 1) Fie g : R R, de perioada 2, denita prin g(x) = x
2
pentru x [, ].
Conform Criteriului Dirichlet-Jordan, seria Fourier asociata converge uniform catre g pe
R. Cum g este para, seria Fourier va numai de cosinusuri. Folosind formulele pentru
coecient ii Fourier a
n
si b
n
, obt inem a
0
=
2
3
si a
n
=
4(1)
n
n
2
, n = 1, 2, . . . , de unde se
deduce (11.19).
Fie acum h : R R, de perioada 2, data prin h(x) = x daca x (, ) si h() = 0.
Cum h este local integrabila pe R, iar pentru orice x ,= (2k +1), k Z, f este derivabila,
seria sa Fourier converge si are drept suma h. Seria Fourier va numai de sinusuri, iar
b
n
=
2(1)
n
n
2
, n = 1, 2, . . . , de unde deducem (11.20).
2) Avem

(1)
n
cos nx
n
2
+a
2

1
n
2
+a
2
, x R.
Cum

n=1
1
n
2
+a
2
converge, deducem ca f exista ca ind suma unei serii de funct ii uniform
convergente pe [, ], si ca f este continua pe [, ].
Fie u : [, ] R denita prin f(x) =
x
2
4


2
12
+a
2
u(x), adica
u(x) =

n=1
u
n
(x), unde u(x) = (1)
n+1
cos nx
n
2
(n
2
+a
2
)
.
Se verica faptul ca seriile

n=1
u
n
,

n=1
u

n
,

n=1
u

n
converg uniform pe [, ], ceea ce
antreneaza faptul ca u este de clas a (
2
, precum si relat iile:
u

(x) =

n=1
(1)
n
sin nx
n(n
2
+a
2
)
;
u

(x) =

n=1
(1)
n
cos(nx)
n
2
+a
2
= f(x).
Siruri si serii de funct ii: serii Taylor, serii Fourier 151
Rezulta ca f este de clasa (
2
, si ca f este solut ia pe [, ] a ecuat iei diferent iale
y

a
2
y =
1
2
. (11.21)
O solut ie particulara a ecuat iei (11.21) este x
1
2a
2
. T inand cont de paritatea
lui f, deducem ca exista o constanta A astfel ncat, pentru orice x [, ], f(x) =
Ach ax
1
2a
2
. Cum u

() = 0, deducem ca f

() =

2
. Totodata, f

() = Aa sh a. De
aici, A =

2a sh a
si

n=1
(1)
n
cos nx
n
2
+a
2
=

a
ch ax
sh a

1
2a
2
, x (, ).
3) Rezulta ca f

(x) =

2
shax
sha
. Pe de alta parte, f

(x) =
x
2
+a
2
u

(x). Restrangandu-ne
la (, ) si folosind 2), avem
f

(x) =

n=1
(1)
n+1
sin nx
n
_
1
1
n
2
+a
2
_
.

In nal,

n=1
(1)
n+1
nsin nx
n
2
+a
2
=

2
sh ax
sh a
.
Problema 11.19 Sa se demonstreze ca, pentru orice x
_

4
,

4

, are loc egalitatea:


sec x =
4

ln(1 +

2)
+
8

n=1
_
ln(1 +

2) + 2
2n1

k=0
(1)
k+1
2k + 1
sin(2k + 1)

4
_
cos 4nx.
Solut ie. Funct ia f :
_

4
,

4

R, f(x) = sec x este para si verica toate condit iile


din criteriul lui Dirichlet, decieste dezvoltabila n serie Fourier numai de cosinusuri pe
acest interval. Avem
a
0
=
8

4
_
0
sec x dx, a
n
=
8

4
_
0
sec xcos 4nx dx, n N.
Folosind schimbarea de variabila tg x = t, deducem a
0
=
8

ln(1 +

2). Pentru calculul lui


a
n
, folosim identitatea
cos 4nx
cos x
= 2 cos(4n 1)x 2 cos(4n 3)x +
cos 4(n 1)x
cos x
,
de unde deducem ca
a
n
=
16

_
1
4n 1
sin(4n 1)

4

1
4n 3
sin(4n 3)

4
_
+a
n1
.
Adunand aceste relat ii, obt inem ca a
n
=
16

2n1

k=0
(1)
k+1
2k + 1
sin(2k + 1)

4
+a
0
. De aici, con-
cluzia.
152
Problema 11.20 1) Sa se arate ca funct ia f(x) =
1
2 + cos x
este dezvoltabila n serie
Fourier convergenta pe toata axa reala si sa se deduca aceasta dezvoltare.
2) Deducet i valoarea integralei

_
0
cos nx
2 + cos x
dx.
Solut ie. 1) Funct ia f verica condit iile criteriului Dirichlet. Se observa ca f este o
funct ie para, de perioada 2.

In consecint a, funct ia f este dezvoltabila n serie Fourier
decosinusuri pe toata axa reala.
Fie
f(x) =
a
0
2
+

n=1
a
n
cos nx. (11.22)
Avem
a
0
=
2

_
0
dx
2 + cos x
=
2

3
3
.

Inmult ind ambii membri ai relat iei (11.22) cu 2(2 + cos x) obt inem
2 = 2a
0
+a
0
cos x + 4

n=1
a
n
cos nx +

n=1
2a
n
cos xcos nx
= 2a
0
+a
0
cos x + 4

n=1
a
n
cos nx +

n=1
a
n
[cos(n + 1)x + cos(n 1)x.
Cum funct ia g(x) = 2 este para, este dezvoltabila n serie Fourier de cosinusuri pe toata
axa reala. T inand seama de egalitatea a doua serii Fourier, avem
2 = 2a
0
+a
1
0 = a
0
+ 4a
1
+a
2
0 = 4a
k
+a
k+1
+a
k1
, k = 1, 2, . . .
Sirul (a
n
) verica deci recurent a liniara a
k
= 4a
k1
a
k2
, cu a
0
=
2

3
3
si a
1
=
6 4

3
3
.
Se deduce ca a
k
=
2

3
3
(

3 2)
k
, k = 1, 2, . . .
Deci
f(x) =

3
3
+
2

3
3

n=1
(

3 2)
n
cos nx.
2) Deoarece a
n
=
2

_
0
cos nx
2 + cos x
dx, rezulta ca

_
0
cos nx
2 + cos x
dx =

3
3
(

3 2)
n
.
Problema 11.21 (Formula lui Poisson) Sa se arate ca funct ia f : R R, f(x) = 1
2cos x +
2
, unde [[ , = 1, este strict pozitiva pentru orice x R, iar funct ia
g : R R, g(x) =
1
2
1 2cos x +
2
, [[ < 1
este dezvoltabila n serie Fourier pe R; sa se determine apoi dezvoltarea funct iei g.
Siruri si serii de funct ii: serii Taylor, serii Fourier 153
Solut ie. Folosind inegalitatea
1 2cos x +
2
(1 [[
2
), x R,
si faptul ca [[ ,= 1, obt inem ca f > 0 pe R. Funct ia g este periodica de perioada 2,
derivabila cu derivata continua pe R.

In consecint a, verica condit iile din criteriul lui
Dirichlet, deci este dezvoltabila n serie Fourier pe R. Seria Fourier converge chiar uniform
pe R catre g. Cum g este para, seria Fourier asociata ei va numai de cosinusuri:
g(x) =
a
0
2
+

n=1
a
n
cos nx. (11.23)

Intrucat integralele care apar n calculul coecient ilor Fourier implica unele calcule destul
de complicate, vom evita acest calcul, dupa cum se va vedea n continuare. Amplicam
relat ia (11.23) cu 2(1 2cos x +
2
), si obt inem
2(1
2
) = a
0
(1 2cos x +
2
) + 2(1 +
2
)

n=1
a
n
cos nx
2

n=1
a
n
[cos(n + 1)x cos(n 1)x].
De aici deducem
2(1
2
) = a
0
(1 +
2
) 2a
1
+ 2

n=1
[(1 +
2
)a
n
a
n1
a
n+1
] cos nx. (11.24)
Funct ia constanta h(x) = 2(1
2
) poate considerata ca ind periodica de perioada
2, para, deci dezvoltabila n serie de cosinusuri pe R n mod unic. Deci, (11.24) e chiar
dezvoltarea funct iei hn serie Fourier pe R. Dar aceasta funct ie are tot i coecient ii Fourier
nuli, n afara de primul care este 4(1
2
). Prin urmare,
a
0
(1 +
2
) 2a
1
= 4(1
2
) si (11.25)
(1 +
2
)a
n
a
n1
a
n+1
= 0, n N

.
Coecientul a
0
al funct iei g se calculeaza direct, ind
a
0
=
2

_
0
1
2
1 2cos x +
2
dx = 2.
Folosind (11.25), deducem ca a
1
= 2 si, prin recurent a, a
n
= 2
n
.

In concluzie,
1
2
1 2cos x +
2
= 1 + 2

n=1

n
cos nx, x R. (11.26)
Ultima formula se mai numeste formula lui Poisson.
Problema 11.22 (Integrala Poisson) Fie (0, 1), f o funct ie continua de perioada 2,
a
n
si b
n
coecient ii sai Fourier. Notam prin
f(x, r) =
a
0
2
+

n=1

n
(a
n
cos nx +b
n
sin nx) .
154
Sa se arate ca seria
a
0
2
+

n=1

n
(a
n
cos nx +b
n
sin nx) (11.27)
este absolut convergenta si ca are loc relat ia
f(x, r) =
1
2

f(t)
1
2
1 2cos(t x) +
2
dt.
(f(x, r) poarta numele de integrala lui Poisson).
Solut ie. Deoarece a
n
, b
n
0, sirurile (a
n
) si (b
n
) sunt marginite. Exista atunci M > 0
astfel ncat, pentru orice n N, [a
n
[ M si [b
n
[ M. Cum
[
n
(a
n
cos nx +b
n
sin nx)[ 2M
n
si cum (0, 1), conform criteriului Weierstrass, rezulta convergent a uniforma a seriei
(11.27).
T inand acum cont de formulele lui a
n
si b
n
, obt inem
f(x, r) =
1
2

f(t)dt +
1

n=1

f(t) cos n(t x)dt. (11.28)


Dar cum seria
1
2
+

n=1

n
cos n(tx) este absolut si uniform convergenta, putem integra
termen cu termen relat ia (11.28) si obt inem
f(x, r) =
1

f(t)
_
1
2
+

n=1

n
cos n(t x)
_
dt.
Folosind si identitatea (11.26) de la problema anterioara, deducem concluzia.
Problema 11.23 Fie (g
n
) un sir de funct ii reale denit pe R prin:
g
n
(x) =
_

_
n, daca x = 2k, k Z
(1)
n1
n, daca x = (2k + 1), k Z
sin nx
sin x
, daca x , Z.
Sa se arate ca seria

n=1

n
g
n
, unde [[ < 1, converge uniform pe R catre o funct ie continua
si sa se determine aceasta funct ie.
Solut ie. Se observa ca funct iile g
n
sunt continue pe R. Cum
g
n
(x) = 2 cos(n 1)x +g
n2
(x), x R, n 3, (11.29)
rezulta ca [g
n
(x)[ n, x R, n N. Atunci [
n
g
n
[ n[[
n
, x R, n N. Cum
seria

n=1
n[[
n
este evident convergenta, conform criteriului lui Weierstrass, seria

n=1

n
g
n
Siruri si serii de funct ii: serii Taylor, serii Fourier 155
converge uniform pe R.

In baza transferului de continuitate, suma ei este continua.

In
virtutea relat iei (11.29), avem

n=3

n
g
n
(x) = 2

n=3

n
cos(n 1)x +

n=3

n
g
n2
(x), x R, (11.30)
iar seriile din membrul drept sunt de asemenea uniform convergente pe R.

Insa g
1
=
si
2
g
2
= 2
2
cos x. Folosind aceste relat ii si (11.30), obt inem

n=1

n
g
n
(x) = + 2

n=1

n
cos nx +
2

n=1

n
g
n
(x), x R,
de unde

n=1

n
g
n
(x) =

1
2
+
2
1
2

n=1

n
cos nx, x R.
Folosind si relat ia (11.26), obt inem ca funct ia suma ceruta este

1 2cos x +
2
.
Problema 11.24 Fie f si F doua funct ii la patrat integrabile denite pe [l, l] si
f(x)
a
0
2
+

n=1
(a
n
cos
nx
l
+b
n
sin
nx
l
),
F(x)
A
0
2
+

n=1
(A
n
cos
nx
l
+B
n
sin
nx
l
)
seriile Fourier atasate lor. Sa se arate ca
1
l
l
_
l
f(x)F(x)dx =
a
0
A
0
2
+

n=1
(a
n
A
n
+b
n
B
n
).
Solut ie. Seriile Fourier atasate funct iilor f +F si f F sunt
f(x) +F(x)
a
0
+A
0
2
+

n=1
[(a
n
+A
n
) cos
nx
l
+ (b
n
+B
n
) sin
nx
l
],
f(x) F(x)
a
0
A
0
2
+

n=1
[(a
n
A
n
) cos
nx
l
+ (b
n
B
n
) sin
nx
l
].
Deoarece f si F sunt funct ii la patrat integrabile, f +F si f F sunt la patrat integrabile.
Egalitatea lui Parseval ne conduce la
1
l
l
_
l
[f(x) +F(x)]
2
dx =
(a
0
+A
0
)
2
2
+

n=1
[(a
n
+A
n
)
2
+ (b
n
+B
n
)
2
],
1
l
l
_
l
[f(x) F(x)]
2
dx =
(a
0
A
0
)
2
2
+

n=1
[(a
n
A
n
)
2
+ (b
n
B
n
)
2
].
Scazand ultimele egalitat i, obt inem egalitatea ceruta.
156
Problema 11.25 Fie (0, 1) si f : R R periodica de perioada 2, cu
f(x) =
sh x
[x[

dac a x (, ) 0, f(0) = f() = 0.


1) Aratat i ca seria Fourier asociata lui f este de forma

n=1
b
n
sin nx, si ca b
n
=
O
_
1
n
1
_
.
2) Studiat i convergent a seriei Fourier de mai sus.
Solut ie. 1) Deoarece f este periodica de perioada 2 este local integrabila pe R, deci
seria sa Fourier exista.

In plus, deoarece f este impara, seria Fourier va o serie numai
de sinusuri. Avem
b
n
=
2

_
0
sh t
sin nt
t

dt, n N

.
Putem scrie:

2
b
n
= lim
x0
+

_
x
sh t

(t)dt, unde este aplicat ia de clasa (


1
: t
t
_

sin nu
u

du.
Integrand prin part i obt inem, pentru orice x (0, ] :

_
x
sh t

(t)dt = sh t (t)

_
x
ch t (t)dt.
Cum
0
_

sin nu
u

du, < 1 converge, deducem ca


b
n
=
2

_
0
ch t ((t))dt.
Facand schimbarea de variabila v = nu, avem: (t) =
1
n
1
n
_
nt
sin v
v

dv. Aplicat ia con-


tinua : x
x
_
1
sin v
v

dv din R

+
n R se poate prelungi prin continuitate la R
+
.
Folosind convergent a integralei

_
1
sin v
v

dv, ea admite o limita nita la . Exista deci


M = sup
xR
+
[(x)[ . Deducem atunci ca
[b
n
[ =
2


1
n
1
_
_

_
0
ch t ((n) (nt))dt
_
_

2M sh
1
n
1
.
2) Pentru orice x
0
, Z, f este derivabila n x
0
. Atunci seria Fourier asociata lui
f converge n x
0
, avand drept suma f(x
0
). Pentru x
0
Z, seria Fourier este vizibil
convergenta n x
0
, cu suma 0, iar n punctele de acest tip f(x
0
) = 0.

In concluzie, seria
Fourier asociata lui f converge simplu pe R, avand drept suma f.
Siruri si serii de funct ii: serii Taylor, serii Fourier 157
Problema 11.26 Fie (b
n
)
nN
un sir descrescator de numere reale, cu limita 0; acestui
sir i asociem seria trigonometrica

n=1
f
n
, unde f
n
(x) = b
n
sin nx.
1) Aratat i ca seria converge simplu pe R, si uniform pe orice interval compact din R
care nu cont ine multipli de 2.
2) Stabilit i echivalent a urmatoarelor armat ii:
(i)

n=1
f
n
converge uniform pe R;
(ii) b
n
= O
_
1
n
_
n vecinatatea lui .
3) Presupunem ca b
n
= O
_
1
n
_
n vecinatatea lui . Aratat i ca suma seriei

n=1
f
n
este
marginita si local integrabila pe R; gasit i seria Fourier asociata funct iei suma.
Solut ie. 1) (a) Pentru tot i x Z, seria numerica

n=1
f
n
(x), nula, este convergenta.
Notam S
n
(x) :=
n

k=1
sin kx, n N

, x R. Atunci (folosind transformarea lui Abel) putem


scrie pentru orice n, p N
s

k=1
f
n+k
(x) =
s

k=1
(b
n+k
b
n+k+1
)S
n+k
(x) b
n+1
S
n
(x) +b
n+p+1
S
n+p
(x).
Folosind formula clasica S
n
(x) =
sin
nx
2
sin
(n+1)x
2
sin
x
2
, daca x , 2Z, obt inem ca [S
n
(x)[
1

sin
x
2

, n N, x , 2Z. T inand cont si de faptul ca b


n+k
b
n+k+1
0, obt inem ca
pentru orice x R 2Z

k=1
f
n+k
(x)

k=1
(b
n+k
b
n+k+1
) [S
n+k
(x)[ +b
n+1
[S
n
(x)[ +b
n+p+1
[S
n+p
(x)[

2b
n+1

sin
x
2

, n, p N. (11.31)
Deci, seria numerica

n=1
f
n
(x) satisface criteriul lui Cauchy, deci este convergenta.

In concluzie,

n=1
f
n
este convergenta punctual pe R. Fie f suma sa. Se observa ca f
este periodica de perioada 2 si impara.
(b) Periodicitatea lui f permite reducerea problemei la a arata convergent a uniforma a
lui

n=1
f
n
pe un interval de forma [, 2 ], cu (0, ). Avem, n virtutea monotoniei
funct iei sin pe intervalul
_

2
,

2

,
sup
x[,2]

k=1
f
n+k
(x)

2b
n+1
sin

2
.
Cum lim
n
b
n
= 0,

n=1
f
n
verica criteriul lui Cauchy de convergent a uniforma pe [, 2],
deci converge uniform pe acest interval.
Mai remarcam, n virtutea transferului de continuitate, ca f este continua pe R 2Z.
158
De asemenea, folosind inegalitatea sin
x
2

x

pentru x [0, ], majorarea (11.31) arata


ca

k=n+1
f
k
(x)

2b
n+1
x
, n N, x (0, ]. (11.32)
2) (i) (ii) : Prin ipoteza,

n=1
f
n
converge uniform pe R. Avem, n particular,
lim
n
_
sup
xR

2n

k=n+1
b
k
sin kx

_
= 0,
de unde
lim
n
_
2n

k=n+1
b
k
sin
k
4n
_
= 0.
Cum, pentru orice numar natural din intervalul [n + 1, 2n], avem ca
0 b
2n
sin

4
b
k
sin
k
4n
,
rezulta ca lim
n
(nb
2n
) = 0. Cum 0 b
2n+1
b
2n
, avem si ca lim
n
(nb
2n+1
) = 0. Deducem
de aici ca lim
n
(nb
n
) = 0.
(ii) (i) : Prin ipoteza, lim
n
(nb
n
) = 0.
Fie > 0. Exista atunci n
0
N

astfel ncat 0 nb
n
, pentru orice n n
0
.
Consideram x (0, ] si n n
0
. Notand cu p partea ntreaga a lui

x
, putem scrie:

k=n
f
k
(x)

n+p1

k=n
[f
k
(x)[ +

k=n+p
f
k
(x)

.
Majorand [sin kx[ prin kx, x prin

p
, si kb
k
prin , obt inem
n+p1

k=n
[f
k
(x)[ =
n+p1

k=n
b
k
[sin kx[
n+p1

k=n
b
k
kx
n+p1

k=n

p
= .
Folosind acum (11.32), si majorand

x
prin n +p = n +
_

x
_
, obt inem ca

k=n+p
f
k
(x)

2b
n+p
x
2(n +p)b
n+p
2.

In concluzie, pentru orice x (0, ], avem

k=n
f
k
(x)

( + 2).
Cum aceasta inegalitate este satisfacuta n mod trivial n punctul 0, folosind de asemenea
paritatea si periodicitatea, este satisfacuta n orice x R. Rezulta cu seria

n=1
f
n
converge
uniform pe R.
Siruri si serii de funct ii: serii Taylor, serii Fourier 159
3) Aplicam concluzia de la punctul 2). Stim ca M > 0 astfel ncat, pentru orice
n N

, nb
n
M. Fie x (0, ]. Notand iarasi cu p partea ntreaga a lui

x
, avem ca
[f(x)[
s

k=1
[f
k
(x)[ +

k=p+1
f
k
(x)

.
Majorand din nou [sin kx[ prin kx, x prin

p
, si kb
k
prin M, obt inem ca
s

k=1
[f
k
(x)[ M.
Folosind din nou (11.32), si majorand

x
prin p + 1, avem

k=p+1
f
k
(x)

2b
p+1
x
2(p + 1)b
p+1
2M.
Inegalitatea [f(x)[ ( + 2)M este deci adevarata pentru orice x (0, ]; cum este
evident satisfacuta n x = 0, este adevarata pe Rn virtutea paritat ii si a periodicitat ii lui
f. Aplicat ia f : R R este deci m arginita.
Restrict ia lui f la intevalul [0, 2] este marginita si are cel mult doua puncte de dis-
continuitate (0 si 2); ea este deci integrabila; rezulta ca f este local integrabila si ca este
dezvoltabila n serie Fourier. Seria Fourier asociata va o serie numai de sinusuri, pe care
o vom nota

n=1
c
n
sin nx. Calculam c
m
pentru ecare m N

xat. Avem

2
c
m
=

_
0
f(t) sin mt dt =

_
0
_

n=1
g
n
(t)
_
dt,
unde g
n
(t) = b
n
sin nt sin mt. Este evident ca

n=1
g
n
este convergenta simplu pe [0, ] si are
ca suma funct ia x f(x) sin mx. Vom arata ca aceasta convergent a este uniforma.
Studiem T
n
(x) :=

k=n+1
g
k
(x) = sin mx

k=n+1
f
k
(x). T
n
(0) = 0 e trivial. Pentru x
(0, ] folosim (11.32) si obt inem
[T
n
(x)[ = [sin mx[

k=n+1
f
k
(x)

mx
2b
n+1
x
= 2mb
n+1
.
Rezulta sup
x[0,]
[T
n
(x)[ 2mb
n+1
. Dar b
n+1
0.
Folosind acum convergent a uniforma, putem scrie

2
c
m
=

n=1
_
_

_
0
b
n
sin nxsin mx dx
_
_
.
Cum

_
0
sin nxsin mx dx =
mn

2
, rezulta ca b
m
= c
m
.
Seria Fourier asocita lui f este deci

n=1
f
n
.
160
Problema 11.27 Fie
p
:=
1

2p

n=1
1
n
2p
, p N

. Aratat i ca
p
Q

+
.
Solut ie. Fie f
2k1
, f
2k
, p N

, periodice de perioada 2 de la R la R, denite prin


f
2k1
(x) = x
2k1
, daca x (, ), f
2k1
() = 0,
f
2k
(x) = x
2k
, daca x (, ].
Aceste funct ii sunt local integrabile si admit (respectiv) dezvoltarile n serii Fourier

n=1
b
k
n
sin nx si
a
k
0
2
+

n=1
a
k
n
cos nx.
Folosind formulele lui b
k
n
si a
k
n
, obt inem (prin integrare prin part i) ca, pentru orice n N

,
b
k+1
n
= (1)
n+1
2
2k
n

2k(2k + 1)
n
2
b
k
n
,
a
k
n
=
2k
n
b
k
n
.
Prin calcul direct, gasim ca b
1
n
= (1)
n+1
2
n
. Prin recurent a dupa k N

, se deduce ca
exista
k,l
Z si
k,l
Z, independente de n, astfel ncat
(1)
n
b
k
n
=
k1

l=0

k,l

2l
n
2k12l
; (1)
n
a
k
n
=
k1

l=0

k,l

2l
n
2k2l
,
pentru orice n, k N

. De asemenea, obt inem direct ca a


k
0
=
2
2k
2k + 1
.
Pentru k N

xat, avem, din formula lui Parseval:


1
2

n=1
(b
k
n
)
2
=
1

_
0
t
4k2
dt =

4k2
4k 1
, (11.33)
1
2

n=1
(a
k
n
)
2
=

4k
4k + 1

1
4
(a
k
0
)
2
.
Pentru k = 1, deducem ca

1
=
1
6
si
2
=
1
90
. (11.34)
Pentru k 2, avem, notand
k
= 0, 1, . . . , k 1
2
:

n=1
(b
k
n
)
2
=

(l,l

)
k

k,l

k,l

2(l+l

n=1
1
n
4k22(l+l

)
.
Din (11.33) si din denit ia lui
p
, avem ca

(l,l

)
k

k,l

k,l

2k1(l+l

)
Q.

In acelasi mod

(l,l

)
k

k,l

k,l

2k(l+l

)
Q.
Folosind (11.34), deducem prin recurent a ca
p
Q

+
, p N

.
Siruri si serii de funct ii: serii Taylor, serii Fourier 161
Problema 11.28 (Teorema lui Riemann) 1) Fie f : [a, b] R o funct ie integrabila n
sens propriu sau absolut integrabil a n sens impropriu pe intervalul compact [a, b]. Sa se
arate ca:
lim
t
b
_
a
f(x) sin tx dx = 0 si lim
t
b
_
a
f(x) cos tx dx = 0.
2) Fie g : [l, l] R, integrabila n sens propriu sau absolut integrabila n sens
impropriu pe [l, l]. Aratat i ca a
n
si b
n
, coecient ii Fourier corespunzatori lui g, satisfac
lim
n
a
n
= lim
n
b
n
= 0. Deducet i un criteriu necesar pentru ca o serie trigonometrica sa
e serie Fourier.
Solut ie. 1) Observam ca, pentru orice interval compact [c, d], avem

d
_
c
sin tx dx

cos tc cos td
t

2
[t[
. (11.35)
Presupunem mai ntai ca funct ia f este integrabila n sens propriu pe [a, b]. Fie a = x
0
<
x
1
< < x
n1
< x
n
= b o diviziune a intervalului [a, b]; notam m
i
= inf
x[a,b]
f(x),
M
i
= sup
x[a,b]
f(x) si
i
= M
i
m
i
. Atunci, t inand seama si de (11.35), avem

b
_
a
f(x) sin tx dx

n1

i=0
x
i+1
_
x
i
f(x) sin tx dx

n1

i=0
x
i+1
_
x
i
[f(x) m
i
] sin tx dx +
n1

i=0
m
i
x
i+1
_
x
i
sin tx dx

n1

i=0

i
(x
i
x
i1
) +
2
[t[
n1

i=0
[m
i
[ .
Cum f este integrabila pe [a, b], obt inem din criteriul lui Darboux ca pentru orice > 0,
exista

> 0 astfel ncat, pentru orice diviziune cu norma mai mica decat

, diferent a
sumelor Darboux este mai mica decat

2
. Presupunem ca norma diviziunii alese de noi este
mai mica decat

, si obt inem deci ca


n1

i=0

i
(x
i
x
i1
) <

2
. Notam t

=
4

n1

i=0
[m
i
[ . Atunci
2
t
n1

i=0
[m
i
[ <

2
, t > t

.
Asadar, > 0, t

R, t > t

b
_
a
f(x) sin tx dx

< . Deci, lim


t
b
_
a
f(x) sin tx dx = 0.
Sa presupunem acum ca f este absolut integrabila n sens impropriu pe [a, b]. Cum
[f(x) sin tx[ [f(x)[ , x [a, b], t R, folosind criteriul comparat iei, deducem ca in-
tegrala
b
_
a
f(x) sin tx dx este absolut convergenta n sens impropriu pe [a, b]. Pentru a re-
zolva problema, este sucient sa consideram cazul n care aceasta este de tipul
b0
_
a
f(x)dx,
162
celelalte cazuri rezolvandu-se analog.

Intrucat integrala considerata este convergenta,
> 0, =

(a, b) astfel ncat

b
_

f(x) sin tx dx

<

2
, t R.
Dar cum pe intervalul compact [a, ] stim din demonstrat ia anterioara ca t

R, t > t

_
a
f(x) sin tx dx

<

2
, rezulta ca > 0, =

(a, b), t

R, t > t

b
_
a
f(x) sin tx dx

_
a
f(x) sin tx dx

b
_

f(x) sin tx dx

< .
De aici, lim
t
b
_
a
f(x) sin tx dx = 0. Pentru cealalta integrala, demonstrat iile sunt complet
analoage.
2) Folosind punctul 1), deducem imediat ca a
n
0, b
n
0. Sa remarcam faptul ca
acest lucru reiese si din inegalitatea lui Bessel sau din egalitatea lui Parseval, dar numai
pentru funct iile de patrat integrabil n sens impropriu sau integrabile n sens propriu. Dupa
cum se poate vedea examinand funct ia g : [0, 1] R, g(x) =
1

x
, daca x (0, 1], si g(0) =
0, nu orice funct ie de patrat integrabil n sens impropriu este integrabila n sens impropriu.
Reciproca este adevarata, dupa cum ne arata inegalitatea [h(x)[
1 +h
2
(x)
2
, x [a, b].

In concluzie, un criteriu necesar ca o serie trigonometrica sa e serie Fourier este ca sirurile


(a
n
) si (b
n
) formate din coecient ii ei sa convearga la 0.
Problema 11.29 1) (Nucleele Dirichlet si Fejer) Fie seria divergenta
1
2
+

n=1
cos nx. (11.36)
Calculat i sumele
s
n+1
(x) =
1
2
+
n

k=1
cos kx si
n+1
(x) =
s
0
(x) +s
1
(x) + +s
n
(x)
n + 1
.
Funct ia s
n+1
(x) se numeste nucleul lui Dirichlet, iar funct ia
n+1
(x) se numeste nucleul
lui Fejer.
Aratat i ca pentru orice x ,= 2k, seria (11.36) este (C, 1) sumabila catre 0. Vericat i ca
1

s
n+1
(t)dt =
1

n+1
(t)dt = 1.
(O serie se numeste (C, 1)sumabila daca sirul mediilor aritmetice ale sumelor sale
part iale este convergent.)
2) Fie seria

n=1
sin nx. (11.37)
Aratat i ca este (C, 1)sumabila, cu (C, 1)suma =
_
1
2
, daca x ,= 2k
0, daca x = 2k.
.
Siruri si serii de funct ii: serii Taylor, serii Fourier 163
Solut ie. 1) Se obt ine s
n+1
(x) =
sin(n +
1
2
)x
2 sin
x
2
si

n+1
(x) =
1
n + 1

1
2 sin
x
2
_
sin
x
2
+ sin
3x
2
+ + sin(n +
1
2
)x
_
=
1
n + 1
sin
2
(n +
1
2
)x
2 sin
2 x
2
.
Pentru orice x ,= 2k avem lim
n

n+1
(x) = 0. Vericarea integralelor rezulta direct.
2) Se obt ine
S
n
(x) =
n

k=1
sin kx =
cos
x
2
cos
_
n +
1
2
_
x
2 sin
x
2
,
S
1
(x) +S
2
(x) + +S
n
(x)
n
=
1
2
ctg
x
2

1
n
sin(n + 1)x sin x
4 sin
2 x
2
,
de unde concluzia.
Problema 11.30 (Integralele Dirichlet si Fejer) Fie f : [, ] R o funct ie periodica
de perioada 2. Notam cu s suma seriei Fourier asociate lui f, iar cu s
n
suma part iala
s
n
(x) =
a
0
2
+
n

k=1
(a
k
cos kx +b
k
sin kx) .
Aratat i ca
s
n
(x) =
1
2

_
0
sin
_
n +
1
2
_
u
sin
u
2
[f(x +u) +f(x u)]du, (11.38)

n
(x) =
s
0
(x) +s
1
(x) + +s
n1
(x)
n
=
1
2n

_
0
sin
2 nu
2
sin
2 u
2
[f(x +u) +f(x u)]du. (11.39)
Calculat i apoi diferent ele s
n
(x) s(x) si
n
(x) s(x).
Integralele (11.38) si (11.39) se numesc, respectiv, integralele Dirichlet si Fejer.
Solut ie. Folosind expresiile integrale ale coecient ilor a
k
si b
k
, precum si problema
anterioara, obt inem
s
n
(x) =
1

f(u)
_
1
2
+
n

k=1
cos k(u x)
_
du
=
1

f(u)
sin(2n + 1)
ux
2
2 sin
ux
2
du.
Cumn integrala precedenta funct iile de u care apar sunt periodice de perioada 2, valoarea
integralei nu se modica daca schimbam intervalul de integrare n [x , x + ]. Atunci,
164
folosind si schimbarea de variabila t = u x, obt inem
s
n
(x) =
1

x+
_
x
f(u)
sin(2n + 1)
ux
2
2 sin
ux
2
du =
1

f(x +t)
sin(n +
1
2
)t
2 sin
t
2
dt
=
1

_
_
0
_

f(x +t)
sin(n +
1
2
)t
2 sin
t
2
dt +

_
0
f(x +t)
sin(n +
1
2
)t
2 sin
t
2
dt
_
_
=
1

_
0
[f(x +t) +f(x t)]
sin(n +
1
2
)t
2 sin
t
2
dt,
adica (11.38) este aratata.
Pentru calculul lui
n
(t), t inem seama de (11.38), de proprietatea de aditivitate a
integralei si de formula (folosita mai sus)
sin
x
2
+ sin
3x
2
+ + sin(n +
1
2
)x =
sin
2
(n +
1
2
)x
2 sin
2 x
2
.
Daca f 1 n (11.38), atunci tot i coecient ii Fourier asociat i ei sunt 0, n afara de a
0
,
care este 2. Prin urmare, suma part iala de rang n asociata ei este identic egala cu 1. Din
relat ia (11.38) deducem atunci ca
2

_
0
sin(n +
1
2
)t
2 sin
t
2
dt = 1. De aici, folosind aceeasi relat ie,
obt inem
s
n
(x) s(x) =
1

_
0
[f(x +t) +f(x t) 2s(x)]
sin(n +
1
2
)t
2 sin
t
2
dt. (11.40)
Cu un calcul similar care porneste de la formula (11.39), si folosind identitatea
1
2n

_
0
sin
2 nt
2
sin
2 t
2
dt = 1, obt inem

n
(x) s(x) =
1
2n

_
0
[f(x +t) +f(x t) 2s(x)]
sin
2 nt
2
2 sin
2 t
2
dt. (11.41)
Problema 11.31 Fie f ca n problema anterioara. Folosind formula integralei Dirichlet,
sa se arate ca daca f este marginita, exista A, M > 0 astfel ncat are loc evaluarea Lebesgue

a
0
2
+
n

k=1
(a
k
cos kx +b
k
sin kx)

< AM ln n.
Solut ie. Deoarece f este marginita, exista M > 0 astfel ncat [f(x)[ M, x [, ].

In formula (11.38) folosim aceasta inegalitate, precum si faptul ca sin


x
2

x

, x [, ].
Obt inem succesiv
[s
n
(x)[
M

_
0

sin
_
n +
1
2
_
u

sin
u
2
du M

_
0

sin
_
n +
1
2
_
u

u
du
=
M
n +
1
2
(n+
1
2
)
_
0
[sin t[
t
dt.
Siruri si serii de funct ii: serii Taylor, serii Fourier 165
Cum
[sin t[
t
1, daca x (0, 1], lim
t0
+
[sin t[
t
= 1, si cum
[sin t[
t

1
t
, daca x > 1, avem
[s
n
(x)[ M
1
_
0
dt +M
(n+
1
2
)
_
1
dt
t
= M
_
1 + ln
_
n +
1
2
_
t
_
M(1 + ln n + ln 2).
Fie A > 1 +
1 + ln 2
ln 2
. Atunci 1 + ln n + ln 2 < Aln n. De aici, concluzia.
Problema 11.32 (Teorema localizarii) Sa se arate ca, data o funct ie absolut integra-
bila f, comportarea seriei Fourier asociata lui f, ntr-un punct x
0
, depinde exclusiv de
comportarea funct iei f ntr-o vecin atate a acestui punct.
Solut ie. Fie (0, ). Consideram funct ia g : [x
0
, x
0
+] R, data prin
g(x) =
_
f(x), daca x (x
0
, x
0
+)
0, dac a x [x
0
, x
0
) (x
0
+, x
0
+].
Folosind (11.38), obt inem succesiv sumele part iale pentru f si pentru g
s
n
(x
0
) =
1
2

_
0
sin
_
n +
1
2
_
u
sin
u
2
[f(x
0
+u) +f(x
0
u)]du
S
n
(x
0
) =
1
2

_
0
sin
_
n +
1
2
_
u
sin
u
2
[g(x
0
+u) +g(x
0
u)]du
=
1
2

_
0
sin
_
n +
1
2
_
u
sin
u
2
[f(x
0
+u) +f(x
0
u)]du.
Atunci, diferent a acestora este
s
n
(x
0
) S
n
(x
0
) =
1
2

sin
_
n +
1
2
_
u
sin
u
2
[f(x
0
+u) +f(x
0
u)]du.
Cum funct ia
1
sin
u
2
[f(x
0
+ u) + f(x
0
u)] este absolut integrabila pe [, ], rezulta n
baza teoremei lui Riemann (vezi Problema 11.28) ca
lim
n
(s
n
(x
0
) S
n
(x
0
)) = 0.
De aici, se deduce faptul ca pentru > 0 arbitrar de mic, comportarea sumelor s
n
(x
0
)
depinde de comportarea funct iei f n intervalul (x
0
, x
0
+ ), si nu de valorile din
exteriorul acestui interval.
Problema 11.33 (Criteriul lui Dini) Fie f : R R o funct ie periodica de perioada 2 si
integrabila n sens propriu sau absolut integrabila n sens impropriu pe intervalul [, ],
x
0
un punct din acest interval unde f are limite laterale nite si
s
0
=
f(x
0
+ 0) +f(x
0
0)
2
.
166
Sa se arate ca daca exista un numar a (0, ] astfel ncat integrala
a
_
0
[f(x
0
+t) +f(x
0
t) 2s
0
[
t
dt
sa e convergenta, atunci seria Fourier asociata funct iei f converge n punctul x
0
catre s
0
.
Solut ie. Fie funct ia h : [0, ] R, h(t) =
f(x
0
+t) +f(x
0
t) 2s
0
2 sin
t
2
, daca t (0, ],
h(0) = 0. Atunci, cum lim
t0
t
2 sin
t
2
= 1, rezulta ca pentru = 1, > 0, t a.. [t[ < :

t
2 sin
t
2
1

< 1, sau

t
2 sin
t
2

< 2. Fie acum t (0, min, ) . Avem


[h(t)[ =

f(x
0
+t) +f(x
0
t) 2s
0
t

t
2 sin
t
2

<
2 [f(x
0
+t) +f(x
0
t) 2s
0
[
t
.
T inand seama de ipoteza, deducem ca

_
0
h(x)dx este absolut convergenta. Atunci, n vir-
tutea Teoremei lui Riemann si relat iei (11.40), avem
lim
n
(s
n
(x
0
) s
0
) = lim
n
1

_
0
h(t) sin(n +
1
2
)tdt = 0,
de unde concluzia.
Sa mai observam doar ca daca f este continua n x
0
, atunci s
0
= f(x
0
).
Problema 11.34 (Criteriul lui Lipschitz) Fie f : R R o funct ie periodica de perioada
2, integrabila n sens propriu sau absolut integrabila n sens impropriu pe intervalul
[, ] si x
0
R. Daca n punctul x
0
funct ia f este continua si daca exista constantele
pozitive M, si astfel ncat, pentru orice t (0, ),
[f(x
0
t) f(x
0
)[ Mt

,
atunci seria Fourier asociata lui f converge n punctul x
0
catre f(x
0
).
Solut ie. Notam g(t) := f(x
0
+t) +f(x
0
t) 2f(x
0
). Atunci

_
0
[g(t)[
t
dt

_
0
[f(x
0
+t) f(x
0
)[
t
dt +

_
0
[f(x
0
t) f(x
0
)[
t
dt

_
0
2Mt
1
dt =
2M

.
Rezulta ca

_
0
|g(t)|
t
dt este convergenta si deci, n baza criteriului lui Dini (problema 11.33),
concluzia.
Siruri si serii de funct ii: serii Taylor, serii Fourier 167
Problema 11.35 (Teorema lui Fejer) Fie f ca n Problema 11.30. Presupunem ca
lim
n
1
2n

_
0
sin
2 nu
2
sin
2 u
2
[f(x +u) +f(x u) 2s(x)]du = 0.
Sa se arate ca seria Fourier asociat a lui f este (C, 1)sumabila catre s(x).
Daca, n plus, f este marginita, atunci seria Fourier asociata lui f este (C, 1)sumabila
catre
f(x + 0) +f(x 0)
2
n punctele x n care limitele laterale ale funct iei sunt nite, si
(C, 1)sumabila catre f(x) n punctele x n care f este continua.
Solut ie. Prima parte rezulta direct din formula (11.41) si din denit ia
(C, 1)sumabilitat ii.
Pentru partea a doua, demonstrat ia este asemanatoare cu cea a Criteriului Dini. Notam
(u) := f(x +u) +f(x u) 2s(x). Fie x un punct n care f are limitele laterale nite.
Punand s(x) =
f(x + 0) +f(x 0)
2
, constatam ca lim
u0
(u) = 0. Atunci, > 0, =

(0, ), u a.. u : [(u)[ < . Fixand , avem (din (11.41))

n
(x) s(x) =
1
2n

_
0
(u)
sin
2 nu
2
2 sin
2 u
2
du +
1
2n

(u)
sin
2 nu
2
2 sin
2 u
2
du.
Notand cu I
1
si I
2
cele doua integrale de mai sus, folosind faptul ca [(u)[ M pe (, ),
si totodata egalitatea
1
2n

_
0
sin
2 nu
2
sin
2 u
2
du = 1, avem
[I
1
[
1
2n

_
0
sin
2 nu
2
sin
2 u
2
[(u)[ du

2n

_
0
sin
2 nu
2
sin
2 u
2
du =

2
,
[I
2
[
1
2n

sin
2 nu
2
sin
2 u
2
[(u)[ du
M
2nsin
2
2
.

_
0
du =
M
2nsin
2
2
.
Din aceste doua relat ii, deducem c a, pentru n sucient de mare,
[
n
(x) s(x)[ .
De aici, concluzia.
Problema 11.36 Sa se arate ca funct ia f : R R data prin
f(x) =
_
ln

cos
x
2

, daca x ,= (2k + 1), k Z


0, daca x = (2k + 1), k Z
este dezvoltabila n serie Fourier pe R (2k + 1) [ k Z si sa se determine aceasta
dezvoltare.
Solut ie. Funct ia f este periodica de perioada 2 si para, deci este sucient sa studiem
problema pe [0, ]. Cum f este nemarginita pe acest interval, trebuie aratat mai ntai ca f
este absolut integrabila n sens impropriu pe [0, ]. Cum f este continua pe orice interval
de forma [0, x], cu x [0, ), ea este integrabila Riemann pe acest interval. Mai observam,
168
aplicand regula lui lHospital, ca lim
x

x[f(x)[ = 0, de unde, n virtutea criteriului


n , f este absolut integrabila pe [0, ], deci i putem determina coecient ii Fourier. Fie
x
0
[0, ) si e un interval compact [x
0
, x
0
+ ] (, ). Cum f este derivabila
cu derivata continua pe (, ), rezulta ca f este lipschitziana pe [x
0
, x
0
+ ]. Prin
urmare, f verica toate condit iile din Criteriul lui Lipschitz (Problema 11.34), deci seria
Fourier asociata ei converge n punctul x
0
catre f(x
0
). De aici, cum x
0
a fost ales arbitrar
din [0, ), folosind paritatea si periodicitatea funct iei f, rezulta ca este dezvoltabila n serie
Fourier numai de cosinusuri pe R (2k + 1) [ k Z.

In plus,
a
n
=
2

_
0
f(x) cos nx dx =
2

_
_
sin nx
n
ln cos
x
2

0
+
1
2n

_
0
sin nxtg
x
2
dx
_
_
.
Folosind faptul ca lim
x
sin nxln cos
x
2
= 0, facand schimbarea de variabila x = y si
t innd cont de formulele
sin ny cos
y
2
=
1
2
_
sin
_
n +
1
2
_
y + sin
_
n
1
2
_
y
_
,
n

k=1
cos kx =
sin
_
n +
1
2
_
x sin
x
2
2 sin
x
2
,
avem
a
n
=
(1)
n+1
n
_
_

_
0
_
1
2
+
n

k=1
cos ky
_
dy +

_
0
_
1
2
+
n1

k=1
cos ky
_
dy
_
_
=
(1)
n+1
n
.
De aici,
f(x) =
a
0
2
+

n=1
(1)
n+1
n
cos nx, x ,= (2k + 1), k Z.
Pentru x = 0 n relat ia de mai sus, obt inem a
0
= 2

n=1
(1)
n+1
n
= 2 ln 2.

In punctele
de forma (2k + 1), k Z seria din membrul drept este divergenta, deci egalitatea nu are
loc.
Problema 11.37 (Hurwitz) Fie mult imea tuturor curbelor simple si nchise care au
aceeasi lungime l. Presupunand ca ecuat iile lor parametrice pot scrise sub forma seriilor
Fourier
x = x(s) =
a
0
2
+

n=1
_
a
n
cos
2n
l
s +b
n
sin
2n
l
s
_
,
y = y(s) =

0
2
+

n=1
_

n
cos
2n
l
s +
n
sin
2n
l
s
_
,
unde s este chiar arcul de curba (0 s < l), sa se arate ca dintre toate curbele familiei
cercul este curba care nchide aria maxima.
Siruri si serii de funct ii: serii Taylor, serii Fourier 169
Solut ie. Fie . Lungimea sa l si aria A a mult imii pe care o margineste sunt date
de formulele
l =
_

ds =
l
_
0
_
[x

(t)]
2
+ [y

(t)]
2
dt,
A =
_

xdy ydx =
1
2
l
_
0
_
x(t) y

(t) y(t) x

(t)
_
dt.
Derivatele x

(t) si y

(t) sunt date de formulele


x

(t) =
2
l

n=1
_
na
n
sin
2n
l
t +nb
n
cos
2n
l
t
_
,
y

(t) =
2
l

n=1
_
n
n
sin
2n
l
t +n
n
cos
2n
l
t
_
.
Dar, (folosind eventual faptul ca aplicat ia , : L
2
[0, l] L
2
[0, l] R, f, g :=
1
l
l
_
0
f(t)g(t)dt este un produs scalar pe spat iul liniar al claselor de funct ii de patrat integrabil
n sens Lebesgue pe [0, l]), se poate deduce ca
l =
l
_
0
[x

(t)]
2
+ [y

(t)]
2
dt.
Folosind acum formula lui Parseval si cea data de Problema 11.24, obt inem din for-
mulele lungimii si ariei de mai sus ca
l =
2
2
l

n=1
n
2
_
a
2
n
+b
2
n
+
2
n
+
2
n
_
,
A =

n=1
n(a
n

n

n
b
n
) .
Calculam diferent a dintre aria cercului de lungime l si aria A marginita de o curba de
aceeasi lungime cu cercul. Avem
l
2
4
A =

2

n=1
n
2
_
a
2
n
+b
2
n
+
2
n
+
2
n
_

n=1
n(a
n

n

n
b
n
)
=

2

n=1
[(na
n

n
)
2
+ (nb
n
+
n
)
2
+ (n
2
1)(
2
n
+
2
n
)] 0.
Diferent a
l
2
4
A se anuleaza cand a
1
=
1
, b
1
=
1
, a
n
= b
n
=
n
=
n
= 0, n =
2, 3, . . . .

In acest caz aria marginit a de curba devine maxima, iar ecuat iile parametrice
ale acestei curbe sunt
x = x(s) =
a
0
2
+a
1
cos
2
l
s +b
1
sin
2
l
s,
y = y(s) =

0
2
b
1
cos
2
l
s +a
1
sin
2
l
s,
ceea ce reprezinta cercul de ecuat ie implicita
_
x
a
0
2
_
2
+
_
y

0
2
_
2
= 2(a
2
1
+b
2
1
).
Capitolul 12
Funct ii complexe
Denit ii si rezultate
Numere complexe
Se noteaza C mult imea numerelor complexe z = x + iy, unde x, y R iar i
2
= 1.
x = Re z si y = Im z se numesc, respectiv partea reala si partea imaginara a numarului
complex z.
Denit ii. Fie z = x +iy un numar complex. Se noteaza
z = x iy
si se numeste conjugatul numarului complex z. Au loc relat iile:
Re z =
z +z
2
; Im z =
z z
2i
Se noteaza [z[ =
_
(x
2
+y
2
si se numeste modulul numarului complex z.
Denit ie si teorema. Fie z C, z ,= 0. Exista si este unic (, ], numit
argumentul numarului complex z si notat arg z, astfel ncat:
cos =
Re z
[z[
; sin =
Im z
[z[
Intervalul (, ] se poate nlocui cu [0, 2) sau cu orice alt interval de lungime 2.
O expresie explicita a argumentului este:
arg z =
_
2arctg
y
x +[z[
, x +[z[ , = 0
, x +[z[ = 0
Scrierea z = [z[ (cos +i sin ) poarta numele de forma trigonometrica a numarului
z C 0. Orice alegere = arg z + 2k, k Z convine). Deoarece
(cos +i sin )(cos +i sin ) = cos( +) +i sin( +)
deducem formula lui Moivre:
(cos +i sin )
n
= cos n +i sin n, n Z
1.3. Observat ie. Identicand R cu o dreapta (raportata la un reper), urmeaza ca
C = R R se identica cu un plan (raportat la un reper).
170
Funct ii complexe 171
Asadar, z (z) reprezinta simetria fat a de punctul O; z z reprezinta simetria fat a
de axa Ox; z z +a reprezinta translat ia de vector a (a C); z z ( > 0) reprezinta
omotetia de centru O si de raport ; z az (a C, [a[ = 1) reprezinta rotat ia de centru
O si de unghi arg a.

In sfarsit, deoarece
1
z
=
z
[z[
2
urmeaza ca z
1
z
reprezinta inversiunea
de pol O si de putere 1, urmata de simetria fat a de axa Ox.

In unele aplicat ii, este comod sa introducem un punct, notat , C. Nu vom deni
operat ii algebrice pe mult imea C (care va numita planul complex extins). Sirul
(z
n
)
n
din C are limita daca si numai daca [z
n
[ . Evident: z
n

1
z
n
0.
Transformari omograce
Denit ie. Cu ecare numere a, b, c, d C, vericand ad bc ,= 0, asociem funct ia
f : C C , denita astfel: daca c ,= 0, atunci
f(z) =
_

_
az +b
cz +d
, z C d/c
, z = d/c
a/c, z =
respectiv, daca c = 0
f(z) =
_
az +b
d
, z C
, z =
Funct iile de aceasta forma sunt numite transformari omograce, iar z =
d
c
se
numeste polul transformarii.
Observat ii. Este evident ca transformarile omograce sunt funct ii continue. Cerint a
ca ad bc ,= 0 este echivalenta cu faptul ca f este neconstanta.
Fiecare transformare omograca este funct ie olomorfa n C d/c (respectiv n C,
daca c = 0), iar:
f

(z) =
ad bc
(cz +d)
2
,= 0

In particular, transformarile omograce pastreaza unghiurile, n afara polului.


Propozit ie. Mult imea transformarilor omograce formeaza grup fat a de compunerea
funct iilor.
Propozit ie. Fiecare transformare omograca apare ca o compunere de urmatoarele
tipuri: (i) z z + a (translat ie); (ii) z az (a ,= 0, rotat ie n jurul originii, de unghi
arg a si omotetie de centru 0 si de raport [a[); (iii) z z
1
(inversiune de pol 0 si de
putere 1, urmata de simetrie fat a de axa Ox).
Propozit ie. Fiind date doua triplete (z
1
, z
2
, z
3
), (w
1
, w
2
, w
3
) din C , formate
din puncte distincte doua cate doua, exista si este unica o transformare omograca f,
astfel ncat f(z
k
) = w
k
, k = 1, 2, 3.
Lema. Fiecare transformare omograca, diferita de identitate, are cel mult doua
puncte xe.
Propozit ie. Transformarile omograce aplica: cercurile si dreptele care trec prin polul
transformarii n drepte; iar cercurile si dreptele care nu trec prin pol n cercuri.
Propozit ie. Fie
1
,
2
doua cercuri sau drepte. Exista transformari omograce care
sa aplice
1
pe
2
.
172
Probleme
Problema 12.1 Fie f : C C denita prin f(z) =
z
1 +[z[
. Sa se arate ca f stabileste
o biject ie ntre C si o mult ime , care se va determina. Sa se scrie explicit inversa sa.
Solut ie. Din ecuat ia f(z) = w, adica
z
1 +[z[
= w se deduce: [z[ =
[w[
1 [w[
daca
si numai daca [w[ < 1. Revenind n ecuat ia init iala, se obt ine z =
w
1 [w[
. Asadar
= w C[ [w[ < 1 iar f
1
(w) =
w
1 [w[
.
Problema 12.2 Fie z
1
, z
2
, . . . , z
n
C. Sa se arate ca ca exista J 1, 2, . . . , n, astfel
ncat

jJ
z
j

1
4

2
n

k=1
[z
k
[
Berkeley, 1990
Solut ie. Se partit ioneaza mult imea 1, 2, . . . , n n patru submult imi, dupa cum z
j
se
aa n unul din cele patru cadrane determinate de cele doua bisectoare. Va exista deci cel
put in o parte, din cele patru, pe care o notam cu J, pentru care:

jJ
[z
j
[
1
4
n

j=1
[z
j
[
Din faptul ca toate numerele z
j
apart in unui acelasi cadran are loc una din ine-
galitat ile: [Re z
j
[
1

2
[z
j
[ sau [Im z
j
[
1

2
[z
j
[. Mai departe, avem de asemenea

jJ
Re z
j

jJ
[Re z
j
[ sau

jJ
Im z
j

jJ
[Im z
j
[. Inegalitatea ceruta se deduce acum,
observand ca:

jJ
z
j

Re

jJ
z
j

jJ
Re z
j

sau

jJ
z
j

Im

jJ
z
j

jJ
Im z
j

Problema 12.3 Fie a, b, c C necoliniare. Daca (z


n
)
n
este un sir de numere complexe,
avand proprietatea ca ecare din sirurile: ([z
n
a[)
n
; ([z
n
b[)
n
; ([z
n
c[)
n
converg,
atunci (z
n
)
n
este convergent.
Solut ia 1. Sirul (z
n
)
n
rezulta marginit. Daca nu ar convergent, ar avea cel put in
doua subsiruri convergente, cu limite distincte, e acestea z ,= z

. Ar urma: [za[ = [z

a[
si analoagele. Adica a, b, c s-ar aa pe mediatoarea segmentului determinat de z si z

, n
contradict ie cu ipoteza de necoliniaritate.
Solut ia 2. Se obt ine prin calcul efectiv. Sa notam:
a
n
= [z
n
a[; b
n
= [z
n
b[; c
n
= [z
n
c[
Funct ii complexe 173
si:
A = lim
n
a
n
; B = lim
n
b
n
; C = lim
n
c
n
Avem:
_
_
_
z
n
z
n
az
n
az
n
= a
2
n
[a[
2
z
n
z
n
bz
n
bz
n
= b
2
n
[b[
2
z
n
z
n
cz
n
cz
n
= c
2
n
[c[
2
Considerat ca sistem liniar de trei ecuat ii cu necunoscutele: z
n
z
n
, z
n
, z
n
, solut ia este unica,
deoarece ipoteza de necoliniaritate asigura exact ca determinantul sistemului este nenul.
Se obt ine:
z
n
=

1 a
2
n
[a[
2
a
1 b
2
n
[b[
2
b
1 c
2
n
[c[
2
c

1 a a
1 b b
1 c c

de unde rezulta ca sirul (z


n
)
n
este convergent si are limita:

1 A
2
[a[
2
a
1 B
2
[b[
2
b
1 C
2
[c[
2
c

1 a a
1 b b
1 c c

Observat ii.

Intre A, B, C exista o relat ie, care se obt ine scriind ca solut ia z
n
z
n
a
sistemului este produsul solut iilor z
n
si z
n
.
Geometric, [z
n
a[ A nseamna ca sirul (z
n
)
n
are toate punctele de acumulare pe
cercul de centru a si de raza A. Cercurile de centre a, b, c nu pot avea mai mult decat un
punct comun, tocmai datorita ipotezei de necoliniaritate.
Problema 12.4 Sa se arate ca proiect ia stereograca, denita prin:
(x
1
, x
2
, x
3
)
x
1
+ix
2
1 x
3
daca x
3
,= 1
stabileste o biject ie bicontinua ntre planul complex extins C si sfera unitate din R
3
.
Sa se arate ca acesta corespondent a transforma orice cerc de pe sfera ntrun cerc sau
ntr-o dreapta din plan.
Solut ie. Notam cu S sfera unitate din R
3
, de ecuat ie x
2
1
+ x
2
2
+ x
2
3
= 1. Fie P S
punctul de coordonate (0, 0, 1) (polul nord)
Pentru un punct oarecare M SP, de coordonate (x
1
, x
2
, x
3
), dreapta determinata
de punctele P si M are ecuat ia
X
x
1
=
Y
x
2
=
Z 1
x
3
1
deci intersecteaza planul x
3
= 0 n punctul
_
x
1
1 x
3
,
x
2
1 x
3
, 0
_
. Vom deni deci aplicat ia
: S C prin
(x
1
, x
2
, x
3
) =
_
_
_
x
1
+ix
2
1 x
3
, daca x
3
,= 1
, daca x
3
= 1
174
Prin calcul sau geometric, se constata ca este o biject ie. Inversa este : C S
denita astfel:
(z) =
_
z +z
1 +[z[
2
,
z z
i(1 +[z[
2
)
,
[z[
2
1
[z[
2
+ 1
_
; () = (0, 0, 1)
Deoarece
lim
x
3
1
(x
1
, x
2
, x
3
) S

x
1
+i.x
2
1 x
3

2
= lim
x
3
1
(x
1
, x
2
, x
3
) S
1 +x
3
1 x
3
=
iar
lim
z
_
z +z
1 +[z[
2
,
z z
i(1 +[z[
2
)
,
[z[
2
1
[z[
2
+ 1
_
= (0, 0, 1)
rezulta ca este continua.
Geometric este evident ca orice cerc de pe sfera, ce trece prin P, este transformat
ntro dreapta din plan si reciproc. Scriind ecuat ia unui cerc de pe sfera ca intersect ia cu
planul

1
x
1
+
2
x
2
+
3
x
3
=
0
, unde
2
1
+
2
2
+
2
3
= 1 si 0
0
< 1
urmeaza ca imaginea n plan are ecuat ia:

1
(z +z) i.
2
(z z) +
3
([z[
2
1) =
0
([z[
2
+ 1)
ceea ce reprezinta un cerc sau o dreapta, dupa cum
0
,=
3
sau nu. Reciproc, dat ind
un cerc sau o dreapta din plan, de ecuat ie Azz + Bz + Bz + C = 0, coecient ii
k
se
determina unic, cu proprietat ile

2
1
+
2
2
+
2
3
= 1 si 0
0
< 1
Problema 12.5 Fie z
1
, z
2
, z
3
, z
4
C patru puncte distincte. Notam:
(z
1
, z
2
, z
3
, z
4
) =
z
1
z
2
z
1
z
4
:
z
3
z
2
z
3
z
4
(numit biraportul celor patru puncte).
(i) Sa se verice ca, pentru orice transformare omograca f, are loc:
(f(z
1
), f(z
2
), f(z
3
), f(z
4
)) = (z
1
, z
2
, z
3
, z
4
)
(ii) Fie z
1
, z
2
, z
3
, z
4
C respectiv w
1
, w
2
, w
3
, w
4
C puncte distincte. Sa
se arate ca exista o transformare omograca f astfel ncat f(z
k
) = w
k
, k = 1, 4 daca si
numai daca : (z
1
, z
2
, z
3
, z
4
) = (w
1
, w
2
, w
3
, w
4
).
(iii) Fie f o transformare omograca cu f
3
,= I. Sa se arate ca biraportul
(z, f(z), f
2
(z), f
3
(z)) nu depinde de z (daca z nu este punct x pentru f).
Solut ie. (i) Fie g (unica) transformare omograca, care duce (f(z
2
), f(z
3
), f(z
4
)) n
(0, 1, ). Se stie ca:
g(z) = (z, f(z
2
), f(z
3
), f(z
4
))
deci:
g(f(z)) = (f(z), f(z
2
), f(z
3
), f(z
4
))
Pe de alta parte, g f este transformarea omograca, care duce (z
2
, z
3
, z
4
) n (0, 1, ).
Asadar:
Funct ii complexe 175
(g f)(z) = (z, z
2
, z
3
, z
4
)
adica tocmai concluzia.
(ii) O implicat ie a fost demonstrata mai sus. Reciproc, daca birapoartele sunt egale,
sa notam f
1
, f
2
transformarile omograce care duc, (z
2
, z
3
, z
4
), respectiv (w
2
, w
3
, w
4
) n
(0, 1, ). Atunci:
f
1
(z
1
) = (f
1
(z
1
), 0, 1, ) = (f
1
(z
1
), f
1
(z
2
), f
1
(z
3
), f
1
(z
4
)) =
= (z
1
, z
2
, z
3
, z
4
) = (w
1
, w
2
, w
3
, w
4
) = (f
2
(w
1
), f
2
(w
2
), f
2
(w
3
), f
2
(w
4
)) =
= (f
2
(w
1
), 0, 1, ) = f
2
(w
1
)
Rezulta ca f = f
1
2
f
1
convine.
(iii) Sa presupunem ca f are doua puncte xe distincte z
1
, z
2
C . Fie z, w
C, diferite de z
1
si de z
2
. Exista transformarea omograca g, care are z
1
si z
2
puncte
xe, iar g(z) = w. g comuta cu f , deci:
(z, f(z), f
2
(z), f
3
(z)) = (g(z), g(f(z)), g(f
2
(z)), g(f
3
(z))) =
= (w, f(w), f
2
(w), f
3
(w))
Cazul cand punctele xe sunt confundate se justica analog.
Observat ie. Se observa ca funct ia considerata este continua de z. Daca sirul (f
n
(z))
n
are limita, deducem concluzia. Valoarea constanta este:
1
1 +
bc ad
(a +d)
2
si se obt ine pentru z = .
Problema 12.6 Punctele distincte z
1
, z
2
, z
3
, z
4
C sunt coliniare sau conciclice
daca si numai daca (z
1
, z
2
, z
3
, z
4
) R.
Solut ie. Fie f transformarea omograca avand proprietatea ca duce (z
2
, z
3
, z
4
) n
(0, 1, ), adica f(z) = (z, z
2
, z
3
, z
4
) = (f(z), 0, 1, ). Urmeaza ca z
1
, z
2
, z
3
, z
4
sunt col-
iniare sau conciclice f(z
1
), 0, 1, sunt coliniare sau conciclice f(z
1
) R
(z
1
, z
2
, z
3
, z
4
) R.
Observat ii. Condit ia de coliniaritate se obt ine luand z
4
= . Deci z
1
, z
2
, z
3
sunt
coliniare daca si numai daca
z
1
z
2
z
1
z
3
R.
Condit ia (z
1
, z
2
, z
3
, z
4
) > 0 revine la faptul ca z
1
, z
2
se aa pe acelasi arc determinat
de z
3
, z
4
.
Problema 12.7 Fie b R si P un polinom cu toate radacinile reale. Se deneste polino-
mul Q prin:
Q(z) = P(z +ib) +P(z ib)
Sa se arate ca si Q are toate radacinile reale
176
Solut ie. Cazul b = 0 ind banal, putem presupune b > 0. Notand x
k
R radacinile
lui P, se scrie P(z) = A(z x
1
) . . . (z x
n
) de unde
Q(z) = A[(z +ib x
1
) . . . (z +ib x
n
) + (z ib x
1
) . . . (z ib x
n
)]
Daca z = x
k
+ ib, atunci z + ib x
j
= 2ib + x
k
x
j
,= 0, deci n acest caz Q(z) ,= 0.

In
rest:
Q(z) = A(z ib x
1
) . . . (z ib x
n
)
_
z +ib x
1
z ib x
1

z +ib x
n
z ib x
n
+ 1
_
Notand a
k
= x
k
ib, constatam ca Im a
k
= b < 0.
Un calcul direct arata ca:

z a
z a

2
1 =
4(Im z)(Im a)
[z a[
2
Deci, pentru ecare k = 1, n si pentru Im z > 0:

z +ib x
k
z ib x
k

> 1
ceea ce arata ca si n acest caz Q(z) ,= 0. Analog se rat ioneaza daca Im z < 0. Ramane
deci singura posibilitate: Q(z) = 0 = z R.
Problema 12.8 Fie D C deschis, nevid, f : D C o funct ie olomorfa cu proprietatea
ca f

(z) ,= 0, z D si care veric a relat ia:


f

(z)
f

(z)

3
2
_
f

(z)
f

(z)
_
2
= 0, z D
Sa arate ca f este o transformare omograca.
Solut ie. Punand = f

, ecuat ia devine:


3
2
_

_
2
= 0
Punand

= g (corect denita, datorita ipotezei), se obt ine

= g

.+g.

, deci ecuat ia
devine 2g

= g
2
. Aceasta ecuat ie admite evident solut ia g 0, care conduce la f(z) =
az + b.

In rest, notand g =
1
h
(cu except ia unor puncte izolate), ecuat ia devine h

=
1
2
.
Acest caz conduce la

(z)
(z)
=
1

1
2
z +a
de unde se obt ine usor concluzia.
Observat ie. Expresia:
S
f
(z) =
f

(z)
f

(z)

3
2
_
f

(z)
f

(z)
_
2
se numeste derivata Schwarziana a funct iei f. Se verica imediat ca orice transformare
omograca f satisface S
f
0.
Funct ii complexe 177
Problema 12.9 Fie f o transformare omograca. Sa se arate ca, daca f admite n C
doua puncte xe distincte (notate z
1
, z
2
), atunci exista C, ,= 0 astfel ncat:
f(z) z
1
f(z) z
2
=
z z
1
z z
2
Daca f admite n C puncte xe confundate (notate z
0
), atunci exista C astfel ncat
1
f(z) z
0
=
1
z z
0
+
Sa se formuleze si sa se justice rezultatele corespunzatoare pentru restul cazurilor.
Utilizand acest rezultat, sa se arate ca, daca f admite n C doua puncte xe distincte,
atunci sirul (z
n
)
n
, denit prin recurent a astfel: z
0
C, z
n+1
= f(z
n
) are limita, n afara
cazului n care
ad bc
(a +d)
2
(1/4, ].
Solut ie. Punctele xe ale transformarii omograce f(z) =
az +b
cz +d
ind radacinile
ecuat iei cz
2
+ (d a)z b = 0 distingem cazurile:
1) c = d a = b = 0; n acest caz f(z) = z, z C si orice punct este x.
2) c = d a = 0; b ,= 0. Se obt in translat iile f(z) = z + . Se poate admite, prin
convent ie, ca este punct x dublu.
3) c = 0, d a ,= 0. Exista un singur punct x z
0
C, al doilea ind . Scriind
f(z) = z +, f(z
0
) = z
0
= z
0
+, obt inem scrierea:
f(z) z
0
= (z z
0
)
( cu ,= 0). Este vorba deci de rotat ii si omotetii n jurul punctului x z
0
.
4) c ,= 0. Aici distingem doua subcazuri, dupa cum ecuat ia are radacini distincte sau
nu.
(i) Daca = (d a)
2
+ 4bc = 0, e z
0
unicul punct x (dublu). Atunci:
1
f(z) z
0

1
z z
0
=
1
az +b
cz +d

az
0
+b
cz
0
+d

1
z z
0
=
=
c(cz
0
+d)z +cdz
0
+d
2
ad +bc
(ad bc)(z z
0
)
este constant, deoarece:
(z
0
)c(cz
0
+d) (cdz
0
+d
2
ad +bc) = 0
folosind faptul ca z
0
=
a d
2c
.
(ii) Daca ,= 0, e z
1
,= z
2
C radacinile ecuat iei.

In loc sa vericam prin calcul ca
f(z) z
1
f(z) z
2
:
z z
1
z z
2
este constant, sa observam ca f este (unica) transformare omograca care aplica z
k
n z
k
pentru k = 1, 2 iar f() = a/c. Astfel, f are forma:
f(z) z
1
f(z) z
2
:
f() z
1
f() z
2
=
z z
1
z z
2
178
deci =
f() z
1
f() z
2
.
Pentru studiul sirului, sa notam cu l
1
, l
2
cele doua puncte xe. Din relat ia:
f(z) l
1
f(z) l
2
=
z l
1
z l
2
deducem ca:
z
n
l
1
z
n
l
2
=
n
z
0
l
1
z
0
l
2
de unde:
z
n
=
l
1
(z
0
l
2
)
n
l
2
(z
0
l
1
)
(z
0
l
2
)
n
(z
0
l
1
)
Acest sir este convergent daca si numai daca [[ ,= 1, condit ie echivalenta cu cea din enunt .
Problema 12.10 (Teorema lui Lucas) Fie P o funct ie polinomiala. Sa se arate ca
radacinile lui P

se gasesc n cel mai mic poligon convex ce cont ine radacinile lui P.
Solut ie. Fiecare poligon convex este intersect ia unui numar nit de semi-plane. Este
deci sucient sa aratam ca, daca toate radacinile lui P, sa le notam z
1
, . . . , z
n
, se aa
ntr-un anumit semi-plan, atunci P

nu se anuleaza n nici un punct din celalalt semi-plan.


Este convenabil sa descriem semi-planul prin z C [ Im a(z b) 0, cu a, b C, a ,= 0.
Presupunem deci ca Im a(z
k
b) 0, k = 1, n. Fie z din celalalt semi-plan, adica
Im a(z b) > 0. Deoarece:
P

(z)
P(z)
=
n

k=1
1
z z
k
deducem:
Im
P

(z)
aP(z)
=
n

k=1
Im
1
a(z z
k
)
=
n

k=1
1
[a(z z
k
)[
2
Im a(z z
k
) =
=
n

k=1
1
[a(z z
k
)[
2
[Im a(z b) Im a(z
k
b)] < 0

In particular P

(z) ,= 0.
Observat ie. Analizand calculul facut, constatam ca radacinile lui P

se aa chiar n
interiorul poligonului, cu except ia urmatoarelor situat ii:
(i) P are radacini multiple.
(ii) P are toate radacinile coliniare.
Problema 12.11 Un polinom cu coecient i reali se numeste stabil daca toate radacinile
sale au partea reala negativa.
a) Daca P este stabil, atunci si P

este stabil.
b) (i) Daca P(X) = X
2
+ aX + b atunci P este stabil daca si numai daca a > 0 si
b > 0.
(ii) Daca P(X) = X
3
+ aX
2
+ bX + c atunci P este stabil daca si numai daca a >
0, b > 0, c > 0 si a.b > c.
(iii) Daca P(X) = X
4
+aX
3
+bX
2
+cX +d atunci P este stabil daca si numai daca
a > 0, b > 0, c > 0, d > 0 si abc > a
2
d +c
2
.
c) Fie P(X) = a
n
X
n
+a
n1
X
n1
+. . . +a
1
X +a
0
un polinom cu coecient i reali. Sa
se arate ca, daca P este stabil, atunci a
k
a
k+3
< a
k+1
a
k+2
, pentru orice k = 0, 1, . . . , n3.
Funct ii complexe 179
IMC, 2003
Solut ie. a) Consecint a a teoremei lui Lucas.
b) (i) Daca P are radacini reale, condit ia este ca ambele sa e negative, ceea ce este
echivalent cu a > 0 si b > 0. Daca P are radacini nereale, e acestea z
1
si z
2
, atunci
b = z
1
z
2
> 0 iar a = z
1
+z
2
= 2Re z
1
< 0.
(ii) P admite cel put in o radacina reala, e aceasta x
1
. Scriind:
P(X) = (X x
1
)(X
2
+pX +q)
deducem ca P este stabil daca si numai daca x
1
< 0, p > 0, q > 0.

Insa a = p x
1
; b =
q px
1
; c = qx
1
. Echivalent a cu proprietat ile din enunt este acum imediata.
(iii)

In acest caz scriem
P(X) = (X
2
+pX +q)(X
2
+rX +s),
deci condit ia este p > 0, q > 0, r > 0, s > 0. Pe de alta parte, a = p +r; b = s +pr +q;
c = ps +qr; d = qs. T inand seama ca
abc a
2
d c
2
= pr
_
(q s)
2
+ (p +r)(ps +qr)

echivalent a cu proprietat ile din enunt rezulta din nou cu usurint a.


c) Scriem descompunerea polinomului P:
P(X) =

i
(k
i
X +l
i
)

j
(p
j
X
2
+q
j
X +r
j
)
unde k
i
, l
i
, p
j
, q
j
, r
j
R. Din ipoteza, pentru ecare i, k
i
si l
i
au acelasi semn; iar pentru
ecare j, p
j
, q
j
, r
j
au de asemenea acelasi semn.

Inmult ind eventual P cu 1, putem
presupune ca are tot i coecient ii pozitivi. Pentru simplicarea notat iilor, extindem sirul
coecient ilor astfel: a
n+1
= a
n+2
= . . . = 0 si a
1
= a
2
= ... = 0. Demonstrat ia se
face prin induct ie, pentru 1 k n 2. Cazurile n = 2, 3 au fost deja vericate. Fie
n 3 si sa presupunem ca armat ia este valabila pentru toate valorile mai mici ale lui n.
Sa consideram un factor al lui P, de forma X
2
+ pX + q unde p si q sunt numere reale
pozitive. Adica P(X) = (X
2
+pX +q)(b
n2
X
n2
+... +b
1
X +b
0
) = (X
2
+pX +q)Q(X).
Toate radacinile polinomului Q au partea reala negativa, deci conform ipotezei inductive
avem b
k+1
b
k+2
< b
k
b
k+3
, 1 k n 4. Denind analog b
n1
= b
n
= ... = 0 si
b
1
= b
2
= ... = 0 , inegalitatea precedenta are loc pentru toate valorile lui k. Sa aratam
acum a
k+1
a
k+2
> a
k
a
k+3
. Pentru k = 1 sau k = n 2 este evident, deoarece a
k+1
a
k+2
este pozitiv iar a
k
a
k+3
= 0. Mai departe, presupunem 0 k n 3. Dar
a
k+1
a
k+2
a
k
a
k+3
= (qb
k+1
+pb
k
+b
k1
)(qb
k+2
+pb
k+1
+b
k
)
(qb
k
+pb
k1
+b
k2
)(qb
k+3
+pb
k+2
+b
k+1
) = (b
k1
b
k
b
k2
b
k+1
)+
+p(b
2
k
b
k2
b
k+2
) +q(b
k1
b
k+2
b
k2
b
k+3
) +p
2
(b
k
b
k+1
b
k1
b
k+2
)+
+q
2
(b
k+1
b
k+2
b
k
b
k+3
) +pq(b
2
k+1
b
k1
b
k+3
)
Aratam acum ca ecare din cei sase termeni este pozitiv, iar cel put in unul este strict
pozitiv. Din ipoteza inductiva p
2
(b
k
b
k+1
b
k1
b
k+2
) > 0 iar b
k1
b
k
b
k2
b
k+1
0 si
q
2
(b
k+1
b
k+2
b
k
b
k+3
) 0. Pentru a conrma semnul expresiei p(b
2
k
b
k2
b
k+2
) scriem
b
k1
(b
2
k
b
k2
b
k+2
) = b
k2
(b
k
b
k+1
b
k1
b
k+2
) +b
k
(b
k1
b
k
b
k2
b
k+1
) 0. If b
k1
> 0 se
180
obt ine b
2
k
b
k2
b
k+2
0. Daca nu, din b
k1
= 0 rezulta e b
k2
= 0 sau b
k+2
= 0. In ambele
cazuri b
2
k
b
k2
b
k+2
= b
2
k
0. Deci, p(b
2
k
b
k2
b
k+2
) 0. Similar, pq(b
2
k+1
b
k1
b
k+3
) 0.
Semnul expresiei q(b
k1
b
k+2
b
k2
b
k+3
) se verica analog. Consideram b
k+1
(b
k1
b
k+2

b
k2
b
k+3
) = b
k1
(b
k+1
b
k+2
b
k
b
k+3
) + b
k+3
(b
k1
b
k
b
k2
b
k+1
) 0. Discutam din nou
cazurile: b
k+1
> 0, cand putem mpart i prin b
k+1
. Altfel, e b
k2
= 0 sau b
k+3
= 0. In
ambele cazuri, obt inem b
k1
b
k+2
b
k2
b
k+3
0, ceea ce ncheie demonstrat ia.
Problema 12.12 Pentru n 1 notam:
E
n
(z) = (1 z) exp
_
z +
z
2
2
+. . . +
z
n
n
_
Sa se arate ca [1 E
n
(z)[ [z[
n+1
, [z[ < 1
Solut ie. Consideram dezvoltarea n serie de puteri:
E
n
(z) = 1 +

k=1
a
k
z
k
Prin derivare, se obt ine:
z
n
exp
_
z +
z
2
2
+. . . +
z
n
n
_
=

k=1
ka
k
z
k1
Identicand coecient ii, deducem ca a
1
= a
2
= . . . = a
n
= 0 iar a
n+k
0, k 1. Pe de
alta parte
0 = E
n
(1) = 1 +

k=1
a
n+k
conduce la:

k=1
[a
n+k
[ =

k=1
a
n+k
= 1

In concluzie, daca [z[ 1, obt inem:


[1 E
n
(z)[ =

k=1
a
n+k
z
n+k

= [z[
n+1

k=1
a
n+k
z
k1

[z[
n+1

k=1
[a
n+k
[ = [z[
n+1
Problema 12.13 Sa se justice ca exista o funct ie olomorfa pe mulctimea U = z
C[[z[ > 4 a carei derivata este
z
(z 1)(z 2)(z 3)
.
Exista o funct ie olomorfa n U a carei derivata este
z
2
(z 1)(z 2)(z 3)
?
Berkeley, 1978
Funct ii complexe 181
Solut ie. Deoarece daca G(z) = F
_
1
z
_
, atunci G

(z) =
1
z
2
F

_
1
z
_
, prima chestiune
revine la a gasi o funct ie G, olomorfa n discul centrat n 0 si de raza
1
4
, astfel ncat
G

(z) =
1
(1 z)(1 2z)(1 3z)
O asemenea funct ie exista, orice funct ie olomorfa ntr-un disc admit and primitive.
La a doua ntrebare raspunsul este negativ, caci
1
z(1z)(12z)(13z)
nu este olomorfa n
disc.
Problema 12.14 Fie f : C C o funct ie olomorfa, cu proprietatea [f(z)[ = [ sin z[. Sa
se arate ca exista o constanta C de modul 1 astfel ncat f(z) = C sin z, z C.
Solut ie. Funct ia
f(z)
sin z
este olomorfa n mult imea deschisa si conexa D := C k[k
Z . Deoarece

f(z)
sin z

= 1, z D, concluzia rezulta din urmatorul rezultat general:


Lema Fie D C deschis conex, iar f : D C o funct ie olomorfa. Daca [f[ este
constanta n D, atunci funct ia f este constanta n D.
Demonstrat ie. Daca [f[ = 0, atunci f = 0 n D. Fie deci [f[
2
= u
2
+ v
2
= C ,= 0.
Deducem:
_

_
u
u
x
+v
v
x
= 0
u
u
y
+v
v
y
= 0
Folosind condit iile CauchyRiemann, se obt ine:
_

_
u
u
x
v
u
y
= 0
u
u
y
+v
u
x
= 0
De unde (u
2
+v
2
)
u
x
= 0 si deci concluzia.
La acelasi rezultat se ajunge cu mai put ine calcule, daca folosim o transformare omo-
graca T, care aplica cercul [z[ = C pe axa reala (de ex. Tz := i
zC
z+C
). Astfel, funct ia
olomorfa T f ia numai valori reale. Avand partea imaginara identic 0, condit iile Cauchy-
Riemann asigura ca partea reala este constanta.
Observat ie. De fapt, funct ia
f(z)
sin z
este olomorfa n C, deoarece f(k) = 0, ceea ce
arata ca ecare punct k este o singularitate aparenta. Astfel, aceasta funct ie olomorfa
este si maginita n C iar concluzia rezulta pe baza teoremei lui Liouville.
Problema 12.15 Pentru ecare numar complex z , 0, 1 denim
f(z) :=

1
(log z)
4
suma ind extinsa la toate ramurile logaritmului. Sa se arate ca
f(z) =
z
3
+ 4z
2
+z
6(z 1)
4
z , 0, 1.
182
IMC, 2004
Solut ia 1. Este vorba despre seria

kZ
1
(ln [z[ +i arg z + 2ki)
4
cu arg z (, ]. Seria modulelor este

kZ
1
((ln [z[)
2
+ (arg z + 2k)
2
)
2
Aceasta serie este evident convergenta, deci n seria init iala nu conteaza ordinea de sumare.
De asemenea, seria este uniform convergenta pe compacte din C0, 1. In adevar, pentru
ecare compact din C 0, 1, exista 0 < < 1 < A < pentru care z K
[z[ A. Pentru orice k ,= 0, putem majora temenul general cu
1
(C
2
+ (2[k[ 1)
2
)
2
unde C := min (ln , ln A).
Avand n vedere ca la traversarea semiaxei negative, determinarile se permuta ciclic,
deducem ca suma seriei este chiar funct ie olomorfa n C 0, 1.
Studiem natura singularitat ilor izolate 1, 0 si .
Observam ca 1 este pol de ordin 4: n afara de termenul corespunzator lui k = 0, tot i
sunt funct ii olomorfe n discul centrat n 1 si de raza 1.
Pentru observam urmatoarea majorare, pentru [z[ > 1:

kZ
1
((ln [z[)
2
+ (arg z + 2k)
2
)
2

1
2

kZ
_
(2k+1)
(2k1)
dt
((ln [z[)
2
+t
2
)
2
=
=
1
2
_

dt
((ln [z[)
2
+t
2
)
2
=
1
2(ln [z[)
3
_

dt
(1 +t
2
)
2

C
(ln [z[)
3
De aici rezulta ca lim
z
f(z) = 0 astfel ca este o singularitate aparenta pentru f, cu
valoarea 0.
Pentru z = 0 este sucient sa observam ca ecare determinare pentru log
1
z
este o alta
determinare pentru (log z). Adica are loc relat ia f(1/z) = f(z), ceea ce arata ca z = 0
este de asemenea o singularitate aparenta pentru f, cu valoarea 0.
Astfel f este funct ie meromorfa n planul complex extins, deci este o funct ie rat ionala.
Avand doar pol de ordin 4n z = 1, deducem ca exista P polinom astfel ncat f(z) =
P(z)
(z1)
4
.
Deoarece este singularitate aparenta cu valoarea 0, rezulta ca P este polinom de grad
< 4. z = 0 ind de asemenea singularitate aparenta cu valoarea 0, rezulta ca termenul liber
al lui P este 0. Tinand cont si de relat ia f(1/z) = f(z), deducem ca P(z) = az
3
+bz
2
+az.
Pentru a determina valorile a = 1/6, b = 2/3 exista mai multe posibilitat i.
Prima posibilitate este sa dam valori; pentru z = 1 scriem
f(1) =

kZ
1
((2k 1))
4
= 2
4

n=1
1
(2n 1)
4
=
1
48
de unde 2a b = 1/3; sa observam ca putem calcula
lim
z1
(z 1)
4
f(z) = 1
Funct ii complexe 183
de unde 2a +b = 1.
Cu mai multe calcule, putem determina partea principala a dezvoltarii Laurent pentru
f n z = 1. Folosind scrierea
ln(1 +w) = w
w
2
2
+
w
3
3

w
4
4
+. . .
deducem partea principala pentru
1
(ln(1+w))
4
ca ind
w
4
+ 2w
3
+
7
6
w
2
+
1
6
w
1
iar aceasta este si partea principal a pentru f(1 +w). Astfel
f(z) =
1 + 2(z 1) +
7
6
(z 1)
2
+
1
6
(z 1)
3
(z 1)
4
=
z
3
+ 4z
2
+z
6(z 1)
4
O alta posibilitate este sa observam ca are loc relat ia f(z) + f(z) = 16f(z
2
): de-
terminarile pentru log(z
2
) = log((z)
2
) sunt 2 log(z) si 2 log(z). Din aceasta observat ie
obt inem b = 4a.
O cale complet diferita, care sugereaza si o a doua solut ie, este sa plecam de la
g(z) :=

1
(log z)
2
Ca mai sus, obt inem
g(z) =
dz
(z 1)
2
iar = 1 se gaseste usor cu una din metodele deja utilizate. Mai departe se deriveaza de
doua ori.
Solut ia 2. Pornim de la dezvoltarea cotangentei n fract ii simple
cotg w =
1
w
+

n=1
_
1
w +n
+
1
w n
_
care se poate scrie
1
2i
cotg
w
2i
=
1
w
+

n=1
_
1
w + 2ni
+
1
w 2ni
_
Inlocuind w = ln [z[ +i. arg z avem
1
2i
cotg
ln z
2i
=
1
2i
i
e
2i
ln z
2i
+ 1
e
2i
ln z
2i
1
=
1
2
+
1
z 1
Aici denit ia sumarii este esent ial a!
Prin derivari succesive, deducem:

1
(log z)
2
= z
_
1
2
+
1
z 1
_

=
z
(z 1)
2

1
(log z)
3
=
z
2
+z
2(z 1)
2

1
(log z)
4
=
z
3
+ 4z
2
+z
6(z 1)
4
184
Problema 12.16 Fie p(z) = a
0
+ a
1
z + a
2
z
2
+ . . . + a
n
z
n
un polinom cu coecient i
complecsi. Fie (c
k
)
k=0,...,n
un sir convex (adica 2c
k
c
k1
+ c
k+1
pentru ecare k =
1, 2, ..., n 1) si 1 = c
0
c
1
. . . c
n
0. Denim polinomul q(z) = c
0
a
0
+ c
1
a
1
z +
c
2
a
2
z
2
+. . . +c
n
a
n
z
n
. Sa se arate ca
max
|z|1
[q(z)[ max
|z|1
[p(z)[
IMC, 2009
Solut ie. Din principiul de maxim, avem
M
p
:= max
|z|=1
[p(z)[ = max
|z|1
[p(z)[
Avem de aratat ca M
q
M
p
. Observam pentru nceput ca putem presupune c
n
= 0.
In adevar, daca c
n
= 1, atunci p = q si armat ia este banala. In rest, q(z) = c
n
p(z) +
(1 c
n
)r(z), unde r(z) =
n

j=0
c
j
c
n
1 c
n
a
j
z
j
. Sirul c

j
:=
c
j
c
n
1 c
n
satisface condit iile si n plus
c

n
= 0. Iar daca stim ca M
r
M
p
, atunci rezulta
M
q
= [q(z
0
)[ c
n
[p(z
0
)[ + (1 c
n
)[r(z
0
)[ c
n
M
p
+ (1 c
n
)M
r
M
p
Revenim la demonstrat ie, n cazul c
n
= 0. Pe baza formulei lui Cauchy:
a
j
=
1
2i
_
|z|=1
p(z)
z
j+1
dz =
1
2
_
|z|=1
p(z)z
j
[dz[
Folosind aceasta formula, putem exprima polinomul q astfel:
2q(w) =
n

j=0
c
j
_
_
|z|=1
p(z)z
j
[dz[
_
w
j
Adunand expresii similare, dar care sunt 0 datorita olomorei, pentru n j 1,
obt inem n continuare:
2q(w) =
n

j=n
c
|j|
_
_
|z|=1
p(z)z
j
[dz[
_
w
j
=
_
|z|=1
_
_
n

j=n
c
|j|
(
w
z
)
j
_
_
p(z)[dz[
Sa introducem notat ia
K(u) :=
n

j=n
c
|j|
u
j
= c
0
+
n

j=1
c
j
Re u
j
Pentru a termina demonstrat ia, aratam ca funct ia K ia numai valori pozitive. Pentru
aceasta, se verica prin induct ie scrierea
K(u) =
n

k=1
d
k
F
k
(u)
unde d
k
:= c
k1
2c
k
+c
k+1
0 iar
F
k
(u) :=
k1

j=k+1
(k [j[)u
j
Funct ii complexe 185
In sfarsit, scrierea
F
k
(u) = (1 +u +. . . +u
k1
)(1 +u
1
+. . . +u
(k1)
) =

1 +u +. . . +u
k1

2
0
arata ca F
k
0, deci K 0.
Deoarece avem
_
|z|=1
[K(u)[[du[ =
_
|z|=1
K(u)[du[ = 2c
0
= 2
putem naliza:
2[q(w)[ =

_
|z|=1
K
_
w
z
_
p(z)[dz[

_
|z|=1
[K
_
w
z
_
[[p(z)[[dz[
M
p
_
|z|=1
[K(u)[[du[ = 2M
q
Problema 12.17 Notam D = C z C [ Re z = 0, [Im z[ 1 si denim f : D C
prin:
f(z) =
1
2i
ln
1 +iz
1 iz
(determinarea principala).
(i) Sa se verice ca funct ia f este olomorfa n D iar f

(z) =
1
z
2
+ 1
, z D.
(ii) Daca x R, atunci f(x) = arctg x.
(iii) Notam D
1
= w C [ Re w (/2, /2). Sa se arate ca f = ( tg [
D
1
)
1
Solut ie. (i) Pentru ca f sa e denita si olomorfa, se impun condit iile: z ,= i si
1 +iz
1 iz
, (, 0], care sunt echivalente cu z D.
f

(z) =
1
2i
_
1 +iz
1 iz
_

1 +iz
1 iz
=
1
1 +z
2
(ii) Deoarece

1 +ix
1 ix

= 1, deducem ca:
f(x) =
1
2i
ln
1 +ix
1 ix
=
1
2i
_
ln

1 +ix
1 ix

+i arg
1 +ix
1 ix
_
=
1
2
arg
1 +ix
1 ix
Daca notam = arg
1 +ix
1 ix
, atunci (, ), cos =
1 x
2
1 +x
2
, sin =
2x
1 +x
2
, de unde
/2 (/2, /2) si tg /2 =
sin
1 + cos
= x, adica exact /2 = arctg x.
(iii) Daca z D, s-a observat c a
1 +iz
1 iz
, (, 0], deci
Re f(z) =
1
2
arg
1 +iz
1 iz
(/2, /2)
186
adica f(z) D
1
, z D. Astfel, putem calcula:
tg f(z) =
e
2if(z)
1
i(e
2if(z)
+ 1)
=
1 +iz
1 iz
1
i
_
1 +iz
1 iz
+ 1
_ = z, z D
Reciproc, ecuat ia tg w = iy (y R) conduce la e
2iw
=
1 y
1 +y
. Daca [y[ 1, atunci
1 y
1 +y
0, ceea ce arata ca w D
1
= tg w D. Astfel, pentru w D
1
putem calcula:
f(tg w) =
1
2i
ln
1 +itg w
1 itg w
=
1
2i
ln
cos w +i sin w
cos w +i sin w
=
1
2i
ln e
2iw
= w.
Observat ie. Aceste proprietat i justica denirea funct iei arctg ca f de mai sus.
Problema 12.18 Sa se calculeze
_
2
0
cos
2n
xdx
Solut ie. Folosind cos x =
e
ix
+e
ix
2
gasim:
_
2
0
cos
2n
xdx = 2
2n
_
2
0
(e
ix
+e
ix
)
2n
dx =
= 2
2n
_
2
0
2n

k=0
C
k
2n
e
i(2n2k)x
dx = 2
2n1
C
n
2n
Problema 12.19 Sa se gaseasca toate funct iile f, denite si continue n mult imea [z[ 2,
olomorfe n mult imea [z[ < 2 si care verica relat ia:
1
2i
_
||=2
( +
1

)f()
d
( z)
2
= z, [z[ < 2
Solut ie. Descompunand n fract ii simple (sau cu teorema reziduurilor) relat ia din
enunt se scrie:
f(0)
z
2
+ (1
1
z
2
)f(z) + (z +
1
z
)f

(z) = z
sau:
_
(z +
1
z
)f(z)
_

=
_
z
2
2
+
f(0)
z
_

de unde:
(z +
1
z
)f(z) =
z
2
2
+
f(0)
z
+C
deci:
f(z) =
z
3
+ 2Cz + 2f(0)
2(z
2
+ 1)
Punand condit ia de olomore ([z[ < 2), se obt ine f(0) = i(
1
2
C), de unde C =
1
2
si
f(0) = 0. Deci f(z) = z, [z[ 2 este singura funct ie care convine.
Funct ii complexe 187
Problema 12.20 Fie dezvoltarea n serie de puteri
ln(1 +e
z
) =

n=0
a
n
z
n
(i) Sa se ae raza de convergent a a seriei de puteri.
(ii) Sa se arate ca a
0
= ln 2; a
1
= 1/2 iar a
2n+1
= 0, n 1.
(iii) Sa se stabileasca o formula de recurent a pentru coecient ii a
2n
.
Solut ie. (i) ln(1 + e
z
) este funct ie olomorfa n C, mai put in semi-dreptele Re z >
0, Im z = (2k + 1). Deducem ca raza de convergent a a seriei de puteri este .
(ii) Prin derivare, se obt ine funct ia
e
z
1 +e
z
, care este para, cu valoarea n 0 egala cu
1
2
.
Sau, se poate folosi identitatea: ln(1 +e
z
) ln(1 +e
z
) = z.
(iii) Relat ia de recurent a se stabileste, identicand n expresia derivatei:
e
z
1 +e
z
=

n=1
na
n
z
n1
, [z[ <
adica:

n=0
z
n
n!
= (2 +

n=1
z
n
n!
)(

n=1
na
n
z
n1
), [z[ <
de unde a
2
= 1/8 iar
2na
2n
+
n 1
2!
a
2n2
+
n 2
4!
a
2n4
+. . . +
1
(2n 2)!
a
2
=
1
4(2n 1)!
Problema 12.21 Fie f : C C o funct ie olomorfa, cu proprietatea ca exista n N,
astfel ncat f(ze
2i/n
) = f(z), z C. Sa se arate ca exista o funct ie olomorfa g : C C,
astfel ncat f(z) = g(z
n
), z C
Solut ie. Deoarece f(z) =

m=0
a
m
z
m
, ipoteza arata ca:
f(z) = f(ze
2i/n
) =

m=0
a
m
z
m
e
2mi/n
Identicand coecient ii, se obt ine ca a
m
,= 0 daca si numai daca n divide m, adica f(z) =

m=0
a
mn
(z
n
)
m
.
Problema 12.22 (i) Fie n N xat. Sa se arate ca funct ia:
f(z) =
sin z
z(1 z)(1
z
2
) . . . (1
z
n
)
are doar singularitat i aparente n C.
Rezulta ca exista o (unica) funct ie olomorfa g : C C, astfel ncat g
2
(z) = f(z),
z C si g(0) = 1.
188
(ii) Fie d dreptunghiul cu varfurile: 1 + iA si n +1 iA (unde (0, 1)). Sa
se arate ca:
_
d
g(z)
sin z
dz = 2i
n

k=0
(1)
k
_
C
k
n
(iii) Folosind o majorare convenabila a integralei, sa se arate ca
lim
n
n

k=0
(1)
k
_
C
k
n
= 0
Solut ie. (i) Deoarece lim
z0
sin z
z
= 1 iar
k = 1, n : lim
zk
sin z
1
z
k
= (1)
k+1
k
deducem ca f are doar singularitat i aparente n C.

In plus,
lim
zk
f(z) = C
k
n
C ind simplu conex, existent a (si unicitatea ) lui g sunt cunoscute.
(ii) Cu cele demonstrate, funct ia
g(z)
sin z
are n interiorul dreptunghiului d numai sin-
gularitat i izolate n k = 0, 1, . . . , n, ecare ind pol simplu. Deci:
Rez
_
g(z)
sin z
, k
_
=
g(k)
(1)
k

=
(1)
k

_
C
k
n
.
(iii) Pe laturile orizontale z = x iA, deci:
[ sin z[
1
2
(e
A
1) si [1
z
k
[
A
k
1
Rezulta:

g(z)
sin z

2
=

1
sin z z(1 z) (1
z
n
)

1
1
2
(e
A
1)A (
A
n
1)
deci integralele pe cele doua laturi orizontale tind la 0 cand A .
Evaluarea pe latura verticala Re z = n+1 se reduce la cea pe latura Re z = 1+,
prin schimbarea z n z.

Intr-adevar sin (n z) = (1)
n+1
sin z, iar:
(n z)(1 n +z) (1
n z
n
) = (1
z
n
)(1
z
n 1
) z(1)
n1
.
Acum, integrala pe latura vertical a ramasa, se majoreaza prin:

_
A
A
g(1 + +iy)idy
sin (1 + +iy)

_
A
A

g(1 + +iy)
sin (1 + +iy)

dy
Deoarece integrala
_

dy
[ sin (1 + +iy)[
este convergenta, ramane sa minoram:
[z(1 z)(1
z
2
) . . . (1
z
n
)[ (1 )(2 ) . . . (1
1
k
) . . . (1
1
n
)
Funct ii complexe 189
Sa notam = 1 (0, 1). Folosind faptul ca ln(1 +

k
)

k


2
2k
2
, precum si faptul
ca 1 +
1
2
+. . .
1
n
ln n , deducem ca exista o constanta C (independenta de ), astfel
ncat Cn

minoreaza produsul. Se obt ine astfel ncadrarea:


0
n

k=0
(1)
k
_
C
k
n

C

n
Observat ie. Evident, pentru n impar
n

k=0
(1)
k
_
C
k
n
= 0.
Problema 12.23 Sa se arate ca:
1
2i
_
|z|=1
e
x
2
(z +
1
z
)
dz =

n=0
_
x
2
_
2n+1
1
n!(n + 1)!
Solut ie. Avem:
1
2i
_
|z|=1
e
x
2
(z +
1
z
)
dz = Rez
_
_
e
x
2
(z +
1
z
)
, 0
_
_
Reziduul se determina scriind:
e
x
2
(z +
1
z
)
=
_

n=0
x
n
z
n
2
n
n!
__

n=0
x
n
2
n
n!z
n
_
si identicand coecientul lui z
1
.
Problema 12.24 Integrand funct ia:
1
z
3
cos z
pe patratul
n
de varfuri n in, sa se
determine

n=0
(1)
n
(2n + 1)
3
.
Solut ie. Aplicand teorema reziduurilor, avem:
_

n
dz
z
3
cos z
= 2i
_
n1

k=n
Rez(f, k +
1
2
) + Rez(f, 0)
_
=
= 2i
_
2
n1

k=0
(1)
k
(k +
1
2
)
3
+

2
2
_
caci:
Rez(f, 0) =
1
2!
_
d
2
dz
2
1
cos z
_
z=0
=
_

2
2 cos
3
z
_
z=0
Apoi
[ cos z[
2
=
1
4
_
e
2y
+e
2y
+ 2 cos 2x

.
Deci, daca x = n, atunci
[ cos z[
2
=
1
4
(e
y
+e
y
)
2
1.
Daca y = n, atunci, pentru n sucient de mare [ cos z[
2
1. Deci
_

n
dz
z
3
cos z
0
pentru n , de unde rezulta ca suma ceruta este

3
32
.
190
Problema 12.25 Integrand funct ia z
1
e
iz
(determinarea principala, (0, 1)) pe
curba formata din arcele: segmentul [, A]; arcul de cerc de centru 0 si de raza A,
situat n primul cadran; segmentul de extremitat i iA si i; arcul de cerc de centru 0 si de
raza (unde 0 < < A < ), sa se calculeze valorile integralelor:
_

0
x
1
sin x dx ;
_

0
x
1
cos x dx
Solut ie. Funct ia ind olomorf a n interiorul curbei , rezulta ca:
0 =
_

z
1
e
iz
dz =
_
A

x
1
e
ix
dx +
_
C
A
z
1
e
iz
dz

_
A

x
1
e
x
e
i./2
dx
_
C
z
1
e
iz
dz
Cu lema lui Jordan, deducem ca:
lim
A
_
C
A
z
1
e
iz
dz = 0
Apoi:

_
C
z
1
e
iz
dz

2
0
cand 0. Egaland part ile reala si imaginara, se obt ine:
_

0
x
1
cos x dx = cos

2
()
_

0
x
1
sin x dx = sin

2
()
Problema 12.26 Fie 1 < a < b. Integrand funct ia
_
z +
1
z
_
a
z
b1
pe curba nchisa
formata din arcele: arcul de cerc de centru 0 si de raza 1, situat n semiplanul Re z > 0;
segmentele de extremitat i i(1 ) si i, respectiv i(1 +) si i; arcele de cerc de centre
0, i si i de raza (unde 0 < < 1), sa se arate ca:
_
2
0
cos
a
t cos bt dt =
(a + 1)
2
a+1
(
a+b
2
+ 1)(
ab
2
+ 1)
Solut ie. Funct ia data ind olomorfa n domeniul limitat de curba , gasim:
0 =
_

i
(z +
1
z
)
a
z
b1
dz
_
1

(
1
x
x)
a
x
b1
e
i/2(ba1)
dx

0
(z +
1
z
)
a
z
b1
dz +
_
1

(
1
x
x)
a
x
b1
e
i/2(ab+1)
dx+
_
/2
/2
cos
a
t(cos(b 1)t +i sin(b 1)t)i(cos t +i sin t)dt
_

i
(z +
1
z
)
a
z
b1
dz
Funct ii complexe 191
Apoi:

i
(z +
1
z
)
a
z
b1
dz

C.

i
(z +
1
z
)
a
z
b1
dz

0
(z +
1
z
)
a
.z
b1
dz

ba1
de unde:
_
2

2
cos
a
t cos bt dt = 2 sin

2
(b a 1)
_
1
0
(x
1
x
)
a
x
b1
dx
si deci rezultatul propus.
Problema 12.27 (i) Integrand funct ia
e
3iz
3e
iz
+ 2
z
3
pe curba formata din arcele: segmentele [A, ] si [, A] si semicercurile centrate n 0,
de raze respectiv si A, situate n semiplanul Im z > 0,, sa se arate ca:
_

0
sin
3
x
x
3
dx =
3
8
(ii) Integrand funct ia:
1 e
iz
z
2
(z
2
+ 1)
pe curba de mai sus, sa se arate ca :
_

1 cos x
x
2
(x
2
+ 1)
dx =

e
(iii) Integrand funct ia:
e
2aiz
e
2biz
z
2
, a > 0, b > 0
pe curba de mai sus, sa se deduc a valoarea integralei (Froullani):
_

0
cos 2ax cos 2bx
x
2
dx
Solut ie. (i) Daca x R, atunci:
e
3ix
3e
ix
+ 2
x
3
=
cos 3x 3 cos x + 2
x
3
+i
sin
3
x
x
3
Deducem ca:
0 =
_

e
3iz
3e
iz
+ 2
z
3
dz =
_

A
_
cos 3x 3 cos x + 2
x
3
+i
sin
3
x
x
3
_
dx+
192
+
_
A

_
cos 3x 3 cos x + 2
x
3
+i
sin
3
x
x
3
_
dx
_
C

e
3iz
3e
iz
+ 2
z
3
dz+
+
_
C
A
e
3iz
3e
iz
+ 2
z
3
dz
Insa:

_
C
A
e
3iz
3e
iz
+ 2
z
3
dz

A
6
A
3
0
cand A . Deoarece:
e
3iz
3e
iz
+ 2
z
3
=
3
z
+f(z)
cu f olomorfa ntr-o vecinatate a originii, urmeaza (cf. teorema semireziduurilor):
_
C
e
3iz
3e
iz
+ 2
z
3
dz = (3)i +
_
C
f(z)dz
iar _
C
f(z)dz 0
cand 0. Se obt ine astfel:
_

0
sin
3
x
x
3
dx =
1
2
_

sin
3
x
x
3
dx =
1
8
_

sin 3x 3 sin x
x
3
dx =
3
8
(ii) Aplicand teorema reziduurilor, obt inem:
_

1 e
iz
z
2
(z
2
+ 1)
dz = 2iRez(
1 e
iz
z
2
(z
2
+ 1)
, i) = 2i
1 e
1
(1)2i
Pe de alta parte:
_

1 e
iz
z
2
(z
2
+ 1)
dz =
_

A
1 cos x i sin x
x
2
(x
2
+ 1)
dx
_
C
1 e
iz
z
2
(z
2
+ 1)
dz+
+
_
A

1 cos x i sin x
x
2
(x
2
+ 1)
dx +
_
C
A
1 e
iz
z
2
(z
2
+ 1)
dz
Deoarece
1 e
iz
z
2
(z
2
+ 1)
=
1 (1 +iz +z
2
f(z))
z
2
(z
2
+ 1)
=
i
z

f(z)
z
2
+ 1
deducem ca:
_
C
1 e
iz
z
2
(z
2
+ 1)
dz = i
_
C
dz
z

_
C
f(z)
z
2
+ 1
dz

Insa:

_
C
f(z)
z
2
+ 1
dz

M 0
cand 0, iar:
_
C
dz
z
= i
Funct ii complexe 193
Deoarece:

_
C
A
1 e
iz
z
2
(z
2
+ 1)
dz

A
M
A
4
0
cand A , deducem ca valoarea integralei este

e
.
(iii) Avem:
0 =
_

e
2aiz
e
2biz
z
2
dz =
_

A
e
2aix
e
2bix
x
2
dx
_
C
e
2aiz
e
2biz
z
2
dz+
+
_
A

e
2aix
e
2bix
x
2
dx +
_
C
A
e
2aiz
e
2biz
z
2
dz
Deoarece:
e
2aiz
e
2biz
z
2
=
2i(a b)
z
+f(z)
cu f olomorfa ntr-o vecinatate a originii, deducem ca:
lim
0
_
C
e
2aiz
e
2biz
z
2
dz = i 2i(a b)
Apoi:

_
C
A
e
2aiz
e
2biz
z
2
dz

A
M
A
2
0
cand A . Rezulta ca valoarea integralei propuse este (b a).
Problema 12.28 Integrand funct ia
e
az
(e
z
+ 1)
2
, (a (0, 2)) pe dreptunghiul de varfuri
A, A+ 2i, sa se ae valoarea integralei:
_

e
ax
dx
(e
x
+ 1)
2
.
Solut ie. Conform teoremei reziduurilor:
_

e
az
(e
z
+ 1)
2
dz = 2iRez(f, i)
Pentru a calcula reziduul n polul de ordinul 2, se efectueaza schimbarea z i = w si
deci: Rez(f, i) = e
ai
Rez
_
e
aw
(1 e
w
)
2
, 0
_
. Scriind:
e
aw
(1 e
w
)
2
=
p
w
2
+
q
w
+. . .
se gaseste:
1 +aw +. . . = (1 +w +. . .)(p +qw +. . .)
deci prin identicare: Rez(f, i) = e
ai
(a 1). Pe de alta parte:
_

e
az
(e
z
+ 1)
2
dz =
_
A
A
e
ax
dx
(e
x
+ 1)
2
+
_
2
0
e
a(A+iy)
(e
A+iy
+ 1)
2
idy

_
A
A
e
a(x+2i)
(e
x+2i
+ 1)
2
dx
_
2
0
e
a(A+iy)
(e
A+iy
+ 1)
2
idy
Ipotezele asigura ca (1e
2ia
)
_

e
ax
dx
(e
x
+ 1)
2
= 2ie
ai
(a1) deci valoarea integralei este:
(1 a)
sin a
pentru a ,= 1. Pentru a = 1, valoarea 1 se poate gasi si prin calcul direct.
194
Problema 12.29 Integrand funct ia
ln z
z
2
1
(determinarea principala), pe curba formata
din arcele: segmentul [, A]; arcul de cerc de centru 0 si de raza A, situat n primul cadran;
segmentul de extremitat i iA si i; arcul de cerc de centru 0 si de raza (unde 0 < <
A < ), sa se arate ca :
_

0
ln x
x
2
1
dx =

2
4
Solut ie. Se arata imediat ca funct ia
ln z
z
2
1
are o singularitate aparenta n 1, deci:
0 =
_

ln zdz
z
2
1
=
_
A

ln xdx
x
2
1
+
_
C
A
ln zdz
z
2
1

_
A

ln iy
y
2
1
idy
_
C
ln zdz
z
2
1
Apoi:

_
C
A
ln z
z
2
1
dz


2
A
ln A+

2
A
2
1
0 cand A

_
C
ln z
z
2
1
dz

ln +

2
1
2
0 cand 0
Deci:
_

0
ln x
x
2
1
dx = i
_

0
ln y
y
2
+ 1
dy +

2
_

0
dy
y
2
+ 1
=

2
4
(prima integrala ind nula n mod necesar).
Problema 12.30 Integrand funct ia
ln
2
z
1 z
3
(determinarea principala), pe cercul centrat
n 0, de raza A, cu o taietura pe semiaxa reala negativa, sa se ae valoarea integralei
_

0
ln x
x
3
+ 1
dx.
Solut ie. Pe de o parte:
_

ln
2
z
1 z
3
dz = 2i
_
Rez(
ln
2
z
1 z
3
, 1) + Rez(
ln
2
z
1 z
3
, e
2i
3
)+
+Rez(
ln
2
z
1 z
3
, e
4i
3
)
_
=
= 2i
_
(
2i
3
)
2
3e
4i
3
+
(
2i
3
)
2
3e

4i
3
_
=
8
3
i
27
2 cos
4
3
=
8
3
i
27
Pe de alta parte:
_

ln
2
z
1 z
3
dz =
_

A
[ln(x) +i]
2
1 x
3
(dx)
_
C
ln
2
z
1 z
3
dz

_

A
[ln(x) i]
2
1 x
3
(dx) +
_
C
A
ln
2
z
1 z
3
dz
Deoarece:

_
C
ln
2
z
1 z
3
dz

2
(ln +)
2
1
3
0 pentru 0
Funct ii complexe 195
iar

_
C
A
ln
2
z
1 z
3
dz

2A
(ln A+)
2
A
3
1
0 pentru A
deducem ca:

8
3
i
27
=
_
0

4i ln(x)
1 x
3
(dx) = 4i
_

0
ln x
x
3
+ 1
dx
adica:
_

0
ln x
x
3
+ 1
dx =
2
2
27
.
Capitolul 13
Combinatorica si grafuri
Combinatorica
Notat ii
N - mult imea numerelor naturale
[A[ - cardinalul mult imii A
x| - partea ntreaga a numarului x

_
1 2 . . . n
p(1) p(2) . . . p(n)
_
- permutare a mult imii X = 1, 2, . . . , n, unde p : X
X este o aplicat ie bijectiva
n! = 1 2 3 . . . n - numarul n-factorial, pentru n N, n 1
Prin denit ie, 0! = 1.
[n]
k
- numarul aranjamentelor de n luate cate k, pentru n, k N, n k

_
n
k
_
- numarul combinarilor de n luate cate k, pentru n, k N, n k
Denit ii si rezultate
Denit ie. Se numeste permutare a mult imii X = 1, 2, . . . , n orice aplicat ie bijectiva
p : X X.
Daca X este o mult ime oarecare, atunci orice aplicat ie bijectiva p : X X se numeste
permutare a mult imii X.
Denit ie. Se numeste partit ie a unei mult imi X, o descompunere a lui X sub forma
X = A
1
A
2
A
k
, unde mult imile nevide A
1
, . . . , A
k
sunt doua cate doua disjuncte
si se numesc clasele partit iei.
O partit ie este de tipul 1
k
1
2
k
2
. . . n
kn
daca ea cont ine k
1
clase cu un element, k
2
clase
cu doua elemente, . . . , k
n
clase cu n elemente.
Observat ie.

Intr-o partit ie nu conteaza ordinea de scriere a claselor si nici ordinea
de scriere a elementelor n ecare clasa.
Denit ie. Se numeste partit ie a unui ntreg n o scriere a numarului n sub forma
n = n
1
+n
2
+ +n
k
, unde numerele naturale n
1
, n
2
, . . . , n
k
se numesc part ile partit iei
si verica inegalitat ile n
1
n
2
n
k
1.
196
Combinatorica si grafuri 197
Denit ie. Pentru o mult ime nita X, astfel ncat [X[ = n si 1 k n, se numeste
aranjament de n elemente luate cate k orice submult ime ordonata alcatuita din k
elemente ale mult imii X.
Teorema. Numarul de aranjamente de n elemente luate cate k ale mult imii X este
[n]
k
= n(n 1) (n k + 1), 1 k n.
Observat ie. Numarul [n]
k
se poate exprima sub forma
[n]
k
=
n!
(n k)!
, n, k N, 1 k n.
Prin denit ie, [n]
0
= 1.
Observat ie. Numarul de posibilitat i de a introduce k bile, numerotate de la 1 la k
n n urne, numerotate de la 1 la n este [n]
k
.
Denit ie. Pentru o mult ime nita X, astfel ncat [X[ = n, se numeste permutare de
n elemente un aranjament de n elemente luate cate n.
Observat ie. Numarul permutarilor de n elemente este [n]
n
= n!.
Teorema. Daca X, Y sunt mult imi nite astfel ncat [X[ = m si [Y [ = n, unde
m, n N

, atunci numarul funct iilor f : X Y este n


m
.
Observat ie. Daca X este formata din m bile, iar mult imea Y este formata din n
cutii, atunci n
m
reprezinta numarul tuturor posibilitat ilor de a aseza bilele n cutii (n
aceeasi cutie pot introduse mai multe bile).
Teorema. Daca X, Y sunt mult imi nite astfel ncat [X[ = m si [Y [ = n, unde
m, n N

, m n, atunci numarul funct iilor injective f : X Y este [n]


m
.
Observat ie.

In cazul particular f : X Y cu [X[ = [Y [ = n, orice aplicat ie
injectiva este bijectiva si deci numarul de aplicat ii bijective de la X la Y este [n]
n
= n!.
Denit ie. Pentru o mult ime nita X, astfel ncat [X[ = n si 0 k n, se numeste
combinare de n elemente luate cate k orice submult ime a lui X formata din k
elemente.
Teorema. Numarul de combinari de n elemente luate cate k ale mult imii X este
_
n
k
_
=
n!
k!(n k)!
, 0 k n.
Teorema (formula de recurent a). Daca 1 k n, atunci
_
n
k
_
=
_
n 1
k
_
+
_
n 1
k 1
_
Teorema (binomul lui Newton). Daca a, b C si n N, atunci are loc formula
(a +b)
n
=
n

k=0
_
n
k
_
a
k
b
nk
.
Teorema (principiul includerii si al excluderii). Pentru orice n mult imi nite
X
1
, . . . , X
n
avem
[X
1
X
n
[ =

[X
1
[

[X
1
X
2
[ +

[X
1
X
2
X
3
[ + +
198
(1)
n
[X
1
X
n
[.
Teorema. Daca X, Y sunt mult imi nite astfel ncat [X[ = m si [Y [ = n, unde
m, n N

, m n atunci numarul funct iilor surjective f : X Y este


n
m

_
n
1
_
(n 1)
m
+
_
n
2
_
(n 2)
m

_
n
3
_
(n 3)
m
+ + (1)
n1
_
n
n 1
_
.
Denit ie. Se numeste funct ie generatoare asociata unui sir de numere (a
n
) ca ind
suma seriei

n=0
a
n
x
n
, n ipoteza ca seria este convergenta pe R sau pe intervale din R.
Denit ie. Se deneste numarul lui Catalan ca ind numarul de moduri n care se pot
pune parantezele ntr-un produs neasociativ de n factori, scrisi n ordinea a
1
, a
2
, . . . , a
n
.
Teorema. Numarul lui Catalan are expresia
T
n
=
1
n
_
2n 2
n 1
_
,
pentru n N, n 1.
Grafuri
Notat ii
[V [ numarul elementelor mult imii nite V
(V, U) G graf
O(G) ordinul grafului G
dim(G) dimensiunea grafului G
gr (G) grosimea grafului G
d (G) diametrul unui graf conex G
(G) raza grafului G
(G) numarul cromatic al grafului G
q (G) indicele cromatic al grafului G
(x
i
, x
j
) arc al grafului G
[x
i
, x
j
] muchie a grafului G
A mult imea arcelor grafului G
D (x
1
, x
2
, . . . , x
p
) drum al grafului G
l (D) lungimea drumului D
g (x) gradul varfului x
g

(x) gradul de intrare al varfului x


g
+
(x) gradul de iesire al v arfului x
Combinatorica si grafuri 199
Denit ii si rezultate
Denit ie. Consideram V = x
1
, . . . , x
n
, n N

, o mult ime. Numim graf neorientat


perechea ordonata de mult imi (V, U) G, U T
2
(V ), unde T
2
(V ) reprezinta mult imea
part ilor cu doua elemente ale lui V .
Elementele mult imii V se numesc varfuri sau noduri ale grafului G iar elementele
mult imii U se numesc muchii.
Numarul elementelor mult imii V, notat O(G) se numeste ordinul grafului G, iar
numarul elementelor mult imii U, notat dim(G) se numeste dimensiunea grafului G.

In cazul n care x
i
, x
j
U, vom nota aceasta mult ime cu [x
i
, x
j
] , iar x
i
si x
j
se
numesc extremitat ile acestei muchii.
Daca [x
i
, x
j
] U, spunem ca varfurile x
i
si x
j
sunt adiacenten graful Gsi ca varfurile
x
i
si x
j
sunt incidente cu muchia [x
i
, x
j
] .
Denit ie. Numim graf orientat perechea ordonata de mult imi (V, U) G, unde U
este formata din perechi ordonate de elemente din V, numite arce.
Daca x
i
, x
j
U, vom nota acest arc cu (x
i
, x
j
). De asemenea, notand cu u
ij
(x
i
, x
j
)
un arc oarecare vom spune ca x
i
este extremitatea init iala, iar x
j
este extremitatea
nala a arcului u
ij
, despre care vom spune ca este orientat de la x
i
la x
j
.
Pentru un graf orientat G, un sir de varfuri D = x
1
, x
2
, . . . , x
p1
, x
p
cu proprietatea
ca: (x
1
, x
2
) ; (x
2
, x
3
) ; . . . ; (x
p1
, x
p
) U se numeste drum.
Numarul arcelor care compun drumul D, notat cu l (D) se numeste lungimea dru-
mului.
Drumul D cu proprietatea ca x
1
= x
p
si toate arcele (x
1
, x
2
) ; (x
2
, x
3
) ; . . . ; (x
p1
, x
p
)
sunt distincte doua cate doua se numeste circuit. Un circuit format dintr-un singur arc
se numeste bucla.
Pentru un graf neorientat G, un sir de varfuri L = [x
1
, x
2
, ..., x
p
] cu proprietatea ca
oricare doua varfuri vecine sunt adiacente, adica [x
1
, x
2
]; [x
2
, x
3
]; . . . ; [x
p1
, x
p
] U se
numeste lant .
Varfurile x
1
si x
p
se numesc extremitat ile lant ului L, iar p 1 este lungimea acestui
lant . Daca varfurile x
1
, x
2
, . . . , x
p
sunt distincte doua cate doua, lant ul L se numeste
elementar.
Pentru un graf neorientat G, un lant L = [x
1
, x
2
, . . . , x
p
] cu proprietatea ca x
1
= x
p
si toate muchiile [x
1
, x
2
]; [x
2
, x
3
]; . . . ; [x
p1
, x
p
] distincte doua cate doua se numeste ciclu.
Daca ntr-un graf G suprimam unul sau mai multe arce (muchii) se obt ine un nou graf
numit graf part ial al grafului G. Daca ntr-un graf G suprimam unul sau mai multe
varfuri, mpreuna cu arcele (muchiile) care intra sau ies din ele, obt inem un nou graf
numit subgraf al grafului G.
Un ciclu (circuit) hamiltonian pentru un graf G este un ciclu (circuit) elementar
care cont ine toate varfurile grafului. (trece o data si numai o data prin toate varfurile
grafului)
Un ciclu (circuit) eulerian al unui graf G este un ciclu (circuit) care foloseste toate
muchiile (arcele) grafului G, adica trece o data si numai o data prin ecare muchie (arc)
a grafului G.
Denit ie. Numim graf hamiltonian un graf neorientat care cont ine un ciclu hamilto-
nian sau un graf orientat care cont ine un circuit hamiltonian.
Un lant (drum) elementar al unui graf G care cont ine toate varfurile grafului se numeste
lant (drum) hamiltonian.
Un drum se numeste eulerian daca trece o data si numai o data prin ecare arc
(muchie) al grafului.
200
Un drum se numeste simplu daca trece o singura data printr-un acelasi arc al grafului.
Denit ie. Un graf G cu proprietatea ca oricare doua varfuri sunt extremitat ile unui
lant al lui G se numeste graf conex.
Denit ie. Un graf G se numeste tare conex daca oricare ar x
i
si x
j
doua varfuri ale
sale, exista un drum de la x
i
la x
j
si un drum de la x
j
la x
i
.
Un graf orientat este simetric daca (x
i
, x
j
) U (x
j
, x
i
) U, pentru orice arc al
grafului si antisimetric daca (x
i
, x
j
) U (x
j
, x
i
) / U, pentru orice varfuri x
i
, x
j
V.
Denit ie. Orice graf conex si fara cicluri se numeste arbore.
Daca G este un graf conex si x este un varf oarecare al acestuia prin excentrici-
tatea acestuia nt elegem numarul e (x) = max
yV
d (x, y) , unde d (x, y) este distant a dintre
varfurile x si y, adica lungimea minima a lant urilor dintre x si y.
Diametrul unui graf conex G, notat d (G) este distant a maxima dintre perechile de
varfuri ale lui G.
Centrul unui graf conex G este format din acele varfuri de excentricitate minima.
Raza unui graf conex G este minimul excentricitat ilor varfurilor sale, minim notat
(G) .
Numarul cromatic al unui graf G, notat (G) este numarul minim de culori cu care
pot colorate varfurile lui G, astfel ncat oricare doua varfuri adiacente sa aiba culori
diferite.
Indicele cromatic al unui graf G, notat q (G) este numarul minim de culori cu care
pot colorate muchiile grafului G, astfel ncat oricare doua muchii cu o extremitate comuna
sa e colorate cu culori diferite.
Pentru un graf neorientat G, gradul varfului x, notat g (x) este numarul muchiilor
incidente cu varful x.
Daca G este un graf orientat, gradul de intrare al varfului x, notat g

(x) este
numarul arcelor care intra n varful x (de forma (y, x)), iar gradul de iesire al varfului
x, notat g
+
(x) este numarul arcelor care ies din varful x (de forma (x, z)) si gradul varfului
x este g (x) = g

(x) +g
+
(x) .
Denit ie. Grafurile G
1
(V
1
, U
1
) si G
2
(V
2
, U
2
) se numesc izomorfe daca exista o
biject ie f : V
1
V
2
, astfel ncat [x, y] U
1
daca si numai daca [f (x) , f (y)] U
2
, unde
prin [x, y] este notata muchia care uneste varfurile x si y ale grafului G
1
.
Un izomorsm al unui graf G cu el nsusi se numeste automorsm.
O pcolorare a unui graf G este e o partit ie V = V
1
V
2
... V
p
a mult imii
varfurilor sale astfel ncat oricare doua varfuri din aceeasi clasa sa nu e adiacente, e o
funct ie f : U 1, 2, ..., p astfel ncat [i, j] U sa implice f (i) ,= f (j) .
Denit ie. Numim graf kregulat un graf neorientat pentru care ecare varf x V
are gradul g (x) = k sau un graf orientat cu proprietatea ca g

(x) = g
+
(x) = k, pentru
orice varf x.
Pentru un graf G, un varf de grad zero se numeste varf izolat, iar un varf de gradul
unu se numeste varf terminal.
Reprezentand ecare varf x
i
V printr-un punct n plan si ducand arcele dupa criteriul
stabilit obt inem imaginea grafului.
Denit ie. Numim graf planar un graf G ale carui varfuri se pot reprezenta prin puncte
n plan si ale carui muchii se pot reprezenta prin arce de curba Jordan ce unesc perechi
de puncte care corespund extremit at ilor ecarei muchii, astfel ncat oricare doua arce de
curba au n comun cel mult un punct extremitate.
Denit ie. Un graf G care admite o partit ie a mult imii varfurilor, V = V
1
V
2
V
p
astfel ncat ecare muchie sa aib a extremitat ile n doua clase distincte ale partit iei se
Combinatorica si grafuri 201
numeste graf multipartit.
Un graf multipartit este complet daca oricare pereche de varfuri situate n clase
diferite ale partit iei formeaza o muchie.

In particular, un graf G pentru care exista o partit ie a mult imii varfurilor, V = V


1
V
2
,
V
1
V
2
= , astfel ncat ecare muchie u a grafului are o extremitate n V
1
si cealalta n
V
2
se numeste graf bipartit.
Graful bipartit G este complet daca el cont ine toate muchiile [x, y] unde x V
1
si
y V
2
.
Daca [ V
1
[= p si [ V
2
[= q, graful bipartit complet G se noteaza cu K
p,q
.
Prin K
n
vom nota un graf complet cu n varfuri si care are
_
n
2
_
muchii, deci pentru care
oricare doua varfuri sunt adiacente.
Un graf bipartit complet de forma K
1,p
se numeste graf-stea.
Cea mai mica lungime a unui ciclu elementar al grafului G, notata gr (G) se numeste
grosimea grafului.
Matrice asociate unui graf.
Arcele unui graf care au extremitatea nala ntr-un varf x
i
se numesc arce incidente
interior varfului x
i
, iar cele cau au extremitatea init iala n varful x
i
se numesc arce
incidente exterior varfului x
i
.
Consideram G un graf de ordinul n si sa notam cu (u
j
) arcele acestui graf.
Matricea de incident a asociata grafului G este matricea patratica A = (a
ij
) , unde:
a
ij
=
_
_
_
1, daca arcul u
j
este incident interior varfului x
i
;
0, daca x
i
nu este extremitate a arcului u
j
;
1, daca arcul u
j
este incident exterior varfului x
i
.
Matricea conexiunii directe asociata grafului G este matricea patratica D = (d
ij
) ,
unde:
d
ij
=
_
1, daca exista arcul (x
i
, x
j
) ;
0, daca nu exista arcul (x
i
, x
j
) .
Matricea conexiunii totale asociata grafului G este matricea patratica T = (t
ij
) ,
unde:
t
ij
=
_
1, daca exista cel put in un drum de la x
i
la x
j
, x
i
,= x
j
;
0,n caz contrar.
Teorema lui Chen. Daca graful G cu n varfuri este un graf fara circuite, atunci el
cont ine un drum hamiltonian daca si numai daca matricea T a conexiunii totale atasata
cont ine exact C
2
n
elemente nenule.
Formula lui Cayley. Numarul arborilor cu n varfuri, x
1
, x
2
, . . . , x
n
este egal cu
n
n2
, n 2.
Denit ie. Pentru un graf planar G, o componenta conexa a planului, obt inuta prin
suprimarea din plan a muchiilor si varfurilor reprezentarii plane a lui G se numeste fat a
a reprezentarii plane.
Frontiera oricarei fet e este o curba Jordan nchisa, constand din muchiile unui ciclu
elementar al grafului G.
Formula lui Euler. Daca graful conex si planar G are n varfuri si m muchii, atunci
orice reprezentare planara a sa cont ine mn + 2 fet e.
202
Probleme
Problema 13.1 Sa se arate ca oricare ar 52 de puncte ntr-un patrat de latura 1, exista
trei puncte care pot acoperite cu un disc de raza
1
7
.
Solut ie. Fie punctele M
1
, . . . , M
52
situate ntr-un patrat de latura 1. Vom arata ca
exista trei discuri D(M
i
k
,
1
7
), i
k
1, . . . , 52, pentru k 1, 2, 3, care sa aiba intersect ia
nevida.

In acest caz discul cautat are centrul ntr-un punct din intersect ie, iar raza este
1
7
.
Suprafat a totala acoperita de cele 52 de discuri este S = 52
1
49
=
52
49
. Suprafat a
acoperita de acestea, S

, este cel mult egala cu cea a unui patrat ce are latura cu


2
7
mai
mare decat cea a patratului init ial, adica S

<
81
49
.
Obt inem
S
S

>
52
81
> 2.

In consecint a, exista D(M


i
k
,
1
7
), i
k
1, . . . , 52, k 1, 2, 3 cu intersect ia nevida.
Problema 13.2 Pe suprafat a unui poligon de arie 13 se aseaza 10 poligoane de arie 6. Sa
se arate ca exista 4 poligoane ce se intersecteaza dupa o arie mai mare ca
1
70
.
Solut ie. Fie poligoanele P
1
, . . . , P
10
astfel ncat S(P
k
) = 6, pentru orice k = 1, . . . 10
si poligonul P cu S(P) = 13. Denim mult imile
I
1
=
10
_
k=1
P
k
, I
2
=
10
_
i=j=1
(P
i
P
j
), . . . , I
10
=
10

k=1
P
k
.
Se poate demonstra prin induct ie relat ia
10

k=1
S(P
k
) =
10

k=1
S(I
k
).
Pentru mult imile I
k
, k = 1, . . . 10, sunt evidente inegalitat ile
S(I
1
) S(I
2
) . . . S(I
10
),
de unde obt inem
10

k=1
S(P
k
) 3S(I
1
) + 7S(I
4
),
deci 60 39 + 7S(I
4
), adica S(I
4
) 3.
I
4
cont ine
_
10
4
_
intersect ii de cate 4 poligoane, printre care exista una de arie maxima
notata S
max
, deci:
3 S(I
4
)

S(P
1
P
2
P
3
P
4
)
_
10
4
_
S
max
Din inegalitat ile de mai sus deducem ca S
max
3/
_
10
4
_
=
1
70
.
Problema 13.3 Fiind date numerele naturale nenule m, n, m n, sa se demonstreze ca
m divide numarul
n
m1

k=0
(1)
k
_
n
k
_
.
Combinatorica si grafuri 203
Hungarian Mathematical Olympiad, 2001
Solut ie. Vom aplica formula de recurent a a coecient ilor binomiali.
n
m1

k=0
(1)
k
_
n
k
_
= n
m1

k=0
(1)
k
_
n 1
k
_
+n
m1

k=1
(1)
k
_
n 1
k 1
_
= n
_
m1

k=0
(1)
k
_
n 1
k
_
+
m2

k=0
(1)
k+1
_
n 1
k
_
_
= n(1)
m1
_
n 1
m1
_
= n(1)
m1
(n1)!
(m1)!(nm)!
= m(1)
m1
_
n
m
_
.
Problema 13.4 Sa se calculeze suma
1 2
_
n
2
_
+ 2 3
_
n
3
_
+ + (n 1) n
_
n
n
_
.
Solut ie. Consideram un grup de n persoane dintre care vom alege comitete de cate
k persoane, ecare comitet avand un presedinte si un vicepresedinte. Exista
_
n
k
_
posibilitat i de a alege asemenea comitete, iar pentru ecare comitet presedintele si vi-
cepresedintele pot selectat i n k(k 1) moduri. Numarul comitetelor pe care le putem
forma cu un presedinte si un vicepresedinte este
1 2
_
n
2
_
+ 2 3
_
n
3
_
+ + (n 1) n
_
n
n
_
.
Select ia se poate face si n alta maniera: putem alege presedintele si vicepresedintele
n n(n 1) moduri, dupa care ad augam ceilalt i membri ai comitetului dintre cele n 2
persoane ramase. Deoarece numarul tuturor submult imilor unei mult imi cu n2 elemente
este 2
n2
, obt inem
1 2
_
n
2
_
+ 2 3
_
n
3
_
+ + (n 1) n
_
n
n
_
= n(n 1)2
n2
.
Problema 13.5 Doua sute de student i participa la un concurs de matematica, la care
au de rezolvat 6 probleme. Se stie ca ecare problema a fost rezolvata de cel put in 120 de
participant i. Sa se demonstreze ca exista doi participant i astfel ncat ecare problema a
fost rezolvata de cel put in unul dintre ei.
Solut ie.

Intrucat ecare problema a fost rezolvata de cel put in 120 de participant i, de-
ducem ca exista cel put in 720 de solut ii corecten total. Deoarece sunt doar 200 de student i,
obt inem ca macar un student a rezolvat cel put in patru probleme. Daca a rezolvat 5 sau
6 probleme, atunci el poate face pereche cu oricare alt student. Presupunem ca a rezolvat
exact 4 probleme. Fiecare dintre cele doua probleme ramase a fost rezolvata de cel put in
120 de participant i si cum numarul total este 200, exista cel put in un student care le-a
rezolvat pe amandoua.
204
Problema 13.6 O mult ime S ce cont ine patru numere naturale o numim conexa daca
pentru orice x S, cel put in unul din numerele x 1 sau x + 1 apart ine lui S. Fie C
n
numarul de submult imi conexe ale mult imii 1, 2, . . . n.
(a) Sa se determine C
7
;
(b) Sa se determine o formula generala pentru C
n
.
Romanian Mathematical Olympiad, 2006
Solut ie. Fie S = a, b, c, d mult ime conexa, astfel ncat a < b < c < d. Deoarece
a 1 / S rezulta ca a +1 S, deci b = a +1. Analog, ntrucat d +1 / S, avem d 1 S,
adica c = d 1. Astfel o mult ime care sa satisfaca condit iile problemei este de forma
S = a, a + 1, d 1, d , cu d a > 2.
(a) Submult imile conexe ale mult imii 1, 2, . . . , 7 sunt
1, 2, 3, 4; 1, 2, 4, 5; 1, 2, 5, 6; 1, 2, 6, 7;
2, 3, 4, 5; 2, 3, 5, 6; 2, 3, 6, 7; 3, 4, 5, 6; 2, 4, 6, 7; 4, 5, 6, 7.
(b) Denim diametrul mult imii S = a, a+1, d1, d astfel D = da+1. Este evident
ca D > 3 si D n 1 + 1 = n.

In cazul D = 4 avem n 3 mult imi conexe, daca
D = 5 exista n 4 mult imi conexe s.a.m.d. Astfel obt inem
C
n
= 1 + 2 + + (n 3) =
(n 3)(n 2)
2
.
Problema 13.7 Fie mult imea A astfel ncat [A[ = n
2
si T o familie de submult imi ale lui
A care au n elemente. Presupunem ca oricare doua mult imi din T au cel mult un element
n comun.
(a) Sa se arate ca [T[ n
2
+n;
(b) Pentru n = 3, sa se puna n evident a un exemplu n care [T[ sa ia valoarea maxima.
Romanian TST, 1985
Solut ie. (a) Pentru x A, notam cu k(x) numarul mult imilor B T pentru care
x B. Fie aceste mult imi B
1
, B
2
, . . . , B
k(x)
.
Atunci B
1
x, B
2
x, . . . , B
k(x)
x sunt submult imi disjuncte ale lui Ax, ecare
avand n1 elemente.

Intrucat A x are n
2
1 elemente, obt inem k(x)
n
2
1
n1
= n+1.
Repetand rat ionamentul orice x A si sumand, obt inem

xA
k(x) n
2
(n + 1).
Dar

xA
k(x) =

BF
[B[ = n[T[.
Atunci n[T[ n
2
(n + 1), de unde [T[ n
2
+n.
Combinatorica si grafuri 205
(b)

In cazul n care [A[ = 3
2
, asezam elementele 1, 2, . . . , 9 ntr-o matrice
_
_
1 2 3
4 5 6
7 8 9
_
_
si alegem ca mult imi ale lui T cele formate cu factorii produselor care apar n calculul
determinantului de ordin 3 al matricei de mai sus. Deci,
T = 1, 2, 3, 4, 5, 6, 7, 8, 9, 1, 4, 7, 2, 5, 8, 3, 6, 9,
1, 5, 9, 2, 6, 7, 3, 4, 8, 3, 5, 7, 2, 4, 9, 1, 6, 8 .

In acest caz [T[ = 3


2
+ 3 = 12.
Problema 13.8 Se noteaza prin s
n,m,r
numarul funct iilor f : X Y, unde [X[ =
n, [Y [ = m, cu proprietatea ca exista Z Y, [Z[ = r si astfel ncat f(X) Z.
Sa se arate ca
s
n,m,r
= m
n

_
r
1
_
(m1)
n
+
_
r
2
_
(m2)
n
+ (1)
n
(mr)
n
.
Solut ie. Fie Z = y
1
, . . . y
r
si pentru ecare i = 1, . . . , r notam cu A
i
= f : X
Y, y
i
/ f(X). Obt inem s
n,m,r
= m
n
[A
1
A
2
A
r
[. Dar A
i
reprezinta mult imea
funct iilor denite pe X, cu valori n Y y
i
, deci [A
i
[ = (m 1)
n
. Mult imea A
i
A
j
cont ine funct iile denite pe X, cu valori n Y y
i
, y
j
si deci [A
i
A
j
[ = (m 2)
n
.

In general, [A
i
1
A
i
2
A
i
k
[ = (m k)
n
, unde 1 i
1
< i
2
< < i
k
r.
Deoarece exista
_
r
k
_
submult imi de indici i
1
, i
2
, . . . , i
k
, rezulta ca ecare suma de forma

1i
1
<i
2
<<i
k
r
[A
i
1
A
i
2
A
i
k
[ cont ine
_
r
k
_
termeni, ecare ind egal cu (mk)
n
.
Obt inem
s
n,m,r
= m
n
[A
1
A
2
A
r
[
= m
n

_
r
1
_
(m1)
n
+
_
r
2
_
(m2)
n
+
+(1)
r
(mr)
n
.
Problema 13.9 Fie A un alfabet format din n litere identice: a
1
, a
1
; a
2
, a
2
; . . . ; a
n
, a
n
,
perechile diferite constand din litere diferite. Se formeaza toate cuvintele care folosesc
toate cele 2n litere din alfabetul A, astfel sa nu apara doua litere identice vecine.
Sa se arate ca numarul acestor cuvinte este egal cu:
1
2
n
[(2n)!
_
n
1
_
2(2n 1)! +
_
n
2
_
2
2
(2n 2)! + (1)
n
2
n
n!].
Solut ie. Numarul tuturor cuvintelor care folosesc toate cele 2n litere din alfabetul A este
egal cu
(2n)!
(2!)
n
=
(2n)!
2
n
,
deoarece literele identice pot permutatentre elen (2!)
n
= 2
n
moduri distincte, rezultand
un acelasi cuvant format cu cele 2n litere din A.
Sa notam cu A
i
mult imea cuvintelor formate cu cele 2n litere din A pentru care cele
doua litere a
i
sunt vecine. Rezulta ca numarul cautat este egal cu
(2n)!
2
n
[A
1
A
2
A
n
[.
206
Pentru a evalua [A
1
A
2
A
n
[ aplicam principiul includerii-excluderi:
[A
1
A
2
A
n
[ =

n
i=1
[A
i
[

1i<jn
[A
i
A
j
[
+(1)
k1

1i
1
<<i
k
n
[A
i
1
A
i
2
A
i
k
[ +. . .
+(1)
n1
[
n
i=1
A
i
[.
Sa calculam n cazul general numarul de elemente din A
i
1
A
i
2
A
i
k
si sa
aratam ca acesta nu depinde de alegerea indicilor 1 i
1
< < i
k
n. Daca un cuvnt
apart ine acestei mult imi nseamna ca el apart ine ecareia din mult imile A
i
1
, A
i
2
, . . . , A
i
k
,
deci literele a
i
1
, a
i
1
; a
i
2
, a
i
2
; . . . ; a
i
k
, a
i
k
sunt vecine. Cuvintele pentru care cele k perechi
de litere sunt vecine se obt in n modul urmator: se formeaza toate cuvintele avnd 2n k
litere care formeaza un alfabet obt inut din A prin suprimarea catei unei litere dintre
a
i
1
, a
i
2
, . . . , a
i
k
. Apoi n ecare cuvant astfel format se dubleaza literele a
i
1
, a
i
2
, . . . , a
i
k
,
adaugand dupa litera a
i
j
o alta litera egala cu a
i
j
, pentru ecare j = 1, . . . , n. Deci
obt inem:
[A
1
A
2
A
n
[ =
(2n k)!
(2!)
nk
=
2
k
(2n k)!
2
n
.
Deoarece indicii i
1
, . . . , i
k
cu 1 i
1
< < i
k
n pot alesi n
_
n
k
_
moduri, rezulta
ca numarul cuvintelor care nu cont in doua litere identice vecine este
(2n)!
2
n

_
n
1
_
2(2n 1)!
2
n
+
_
n
2
_
2
2
(2n 2)!
2
n
+
(1)
n
2
n
n!
2
n
.
Problema 13.10 Daca p este o permutare a mult imii X = 1, . . . , n, spunem ca numarul
i este un punct x al permutarii p, daca p(i) = i, 1 i n. Sa se arate ca numarul D(n)
al permutarilor fara puncte xe ale mult imii X este egal cu
D(n) = n!
_
1
1
1!
+
1
2!

1
3!
+ +
(1)
n
n!
_
.
Care este numarul permutarilor a n obiecte cu p puncte xe ?
Solut ie. Notam cu A
i
mult imea celor (n 1)! permutari care admit un punct x n i
si vom determina numarul permut arilor care admit cel put in un punct x. Acest numar
este egal cu [A
1
A
2
A
n
[ si, conform principiului includerii-excluderii, se obt ine din
relat ia:
[A
1
A
2
A
n
[ =
n

i=1
[A
i
[

1i<jn
[A
i
A
j
[ + + (1)
n1
[
n
i=1
A
i
[.
Dar [A
i
1
A
i
2
A
i
k
[ = (nk)!, deoarece o permutare din mult imea A
1
A
2
A
n
reprezinta puncte xe n pozit iile i
1
, i
1
, . . . , i
k
, celelalte pozit ii cont inand o permutare a
celor n k elemente ramase, al caror numar este (n k)!. Dar pozit iile i
1
, i
1
, . . . , i
k
pot
alese din mult imea celor n pozit ii n
_
n
k
_
moduri, deci

n
_
i=1
A
i

=
_
n
1
_
(n 1)!
_
n
2
_
(n 2)! + + (1)
n
_
n
n
_
.
Combinatorica si grafuri 207
Astfel,
D(n) = n!

n
_
i=1
A
i

= n!
_
n
1
_
(n 1)! + + (1)
n
_
n
n
_
,
de unde se obt ine expresia lui D(n).
Rezulta ca lim
n
D(n)
n!
=
1
e
, deci pentru n sucient de mare probabilitatea ca o per-
mutare a n elemente aleasa aleator sa nu aiba puncte xe este apropriata de
1
3
. Deoarece
cele p puncte xe (0 p n) pot alese n
_
n
p
_
moduri si celelalte n p puncte
nu mai sunt xe, rezulta ca numarul permutarilor din o
n
cu p puncte xe este egal cu
_
n
p
_
D(n p), deoarece pentru ecare alegere a celor p puncte xe exista D(n p)
permutari ale obiectelor ramase, fara puncte xe, daca se deneste D(0) = 1.
Problema 13.11 Fie X = 1, 2, . . . , n si D(n) numarul permutarilor mult imii X fara
puncte xe. Daca E(n) reprezinta numarul permutarilor pare ale mult imii X fara puncte
xe, aratat i ca
E(n) =
1
2
(D(n) + (1)
n1
(n 1)).
Solut ie. Conform problemei anterioare, avem
D(n) = n!
_
1
1
1!
+
1
2!

1
3!
+ +
(1)
n
n!
_
.
Pentru a obt ine expresia lui E(n) vom nota cu A
i
mult imea permutarilor pare p S
n
astfel ncat p(i) = i. Deoarece S
n
cont ine
1
2
n! permutari pare, rezulta ca
E(n) =
1
2
n!

n
i=1
A
i

=
1
2
n!

n
i=1
[A
i
[ +

1i<jn
[A
i
A
j
[ + + (1)
n

n
i=1
A
i

.
Deoarece [A
i
1
A
i
2
A
i
k
[ =
1
2
(nk)!, rezulta n mod analog ca pentru D(n), ca
E(n) =
1
2
n!
_
n
1
_
1
2
(n 1)! + + (1)
n1
_
n
n 1
_
+ (1)
n
=
1
2
(D(n) + (1)
n1
(n 1)).
Problema 13.12 Vericat i ca:

n=0
D(n)
t
n
n!
=
e
t
1t
;
D(n + 1) = (n + 1)D(n) + (1)
n+1
;
D(n + 1) = n(D(n) +D(n 1)).
Solut ie. Conform unei probleme anterioare, avem:
D(n)
n!
= 1
1
1!
+
1
2!
+
(1)
n
n!
,
Pe de alta parte, se cunoaste dezvoltarea:
e
t
t 1
=
_
1
t
1!
+
t
2
2!
. . .
_
(1 +t +t
2
+. . . ).
Din relat iile de mai sus deducem ca
D(n)
n!
este coecientul lui t
n
din dezvoltarea funct iei
e
t
t1
si astfel se obt ine prima relat ie din enunt ul problemei.
208
Pentru a obt ine relat iile de recurent a pornim de la formula
D(n) = n!
_
1
1
1!
+
1
2!

1
3!
+ +
(1)
n
n!
_
.
Este evident ca
D(n + 1) = (n + 1)!(1
1
1!
+
1
2!

1
3!
+ +
(1)
n
n!
+
(1)
n+1
(n+1)!
)
= (n + 1)n!(1
1
1!
+
1
2!

1
3!
+ +
(1)
n
n!
) + (n + 1)!
(1)
n+1
(n+1)!
,
de unde
D(n + 1) = (n + 1)D(n) + (1)
n+1
.

In continuare, prin adunarea relat iei obt inute mai sus cu


D(n) = nD(n 1) + (1)
n
,
se obt ine D(n +1) +D(n) = n(D(n) +D(n 1)) +D(n) +(1)
n+1
+(1)
n
, ceea ce este
echivalent cu
D(n + 1) = n(D(n) +D(n 1)).
Problema 13.13 Care este numarul u
n
de moduri n care putem urca o scara cu n trepte,
stiind ca la ecare pas urcam o treapta sau doua trepte?
Solut ie. Este evident ca numarul u
n
reprezinta n acelasi timp numarul de scrieri ale
numarului natural n ca o suma a numerelor 1 sau 2, doua scrieri ind distincte daca ele
difera si prin ordinea termenilor.
De exemplu
2 = 1 + 1
3 = 2 + 1 = 1 + 2 = 1 + 1 + 1
4 = 2 + 2 = 2 + 1 + 1 = 1 + 2 + 1 = 1 + 1 + 2 = 1 + 1 + 1 + 1.
Primul termen este 1 sau 2.

In primul caz numarul de scrieri este egal cu u


n1
, deoarece
restul termenilor sunt egali cu 1 sau cu 2 si au suma n1, iar n al doilea caz numarul de
scrieri este egal cu u
n2
. Deci am obt inut recurent a
u
n
= u
n1
+u
n2
,
cu valorile init iale u
1
= 1, u
2
= 2. Denim u
0
= 1 si astfel obt inem sirul lui Fibonacci,
deci u
n
= F
n
. Atasam ecuat ia caracteristica r
2
= r + 1, care are solut iile r
1
=
1+

5
2
si
r
2
=
1

5
2
.
Solut ia generala este
u
n
= c
1
r
n
1
+c
2
r
n
2
,
unde constantele c
1
si c
2
se detrmina din condit iile init iale u
0
= 1 si u
1
= 1.
Astfel se obt ine sistemul
_
C
1
+C
2
= 1
1+

5
2
C
1
+
1

5
2
C
2
= 1,
care are solut iile C
1
=

5+1
2

5
si C
2
=

51
2

5
.
Astfel obt inem
u
n
= F
n
=

5 + 1
2

5
_
1 +

5
2
_
n
+

5 1
2

5
_
1

5
2
_
n
=
Combinatorica si grafuri 209
=
1

5
__
1 +

5
2
_
n+1

_
1

5
2
_
n+1
_
,
pentru orice n 0.
Daca notam f(x) =

n=0
F
n
x
n
, obt inem
xf(x) =

n=1
F
n1
x
n
si x
2
f(x) =

n=2
F
n2
x
n
.
T inand cont de relat ia de recurent a a lui u
n
deducem:
f(x) xf(x) x
2
f(x) = F
0
+ (F
1
F
0
)x +

n=2
(F
n
F
n1
F
n2
)x
n
= F
0
= 1,
de unde
f(x) =
1
1 x x
2
.
Observat ii. 1) Funct ia f(x) =
1
1xx
2
este funct ia generatoare a sirului lui Fibonacci.
2) Problema de mai sus poate enunt ata si n modul urmator:
Sa se determine numarul de moduri de pardosire a unei alei de dimensiuni 2 n cu placi
de dimensiune 2 1.
Problema 13.14 Sa se arate ca funct ia generatoare a numerelor lui Catalan T
n
este data
de egalitatea:
f(x) = T
1
x +T
2
x
2
+ +T
n
x
n
+ =
1

1 4x
2
.
Sa se gaseasca pe aceasta cale expresia numerelor T
n
.
Solut ie. Numarul lui Catalan se deneste ca ind numarul de moduri n care se pot
pune parantezele ntr-un produs neasociativ de n factori, scrisi n ordinea x
1
, x
2
, . . . , x
n
.
Daca exista o singura pereche de paranteze care nu sunt cont inute n alte paranteze,
atunci aceasta pereche cont ine n interior produsul factorilor x
2
, . . . , x
n
, ramanand n
exterior factorul x
1
, sau cont ine produsul x
1
, . . . , x
n1
, ramanand nafara factorul x
n
.
Daca exista doua perechi de paranteze care nu sunt cont inute n alte paranteze, rezulta
ca aceste perechi cont in produsul factorilor x
1
, . . . x
k
, respectiv x
k+1
, . . . , x
n
, unde 2 k
n2. Deoarece ntr-un produs de k, respectiv nk factori putem pune parantezele n T
k
,
respectiv T
nk
moduri, rezulta relat ia
T
n
=
n1

k=1
T
k
T
nk
,
cu T
1
= 1.
Daca notam f(x) = T
1
x +T
2
x
2
+ +T
n
x
n
+ =
1

14x
2
, atunci
f
2
(x) = T
2
1
x
2
+ (T
1
T
2
+T
2
T
1
)x
3
+ + (
n1

k=1
T
k
T
nk
)x
n
+ = f(x) x,
210
t inand seama de relat ia de recurent a obt inuta si de datele init iale T
1
= T
2
= 1. Solut iile
ecuat iei de gradul al doilea f
2
(x) f(x) x = 0 sunt f(x) =
1

14x
2
. Presupunem ca
x <
1
4
si ntrucat f(0) = 0, obt inem
f(x) =
1

1 4x
2
.
Vom dezvolta n serie de puteri funct ia

1 4x, folosindu-ne de formula generalizata


a binomului lui Newton.
(x +a)

= a

+a
1
x +
(1)
2!
a
2
x
2
+ +
+
(1)...(k+1)
k!
a
k
x
k
+ ,
unde a > 0.
Aceasta serie este convergenta pentru orice x care verica [x[ < a. Daca este un
numar ntreg si pozitiv, numai un numar nit de termeni ai seriei sunt nenuli, iar dez-
voltarea obt inuta este formula binomului lui Newton.
Pentru a dezvolta (1 4x)
1
2
n serie de puteri ale lui x vom nota y = 4x, =
1
2
si
vom dezvolta binomul (y + 1)
1
2
. Se obt ine coecientul lui x
n
egal cu
(1/2)(1/2)(3/2)(1/2n+1)
n!
(4)
n
= (1)
n1 1
2
n
13(2n3)
n!
(1)
n
2
2n
=
=
(2n2)!
2
n1
(n1)!n!
2
n
= 2
(2n2)!
n(n1)!(n1)!
=
2
n
_
2n 2
n 1
_
.
Dar T
n
este coecientul lui x
n
din dezvoltarea lui f(x), deci se obt ine nmult ind cu
1
2
coecientul lui x
n
din dezvoltarea lui (1 4x)
1
2
. Astfel, avem T
n
=
1
n
_
2n 2
n 1
_
.
Problema 13.15 Sa se arate ca numarul sirurilor (x
1
, x
2
, . . . , x
2n2
) cu x
i
1, 1,
pentru i = 1, 2, . . . , 2n 2 si care satisfac condit iile:
(a) x
1
+x
2
+ +x
k
0 pentru orice 1 k 2n 2;
(b) x
1
+x
2
+ +x
2n2
= 0
este egal cu
1
n
_
2n2
n1
_
.
Solut ie. Vom determina numarul de siruri de litere care cont in litera a de k ori, litera
b de m ori si care au proprietatea
(P) : Pentru orice i, 1 i m + k, numarul de litere a n primele i litere ale sirului este
mai mare sau egal cu numarul de litere b.
Numarul acestor siruri este nenul daca si numai daca este vericata condit ia k m > 0.
Numarul sirurilor de litere care cont in litera a de k ori si litera b de m ori este egal cu
P(m, k) =
_
m+k
m
_
. Pentru a rezolva problema este sucient sa determinam numarul sirurilor
care nu verica proprietatea (P), deoarece numarul cautat se obt ine prin scaderea acestui
numar din
_
m+k
m
_
.
Vom arata ca numarul sirurilor formate din m litere b si k litere a care nu verica
proprietatea (P) este egal cu P(m1, k +1) =
_
m+k
m1
_
, adica este egal cu numarul tuturor
sirurilor formate din m1 litere b si din k + 1 litere a. Pentru aceasta vom considera un
sir format din m litere b si k litere a care nu verica (P). Va exista o pozit ie cu numarul
2s +1, unde s 0, astfel ncat sirul considerat cont ine litera b n pozit ia 2s +1, iar n fat a
acestei pozit ii exista un numar egal de litere a si b, numar egal cu s. Vom considera cel
mai mic indice cu aceasta proprietate si vom adauga litera a n fat a acestui sir, obt inand
Combinatorica si grafuri 211
astfel un sir format din m litere b si k +1 litere a. Prima litera a sirului astfel obt inut este
a, iar printre primele 2s + 2 litere exista un numar egal de litere a si b. Schimbam literele
a si b ntre ele pe primele 2s + 2 pozit ii ale sirului.
Deoarece n primele 2s + 2 pozit ii numarul de litere a a fost egal cu numarul literelor
b, nu se va schimba numarul total de litere de ecare fel si obt inem un sir format din m
litere b si k + 1 litere a. Prima litera este acum b.
Astfel am asociat ecarui sir din m litere b si k litere a care nu satisfac (P), un sir
format din m litere b si k + 1 litere a, care ncep cu litera b. Se demonstreaza imediat ca
aceasta aplicat ie este injectiva. (Se considera doua siruri diferite care nu satisfac (P) si
care difera pe o pozit ie de rang p 2s + 1 sau p > 2s + 1.)
Vom demonstra ca n acest mod este posibil sa se obt ina orice sir format din m litere
b si k +1 litere a, care ncep cu litera b, deci aplicat ia este si surjectiva. Sa consideram un
astfel de sir. Deoarece presupunem m k sau m < k + 1, va exista o pozit ie pana la care
exista un numar egal de litere a si b, pentru ca sirul ncepe cu litera b.
Daca pana la prima pozit ie cu aceste proprietat i nlocuim literele a si b ntre ele si
suprimam prima litera a, vom gasi un sir format din k litere a si m litere b care nu verica
(P).
Daca aplicam acestui sir transformarea descrisa gasim sirul de la care am plecat. Da-
torita acestei biject ii numarul sirurilor cu m litere b si k litere a care nu verica (P) este
egal cu numarul sirurilor cu m litere b si k + 1 litere a care ncep cu b. Daca suprimam
prima litera b gasim toate sirurile formate din m1 litere b si k+1 litere a, al caror numar
este egal cu P(m1, k + 1) =
_
m+k
m1
_
.

In concluzie, numarul sirurilor care satisfac (P) este egal cu


_
m+k
m
_

_
m+k
m1
_
=
k m+ 1
k + 1
_
m+k
m
_
.
Pentru k = m = n 1 se obt ine numarul T
n
=
1
n
_
2n2
n1
_
.
Acest numar reprezinta solut ia problemei, deoarece daca l nlocuim pe 1 cu litera a
si pe 1 cu litera b, condit ia (a) exprima faptul ca numarul literelor a este cel put in egal
cu numarul literelor b n primele k pozit ii ale sirului, pentru 1 k 2n 2. Condit ia (b)
exprima faptul ca numarul literelor a este egal cu numarul literelor b si ambele sunt egale
cu n 1.
Problema 13.16 O triangulat ie a unui poligon convex A
1
A
2
. . . A
n+1
cu n + 1 varfuri
este o mult ime formata din n2 diagonale care nu se intersecteazan interiorul poligonului
ci numai n varfuri si care mpart suprafat a poligonului n n 1 triunghiuri.
Sa se arate ca numarul de triangulat ii ale unui poligon convex cu n + 1 varfuri este
egal cu
T
n
=
1
n
_
2n 2
n 1
_
.
Solut ie. Vom arata ca exista o biject ie de la triangulat iile unui poligon convex cu n + 1
vrfuri la mult imea scrierilor cu paranteze ale unui produs de n factori n ordinea x
1
, . . . , x
n
.
Daca poligonul cu n+1 varfuri este A
1
A
2
. . . A
n+1
ne vom deplasa din A
1
n A
2
s.a.m.d.
pe laturile poligonului pana cand ajungemn A
n+1
, obt inand o scriere cu paranteze a unui
produs cu n factori, dupa regulile urmatoare:
-cand ne deplasam pe o latura scriem un nou factor x
i
, n ordinea x
1
, . . . , x
n
;
-cand ajungem ntr-un varf la care sosesc anumite diagonale ale triangulat iei, scriem
un numar de paranteze de nchidere egal cu numarul de diagonale care au o extremitate
212
n acel varf si pentru care cealalta extremitate a fost parcursa si un numar de paranteze
de deschidere egal cu numarul de diagonale care sunt incidente n acel varf si pentru care
cealalta extremitate nu a fost vizitata.
Este evident ca aceasta corespondent a este injectiva.
Pentru a arata ca este si surjectiva, vom considera un produs cu paranteze a n factori
n ordinea x
1
, . . . , x
n
. Acest produs cont ine n 2 paranteze de deschidere si tot atatea de
nchidere.
Fiecarei paranteze de deschidere i corespunde o unica paranteza de nchidere. Pentru
ecare pereche formata dintr-o paranteza de deschidere si una de nchidere vom considera
prima litera care se gaseste la dreapta parantezei de deschidere, e x
i
si prima litera care
se gaseste la stanga parantezei de nchidere, e x
j
si vom duce diagonala A
i
A
j+1
.
Deoarece ecare pereche de paranteze cont ine n interior un produs a doi factori si
parantezele sunt corect puse, cele n 2 diagonale ale poligonului constituie o triangulat ie
a acestuia.
Aplicand acestei triangulat ii corespondent a descrisa, gasim produsul cu paranteze a n
factori de la care am plecat, ceea ce demonstreaza ca ea este o biject ie.

In concluzie, numarul de triangulat ii este egal cu numarul lui Catalan, T


n
.
Problema 13.17 Sa se arate ca numarul funct iilor crescatoare
f : 1, . . . , n 1, . . . , n
care satisfac condit ia f(x) x, pentru orice 1 x n, este egal cu numarul lui Catalan
T
n+1
=
1
n + 1
_
2n
n
_
.
Solut ie. Considerand un sistem de axe xOy, vom desena dreptele de ecuat ii x = k, y = l
unde 0 k, l n sunt numere ntregi si vom considera punctele de intersect ie ale acestor
drepte situate n primul cadran, pe prima bisectoare si sub prima bisectoare.
Pentru ecare funct ie crescatoare f : 1, . . . , n 1, . . . , n vom construi un drum
n aceasta ret ea n felul urmator: daca ne aam n punctul M(i, f(i)), ne deplasam n
punctul M
1
(i +1, f(i)) pe un segment orizontal, apoi ne deplasam pe un segment vertical
pana cand ajungemn punctul M
2
(i+1, f(i+1)). Daca f(i+1) = f(i), obt inem M
2
= M
1
;
n caz contrar, deplasarea are loc n sus, deoarece f(i + 1) > f(i).
Efectuand aceste deplasari pentru i = 1, . . . , n 1 obt inem un drum ascendent n
aceasta ret ea, de extremitat i (1, 0) si (n, f(n)). Unind apoi originea O cu punctul (1, 0)
printr-un segment orizontal si n cazul cand f(n) < n, punctul (n, f(n)) cu punctul A(n, n)
situat pe prima bisectoare, printr-un sir de segmente verticale, obt inem un drum de ex-
tremitat i O(0, 0) si A(n, n).
Este evident ca drumul astfel obt inut este compus din n segmente orizontale si n
segmente verticale, nu are coborasuri cand ne deplasam de la O catre A si este situat sub
prima bisectoare. Am obt inut astfel o corespondent a bijectiva ntre mult imea funct iilor
crescatoare f : 1, . . . , n 1, . . . , n care satisfac condit ia f(x) x, pentru orice
1 x n, si mult imea drumurilor de extremitat i O si A cu proprietat ile ment ionate.
Injectivitatea acestei aplicat ii este evidenta, deoarece la funct ii diferite corespund drumuri
diferite. Pentru a arata ca este si surjectiva, sa consideram un drum cu extremitat ile O si
A, cu proprietat ile ment ionate. Denim funt ia f
d
prin
f
d
= max j[ (i, j) d,
Combinatorica si grafuri 213
pentru orice i = 1, . . . , n.

In acest caz imaginea funct iei f


d
prin corepondent a descrisa este
chiar drumul d, ceea ce demonstreaza surjectivitatea acestei corespondent e.
Pentru a numara drumurile ascendente de lungime 2n de extremitat i O si A(n, n), sit-
uate sub prima bisectoare, se observa ca exista o corespondent a biunivoca ntre mult imea
drumurilor si mult imea sirurilor (x
1
, x
2
, . . . , x
2n
) cu x
i
= 1 sau x
i
= 1, pentru 1 i 2n,
care satisfac condit iile:
(a) x
1
+x
2
+ +x
k
0, pentru orice k = 1, . . . , n si
(b) x
1
+x
2
+ +x
2n
= 0
Pentru a deni aceasta corespondent a sa ne deplasam pe un drum ascendent d de la O la
A. Drumul d se poate scrie ca un sir de segmente de lungime 1, d = (s
1
, s
2
, s
2n
), ordinea
indicilor indicand ordinea de parcurgere a segmentelor cand ne deplasam de la O la A. Sirul
asociat drumului se obt ine din sirul (s
1
, s
2
, s
2n
) scriind n locul ecarui segment orizontal
numarul 1 si n locul ecarui segment vertical numarul 1.
Condit ia (a) exprima faptul ca drumul d nu poate trece prin puncte situate deasupra
primei bisectoare a axelor, iar condit ia (b) faptul ca el cont ine n segmente orizontale si n
segmente verticale, deci faptul ca el se termina n punctul A.
Conform unei probleme anterioare, numarul sirurilor (x
1
, . . . , x
2n
) formate din 1 si 1
cu proprietat ile enunt ate este egal cu T
n+1
=
1
n+1
_
2n
n
_
.
Problema 13.18 Sa se arate ca numarul sirurilor (a
1
, a
2
, . . . a
2n+1
) formate din ntregi
nenegativi, cu proprietatea ca a
1
= a
2n+1
= 0 si [a
i
a
i+1
[ = 1 pentru i = 1, 2, . . . , 2n,
este egal cu numarul lui Catalan
T
n+1
=
1
n + 1
_
2n
n
_
.
Solut ie. Conform unei probleme anterioare, numarul sirurilor (x
1
, x
2
, . . . , x
2n
) cu termeni
1 sau 1, care verica condit iile x
1
+ x
2
+ + x
k
0 pentru orice 1 k 2n si
x
1
+x
2
+ +x
2n
= 0 este egal cu
1
n+1
_
2n
n
_
.
Sa observam ca exista o biject ie de la mult imea sirurilor x = (x
1
, x
2
, . . . , x
2n
) la
mult imea sirurilor a = (a
1
, a
2
, . . . , a
2n+1
) care verica condit iile date, denita prin
f(x) = a daca
a
1
= 0,
a
2
= x
1
= 1,
a
3
= x
1
+x
2
,
. . .
a
k+1
= x
1
+x
2
+ +x
k
, pentru 1 k 2n,
deci a
2n+1
= 0.
Problema 13.19 Fie X o colect ie de n obiecte (n 1), nu neaparat distincte. Daca
n a
2
+1, cu a numar ntreg nenegativ, aratat i ca are loc cel put in unul din urmatoarele
doua cazuri:
(a) cel put in a + 1 obiecte sunt identice;
(b) cel put in a + 1 obiecte sunt distincte doua cate doua.
214
Solut ie. Prin reducere la absurd, presupunem ca (a) si (b) nu au loc. Rezulta ca X cont ine
cel mult a obiecte distincte doua cate doua si ecare dintre acestea sunt prezente n cel
mult a copii, deci X are cel mult a
2
n 1 obiecte, ceea ce este absurd.

In consecint a, concluzia problemei este adevarata.


Observat ie. Problema de mai sus este de acelasi tip cu urmatoarele:
(a) Din orice sir de n
2
+1 numere reale se poate extrage sau un subsir crescator cu n+1
termeni, sau un subsir descrescator cu n + 1 termeni.
(b) Se dau n
2
+1 intervale pe o dreapta, atunci sau exista n+1 intervale dintre ele astfel
ncat oricare doua sunt disjuncte, sau exista n + 1 cu intersect ia nevida.
Problema 13.20 Fie A = (
i
)
1in
, B = (
i
)
1in
, C = (
i
)
1in
trei partit ii ale unei
mult imi nite M.
Daca pentru orice i, j, k exista inegalitatea
[A
i
B
j
[ +[A
i
C
k
[ +[B
j
C
k
[ n,
atunci [M[
n
3
3
, egalitatea putand avea loc n cazul n 0 ( mod 3).
Solut ie. Sumand inegalitat ile date dupa j = 1, 2, . . . , n obt inem
[A
i
[ +n[A
i
C
k
[ +[C
k
[ n
2
.
Adunam inegalitat ile obt inute pentru i = 1, . . . , n si avem
[M[ + 2n[C
k
[ n
3
.

Insumand si dupa k deducem ca n[M[ + 2n[M[ n


4
, de unde [M[
n
3
3
.
Daca n 0 (mod 3) aceasta margine inferioara poate atinsa.

Intrucat [M[
n
3
3
,
consideram urmatoarea partit ie a lui M :
M = A
1
1
A
1
2
A
1
n
A
2
1
A
2
2
A
2
n
A
n
1
A
n
2
A
n
n
,
astfel ncat [A
j
i
[ = n/3 pentru orice 1 i, j n.
Daca notam A
i
=
n
_
j=1
A
i
j
, B
i
=
n
_
j=1
A
j
i
si C
i
=
n
_
j=1
A
j
i+j1 (mod n)
, atunci partit iile
(A
i
), (B
i
), (C
i
), 1 i n, verica egalitatea
[A
i
B
j
[ +[A
i
C
k
[ +[B
j
C
k
[ = n,
deoarece pentru i, j, k sunt vericate relat iile
[A
i
B
j
[ = [A
i
C
k
[ = [B
j
C
k
[ =
n
3
.

In acest caz avem [M[ =


n
3
3
.
Problema 13.21 O funct ie f : X X se numeste idempotenta daca f(f(x)) = f(x),
pentru orice x X.
Daca mult imea X are n elemente, sa se arate ca numarul i(n) al funct iilor idempotente
f : X X este dat de relat ia:
i(n) =
n

k=1
_
n
k
_
k
nk
.
Combinatorica si grafuri 215
Solut ie. Vom arata pentru nceput ca f este idempotenta daca si numai daca funct ia
g : Y Y, unde Y = f(X) si g(x) = f(x) pentru orice x Y, este funct ia identica.
Consideram un element arbitrar x Y = f(X), atunci exista z X astfel ncat
x = f(z) si ntrucat f este idempotenta avem:
g(x) = f(x) = f(f(z)) = f(z) = x,
pentru orice element arbitrar x Y.
Daca g este funct ia identica, deducem:
f(f(x)) = f(y) = g(y) = y = f(x),
pentru orice x X, deci f este idempotenta.
Daca notam [Y [ = k, rezulta 1 k n.
Mult imea Y poate aleasa n
_
n
k
_
moduri, restrict ia funct iei f la Y este funct ia
identica, iar numarul funct iilor h : X Y Y este egal cu k
nk
. Deoarece f este identica
pe Y rezulta ca f este unic determinata de restrict ia sa h la X Y. Astfel este justicata
formula pentru i(n).
Problema 13.22 Un lant de lungime nn familia partit iilor unei mult imi X cu n elemente
este un sir de partit ii distincte dou a cate doua care verica:
P
1
< P
2
< < P
n
.
Partit ia P
1
are o singura clasa formata din X, iar P
n
are n clase care cont in ecare cate
un singur element al lui X.
Sa se arate ca numarul lant urilor de lungime nn familia partit iilor lui X cu n elemente
este egal cu
(n 1)!n!
2
n1
.
Solut ie. Fiecare partit ie P
k
, cu 1 k n 1, are k clase si se obt ine din P
k+1
prin
unicarea a doua clase ntr-una singura. Deci, daca P
n
, P
n1
, . . . , P
k+1
sunt xate, partit ia
P
k
poate aleasa n
_
k+1
2
_
moduri.
Numarul lant urilor de lungime n este egal cu
n1

k=1
_
k + 1
2
_
=
(n 1)!n!
2
n1
.
Problema 13.23 Sa se arate ca:

A
1
,...A
k
[A
1
A
2
A
k
[ = n(2
k
1)2
k(n1)
,
unde suma se face dupa toate alegerile submult imilor A
1
, . . . A
k
ale unei mult imi X cu n
elemente.
Solut ie. Fie Y X cu [Y [ = p. Y poate scris ca o reuniune a k mult imi, Y =
A
1
A
2
A
k
n (2
k
1)
p
moduri diferite.

Intr-adevar, ecare dintre cele p elemente
ale lui Y poate apart ine unui num ar de 2
k
1 familii nevide de submult imi A
1
, . . . A
k
.
216
Deoarece Y poate aleasa n
_
n
p
_
moduri diferite, rezulta ca

[A
1
A
k
[ =
n

p=1
p
_
n
p
_
(2
k
1)
p
= n(2
k
1)
n

p=1
_
n 1
p 1
_
(2
k
1)
p1
= n(2
k
1)2
k(n1)
.
Problema 13.24 Sa se demonstreze ca:

[A
1
A
k
[ = (2
k
1)

[A
1
A
k
[,
unde sumele se efctueaza dupa toate alegerile posibile ale submult imilor A
1
, . . . A
k
ale unei
mult imi X.
Solut ie. T inand cont de problema anterioara, trebuie sa mai aratam ca

[A
1
A
k
[ =
n2
k(n1)
, daca [X[ = n.
Deoarece operat ia de trecere la complementara:
C(A
1
A
k
) = CA
1
CA
k
stabileste o biject ie ntre familia mult imilor de forma A
1
A
k
si familia mult imilor
de forma A
1
A
k
, iar
[CA
1
CA
k
[ = n [A
1
A
k
[,
putem scrie

[A
1
A
k
[ =

(n [A
1
A
k
[) = n2
nk
n(2
k
1)2
k(n1)
= n2
k(n1)
,
adica ceea ce trebuie aratat.
S-a t inut seama ca ecare din sumele scrise cont ine 2
nk
termeni, iar ecare din mult imile
A
1
, . . . A
k
poate aleasa dintre submult imile lui X n 2
n
moduri.
Problema 13.25 Fie X o mult ime nita si E
1
, . . . , E
m
o familie de submult imi ale lui
X cu proprietatea ca oricare doua mult imi distincte E
i
si E
j
nu au exact un element n
comun si [E
i
[ 2 pentru i = 1, . . . , m.
Sa se arate ca putem colora elementele din X cu doua culori, astfel ncat nicio mult ime
E
i
sa nu aiba toate elementele de o aceeasi culoare, pentru 1 i m.
Solut ie. Fie X = x
1
, . . . , x
n
. Vom colora aceste elemente cu una din culorile a sau
b, astfel ncat nicio mult ime E
i
sa nu e monocromatica. Sa coloram elementul x
1
cu
culoarea a. Presupunand ca am colorat elementele x
1
, . . . , x
i
, cu 1 < i < n, cu culorile a
sau b astfel ncat sa nu apara nicio mult ime E
k
monocromatica, sa consideram cazul cand
acest proces nu mai poate continua. Deci nu putem colora elementul x
i+1
cu culoarea a,
deoarece exista o mult ime E x
1
, . . . , x
i+1
cu x
i+1
E care are toate elementele diferite
de x
i+1
colorate cu culoarea a. Elementul x
i+1
nu poate colorat nici cu culoarea b, deci
exista o mult ime F x
1
, . . . , x
i+1
cu x
i+1
F, care are toate elementele diferite de x
i+1
colorate cu culoarea b. Obt inem ca E, F sunt doua mult imi distincte dintre E
1
, . . . , E
m
si
Combinatorica si grafuri 217
E F = x
i+1
, ceea ce contrazice ipoteza. Deci putem colora elementul x
i+1
cu culoarea
a sau cu culoarea b, astfel ncat sa nu apara mult imi E
k
monocromatice.
Am demonstrat astfel prin induct ie ca acest proces de colorare poate continua pana
cand reusim sa coloram toate elementele din X cu doua culori, astfel ncat sa nu apara
nicio mult ime monocromatica.
Problema 13.26 Sa se arate ca numarul P(n, m) al partit iilor ntregului n n m part i
satisface recurent a:
P(n +k, k) = P(n, 1) +P(n, 2) + +P(n, k),
iar P(n, 1) = P(n, n) = 1
Solut ie. Partit iile numarului n cu cel mult k part i formeaza o mult ime cu P(n, 1) +
P(n, 2) + +P(n, k) elemente. Fiecare partit ie a lui n cu cel mult k part i poate scrisa
sub forma
n = a
1
+a
2
+ +a
m
+ 0 + + 0,
unde suma cont ine k termeni si a
1
a
2
a
m
1, cu 1 m k.
Din aceasta exprimare a lui n putem obt ine o partit ie a lui n +k cu k part i astfel:
n +k = (a
1
+ 1) + (a
2
+ 1) + + (a
m
+ 1) + 1 + + 1,
unde suma cont ine k termeni si a
1
+ 1 a
2
+ 1 a
m
+ 1 1.
Aplicat ia astfel denita este o inject ie, deoarece unor partit ii diferite ale lui n cu cel
mult k part i le corespund partit ii diferite ale lui n + k cu k part i. Ea este si surject ie,
deoarece orice partit ie a lui n + k cu k part i provine din acea partit ie a lui n cu m k
part i obt inuta prin scaderea unei unitat i din ecare termen al partit iei lui n+k si ret inand
primii termeni diferit i de zero. Existent a unei biject ii ntre mult imea partit iilor lui n cu
cel mult k part i si mult imea partit iilor lui n + k cu k part i justica recurent a data,
care permite calculul prin recurent a al tuturor numerelor P(n, k), plecand de la valorile
P(n, 1) = P(n, n) = 1, pentru orice n, si P(n, k) = 0 pentru n < k.
Problema 13.27 Sa se arate ca numarul P(n) al partit iilor lui n si P(n, m) al partit iilor
ntregului n n m part i, verica relat ia:
P(n, m) = P(n m) pentru m
n
2
.
Solut ie. Fiecarei partit ii a lui n n m part i, de forma n = n
1
+ n
2
+ + n
m
, cu n
1

n
2
n
m
1, i corespunde o partit ie a lui n m, obt inuta din scrierea
n m = (n
1
1) + (n
2
1) + + (n
m
1),
prin suprimarea eventualilor termeni nuli.
Aplicat ia astfel denita este injectiva. Vom demonstra ca ea este si surjectiva pentru
m
n
2
.

Intr-adevar, plecand de la o partit ie
n m = r
1
+ +r
k
a lui n m, obt inem k m, deoarece n caz contrar am avea k m + 1, deci n m
k m+ 1, ceea ce implica m
n1
2
. Dar acest rezultat contrazice ipoteza m
n
2
.
Adunand cate o unitate la ecare din termenii de mai sus si adaugand m k 0
termeni egali cu 1, obt inem o partit ie a lui n cu m part i
n = (r
1
+ 1) + + (r
k
+ 1) + 1 + + 1,
a carei imagine prin aplicat ia descrisa este partit ia lui n m de la care am plecat.
218
Problema 13.28 Sa se justice expresiile urmatoarelor funct ii generatoare:
(a) Funct ia generatoare a numerelor P(n) ale tuturor partit iior ntregului n este:

n=0
P(n)x
n
=
1
(1 x)(1 x
2
)(1 x
3
)
,
unde P(0) = 1;
(b) Funct ia generatoare a numerelor P(n, m) ale partit iior ntregului n n m part i este:

n=0
P(n, m)x
n
=
x
m
(1 x)(1 x
2
)(1 x
3
) (1 x
m
)
;
(c) Funct ia generatoare a numerelor partit iilor lui n n part i impare este
1
(1 x)(1 x
3
)(1 x
5
)
;
(d) Funct ia generatoare a numerelor partit iilor lui n n part i distincte doua cate doua
este
(1 +x)(1 +x
2
)(1 +x
3
) ;
(e) Funct ia generatoare a numerelor partit iilor lui nn part i impare, distincte doua cate
doua este
(1 +x)(1 +x
3
)(1 +x
5
) .
Solut ie. Vom da demonstrat ia doar n cazul (a), n celelalte cazuri ea facandu-se analog.
Fie dezvoltarea
1
(1 a
1
x)(1 a
2
x
2
) (1 a
k
x
k
)
=
(1 +a
1
x +a
2
1
x
2
+ )(1 +a
2
x
2
+a
2
2
x
4
+ ) . . . (1 +a
k
x
k
+a
2
k
x
2k
+ ) =
1 +a
1
x + (a
2
1
+a
2
)x
2
+ + (a

1
1
a

2
2
a

k
k
+ )x
n
+
Se observa ca termenul a

1
1
a

2
2
a

k
k
care intra n coecientul lui x
n
are proprietatea
ca
1
+ 2
2
+ +k
k
= n, deci el deneste o partit ie a lui n si anume:
n = k +k + +k
. .

k
+ + 2 + 2 + + 2
. .

2
+1 + 1 + + 1
. .

1
.
T inand cont de modul de desfacere a parantezelor, se observa ca n acest fel exponent ii
literelor care apar n coecientul lui x
n
genereaza fara repetit ie toate partit iile lui n, deci
daca facem a
1
= a
2
= = 1, coecientul lui x
n
va tocmai numarul partit iilor lui n,
adica P(n).
Din (a) deducem (c) deoarece
2
=
4
= = 0
Demonstrat ia punctelor (d) si (e) se face n mod analog.
Problema 13.29 Sa se determine toate perechile de numere ntregi pozitive (a, b) cu pro-
prietatea ca exista o descompunere a mult imii numerelor ntregi pozitive n doua mult imi
A si B, astfel ncat aA = bB.
Combinatorica si grafuri 219
IMC, 2003
Solut ie. Este evident ca a ,= b, deoarece A si B sunt disjuncte.
Fie a, b o solut ie a problemei, pentru care avem o descompunere a mult imii numerelor
ntregi data de mult imile A si B, astfel ncat aA = bB. Daca notam d = (a, b) cel mai mare
divizor comun al numerelor a si b, atunci a = da
1
, b = bb
1
, (a
1
, b
1
) = 1 si a
1
A = b
1
B.
Rezulta ca a
1
, b
1
este o solut ie, deci este sucient sa determina solut iile a, b doar n
cazul (a, b) = 1.
Daca 1 A, atunci a aA = bB, de unde obt inem ca b este un divizor al lui a.
Analog, daca 1 B, atunci a este un divizor al lui b. Deci, pentru toate solut iile, unul
dintre numerele a, b este un divizor al celuilalt.
Sa demonstram ca daca n 2, atunci perechea (1, n) este o solut ie a problemei.
Pentru ecare ntreg pozitiv k, e f(k) cel mai mare ntreg nenegativ astfel ncat n
f(k)
[ k.
Consideram mult imile
A = k : f(k) este impar, B = k : f(k) este par.
Astfel am obt inut o descompunere a mult imii numerelor ntregi nenegative ce satisface
condit ia A = nB.
Problema 13.30 Pentru un num ar ntreg n 3, se denesc mult imile,
S
n
= (x
1
, x
2
, . . . , x
n
); i, x
i
0, 1, 2
A
n
= (x
1
, x
2
, . . . , x
n
) S
n
; i n 2, [x
i
, x
i+1
, x
i+2
[ ,= 1
si
B
n
= (x
1
, x
2
, . . . , x
n
) S
n
; i n 1, (x
i
= x
i+1
x
i
,= 0) .
Sa se arate ca [A
n+1
[ = 3[B
n
[.
IMC, 2005
Solut ia 1. Extindem denit iile n pentru n = 1, 2. Consideram urmatoarele mult imi
A

n
= (x
1
, x
2
, . . . , x
n
) A
n
; x
n1
= x
n
, A

n
= A
n
A

n
B

n
= (x
1
, x
2
, . . . , x
n
) B
n
; x
n
= 0 , B

n
= B
n
B

n
si notam a
n
= [A
n
[, a

n
= [A

n
[, a

n
= [A

n
[, b
n
= [B
n
[, b

n
= [B

n
[, b

n
= [B

n
[.
Urmatoarele relat ii pentru asiruri sunt evidente:
_
_
_
a
n
= a

n
+a

n
a

n+1
= a

n
a

n+1
= 2a

n
+ 2a

n
,
ceea ce ne conduce la a
n+1
= 2a
n
+ 2a
n1
.
Pentru bsiruri sunt adevarate relat iile:
_
_
_
b
n
= b

n
+b

n
b

n+1
= b

n
b

n+1
= 2b

n
+ 2b

n
,
de unde obt inem b
n+1
= 2b
n
+ 2b
n1
.
Pentru primele valori ale sirurilor (a
n
) si (b
n
) avem
_
a
1
= 3, a
2
= 9, a
3
= 24,
b
1
= 3, b
2
= 8,
220
de unde
_
a
2
= 3b
1
a
3
= 3b
2
.

In continuare, se demonstreaza prin induct ie ca a


n+1
= 3b
n
, pentru orice n 1.
Solut ia 2. Considerand x
i
ca elemente ale lui Z
3
si lucrand modulo 3, obt inem
(x
1
, x
2
, . . . , x
n
) A
n
(x
1
+ 1, x
2
+ 1, . . . , x
n
+ 1) A
n
,
(x
1
+ 2, x
2
+ 2, . . . , x
n
+ 2) A
n
,
ceea ce nseamna ca 1/3 dintre elementele lui A
n
ncep cu 0. Stabilim astfel o biject ie
ntre submult imea vectorilor lui A
n+1
care ncep cu 0 si mult imea B
n
prin
(0, x
1
, x
2
, . . . , x
n
) A
n+1
(y
1
, y
2
, . . . , y
n
) B
n
y
1
= x
1
, y
2
= x
2
x
1
, y
3
= x
3
x
2
, . . . , y
n
= x
n
x
n1
.
(daca y
k
= y
k+1
= 0 x
k
x
k1
= x
k+1
x
k
= 0, cu x
0
= 0, de unde x
k1
= x
k
= x
k+1
,
ceea ce contrazice denit ia lui A
k1
.)
Aplicat ia inversa este denita prin
(y
1
, y
2
, . . . , y
n
) B
n
(0, x
1
, x
2
, . . . , x
n
) A
n+1
x
1
= y
1
, x
2
= y
1
+y
2
, . . . , x
n
= y
1
+y
2
+ +y
n
.
Problema 13.31 Fie V un poligon convex cu n varfuri.
(a) Sa se demonstreze ca daca n este divizibil cu 3, atunci V poate triangulat (adica
mpart it n triunghiuri disjuncte doua cate doua, cu pastrarea varfurilor) astfel ncat
ecare varf al lui V devine v arf al unui numar impar de triunghiuri.
(a) Daca n nu este divizibil cu 3, atunci V poate triangulat astfel ncat exact doua
varfuri ale sale devin varfuri ale unui numar par de triunghiuri.
IMC, 2006
Solut ie. Vom rezolva problema prin induct ie dupa n.

In cazurile n = 3, 4, 5 concluzia
este evidenta.
Presupunem ca este adevarata concluzia problemei pentru n = k si consideram cazul
n = k + 3. Notam varfurile lui V cu P
1
, . . . , P
k+3
. Conform ipotezei inductive pentru
poligonul P
1
. . . P
k
, ecare dintre varfurile acestuia apart ine unui numar impar de tri-
unghiuri, cu except ia a doua varfuri daca n nu este divizibil cu 3. Adaugam triunghiurile
P
1
P
k
P
k+2
, P
k
P
k+1
P
k+2
si P
1
P
k+2
P
k+3
. Astfel atasam doua noi triunghiuri cu varfurile n
P
1
si P
k
, deci paritatea este ment inuta.
Varfurile P
k+1
, P
k+2
, P
k+3
apart in unui numar impar de triunghiuri. Atunci numarul
varfurilor care apart in unui numar par de triunghiuri ramane acelasi ca si pentru poligonul
P
1
P
2
. . . P
k
.
Problema 13.32 Un produs trebuie sa treaca prin sase faze de prelucrare (operat ii)
pentru a ajunge nit. Fie x
i
, i = 1, 6 aceste operat ii. Condit iile tehnice impun respectarea
urmatoarelor restrict ii:
a) Operat ia x
1
trebuie efectuata naintea operat iilor x
2
si x
4
;
b) Operat ia x
2
trebuie efectuat a naintea operat iei x
3
si dupa operat ia x
5
;
c) Operat ia x
4
trebuie efectuata naintea operat iilor x
3
si x
5
;
d) Operat ia x
6
trebuie efectuat a ultima.
Sa se indice ordinea efectuarii operat iilor astfel ncat, trecand o data si numai o data
prin ecare faza de prelucrare, sa se obt ina produsul nit.
Combinatorica si grafuri 221
Solut ie. Vom considera cele sase operat ii drept varfuri ale unui graf, iar faptul ca
operat ia x
i
trebuie efectuata naintea operat iei x
j
va desemnat prin arcul (x
i
, x
j
) .

In aceste condit ii se obt ine graful


Metoda 1.
Utilizam matricea conexiunii directe.

In prima etapa eliminam varful corespunzator coloanei formate numai din zerouri, iar
din matrice linia si coloana corespunzatoare acestuia. Continuam procedeul cu matricea
astfel obt inuta pana la ultimul varf eliminat.
Ordinea fazelor de prelucrare va indicata de varfurile grafului, n ordinea eliminarii.
x
1
x
2
x
3
x
4
x
5
x
6
x
1
x
2
x
3
x
4
x
5
x
6
_
_
_
_
_
_
_
_
0 1 0 1 0 1
0 0 1 0 0 1
0 0 0 0 0 1
0 0 1 0 1 0
0 1 1 0 0 1
0 0 0 0 0 0
_
_
_
_
_
_
_
_
x
1

x
2
x
3
x
4
x
5
x
6
x
2
x
3
x
4
x
5
x
6
_
_
_
_
_
_
0 1 0 0 1
0 0 0 0 1
0 1 0 1 0
1 1 0 0 1
0 0 0 0 0
_
_
_
_
_
_
x
4

x
2
x
3
x
5
x
6
x
2
x
3
x
5
x
6
_
_
_
_
0 1 0 1
0 0 0 1
1 1 0 1
0 0 0 0
_
_
_
_
x
5

x
2
x
3
x
6
x
2
x
3
x
6
_
_
0 1 1
0 0 1
0 0 0
_
_
x
2

x
3
x
6
x
3
x
6
_
0 1
0 0
_
x
3

Asadar ordinea fazelor de prelucrare este indicata de drumul hamiltonian:


D
H
= (x
1
, x
4
, x
5
, x
2
, x
3
, x
6
) .
Metoda 2.
Utilizam matricea conexiunii totale.
T =
_
_
_
_
_
_
_
_
0 1 1 1 1 1
0 0 1 0 0 1
0 0 0 0 0 1
0 1 1 0 1 1
0 1 1 0 0 1
0 0 0 0 0 0
_
_
_
_
_
_
_
_
x
1
x
2
x
3
x
4
x
5
x
6
x
1
x
2
x
3
x
4
x
5
x
6
Observam ca t
ii
= 0, i = 1, 6 si deci graful nu are cicluri.
Numarul elementelor nenule din matricea T este C
2
6
si deci, n graf exista un drum
hamiltonian (conform teoremei lui Chen).
Asezam liniile matricei T dupa ,,puterile de atingere a varfurilor si obt inem matricea
222
T

=
_
_
_
_
_
_
_
_
0 1 1 1 1 1
0 1 1 0 1 1
0 1 1 0 0 1
0 0 1 0 0 1
0 0 0 0 0 1
0 0 0 0 0 0
_
_
_
_
_
_
_
_
x
1
x
4
x
5
x
2
x
3
x
6
Drumul hamiltonian este D
H
= (x
1
, x
4
, x
5
, x
2
, x
3
, x
6
) .
Problema 13.33

Intr-o camera se aa sase persoane. Utilizand teoria grafurilor,
demonstrat i ca exista 3 persoane care se cunosc ntre ele sau 3 persoane care nu se cunosc
ntre ele.
Solut ie. Consideram ca cele sase persoane sunt situate n nodurile unui graf conex cu
sase varfuri.
Luam ca referint a nodul A (persoana A).
F
E
D
C
A
B

Unim doua noduri printr-o muchie verde daca persoanele corespunzatoare se cunosc si
printr-o linie rosie daca ele nu se cunosc.

In camera cele 6 persoane pot n situat ia X cunoaste pe Y sau X nu cunoaste pe Y.


Din A pleaca 5 muchii sunt cel put in 3 muchii de aceeasi culoare (presupunem
verde).
Izolam subgraful A, F, E, C .
Daca F si E s-ar cunoaste (linia verde), atunci avem 3 persoane care se cunosc: A, F, E.
Asadar F si E nu se cunosc (linia rosie).
Rat ionam analog pentru muchiile CE si CF.
Obt inem astfel triunghiul CEF, n varfurile caruia se aa persoane care nu se cunosc
ntre ele.
Problema 13.34

Intr-un plan trasam un numar oarecare de drepte, astfel ncat oricare
trei dintre ele sa nu e concurente. Obt inem un graf planar G considerand punctele de
intersect ie ale dreptelor ca varfuri ale grafului si segmentele dintre punctele de intersect ie
vecine drept muchii ale acestui graf.
Sa se demonstreze ca (G) 3.
Solut ie. Notam coordonatele punctelor de intersect ie, ntr-un sistem ortogonal de axe
n plan cu (x
1
, y
1
) ; (x
2
, y
2
) ; . . . ; (x
n
, y
n
) .
Putem presupune ca direct iile axelor de coordonate sunt astfel alese ncat abscisele
x
1
, x
2
, . . . , x
n
sa e diferite doua cate doua.
Nu restrangem generalitatea presupunand ca x
1
< x
2
< < x
n
.
Vom colora varfurile grafului, n aceasta ordine, succesiv cu trei culori. Daca am colorat
cu 3 culori varfurile de abscise x
1
, x
2
, . . . , x
i1
, varful (x
i
, y
i
) are cel mult doua varfuri
adiacente lui care au fost deja colorate, deoarece nu exista trei drepte concurente.
Combinatorica si grafuri 223
Asadar pentru varful (x
i
, y
i
) gasim o a treia culoare disponibila, pentru i = 2, n, ceea
ce verica inegalitatea ceruta n enunt .
Problema 13.35 Consideram un graf conex G si notam cu d (x, y) distant a dintre
varfurile x si y, adica numarul muchiilor din cel mai scurt lant care uneste pe x cu y
si cu e (x) = max
yV
d (x, y) excentricitatea varfului x.
Centrul grafului G este format din acele varfuri ce au excentricitatea minima, minim
notat cu (G) si numit raza grafului G.
a) Sa se demonstreze ca centrul oricarui arbore este format dintr-un varf sau din doua
varfuri adiacente.
b) Daca G este arbore, sa se demonstreze ca e (x) este o funct ie convexa, n sensul ca
daca y si z sunt varfuri adiacente cu x, atunci:
e (x)
e (y) +e (z)
2
c) Demonstrat i ca pentru orice graf conex avem d (G) 2 (G), unde d (G) este di-
ametrul grafului G.
Solut ie. a) Demonstram mai ntai ca daca suprimam toate varfurile de grad 1 ale
arborelui G, atunci e (x) descreste cu o unitate pentru toate varfurile subgrafului astfel
obt inut.
Toate varfurile la distant a e (x) de x sunt de gradul 1, deci prin suprimarea acestora
e (x) descreste pentru toate varfurile ramase. Prin aceasta operat ie e (x) descreste cu exact
o unitate, deoarece cel mai lung lant care pleaca din x se termina ntr-un varf de gradul 1
al lui G, care este apoi suprimat. Proprietatea este evidenta pentru grafurile cu un singur
varf sau doua si sa presupunem ca ea este adevarata pentru tot i arborii cu cel mult n 1
varfuri.
Consideram ca G are n > 2 varfuri.
Notam cu C
0
mult imea varfurilor x care formeaza centrul lui G. Daca C
0
nu cont ine
nici un varf de gradul 1, suprimam toate varfurile de grad 1 ale arborelui G. Deoarece
pentru toate varfurile ramase x, valoarea e (x) scade cu o unitate, rezulta ca prin aceasta
operat ie se obt ine un nou arbore G

cu acelasi centru C
0
.
Deoarece G

are cel mult n2 varfuri, rezulta conform ipotezei de induct ie ca C


0
este
format dintr-un varf sau din doua varfuri adiacente si demonstrat ia este ncheiata. Daca
C
0
cont ine un varf x de gradul g (x) = 1 rezulta ca x este adiacent cu un singur varf y.
Evident ca y este strict mai apropiat decat x de oricare varf alt al lui G.
Deci e (x) poate minim numai daca e (x) = 1 si G este un arbore format din varfurile
x si y legate printr-o muchie.

In acest caz, C
0
= x, y si proprietatea este demonstrata.
Lant urile de lungime maxima ale unui arbore au intersect ia nevida si aceasta cont ine
centrul arborelui.
b) Fie L un lant de lungime e (x) care pleaca din x. Daca L nu cont ine nici unul dintre
varfurile y sau z, atunci:
e (y) = 1 +e (x) si e (z) = 1 +e (x)
Din acestea obt inem:
e (x) <
e (y) +e (z)
2
Lant ul L nu poate sa cont ina si varful y si varful z deoarece ambele varfuri sunt vecine
cu x.
224
Daca L cont ine pe y, obt inem: e (y) e (x) 1 si e (z) = e (x) + 1, de unde obt inem:
e (x)
e (y) +e (z)
2
c) Aplicat ia d : V V N, d = d (x, y) , denita n enunt pentru orice x, y V este
o metrica pe aceasta mult ime si deci ea satisface inegalitatea triunghiulara. Fie x, y doua
varfuri astfel ncat d (x, y) = d (G) si z un varf de excentricitate minima, egal cu (G) .
Avem:
d (G) = d (x, y) d (x, z) +d (z, y) (G) + (G) = 2 (G)
Problema 13.36 Daca G (V, U) este un arbore si f : V V este o aplicat ie cu pro-
prietatea ca daca [x, y] U, atunci f (x) = f (y), sau [f (x) , f (y)] U, sa se demonstreze
ca f are un punct x sau o muchie xa.
Solut ie. Demonstram proprietatea prin induct ie, dupa numarul varfurilor arborelui
G.
Daca [V [ 1, 2, proprietatea este evidenta. Presupunem ca proprietatea este
adevarata pentru orice arbore cu cel mult n 1 varfuri si sa o demonstram pentru un
arbore G cu n 3 varfuri.
Daca f este o biject ie, atunci pentru orice x ,= y avem f (x) ,= f (y) si [x, y] U implica
[f (x) , f (y)] U, deci f este un automorsm al lui G. Orice varf terminal este dus de f
tot ntr-un varf terminal. Notam cu G

subarborele obt inut din G prin suprimarea tuturor


varfurilor terminale.
Evident G

este nevid deoarece n 3. Notam cu V

mult imea varfurilor lui G

.
Rezulta ca f (V

) = V

si restrict ia lui f la V

, notata f

are aceeasi proprietate ca si


f. Asadar f

si n consecint a f are un punct x sau o muchie xa, conform ipotezei de


induct ie ([V

[ n 2) .
Daca f nu este o biject ie, atunci f (V ) este o submult ime proprie de varfuri ale lui G.
Aceste varfuri induc un subgraf conex al lui G din condit ia impusa lui f, deci f (V )
este mult imea varfurilor unui arbore si [f (V ) [ n 1.
Deoarece f (f (V )) f (V ) putem considera restrict ia lui f la subarborele general de
mult imea de varfuri f (V ) care are aceeasi proprietate ca f. Conform ipotezei de induct ie,
aceasta restrict ie, deci si f, are un punct x sau o muchie xa.
Observat ie. Proprietatea nu mai este adevarata daca G este un graf cu cicluri.
De exemplu, daca G K
3
cont ine varfurile x, y, z, denim f prin f (x) = y, f (y) = z,
f (z) = x.

In acest caz, aplicat ia f nu are nici puncte xe, nici muchii xe.
Problema 13.37 Se noteaza cu a
n
numarul arborilor cu varfurile x
1
, x
2
, . . . , x
n
.
a) Sa se demonstreze ca:
a
n
=
n1

k=1
k
_
n 2
k 1
_
a
k
a
nk
;
b) Sa se demonstreze formula lui Cayley plecand de la aceasta identitate si folosind
una din identitat ile lui Abel.
O. Dziobek, Sitzungsber. Berl. Math. G., 17 (1917), 64-67
Solut ie. a) Pentru orice arbore A cu n varfuri x
1
, x
2
, . . . , x
n
, daca suprimam o muchie
oarecare obt inem doi arbori disjunct i care cont in mpreuna toate varfurile lui A. Marcam
extremitat ile muchiei suprimate. Deoarece A are n 1 muchii, plecand de la tot i cei a
n
Combinatorica si grafuri 225
arbori cu n varfuri obt inem (n 1) a
n
perechi de astfel de arbori cu cate un varf marcat
n ecare arbore. Daca A
1
si A
2
sunt doi arbori disjunct i cu k respectiv nk varfuri, care
cont in mpreuna toate varfurile x
1
, x
2
, . . . , x
n
, putem marca un varf al lui A
1
si un varf al
lui A
2
n k (n k) moduri pentru k = 1, n 1.
Mult imea varfurilor lui A
1
si A
2
pot alese n
_
n1
k1
_
moduri cu condit ia ca un varf
xat, e acesta x
1
sa apart ina arborelui A
1
, pentru a elimina repetit iile.
Putem gasi a
k
, respectiv a
nk
arbori cu mult imea varfurilor n A
1
, respectiv A
2
, deci
numarand n doua moduri perechile de arbori disjunct i care cont in mpreuna varfurile
x
1
, x
2
, . . . , x
n
si au cate un varf marcat n ecare arbore, obt inem:
n1

k=1
_
n 1
k 1
_
a
k
a
nk
k (n k) = (n 1) a
n
. (13.1)
Cum
_
n1
k1
_
=
n1
nk
_
n2
k1
_
, din (13.1) obt inem imediat identitatea ceruta n enunt .
b)

In identitatea (13.1) schimbam ntre ei indicii k si n k, t inem cont de faptul ca
_
n1
k1
_
=
_
n1
nk
_
si aceasta devine:
n1

k=1
_
n 1
k
_
a
k
a
nk
k (n k) = (n 1) a
n
. (13.2)
Adunand membru cu membru (13.1) cu (13.2), obt inem:
n1

k=1
_
n
k
_
a
k
a
nk
k (n k) = 2 (n 1) a
n
. (13.3)
Observat ie. Aceasta identitate se poate deduce si direct daca nu mai xam varful x
1
n A
1
.
Demonstram ca a
n
= n
n2
, prin induct ie dupa n.
Daca n = 1 exista un singur arbore cu un varf si formula se verica.
Presupunem ca a
m
= m
m2
, () m = 1, n 1.
Pentru a demonstra ca a
n
= n
n2
, conform cu (13.3) este sucient sa demonstram ca:
n1

k=1
_
n
k
_
k
k1
(n k)
nk1
= 2 (n 1) n
n2
,
egalitate care este una dintre identitat ile lui Abel.
Problema 13.38 Se da graful G cu n
2
varfuri care corespund patratelor unei table de
sah cu n linii si n coloane (n N

impar).
Un cal de pe tabla de sah este mutat pe un drum n forma de L (doua varfuri se
considera adiacente daca exista o ,,saritura n L a calului de pe un patrat pe celalalt)
Poate un cal de pe tabla de sah, ,,sarind n forma de L, sa treaca o data si numai o
data prin toate cele n
2
patrate ale tablei si sa se ntoarca n punctul de plecare?
Solut ie. Deoarece calul sare de pe un patrat alb pe unul negru si de pe un patrat
negru pe unul alb, rezulta ca acest graf este bipartit.
Graful G considerat cont ine doar cicluri elementare cu un numar impar de varfuri,
deci nu exista un ciclu elementar cu n
2
varfuri.

In concluzie, raspunsul la ntrebare este nu.


226
Problema 13.39 Fie A
1
(V, U
1
) si A
2
(V, U
2
) doi arbori care au aceeasi mult ime
de varfuri V. Daca pentru orice v arf x V, subgraful obt inut din A
1
prin suprimarea
varfului x si a muchiilor incidente cu x este izomorf cu subgraful obt inut din A
2
prin
aceeasi operat ie, sa se demonstreze ca arborii A
1
si A
2
au acelasi diametru.
P.J. Kelly, Pacic J. Math. 7 (1957), 961-968
Solut ie. Notam grafurile obt inute din A
1
, respectiv A
2
prin suprimarea varfului x si
a muchiilor incidente cu x prin A
1x
, respectiv A
2x
.
Deoarece un arbore cu n varfuri are (n 1) muchii, gradul varfului x n arborele A
1
este:
[1] g
1
(x) = [V [ 1 m(A
1x
) , unde prin m(A
1x
) am notat numarul muchiilor din
graful A
1x
.
Analog obt inem:
[2] g
2
(x) = [V [ 1 m(A
2x
)
Deoarece A
1x
si A
2x
sunt izomorfe m(A
1x
) = m(A
2x
) .
Cu aceasta din [1] si [2] deducem ca g
1
(x) = g
2
(x) , pentru orice varf x, deci arborii
A
1
si A
2
au aceleasi varfuri terminale (de gradul unu).
Notam cu T mult imea varfurilor terminale pentru arborii A
1
si A
2
.
Daca [T[ = 2, rezulta ca A
1
si A
2
sunt lant uri de lungime [V [ 1, deci A
1
si A
2
au
acelasi diametru.
Presupunem acum ca [T[ 3.
Consideram L un lant elementar de lungime maxima n arborele A
1
, lungime care este
prin denit ie diametrul lui A
1
, adica d (A
1
) .
Extremitat ile acestui lant sunt doua varfuri terminale din mult imea T.
Mult imea T mai cont ine cel put in un alt varf terminal x care nu apart ine lant ului L.
Conform ipotezei A
1x
este izomorf cu A
2x
. Deoarece lant ul L este cont inut n arborele
A
1x
, rezulta ca exista un lant elementar L

, de aceeasi lungime cu lant ul L si care este


cont inut n arborele A
2x
.
Asadar si arborele A
2
cont ine lant ul elementar L

, de aceeasi lungime cu lant ul L.


Deducem ca d (A
2
) d (A
1
) .

In rat ionamentul precedent schimbam rolul arborilor A


1
si A
2
si deducem ca d (A
1
)
d (A
2
) .
Asadar arborii A
1
si A
2
au acelasi diametru.
Observat ie.

In condit iile date chiar arborii A
1
si A
2
sunt izomor.
Problema 13.40

Intr-o t ara sunt n orase, oricare doua ind unite e printr-o autostrada,
e prin cale ferata. Un turist doreste sa faca un tur complet prin aceasta t ara (sa viziteze
ecare oras o singura data si sa se ntoarca n orasul de unde a pornit). Demonstrat i ca
turistul poate alege orasul de pornire si traseul astfel ncat sa nu schimbe mijlocul de
transport ales mai mult de o data.
Solut ie. Reprezentam t ara printr-un graf complet G cu n varfuri, avand muchiile
colorate cu doua culori n funct ie de traseul ales dintre orasele corespunzatoare varfurilor.
Notam cu c
1
, respectiv cu c
2
culorile alese pentru muchiile corespunzatoare au-
tostrazilor, respectiv caii ferate.
Convenim sa numim un ciclu ,,bun daca el este hamiltonian si are sau toate muchiile
la fel colorate, sau parcurgand ciclul dintr-un oarecare oras ales, culoarea muchiilor se va
schimba o singura data, pentru ca ciclul sa corespunda cerint ei din enunt .
Folosim induct ia dupa n. Pentru n 2, 3 cerint a este ndeplinita.
Combinatorica si grafuri 227
Consideram armat ia adevarat a pentru orice k n si sa demonstram aceasta pentru
k = n + 1. Fie X un varf oarecare al grafului G.
Graful G

, format din varfurile lui G, diferite de X, cont ine un ciclu bun, notat C

(conform ipotezei de induct ie).


Daca acest ciclu este monocromatic, atunci inserand varful Xntre oricare doua varfuri
ale lui C

obt inem un ciclu bun C al lui G. Daca C

este format din muchii de doua


culori, atunci el are forma V
1
V
2
. . . V
k
V
k+1
. . . V
n
V
1
, unde muchiile V
1
V
2
, V
2
V
3
, . . . , V
k1
V
k
sunt de culoare c
1
, iar muchiile V
k
V
k+1
, V
k+1
V
k+2
, . . . , V
n
V
1
sunt de culoare c
2
.
Consideram muchia V
k
X din G.
Daca aceasta are culoarea c
1
, atunci ciclul C V
1
V
2
V
k
XV
k+1
V
n
V
1
este bun.
Daca V
k
X are culoarea c
2
, atunci ciclul C V
1
V
2
V
k1
X V
k
V
k+1

V
n
V
1
este bun.
Problema 13.41 La o petrecere au venit n barbat i si n femei. Daca un barbat doreste sa
danseze cu o femeie, cuplul acestal vom numi ,,compatibil. Compatibilitatea se considera
reciproca. Demonstrat i ca este posibil sa formam n cupluri compatibile daca si numai daca
pentru orice grup de k barbat i exista cel put in k femei compatibile cu cel put in un barbat
dintre cei k barbat i.
P. Hall, 1935
Solut ie. Transpunem enunt ul n limbajul teoriei grafurilor:
Avem un graf bipartit G cu 2n varfuri, mpart ite n doua mult imi A si B cu cate n
elemente.
Din enunt deducem ca pentru oricare k varfuri din A, exista cel put in k varfuri n B
unite cu cel put in unul din cele k varfuri din A. (Vom numi aceasta proprietate ,,condit ia
lemei mariajelor.)
Trebuie sa demonstram ca putem uni ecare varf din A cu un varf din B, astfel ncat
din ecare varf sa porneasca exact o muchie.
Procedam prin induct ie dupa n.
Pentru n 1, 2 , cerint a problemei este evidenta. Presupunem armat ia adevarata
pentru orice graf bipartit cu 2m v arfuri (m < n) si sa demonstram ca ea este adevarata
pentru m = n.
Consideram un varf oarecare X B.
Fie C A, mult imea varfurilor din A unite cu X.
Daca unind un varf Y C cu X, condit ia lemei mariajelor se pastreaza pentru graful
ramas (fara varfurile X si Y ), atunci aplicand induct ia, obt inem concluzia pentru G.
Daca unind Y cu X, condit ia lemei mariajelor nu se pastreaza pentru graful ramas,
atunci exista o submult ime C

A Y pentru care exista o submult ime D

B X
cu mai put in de [C

[ varfuri din B unite cu cel put in un varf din C

Insa conform condit iei


lemei mariajelor pentru C

, avem ca n graful init ial G exista o submult ime D B cu cel


put in [C

[ varfuri unite cu cel put in un varf din C

. Rezulta ca diferent a dintre D si D

consta n varful X, de unde [D

[ = [C

[ 1 si apoi [D[ = [C

[.
T inand cont de faptul ca C

A si D B, ultima relat ie arma faptul ca exista


cateva, sa zicem k < n varfuri din A, astfel ncat exista exact k varfuri n B care pot
unite cu exact cate un varf din cele k varfuri din A.
Deoarece condit ia lemei mariajelor este adevarata pentru G si [D[ = [C

[ deducem ca
ea este adevarata si pentru acest subgraf G

cu 2k varfuri.
Asadar putem aplica induct ia pentru aceste 2k varfuri si le putem uni n k perechi.
228
Ramane sa demonstram ca este adevarata condit ia lemei mariajelor si pentru subgraful
ramas H, format din 2n 2k varfuri. Folosim metoda reducerii la absurd.
Fie A

si B

cele doua mult imi cu cate n k varfuri ale grafului bipartit H. Atunci
condit ia lemei mariajelor nu are loc n acest subgraf.
Fie E A

o submult ime a varfurilor lui H pentru care nu are loc lema mariajelor.
Daca [E[ = p, atunci exista cel mult p 1 varfuri n B

care sunt unite cu un varf din


E.

In graful G (unde condit ia lemei mariajelor are loc) considerand mult imea de varfuri
E C

deducem ca exista cel mult k +[E[ 1 varfuri din B care pot unite cu un varf
din E C

, ceea ce contrazice condit ia lemei mariajelor.


Problema 13.42

Intr-un oras, dintr-o stat ie de metrou se poate ajunge n orice alta
stat ie (eventual prin intermediul altor stat ii).
Demonstrat i ca exista o stat ie ce poate nchisa pentru reparat ii, astfel ncat din orice
stat ie ramasa sa putem ajunge n oricare alta.
Solut ie. Consideram harta orasului ca ind un graf, ale carui varfuri sunt stat iile de
metrou.
Luam un varf oarecare X
0
si construim un sir (x
n
)
nN
n modul urmator:
Init ial x
n
= 1, pentru orice n 1, cu except ia lui x
0
, care este zero.
Apoi pentru k = 1, 2, . . . efectuam urmatorul algoritm: tuturor varfurilor X
n
pentru
care x
n
= 1 si care sunt unite cu cel put in un varf al carui x
n
este (k 1) li se asociaza
un x
n
= k. Dupa ce am asociat ecarui varf un x
n
, luam un varf X
i
astfel ncat x
i
este
maximal.
Vom demonstra ca X
i
poate sters (mpreuna cu muchiile incidente lui) astfel ncat
graful ramas este tot conex.

Intr-adevar luam doua varfuri oarecare X


p
si X
q
. Deoarece x
p
, x
q
x
i
, atunci se poate
ajunge din X
0
n X
p
si X
q
fara a trece prin X
i
.
Atunci ntre X
p
si X
q
exista drumul:
X
p
X
0
X
q
.
Problema 13.43

Intr-o mult ime de 2n+1 persoane, pentru oricare n dintre acestea mai
exista una (nu dintre cele n) care le cunoaste pe toate cele n persoane.
Demonstrat i ca n mult ime exista o persoana care le cunoaste pe toate celelalte.
Rusia, 2001
Solut ie. Corespunzator problemei consideram graful G, ale carui varfuri sunt per-
soanele mult imii.
Demonstram mai ntai ca graful G cont ine un subgraf complet cu cel put in n + 1
varfuri. Sa presupunem, prin reducere la absurd, ca aceasta armat ie nu este adevarata.
Fie k, numarul de varfuri ale subgrafului maximal si complet G

al grafului G.

In cazul k n, adaugam la aceste k persoane alese n mod arbitrar nca nk persoane.


Conform enunt ului mai exista o persoana P care le cunoaste pe toate aceste n persoane
(inclusiv pe primele k).
Dar, atunci varfurile lui G

mpreuna cu P formeaza un subgraf complet cu k + 1


varfuri, contradict ie cu alegerea lui G

.
Asadar k n + 1. Consideram acum un subgraf complet cu n + 1 varfuri. Conform
enunt ului exista o persoana care le cunoaste pe toate cele n persoane ramase. Evident ca
aceasta persoana trebuie sa apart ina subgrafului complet ales, deci ea cunoaste si restul
persoanelor din mult ime.
Combinatorica si grafuri 229
Problema 13.44 17 savant i t in legatura prin e-mail. Ei comunica despre unul dintre trei
subiecte convenite. Demonstrat i ca exista trei savant i care discuta ntre ei acelasi subiect.
OIM, 1964
Solut ie. Consideram savant ii ca varfuri ale unui graf complet G si ,,subiectele lor de
discut ie ca reprezentand trei culori c
1
, c
2
, c
3
.
Fiecare muchie a acestui graf este colorata cu una dintre aceste culori.
Trebuie sa demonstram ca exista un triunghi monocromatic n G. Consideram, n mod
arbitrar un varf X al lui G.
X are 16 vecini si conform principiului lui Dirichlet, exista cel put in sase muchii ce
pornesc din X, colorate cu aceeasi culoare (presupunem ca aceasta este culoarea c
1
).
Consideram ca aceste muchii unesc varful X cu varfurile X
1
, X
2
, . . . , X
6
.
Daca ar exista o muchie X
i
X
j
, i ,= j, i, j = 1, 6 de culoarea c
1
, atunci varfurile X, X
i
,
X
j
formeaza un triunghi monocromatic.
Daca nu exista o astfel de muchie, atunci muchiile dintre X
1
, X
2
, . . . , X
6
sunt colorate
cu doar doua culori diferite (c
2
si c
3
).
Demonstram acum ca exista un triunghi monocolor cu varfurile printre X
1
, X
2
, . . . , X
6
.
Consideram ca acestea sunt varfurile unui hexagon convex, avand laturile si diagonalele
colorate cu una dintre cele doua culori.
Graful corespunzator este complet.
Considerand varful X
1
, dintre cele cinci muchii care pornesc din el, conform principiului
lui Dirichlet, cel put in trei au aceeasi culoare.
Fie ca X
1
X
2
, X
1
X
3
, si X
1
X
4
au culoarea c
2
.
Considerand triunghiul X
2
X
3
X
4
, daca X
2
X
3
, X
2
X
4
sau X
3
X
4
au culoarea c
2
, atunci
mpreuna cu varful X
1
avem un triunghi cu laturile de culoarea c
2
.
Daca nu, rezulta ca X
2
X
3
, X
2
X
4
si X
3
X
4
sunt de culoarea c
3
si deci, triunghiul
X
2
X
3
X
4
este cel cautat.
Problema 13.45 Se considera un poligon convex cu n laturi si n varfurile sale se scriu nu-
merele x
1
, x
2
, . . . , x
n
, iar pe ecare latura se scrie produsul numerelor scrisen extremitat ile
acestei laturi. Fie S suma numerelor scrise pe toate laturile. Demonstrat i inegalitatea:

n 1
_
x
2
1
+x
2
2
+ +x
2
n
_
2S.
Rusia, 2003
Solut ie. Consideram poligonul convex drept un arbore cu n 3 varfuri.
Justicam mai ntai faptul ca orice arbore are un varf terminal (de grad 1), adica un
varf care are un singur vecin.
Un arbore cu n varfuri are n 1 muchii.
Daca din ecare varf ar porni cel put in doua muchii, atunci numarul minim de muchii
ce l-ar putea avea arborele este jumatate din 2 + 2 + + 2
. .
nori
, deoarece ecare muchie este
numarata de doua ori.
Am obt ine n 1 n (fals).
Stergand un varf terminal si muchia adiacenta acestuia, se obt ine tot un arbore.
Vom demonstra inegalitatea din enunt prin induct ie.
Pentru n = 3, inegalitatea devine:
x
2
1
+x
2
2
+x
2
3

2 (x
1
x
2
+x
1
x
3
+x
2
x
3
) ,
230
care se demonstreaza prin calcul direct.
Presupunem ca inegalitatea este adevarata pentru orice arbore cu n varfuri si sa o
demonstram pentru un arbore cu n + 1 varfuri.
Consideram un varf terminal al grafului G si e x numarul scris n acest varf.
Notam cu y x
i
, numarul scris n varful adiacent varfului terminal considerat.
Fie G

graful obt inut din G, fara acest varf si fara aceasta muchie.
Notam cu S

suma tuturor numerelor de pe muchiile lui G

. Trebuie sa demonstram
ca:

n
_
x
2
1
+x
2
2
+ +x
2
_
2S

+ 2xy
Folosind ipoteza de induct ie, este sucient sa demonstram ca:
_
n

n 1
_ _
x
2
1
+x
2
2
+. . . x
2
n
_
+

nx
2
2xy.
Cum

n

n 1 =
1

n+

n1

1
2

n
si x
2
1
+ x
2
2
+ + x
2
n
y
2
, este sucient sa
demonstram ca:
y
2
2

n
+

nx
2
2xy,
inegalitate care se deduce usor din inegalitatea mediilor.
Problema 13.46 Consideram un poligon convex cu n laturi si n care trasam toate diago-
nalele acestuia. Putem sa coloram toate varfurile, laturile si diagonalele acestuia cu cateva
culori, astfel ncat oricare doua laturi sau doua diagonale sau o latura si o diagonala cu
o extremitate comuna sa aiba culori diferite si oricare latura sau diagonala are culoarea
diferita de culoarea varfului ce le uneste.
Determinat i numarul minim posibil de culori necesare.
Italia, 2007
Solut ie. Consideram poligonul ca un graf G complet si neorientat avand n varfuri.
Numarul de culori este mai mare sau egal cu n.

Intr-adevar, din ecare varf pornesc n 1 muchii, colorate n n 1 culori diferite.



In
plus, toate aceste n1 culori diferite sunt diferite si de culoarea varfului din care pornesc.
Init ial sa numerotam varfurile de la 1 la n. Apoi pe ecare muchie dintre doua varfuri
numerotate i si j scriem restul mpart irii lui i + j la n (daca acest rest este zero, scriem
n).

In nal, din ecare varf pornesc n1 muchii de culori diferite. Atunci n varful respectiv
scriem a na culoare (eventual unele varfuri vor colorate la fel, ceea ce nu este n
contradict ie cu enunt ul problemei).
Asadar numarul minim de culori este n.
Problema 13.47

Intre cele 2000 de orase ale unei t ari nu exista drumuri. Demonstrat i
ca ntre orasele t arii pot construite drumuri bidirect ionale astfel ncat din doua orase sa
porneasca cate un drum, din alte doua orase cate doua drumuri, . . . , si din ultimile doua
orase cate 1000 de drumuri.
Sankt-Petersburg, 2001
Solut ia 1. Problema devine un caz particular daca demonstram armat ia generala:
Exista un graf cu 4n varfuri astfel ncat gradul a doua varfuri sa e 1, gradul altor
doua varfuri sa e 2, . . . , gradul ultimilor doua varfuri sa e 2n.
Demonstram prin induct ie dup a numarul de varfuri.
Combinatorica si grafuri 231
Cazul n = 1: Luam 4 varfuri A, B, C, D si unim A cu B, B cu C si C cu D. Astfel A
si D au gradul 1 iar B si C au gradul 2.
Presupunem acum ca armat ia este adevarata pentru un graf cu 4n varfuri si o vom
demonstra pentru un graf cu 4n + 4 varfuri. Fie G un graf cu 4n varfuri care satisface
proprietatea enunt ata.
Notam cu X
1
, X
2
, . . . , X
4n
aceste varfuri. Presupunem ca X
2k1
si X
2k
au gradul k,
pentru k = 1, 2n. Consideram alte patru varfuri A, B, C, D. Unim A cu B, C cu D si B
cu D. Apoi varful B l unim cu 2n varfuri X
i
, unde i este impar, iar pe D l unim cu 2n
varfuri X
i
ale lui G cu i numar par.
Fie G

graful astfel obt inut.


Demonstram ca acesta satisface proprietatea enunt ata.

Intr-adevar el are doua varfuri de gradul 1 (A si C), doua varfuri de gradul 2 (X


1
unit
cu B si X
2
, respectiv X
2
unit cu D si X
1
), . . . , doua varfuri de gradul 2n+1, acestea ind
X
4n1
si X
4n
si doua varfuri de gradul 2n + 2, acestea ind B si D care sunt unite ntre
ele, apoi cu cate 2n varfuri ale lui G si n plus B este unit cu A, iar D este unit cu C.
Solut ia 2.

Impart im cele 2000 orase n 1000 perechi. Unim orasele din aceeasi pereche.
Numerotam perechile de la 1 la 1000.
Al doilea oras din ecare pereche i = 1, 1000 l unim cu primul oras din ecare pereche
j, cu i < j 1000.
Dupa efectuarea acestui procedeu constatam ca din primul oras din perechea k si din
al doilea oras din perechea 1001 k vor porni exact k drumuri.
Problema 13.48 Fiecare dintre cei 2012 deputat i ai parlamentului unei t ari i-a dat o
palma unui alt deputat. Demonstrat i ca se poate forma o comisie parlamentara din 671
deputat i care nu au primit nici o palma.
Moscova, 1994
Solut ie. Consideram deputat ii ca varfurile unui graf orientat G. Un arc ntre varfurile
A si B nseamna ca deputatul A i-a tras o palma deputatului B.
Demonstram cazul general:

In orice graf orientat G cu cel put in 3n 1 varfuri exista
un subgraf cu n varfuri, care nu cont ine niciun arc incident interior cu macar unul dintre
cele n varfuri.
Demonstram prin induct ie dupa n.
Cazul n = 2 este evident.
Presupunem armat ia adevarata pentru un oarecare n si o demonstram pentru n +1.
Consideram ca avem cel put in 3n + 2 varfuri.
Cazul 1. Daca exista un varf X n care nu ajunge nici un arc (n problema aceasta
nseamna ca deputatul X nu a primit nici o palma), atunci consideram graful G

fara
232
acest varf X si varful X

adiacent lui X (si fara muchiile care pornesc din X

). G

are cel
put in 3n > 3n 1 varfuri deci cont ine n varfuri neunite oricare doua printr-un arc.
Adaugand varful X obt inem n + 1 varfuri neunite oricare doua.
Cazul 2. Consideram ca n orice varf ajunge cel put in un arc.
Deoarece din ecare varf porneste exact un arc, rezulta ca n ecare varf ajunge exact
un arc.
Luam oricare varf X, varful A spre care porneste un arc din X si varful B spre care
porneste un arc din X.
Conform induct iei, din varfurile ramase putem selecta n varfuri neunite oricare doua.
Atunci la aceste varfuri l adaugam pe X.
Problema este n cazul particular n = 671.
Capitolul 14
Aritmetica si teoria numerelor
Denit ii si rezultate
Denit ie. Fie a, b, n Z cu n ,= 0. Spunem ca a este congruent cu b modulo n, si
notam a b (mod n), daca a b
.
.
. n.
Observat ie. Pentru a, b Z si n N

, a b (mod n) daca si numai daca a =

b n Z
n
.
Mica teorema a lui Fermat. Daca p este un numar prim, iar a Z cu (a, p) = 1,
atunci a
p1
1 (mod p).
Lema chineza a resturilor. Fie n
1
, n
2
, . . . , n
k
Z

cu proprietatea ca oricare doua


sunt relativ prime si a
1
, a
2
, . . . , a
k
Z. Sistemul de congruent e
_

_
x a
1
(mod n
1
)
x a
2
(mod n
2
)
............................
x a
k
(mod n
k
)
admite solut ii, si, data ind o solut ie x
0
, mult imea solut iilor sistemului este
x
0
+n
1
n
2
n
k
[ Z.
Denit ie. Fie a, n Z cu n ,= 0. Spunem ca a este rest patratic modulo n daca exista
x Z cu proprietatea x
2
a (mod n).
Denit ie. Pentru p > 2 num ar prim si a Z cu (a, p) = 1, denim simbolul lui
Legendre
_
a
p
_
astfel:
_
a
p
_
=
_
1, daca a este rest patratic modulo p
1, daca a nu este rest patratic modulo p.
233
234
Proprietat i ale simbolului lui Legendre. Fie a, b Z si p N prim impar.
1)
_
a
p
_
a
p1
2
(mod p). (Euler)
2) Daca a b (mod p), atunci
_
a
p
_
=
_
b
p
_
.
3)
_
ab
p
_
=
_
a
p
__
b
p
_
.
4)
_
1
p
_
= (1)
p1
2
.
5)
_
2
p
_
= (1)
p
2
1
8
.
Legea de reciprocitate patratica (Gauss). Daca p, q N sunt numere prime,
impare si distincte, atunci
_
p
q
_

_
q
p
_
= (1)
p1
2

q1
2
.
Observat ie. Proprietat ile 2) si 3) arata ca putem interpreta simbolul lui Legendre ca
ind un morsm de grupuri de la Z
p
0 la C

.
Observat ie. Daca p este numar prim impar, a Z si p[ a, vom folosi uneori, pentru
o mai buna sistematizare a calculelor, convent ia
_
a
p
_
= 0.
Denit ie. Funct ia lui Legendre asociaza ecarei perechi alcatuite dintr-un numar
prim p si un numar natural n exponentul e
p
(n) la care apare p n descompunerea n
factori primi a lui n!.
Teorema. e
p
(n) =
[log
p
n]

k=1
_
n
p
k
_
=

k1
_
n
p
k
_
.
Probleme
Problema 14.1 a) Aratat i ca pentru n N sucient de mare putem partit iona un patrat
dat n n patrate.
b) Fie d 2. Aratat i ca exista o constanta N(d) cu proprietatea ca pentru orice numar
natural n N(d) putem partit iona un cub d-dimensional dat n n cuburi d-dimensionale.
IMC, 2000
Solut ie.

Incepem cu urmatoarea
Lema. Daca numerele naturale a si b sunt prime ntre ele, atunci orice numar natural
n > ab a b se poate scrie sub forma n = ax +by, cu x, y N.
Demonstrat ie. Sa presupunem a b. Daca b = 1, armat ia lemei este evidenta.
Presupunem n continuare ca b > 1. Fie n ab a. Atunci numerele n, n a, n
2a, . . . , n (b 1)a parcurg un sistem complet si independent de resturi modulo b, deci
exista x 0, 1, . . . , b 1 si y N astfel ncat n = ax + by. Daca n = ab a k,
k 1, 2, . . . , b1, atunci alegem z 1, 2, . . . , b1 pentru care z =

ka
1
n Z
b
si avem
azk
.
.
. b, deci exista y N pentru care azk = by. De aici rezulta ca n = a(b1z)+by.
Revenind la solut ia propriu-zis a, sa observam ca orice partit ie a unui ,,d-cub (d 2)
n n d-cuburi poate ranata la o partit ie a sa n n + (a
d
1) d-cuburi pentru orice
a N

. Acest lucru se poate realiza pur si simplu prin alegerea unui d-cub din partit ie
Aritmetica si teoria numerelor 235
si partit ionarea acestuia n a
d
d-cuburi. Conform lemei, pentru a demonstra armat iile
problemei este sucient sa gasim o pereche de numere naturale relativ prime de forma
a
d
1; 2
d
1 si (2
d
1)
d
1 reprezinta o astfel de pereche.
Problema 14.2 Fie n N

. Aratat i ca 2
n1
divide numarul

0k<
n
2
_
n
2k + 1
_
5
k
.
IMC, 2008
Solut ie. Se stie ca numarul Fibonacci F
n
este dat de formula
F
n
=
1

5
__
1 +

5
2
_
n

_
1

5
2
_
n
_
.
Folosind formula binomului lui Newton, obt inem
F
n
=
1
2
n1
__
n
1
_
+
_
n
3
_
5 + +
_
n
t
_
5
t1
2
_
, unde t = 2
_
n 1
2
_
+ 1.
Obt inem 2
n1
F
n
=

0k<
n
2
_
n
2k + 1
_
5
k
si problema este rezolvata.
Problema 14.3 Consideram numerele D = d
1
d
2
. . . d
9
, E = e
1
e
2
. . . e
9
si F = f
1
f
2
. . . f
9
scrise n baza 10. Pentru orice i 0, 1, . . . 9, daca nlocuim cifra d
i
a lui D cu e
i
obt inem
un numar divizibil cu 7. De asemenea, pentru orice i 0, 1, . . . 9, daca nlocuim cifra
e
i
a lui E cu f
i
obt inem un numar divizibil cu 7. Aratat i ca pentru orice i 0, 1, . . . 9
avem d
i
f
i
.
.
. 7.
Putnam, 1995
Solut ie. Conform enunt ului, pentru orice i 1, 2, . . . , 9 avem
(e
i
d
i
) 10
9i
+D 0 (mod 7) si (14.1)
(f
i
e
i
) 10
9i
+E 0 (mod 7). (14.2)
Sumand relat iile din (14.1) pentru i 1, 2, . . . , 9, obt inem E D + 9D 0 (mod 7),
adica
E +D 0 (mod 7). (14.3)
Adunand pentru ecare i 1, 2, . . . , 9 relat iile (14.1) si (14.2), obt inem (f
i
d
i
) 10
9i
+
E + D 0 (mod 7). De aici si din (14.3) obt inem, t inand cont de faptul ca (10, 7) = 1,
ca d
i
f
i
.
.
. 7.
Problema 14.4 Aratat i ca nu exista patru puncte n spat iul euclidian astfel ncat
distant a dintre oricare doua sa e numar impar.
Putnam, 1993
236
Solut ia 1. Presupunem ca exista patru astfel de puncte. Pentru x, y R si n N

,
vom folosi notat ia ,,x y (mod n) cu sensul ,,xy este un numar ntreg divizibil cu n.
Alegem un sistem de coordonate n care coordonatele celor patru puncte sunt (0, 0), (a, 0),
(r, s) si (x, y), cu a 2N+1. Patratele numerelor impare ind congruente cu 1 modulo 8,
presupunerea facuta implica relat iile:
r
2
+s
2
1 (mod 8)
(r a)
2
+s
2
1 (mod 8)
x
2
+y
2
1 (mod 8)
(x a)
2
+y
2
1 (mod 8)
(x r)
2
+ (y s)
2
1 (mod 8).
(14.4)
Scazand primele doua dintre aceste relat ii se obt ine 2ar a
2
(mod 8). Prin urmare, r
este un numar rat ional al carui numitor este par si divide 2a. Se arata n mod analog ca x
are aceeasi proprietate.

Inmult im toate coordonatele cu a, reducandu-ne astfel la cazul n
care r si x au numitorul 2. Atunci, congruent a 2ar a
2
(mod 8) este de numere ntregi;
din ea va rezulta r
a
2
(mod 4). Scriind r =
a
2
+ 4b, b Z, obt inem
r
2
=
a
2
4
+ 4ab + 16b
2

a
2
4
(mod 4);
de aici, folosind si prima relat ie din (14.4), deducem ca
s
2
1 r
2
1
a
2
4
(mod 4).

In mod analog se arata ca x


a
2
(mod 4) si ca y
2
1
a
2
4
(mod 4); din penultima relat ie
si din r
a
2
(mod 4) obt inem x r
a
2

a
2
0 (mod 4), deci (x r)
2
16Z. De aici si
din ultima relat ie din (14.4) deducem ca (y s)
2
1 (x r)
2
1 (mod 8). Obt inem
(y +s)
2
2y
2
+ 2s
2
(y s)
2
2(1
a
2
4
) + 2(1
a
2
4
) 1
3 a
2
2 (mod 4).

Inmult ind membru cu membru aceasta congruent a de numere ntregi cu (y s)


2
1
(mod 8), obt inem (y
2
s
2
)
2
2 (mod 4). Daca y
2
s
2
=
u
v
, u, v Z, v ,= 0, rezulta ca
exista k Z astfel ncat u
2
2v
2
= 4kv
2
. De aici, u = 2u
1
, u
1
Z; obt inem 2u
2
1
v
2
= 2kv
2
,
deci v = 2v
1
, v
1
Z, apoi u
2
1
2v
2
1
= 4kv
2
1
. Continuand inductiv, obt inem pentru ecare
j N numerele u
j
, v
j
Z cu proprietatea u
2
j
2v
2
j
= 4kv
2
j
. Din denit ia acestor numere,
avem v
j
= 2v
j+1
pentru tot i j N. De aici, 2
j
[ v pentru orice j N, deci v = 0,
contradict ie.
Solut ia 2. Sa presupunem ca exista patru puncte O, A, B, C n plan astfel ncat
numerele a = OA, b = OB, c = OC, x = BC, y = CA, z = AB sa e ntregi si
impare. Notam cu , respectiv masurile unghiurilor orientate

AOB si

BOC; masura
unghiului orientat

AOC este deci +. Cum cos(+) = cos cos sin sin , rezulta
(cos( +) cos cos )
2
= (1 cos
2
)(1 cos
2
), de unde
1 cos
2
cos
2
cos
2
( +) + 2 cos cos cos( +) = 0. (14.5)
Conform teoremei cosinusului, avem cos =
a
2
+b
2
z
2
2ab
, cos =
b
2
+c
2
x
2
2bc
si cos(+
) =
c
2
+a
2
y
2
2ca
.

Inlocuind n relat ia (14.5) si nmult ind cu 4a
2
b
2
c
2
, obt inem
4a
2
b
2
c
2
c
2
(a
2
+b
2
z
2
)
2
a
2
(b
2
+c
2
x
2
)
2
b
2
(c
2
+a
2
y
2
)
2
+
Aritmetica si teoria numerelor 237
+(a
2
+ b
2
z
2
)(b
2
+ c
2
x
2
)(c
2
+ a
2
y
2
) = 0. De aici rezulta 4 1 1 1 + 1 0
(mod 4), contradict ie.
Solut ia 3. Presupunem ca exista patru puncte cu proprietatea din enunt ; consideram
un sistem de coordonate cu originea n unul dintre ele, si notam cu

v
1
,

v
2
si

v
3
vectorii
care unesc originea cu celelalte trei puncte; volumul V al paralelipipedului determinat de
acesti vectori este nul, deoarece ei sunt coplanari. Deci,
0 = V
2
=


v
1


v
1

v
1


v
2

v
1


v
3

v
2


v
1

v
2


v
2

v
2


v
3

v
3


v
1

v
3


v
2

v
3


v
3

.
Daca punem a = [

v
1
[, b = [

v
2
[, c = [

v
3
[, x = [

v
2


v
3
[, y = [

v
3


v
1
[ si z = [

v
1


v
2
[ si
avem n vedere relat ia
2

v
i


v
j
= [

v
i
[
2
+[

v
j
[
2
[

v
i


v
j
[
2
, (14.6)
obt inem
8V
2
=

2a
2
a
2
+b
2
z
2
a
2
+c
2
y
2
a
2
+b
2
z
2
2b
2
b
2
+c
2
x
2
a
2
+c
2
y
2
b
2
+c
2
x
2
2c
2

,
de unde
0 = 8V
2

2 1 1
1 2 1
1 1 2

4 (mod 8),
contradict ie.
Solut ia 4. Presupunem ca exista patru puncte ca n enunt si denim

v
1
,

v
2
si

v
3
ca
n solut ia 3. Este clar ca

v
i


v
i
1 (mod 8); conform (14.6), avem si 2

v
i


v
j
1 (mod 8)
pentru i ,= j.
Cum printre punctele considerate nu pot exista trei coliniare, rezulta ca vectorii

v
1
,

v
2
si

v
3
sunt doi cate doi necoliniari. Exista prin urmare scalari r, s R cu proprietatea ca

v
3
= r

v
1
+s

v
2
. Atunci,
2

v
1


v
3
= 2r

v
1


v
1
+ 2s

v
1


v
2
2

v
2


v
3
= 2r

v
2


v
1
+ 2s

v
2


v
2
2

v
3


v
3
= 2r

v
3


v
1
+ 2s

v
3


v
2
.
(14.7)
Cum

v
1
si

v
2
sunt necoliniari, avem


v
1


v
1

v
1


v
2

v
2


v
1

v
2


v
2

,= 0.
Prin urmare, primele doua ecuat ii din (14.7), privite ca ecuat ii n r si s, admit solut ie unica
n numere rat ionale. Fie r =
R
T
, s =
S
T
aceasta solut ie, cu R, S, T Z si (R, S, T) = 1.

Inmult ind cu T ecuat iile din (14.7), obt inem T 2R + S (mod 8), T R + 2S (mod 8)
si 2T R + S (mod 8). Adunand primele doua dintre aceste congruent e si scazand-o
pe a treia, gasim ca T este par. Utilizand aceasta informat ie si primele doua congruent e,
deducem ca R si S sunt si ele pare, contradict ie.
Problema 14.5 Fie x, y, z Z cu proprietatea ca S = x
4
+ y
4
+ z
4
se divide prin 29.
Aratat i ca S se divide prin 29
4
.
IMC, 2007
238
Solut ie. Vom arata ca 29 divide x, y si z, de unde rezulta imediat concluzia problemei.
Presupunem ca 29 nu divide x, y si z; pentru a xa ideile, sa consideram ca 29 x. Atunci,
x este inversabil n corpul Z
29
; e w inversul sau. Atunci, numarul (xw)
4
+(yw)
4
+(zw)
4
este la randul sau divizibil cu 29, deci
(yw)
4
1 (zw)
4
(mod 29). (14.8)
Pe de alta parte, exista doar opt puteri 4 modulo 29:
0 0
4
(mod 29),
1 1
4
12
4
17
4
28
4
(mod 29),
7 8
4
9
4
20
4
21
4
(mod 29),
16 2
4
5
4
24
4
27
4
(mod 29),
20 6
4
14
4
15
4
23
4
(mod 29),
23 3
4
7
4
22
4
26
4
(mod 29),
24 4
4
10
4
19
4
25
4
(mod 29),
25 11
4
13
4
16
4
18
4
(mod 29).
De aici rezulta ca ( yw)
4

0,

1,

7,

16,

20,

23,

24,

25, n timp ce

1 ( zw)
4

28,

27,

21,

12,

8,

5,

4,

3 (toate clasele ind modulo 29), ceea ce contrazice


(14.8).
Ramane asadar ca 29 divide x, y si z, ceea ce ncheie demonstrat ia.
Problema 14.6 Fie p > 3 un numar prim si n =
2
2p
1
3
. Aratat i ca n divide 2
n
2.
Vojtech Jarnik, 2002
Solut ie. Cum n = 4
p1
+ 4
p2
+ + 1, putem scrie n = 10101 . . . 101 (p cifre 1) n
binar. Obt inem prin urmare reprezentarea binara
3n = 1111 . . . 111
. .
2p cifre
. (14.9)
Pe de alta parte,
2
n
2 = 1111 . . . 111
. .
n1 cifre
0. (14.10)
Conform micii teoreme a lui Fermat, 2
p1
1 (mod p), deci p[2
2p2
1. Rezulta ca p
divide
2
2p
4
3
=
2
2p
1
3
1, adica p[n 1; cum n este impar, avem si 2p[n 1. Aceasta
relat ie, mpreuna cu (14.9) si (14.10), arata ca 3n[2
n
2.
Problema 14.7 Aratat i ca exista o innitate de perechi (m, n) de numere naturale prime
ntre ele pentru care ecuat ia n x
(x +m)
3
= nx
are trei radacini ntregi distincte.
IMC, 2006
Aritmetica si teoria numerelor 239
Solut ie. Notand y = x +m, ecuat ia devine
y
3
ny +mn = 0.
Notam doua dintre radacinile ei cu u si v; conform relat iilor ntre radacini si coecient i,
cea de-a treia radacina va w = (u+v), si sunt vericate si relat iile uv +uw+vw = n
(de unde u
2
+ uv + v
2
= n) si uv(u + v) = uvw = mn. Prin urmare, uv(u + v) este
divizibil prin u
2
+uv +v
2
. Punand u = kp, v = kq, avem u
2
+uv +v
2
= k
2
(p
2
+pq +q
2
) si
uv(u +v) = k
3
pq(p +q). Alegand p, q prime ntre ele si punand k = p
2
+pq +q
2
, condit ia
u
2
+uv +v
2
[uv(u +v) este ndeplinita. Cu aceste constatari, revenim la notat iile init iale
si punem pentru ecare pereche p, q de numere naturale prime ntre ele m = pq(p + q) si
n = (p
2
+ pq + q
2
)
3
. Obt inem astfel o innitate de ecuat ii (x + m)
3
= nx, ecare avand
trei radacini ntregi distincte: p
3
, q
3
si (p +q)
3
.
Problema 14.8 Demonstrat i ca ecuat ia
a
n+1
(a + 1)
n
= 2001 (14.11)
are solut ie unica n numere naturale nenule.
Putnam, 2001
Solut ie. Fie a, n N

astfel ncat a
n+1
(a +1)
n
= 2001. Cum a divide a
n+1
[(a +
1)
n
1], rezulta ca a[ 2002 = 2 7 11 13.
Cum 2001
.
.
. 3, (14.11) implica a 1 (mod 3). De aici, a
n+1
1 (mod 3), deci (a+1)
n
1
(mod 3). Prin urmare, n este par.
Daca a ar par, atunci a
n+1
(a + 1)
n
(a + 1)
n
(mod 4). Cum n este par, avem si
(a + 1)
n
1 (mod 4). Folosind (14.11), obt inem
1 2001 a
n+1
(a + 1)
n
(a + 1)
n
1 (mod 4),
contradict ie. Ramane ca a este impar, deci a[ 1001 = 7 11 13. Cum a 1 (mod 3),
rezulta ca a[ 7 13. Pe de alta parte, n N

ind par, avem n 2, de unde a a


n+1

(a + 1)
n
= 2001 1 (mod 4). Prin urmare, a 1, 13. Cum pentru niciun n N nu are
loc 1 2
n
= 2001, rezulta a = 13. Este imediat ca a = 13, n = 2 verica (14.11). Pe de
alta parte, daca n > 2 este par, se obt ine
13
n+1
(13 + 1)
n
13
n+1
13 , 1 2001 (mod 8),
deci singura solut ie a ecuat iei (14.11) este a = 13, n = 2.
Problema 14.9 Pentru n N

notam cu d(n) numarul divizorilor sai naturali. Aratat i


ca sirul (d(n
2
+ 1))
nn
0
nu este strict monoton pentru niciun n
0
N.
Vojtech Jarnik, 2003
Solut ie. Sa observam pentru nceput ca daca n este par are loc inegalitatea
d(n
2
+ 1) < n. (14.12)

Intr-adevar, n aceasta situat ie putem grupa divizorii lui n


2
+1 n perechi
_
d,
n
2
+1
d
_
, unde
d < n este impar, ceea ce conduce (ntrucat n
2
+ 1 nu este patrat perfect) la inegalitatea
dorita.
Sa presupunem acum ca exista n
0
N astfel ncat (d(n
2
+ 1))
nn
0
este strict monoton.
Cum d(n
2
+1) este par, obt inem d((n+1)
2
+1) d(n
2
+1)+2 si, inductiv, d((n+k)
2
+1)
d(n
2
+1) +2k pentru orice n n
0
si k N

. Obt inem d(4n


2
0
+1) d(n
2
0
+1) +2n
0
2n
0
,
ceea ce contrazice (14.12).
240
Problema 14.10 Aratat i ca orice numar natural nenul se poate scrie ca suma de unul
sau mai multe numere de forma 2
r
3
s
, cu r, s N, astfel ncat niciunul dintre termenii unei
astfel de sume sa nu e divizor al altuia.
Putnam, 2005
Solut ie. Demonstram armat ia problemei prin induct ie dupa n.
Numarul 1 se reprezinta ca 2
0
3
0
.
Fie n > 1. Presupunem ca toate numerele naturale nenule mai mici sau egale cu n 1
admit reprezentari de tipul din enunt . Daca n este par, atunci obt inem o reprezentare a
lui n nmult ind cu 2 o reprezentare a lui
n
2
. Daca n este impar, notam m = [log
3
n]. Daca
n = 3
m
, am terminat. Altfel, consideram o reprezentare s
1
+s
2
+ +s
k
de tipul din enunt a
numarului
n3
m
2
. Atunci, n = 3
m
+2s
1
+2s
2
+ +2s
k
. Este clar ca niciunul dintre termenii
2s
i
nu divide alt termen de acest tip sau pe 3
m
.

In plus, cum 2s
i
n 3
m
< 3
m+1
3
m
,
obt inem s
i
< 3
m
, deci 3
m
2s
i
. Prin urmare, n admite reprezentari de tipul cerut, ceea ce
ncheie pasul de induct ie si demonstrat ia.
Problema 14.11 Pentru n N

notam cu (n) suma divizorilor naturali ai lui n. Vom


spune ca numarul n este ,,straniu daca (n) 2n si nu exista nicio reprezentare de tipul
n = d
1
+d
2
+ +d
r
, unde r > 1, iar d
1
, d
2
, . . . , d
r
sunt divizori distinct i ai lui n.
Aratat i ca exista o innitate de numere stranii.
Vojtech Jarnik, 2010
Solut ie. Fie n un numar straniu si p > (n) un numar prim. Presupunem ca pn nu
este straniu. Daca 1 = d
1
, d
2
, . . . , d
k
= n sunt divizorii naturali ai lui n, atunci cei ai lui pn
sunt d
1
, d
2
, . . . , d
k
, pd
1
, pd
2
, . . . , pd
k
. Acestia din urma sunt si distinct i, ntrucat (p, n) = 1.
Daca am avea pn = d
i
1
+ +d
ir
+p(d
j
1
+ +d
js
), i
k
, j
l
1, . . . , k, ar rezulta
d
i
1
+ +d
ir
= p(n d
j
1
d
js
). (14.13)
Dar n , d
j
1
, . . . , d
js
, deoarece reprezentarile numerelor stranii nu pot consta ntr-un
singur sumand.

In plus, n ind straniu, avem si nd
j
1
d
js
,= 0. Rezulta ca membrul
drept din (14.13) este multiplu nenul de p; la fel este prin urmare si d
i
1
+ +d
ir
. Acest
lucru este nsa n contradict ie cu p > (n). Ramane deci ca pn este straniu. Prin urmare,
daca am avea un numar straniu n, punand n
1
= n si presupunand construit n
k
, am lua
p > (n
k
) si am obt ine ca n
k+1
= pn
k
este straniu. De aici ar rezulta existent a unui sir
strict crescator de numere stranii, ceea ce ar rezolva problema.
Mai ramane de gasit un exemplu de numar straniu. Sa observam ca, daca consideram un
numar n cu (n) = 2n + 4 si care nu este divizibil nici prin 3, nici prin 4, atunci nu vom
putea sa-l reprezentam pe 4 (deci, nici pe n) ca suma de divizori distinct i ai lui n. n = 2pq
are proprietat ile de mai sus (unde p si q sunt numere prime impare distincte) daca si
numai daca 3(p +1)(q +1) = (2pq) = 4pq +4; ne este deci sucient ca (p 3)(q 3) = 8.
Aceasta egalitate este vericata de p = 5 si q = 7, valori care conduc la numarul straniu
70.
Problema 14.12 Determinat i [S[, unde
S = x N

[ x < 10
2006
si x
2
x
.
.
. 10
2006

IMC, 2006
Aritmetica si teoria numerelor 241
Solut ia 1. Pentru k N

notam S
k
= x N

[ x < 10
k
si x
2
x
.
.
. 10
k
si s(k) = [S
k
[.
Fie x S
k+1
si a
k
a
k1
. . . a
0
scrierea sa zecimala. Este imediat ca a
k1
a
k2
. . . a
0
S
k
.
Fixam acum y = a
k1
a
k2
. . . a
0
S
k
, si consideram x = a
k
a
k1
. . . a
0
, a
k
0, 1, . . . , 9.
Avem x
2
x = (a
k
10
k
+y)
2
(a
k
10
k
+y) = (y
2
y) +a
k
10
k
(2y 1) +a
2
k+1
10
2k
.
Cum exista z Z astfel ncat y
2
y = 10
k
z, rezulta ca x
2
x este divizibil prin 10
k+1
daca si numai daca
z +a
k
(2y 1) 0 (mod 10). (14.14)
Cum y , 3 sau 8 (mod 10), congruent a (14.14) are solut ie unica, ceea ce arata ca ecare
y S
k
provine din exact un element x S
k+1
prin nlaturarea cifrei (eventual, nule)
init iale. Prin urmare, pentru orice k N

exista o corespondent a bijectiva ntre S


k
si
S
k+1
. Rezulta ca s(2006) = s(1). Cum nsa S
1
= 1, 5, 6, avem [S[ = s(2006) = s(1) = 3.
Solut ia 2. Fie x S. Cum x
2
x = x(x 1), iar numerele x si x 1 sunt prime
ntre ele, unul dintre ele trebuie sa e divizibil cu 2
2006
si unul dintre ele (eventual, acelasi)
trebuie sa e divizibil cu 5
2006
. Prin urmare, x trebuie sa satisfaca condit iile x 0 sau
1 (mod 2
2006
) si x 0 sau 1 (mod 5
2006
). Conform lemei chineze a resturilor, ecare din
cele 4 cazuri conduce la o unica solut ie din mult imea 0, 1, . . . , 10
2006
1. Aceste solut ii
sunt distincte doua cate doua, deoarece dau resturi diferite modulo 2
2006
sau 5
2006
. Una
dintre solut ii este 0, care nu se ncadreaza n condit iile din enunt . Prin urmare, [S[ = 3.
Problema 14.13 Pentru a N, notam n
a
= 101a100 2
a
. Aratat i ca pentru a, b, c, d
0, 1, . . . , 99, n
a
+n
b
n
c
+n
d
(mod 10100) implica a, b = c, d.
Putnam, 1994
Solut ie. Cum 2
20
95 (mod 101), iar 2
50
100 (mod 101), rezulta ca ordinul lui 2
n grupul Z
101
0 este 100 (deci, 2 este radacina primitiva modulo 101).
Lema. Daca a, b N sunt astfel ncat 2
a
2
b
(mod 101), atunci a b (mod 100).
Demonstrat ie. Sa presupunem, pentru a xa ideile, ca a b. Daca 101 [ 2
a
2
b
= 2
b
(2
ab

1), atunci 2
ab
1 (mod 101), deci, conform considerat iilor anterioare lemei, a b 0
(mod 100).
Revenind la solut ia propriu-zis a, observam ca, n conformitate cu lema chineza a res-
turilor, relat ia n
a
+n
b
n
c
+n
d
(mod 10100) este echivalenta cu sistemul
_
a +b c +d (mod 100)
2
a
+ 2
b
2
c
+ 2
d
(mod 101).
(14.15)
Cum ordinul lui 2 n Z
101
este 100, din prima relat ie din (14.15) obt inem 2
a+b
2
c+d
(mod 101), deci
2
a
2
b
2
c
2
d
(mod 101). (14.16)
Din (14.16) si a doua relat ie din (14.15) obt inem 2
a
(2
c
+ 2
d
2
a
) 2
c
2
d
(mod 101), de
unde (2
a
2
c
)(2
a
2
d
) 0 (mod 101). Rezulta 2
a
2
c
(mod 101) sau 2
a
2
d
(mod 101).
Aplicand lema, obt inem ca a este congruent cu c sau cu d modulo 100; din (14.15) deducem
ca b este congruent modulo 100 cu cealalta valoare din c, d. Cum a, b, c, d 0, 1, . . . , 99,
congruent ele obt inute sunt de fapt egalitat i.
Problema 14.14 Fie a, b Z si n N

cu proprietatea ca mult imea Zax


n
+by
n
[ x, y
Z este nita. Aratat i ca n = 1.
IMC, 2010
242
Solut ie. Presupunem ca n > 1. Observam ca a si b trebuie sa e relativ prime, deoarece
n caz contrar numerele care nu sunt divizibile prin (a, b) nu pot reprezentate sub forma
ax
n
+by
n
. Remarcam si faptul ca putem nlocui n cu orice divizor prim p al sau.
Daca p = 2, atunci expresia ax
2
+by
2
nu poate avea orice rest modulo 8 (daca b este par,
atunci ax
2
are cel mult trei resturi posibile(mod 8), iar by
2
cel mult doua. Deci, ax
2
+by
2
poate avea cel mult sase resturi modulo 8. Daca a e par, procedam analog, iar daca ab
este impar, atunci ax
2
+ by
2
(x
2
y
2
) (mod 4); (x
2
+ y
2
) nu poate da restul 3
modulo 4, iar (x
2
y
2
) nu poate da restul 2 modulo 4). Acest caz duce prin urmare la
contradict ie.
Daca p 3, cum 0
p
, 1
p
, . . . , (p 1)
p
dau resturi diferite modulo p, iar x
p
(x + kp)
p
(mod p
2
), rezulta ca puterile p de numere ntregi dau exact p resturi modulo p
2
. Prin
urmare, numerele de forma ax
p
+ by
p
pot da cel mult p
2
resturi modulo p
2
. Daca ele nu
dau restul k modulo p
2
, atunci niciun numar din k + p
2
Z nu e reprezentabil sub forma
din enunt , contradict ie. Daca ax
p
+by
p
da toate resturile modulo p
2
, atunci p
2
[ ax
p
+by
p
daca si numai daca p[x si p[y. Rezulta ca p
2
[ ax
p
+ by
p
daca si numai daca p
p
[ ax
p
+ by
p
.
De aici deducem ca niciun numar din p
2
+p
3
Z nu este reprezentabil sub forma ax
p
+by
p
,
contradict ie.
Problema 14.15 Aratat i ca, date ind numerele a, b, c Z, exista n Z pentru care

n
3
+an
2
+bn +c , Z.
Putnam, 1998
Solut ia 1. Presupunem ca P(n) = n
3
+an
2
+bn+c este patrat perfect pentru pentru
n 1, 2, 3, 4. Cum P(2) si P(4) sunt patrate perfecte de aceeasi paritate, diferent a lor,
adica 56 + 12a + 2b, trebuie sa e multiplu de 4. Prin urmare, b trebuie sa e par. Pe
de alta parte, si P(1) si P(3) sunt patrate perfecte de aceeasi paritate, deci diferent a lor,
adica 26 + 8a + 2b, trebuie sa e si ea multiplu de 4. De aici rezulta ca b trebuie sa e
impar, contradict ie.
Solut ia 2. Daca 4b a
2
= 0 si c = 0, atunci n
3
+ an
2
+ bn + c = n(n +
a
2
)
2
, care nu
este patrat perfect pentru niciun n ,=
a
2
care nu este patrat perfect.
Daca 4b a
2
si c nu sunt ambele 0, luam n = 4m
2
. Atunci,
n
3
+an
2
+bn +c = (8m
3
+am)
2
+ (4b a
2
)m
2
+c.
Daca 4b a
2
> 0 sau 4b a
2
= 0 si c > 0, atunci pentru m sucient de mare vom avea
(4b a
2
)m
2
+ c > 0 si (4b a
2
)m
2
+ c < 2(8m
3
+ am) 1, deci n
3
+ an
2
+ bn + c se
aa ntre (8m
3
+ am)
2
si (8m
3
+ am + 1)
2
, neind prin urmare patrat perfect. Daca
4b a
2
< 0 sau 4b a
2
= 0 si c < 0, se arata n mod analog ca n
3
+ an
2
+ bn + c se aa
ntre (8m
3
+am1)
2
si (8m
3
+am)
2
, neind prin urmare patrat perfect.

In problema 14.16 prezentam o generalizare a armat iei problemei 14.15:


Problema 14.16 Daca polinomul P Z[X] are proprietatea ca P(n) este patrat perfect
pentru orice n Z, atunci P este patratul unui polinom din Z[X].
Solut ie. Presupunem ca exista polinoame P care au toate valorile P(n), cu n Z,
patrate perfecte, dar nu sunt ele nsele patrate de polinoame din Z[X]. Este sucient
sa consideram polinoame care nu se divid prin patratul niciunui polinom din Z[X]; pre-
supunem deci ca P are aceasta proprietate. Atunci, discriminantul D al lui P este nenul.
Aritmetica si teoria numerelor 243
Fie m Z pentru care P(m) ,= 0; pentru ecare numar prim p
i
care divide D, notam
e
i
= maxi N [ P(m)
.
.
. p
e
i
i
. Cum P este neconstant, P(m+

p
i
|D
p
e
i
+1
i
) ia valori oricat
de mari pe masura ce creste Z.

In plus, cum
P(m) P(m+

p
i
|D
p
e
i
+1
i
)
.
.
.

p
i
|D
p
e
i
+1
i
,
niciuna dintre aceste valori nu este divizibila cu vreun factor de tipul p
e
i
+1
i
.

In consecint a,
exista numere ntregi n oricat de mari pentru care P(n) are macar un factor prim p D.
Sa consideram o astfel de situat ie. Daca p
2
P(n), atunci P(n) nu este patrat perfect,
contradict ie. Daca p
2
[ P(n), atunci P(n + p) P(n) + pP

(n) pP

(n) (mod p
2
), iar
p P

(n), deoarece p D. De aici, P(n + p)


.
.
. p, dar P(n + p)
.
.
. p
2
, de unde P(n + p) nu
este patrat perfect, contradict ie.
Observat ie. Demonstrat ia arata ceva mai mult decat s-a armat n enunt , si anume:
Daca P Z[X] nu este patratul unui polinom din Z[X], atunci exista numere n Z oricat
de mari pentru care P(n) nu este patrat perfect.
Problema 14.17 Demonstrat i ca exista o innitate de perechi ordonate (a, b) de numere
ntregi cu proprietatea ca pentru orice t N numarul at+b este triunghiular daca si numai
daca t este triunghiular.
Putnam, 1988
Solut ia 1. Reamintim ca numerele triunghiulare sunt cele de tipul t
n
=
n(n+1)
2
, n N.
Se vede usor ca t
3n+1
= 9t
n
+ 1, iar t
3n
t
3n+2
0 (mod 3) pentru orice n N. Prin
urmare, numarul natural t este triunghiular daca si numai daca 9t+1 este triunghiular ().
Consideram funct ia f : R R, f(x) = 9x + 1. Notam f
k
= f f . . . f
. .
k factori
; e a
k
, b
k
N
pentru care f
k
(x) = a
k
x + b
k
. Cum a
k
= 9
k
, aceste perechi sunt distincte. Aplicand
inductiv (), deducem ca toate perechile (a
k
, b
k
) au proprietatea ceruta.
Solut ia 2. Daca t =
n(n+1)
2
, atunci 8t + 1 = (2n + 1)
2
. Reciproc, daca t N are
proprietatea ca 8t + 1 este patrat perfect, acest patrat este al unui numar impar, e el
2n + 1. Rezulta ca t =
n(n+1)
2
. Prin urmare, t N este triunghiular daca si numai daca
8t + 1 este patrat perfect.
Fie un numar natural impar k. Avem k
2
1 (mod 8); de aici deducem ca, pentru t
N, t este triunghiular 8t + 1 este patrat perfect k
2
(8t + 1) este patrat perfect
8
_
k
2
t +
k
2
1
8
_
+ 1 este patrat perfect
_
k
2
t +
k
2
1
8
_
este triunghiular. Prin urmare,
orice pereche (a, b) =
_
k
2
,
k
2
1
8
_
, k 2N + 1, are proprietatea ceruta.
Observat ie. Vom numi ad-hoc o pereche (a, b) de numere ntregi triunghiulara daca
are proprietatea ca, pentru orice numar natural t, t este triunghiular daca si numai daca
at + b este triunghiular. Solut ia 2 arata ca pentru orice numar ntreg impar k perechea
_
k
2
,
k
2
1
8
_
este triunghiulara. Cu alte cuvinte, perechile triunghiulare sunt de forma ((2m+
1)
2
, t
m
), m N.
Vom arata ca, reciproc, orice pereche triunghiulara este de aceasta forma: Fie (a, b) o
pereche triunghiulara. Atunci, pentru orice n N

, at
n
+b este triunghiular, deci 8(at
n
+
b) +1 = 4an
2
+4an +(8b +1) este patrat perfect. Conform observat iei din nalul solut iei
problemei 14.16, exista l Z[X] cu proprietatea
244
4an
2
+ 4an + (8b + 1) = l(n)
2
.
Evident, l este de gradul 1; identitatea anterioara arata ca l = 2

ax+

a. Notand k = l(0),
obt inem a = k
2
= 8b + 1, deci b =
k
2
1
8
. Pentru ca b sa e ntreg, este necesar sa avem k
impar.
Problema 14.18 Denim sirul (x
n
)
n1
astfel: x
n
= n pentru n 1, 2, . . . , 2006 si
x
n+1
= x
n
+ x
n2005
pentru n 2006. Aratat i ca sirul (x
n
)
n
cont ine 2005 termeni con-
secutivi divizibili cu 2006.
Putnam, 2006
Solut ie.

Incepem prin a observa ca, daca un sir de numere ntregi (x
n
)
n1
satisface o
relat ie de recurent a de tipul
x
n
= f(x
n1
, x
n2
, . . . , x
nk
) pentru orice n > k (14.17)
(unde k N

si f Z[X
1
, X
2
, . . . , X
n
] sunt xate), atunci sirul ( x
n
)
n
este periodic modulo
orice N N

de la un rang ncolo. Acest lucru se ntampla deoarece, xand N N

si
notand cu H
N
numarul k-uplurilor ordonate de elemente din Z
N
, printre sistemele
i
=
( x
i+1
, x
i+2
, . . . , x
i+k
), i 0, 1, . . . , H
N
(toate clasele sunt modulo N) vor exista, conform
principiului cutiei, cel put in doua egale. Daca acestea sunt
i
si
j
, sa presupunem, pentru
a xa ideile, ca i < j. Atunci,
x
j+k+1
= f( x
j+k
, x
j+k1
, . . . , x
j+1
) =
= f( x
i+k
, x
i+k1
, . . . , x
i+1
) = x
i+k+1
.
Inductiv, se arata ca x
r+ji
= x
r
pentru orice r i + 1.
Mai observam ca, daca relat ia de recurent a (14.17) poate rescrisa sub forma x
nk
=
g(x
nk+1
, x
nk+2
, . . . , x
n
), g Z[X
1
, X
2
, . . . , X
n
], atunci putem extinde sirul init ial la
. . . , x
2
, x
1
, x
0
, x
1
, x
2
, . . ., acest ,,sir ind, cu argumente similare celor de mai sus, peri-
odic modulo orice N N

.
Cum relat ia de recurent a din enunt se poate rescrie
x
n2005
= x
n+1
x
n
,
sirul dat se prelungeste la . . . , x
2
, x
1
, x
0
, x
1
, x
2
, . . ., care este periodic modulo 2006. Se
constata nsa cu usurint a ca
x
1
= x
0
= = x
2004
= 1 si
x
2005
= x
2006
= = x
4009
= 0.
De aici si din periodicitatea modulo 2006 a ,,sirului . . . , x
2
, x
1
, x
0
, x
1
, x
2
, . . . rezulta
armat ia problemei.
Problema 14.19 Fie a Z. Aratat i ca pentru orice numar prim p polinomul X
4
+a
2

Z
p
[X] este reductibil.
Solut ie. Fie p N un numar prim. Notam f = X
4
+a
2
Z
p
[X].
Daca p [ a, atunci f = X
4
, deci f este reductibil peste Z
p
.
Daca p a, iar p = 2, atunci f = (X + 1)
4
, deci este reductibil.
Aritmetica si teoria numerelor 245
Daca p a, iar p > 2, pot aparea cazurile:
1.
_
1
p
_
= 1. Atunci, exista b Z
p
cu b
2
= 1. Rezulta ca f = (X
2
+ab)(X
2
ab), deci
f este reductibil peste Z
p
.
2.
_
2a
p
_
= 1. Atunci, exista c Z
p
cu c
2
= 2a. Rezulta f = (X
2
+ a)
2
2aX
2
=
(X
2
cX +a)(X
2
+cX +a), deci f este reductibil peste Z
p
.
3.
_
1
p
_
=
_
2a
p
_
= 1. Atunci,
_
2a
p
_
=
_
1
p
__
2a
p
_
= 1. Exista prin urmare d Z
p
cu d
2
= 2a. Atunci, f = (X
2
a)
2
+ 2aX
2
= (X
2
dX a)(X
2
+dX a), deci f este
reductibil peste Z
p
.
Observat ie. Se constata, aplicand de pilda criteriul lui Eisenstein lui f(X + 1), ca
polinomul f = X
4
+ 1 Z[X] este ireductibil. T inand cont si de armat ia problemei,
concluzionam ca exista polinoame ireductibile f Z[X] care sunt reductibile n Z
p
[X]
pentru orice numar prim p.
Problema 14.20 Fie p un numar prim impar. Cate elemente are mult imea
x
2
[ x Z
p
y
2
+ 1 [ y Z
p
?
Putnam, 1991
Solut ia 1. Notam cu S mult imea solut iilor ecuat iei x
2
= y
2
+ 1 (S Z
p
Z
p
).
Schimbarea liniara de coordonate (u, v) = (x + y, x y) a lui Z
p
Z
p
este inversabila,
deoarece

1 1
1 1

= 2 , 0 (mod p). Prin urmare, [S[ este egal cu numarul solut iilor
ecuat iei uv = 1 peste Z
p
, adica p 1.
Problema cere numarul de elemente al imaginii funct iei : S Z
p
, (x, y) = x
2
. Daca
z = x
2
pentru o pereche (x, y) S, atunci
1
(z) = (x, y). Deci,
1
(z) are patru
elemente daca z , 0, 1, doua elemente daca z = 1, si, numai n eventualitatea n care
1 este patrat n Z
p
, doua elemente daca z = 0.

In consecint a, [S[ = 4[(S)[ 2 2c,
unde c este 1 sau 0 dupa cum 1 este sau nu patrat n Z
p
. De aici,
[(S)[ =
p + 1 + 2c
4
. (14.18)
Cum [(S)[ Z, rezulta ca avem c = 1 daca p 1 (mod 4), respectiv c = 0 daca p 3
(mod 4).

Inlocuind n (14.18), deducem ca mult imea din enunt are 1 +
_
p
4
_
elemente.
Solut ia 2. Extinzand denit ia simbolului lui Legendre
_
a
p
_
prin
_
a
p
_
= 0 daca p [ a
si folosind faptul ca numarul de resturi patratice (nenule) modulo p este egal cu cel al
neresturilor patratice, obt inem relat ia
p1

a=0
_
a k
p
_
pentru orice k Z. (14.19)

In continuarea acestei solut ii, vom nota cu [P] 0, 1 valoarea de adevar a propozit iei
P. Notam si Z
2
p
= x
2
[ x Z
p
. Cu aceste notat ii, avem de determinat
N =
p1

a=0
[a Z
2
p
] [a 1 Z
2
p
].
246
Observand ca
[a Z
2
p
] =
1
2
_
1 +
_
a
p
_
+ [a = 0]
_
,
obt inem
N =
1
4
_
1 +
_
1
p
_
+ 1 +
_
1
p
_
+
p1

a=0
_
1 +
_
a
p
___
1 +
_
a 1
p
__
_
=
=
1
2
__
1
p
_
= 1
_
+
1
2
+
1
4
p1

a=0
_
1 +
_
a
p
_
+
_
a 1
p
_
+
_
a
p
__
a 1
p
__
.
Aplicand de doua ori relat ia (14.19), gasim
N =
1
2
__
1
p
_
= 1
_
+
1
2
+
p
4
+
1
4
p1

a=0
_
a
p
__
a 1
p
_
. (14.20)
Pentru k Z, notam S(k) =
p1

a=0
_
a
p
__
a k
p
_
. Pentru a determina N, avem nevoie de
S(1). Daca p k, atunci
S(k) =
p1

b=0
_
kb
p
__
k(b 1)
p
_
=
p1

b=0
_
k
p
__
b
p
__
k
p
__
b 1
p
_
=
=
p1

b=0
_
b
p
__
b 1
p
_
= S(1).

In plus,
p1

k=0
S(k) =
p1

a=0
_
a
p
_
p1

k=0
_
a k
p
_
= 0. Prin urmare,
S(1) =
S(0)
p 1
=
p 1
p 1
= 1. Introducand n (14.20), obt inem
N =
1
2
__
1
p
_
= 1
_
+
p + 1
4
. (14.21)
Cum N Z, rezulta ca daca p 1 (mod 4), atunci
_
1
p
_
= 1, iar daca p 3 (mod 4),
atunci
_
1
p
_
= 1; introducand aceste valori n (14.21) obt inem N = 1 +
_
p
4
_
.
Observat ie. Ambele metode de abordare au condus la redemonstrarea proprietat ii
_
1
p
_
= (1)
p1
2
, prezentata n introducerea capitolului, si care ar putut utilizata
pentru a trage concluzia direct din relat iile (14.18), respectiv (14.21). Am preferat abor-
darea prezentata tocmai pentru a sublinia ca proprietatea ment ionata nu este necesara
pentru completarea rat ionamentului, ci consecint a a acestuia.
Problema 14.21 Aratat i ca pentru orice n N

are loc relat ia


n! =
n

i=1
L
_
1, 2, . . . ,
_
n
i
__
, (14.22)
L(a
1
, a
2
. . . , a
k
) desemnand n aceasta problema cel mai mic multiplu comun al numerelor
ntregi a
1
, a
2
. . . , a
k
.
Putnam, 2003
Aritmetica si teoria numerelor 247
Solut ia 1. Este sucient sa ar atam ca pentru orice numar prim p exponent ii lui p din
descompunerile n factori primi ale celor doi membri ai relat iei (14.22) coincid.
Conform proprietat ilor funct iei lui Legendre (a se vedea introducerea capitolului), expo-
nentul la care apare p n descompunerea lui n! este
n

i=1
_
n
p
i
_
. Acest numar poate inter-
pretat ca ind cardinalul mult imii S a punctelor din primul cadran care au coordonatele
n N

si se aa e pe curba ( data de ecuat ia y = np


x
, e ntre ( si axele de coordonate;
ecare termen
_
n
p
i
_
al sumei reprezinta numarul de puncte din S care au abscisa i.
Pe de alta parte, exponentul lui p din descompunerea n factori primi a lui
m
_
1, 2, . . . ,
_
n
i
__
este
_
log
p
_
n
i
__
=
_
log
p
_
n
i
__
. Acesta este nsa exact numarul de puncte
din S care au ordonata i.

In concluzie,
n

i=1
_
log
p
_
n
i
__
=
n

i=1
_
n
p
i
_
, adica egalitatea dorita.
Solut ia 2. Vom demonstra relat ia ceruta prin induct ie dupa n. Ea este evidenta pentru
n = 1. Pasul de induct ie este imediat daca utilizam identitatea
n =
n1

i=1
L
_
1, 2, . . . ,
_
n
i
_
L
_
1, 2, . . . ,
_
n1
i
_. (14.23)
Ramane deci sa probam aceasta identitate; notam cu P produsul din membrul sau drept.
Remarcam ca al i-lea factor din P este 1 daca
n
i
, N (deci, daca
n
i
nu este divizor al lui n)
sau daca
n
i
este divizor al lui n, dar nu este putere de numar prim (deoarece orice numar
k N

1 care nu este putere de numar prim divide L(1, 2, . . . , k 1)). Pe de alta parte,
daca
n
i
este putere a numarului prim p, atunci cel de-al i-lea factor al lui P este egal cu p.
Fie acum p N un numar prim arbitrar.

Intrucat
n
i
parcurge tot i divizorii proprii ai lui
n, P cont ine cate un factor egal cu p pentru ecare k N

pentru care p
k
[ n.

In plus, P
nu cont ine alt i factori divizibili prin p. Din aceste motive, P coincide cu descompunerea
n factori primi a lui n, ceea ce ncheie demonstrat ia.
Problema 14.22 Pentru R

+
denim S() = [n] [ n N

. Aratat i ca N

nu se
poate scrie ca reuniunea disjuncta a trei mult imi nevide S(), S() si S() cu , , R

+
.
Putnam, 1995
Solut ie. Presupunem ca exista , , R

+
astfel ncat S(), S() si S() sa repre-
zinte o partit ie a lui N

. Atunci, 1 apart ine uneia dintre aceste mult imi, e ea S(). Prin
urmare, exista n N

astfel ncat n < 2. De aici rezuta < 2; pe de alta parte, > 1,


deoarece altfel am avea S() = N

, n contradict ie cu ipoteza.
Consideram m N, m 2, pentru care 1+
1
m
< 1+
1
m1
. Atunci, [k] = k pentru orice
k 1, 2, . . . , m1, iar [m] = m+1, deci m este cel mai mic element din S() S().

In plus, orice doua elemente consecutive ale mult imii S() S() difera prin m sau prin
m+ 1.
Putem presupune, fara a restrange generalitatea, ca m S(). Vom avea deci [] = m.
Fie n un element din S(). Conform celor de mai sus, elemente cele mai apropiate de n
ale lui S() se aa la distant a cel put in m de n, deci distant a dintre ele este de cel put in
2m. Cum nsa [] = m, doua elemente consecutive din S() difera prin cel mult m + 1,
contradict ie.
Observat ie. De fapt, nu exista trei mult imi S(), S() si S() ca n enunt care sa e
disjuncte doua cate doua.
248
Fie N N, N > max, , . Consideram tripletele v
n
=
_
_
n

_
,
_
n

_
,
_
n

__
R
3
,
n 0, 1, . . . , N
3
. Daca divizam cubul unitate din R
3
n N
3
cuburi de latura
1
N
, atunci,
conform principiului cutiei, cel put in doua dintre aceste triplete, e ele v
i
si v
j
, se vor gasi
n acelasi cub din diviziune. Fie k = [i j[. Atunci,
k

este la distant a cel mult


1
N
de un
numar ntreg m, deci [m] este egal cu k 1 sau cu k. Altfel spus, S() cont ine e k 1,
e k. Se arata n mod analog ca S() si S() cont in k 1 sau k. Asadar, cel put in unul
dintre numerele k 1 si k se gaseste n cel put in doua dintre mult imile S(), S() si S().
Prin urmare, aceste mult imi nu pot disjuncte doua cate doua.
Problema 14.23 Fie / N

nevida si N(x) = a /[ a x. Notam cu B mult imea


numerelor naturale nenule b care pot scrise sub forma a a

cu a, a

/. Scriem
B = b
1
, b
2
, . . ., cu b
i
< b
i+1
pentru orice i 1. Aratat i ca daca sirul (b
i+1
b
i
)
iN
este
nemarginit, atunci lim
x
N(x)
x
= 0.
Putnam, 2004
Solut ia 1. Sa presupunem pentru nceput ca exista numere naturale nenule b
0
= 1,
b
1
, . . . , b
n
cu proprietat ile:
(p
n
) Pentru orice i 1, 2, . . . , n, numarul c
i
=
b
i
2b
i1
este natural.
(q
n
) Oricare ar e
1
, . . . , e
n
1, 0, 1, [e
1
b
1
+e
2
b
2
+ +e
n
b
n
[ , B.
Atunci, ecare numar natural a va admite o scriere unica de tipul
a = a
0
b
0
+a
1
b
1
+ +a
n1
b
n1
+mb
n
cu 0 a
i
< 2c
i+1
pentru orice i 0, 1, . . . , n 1. Prin urmare, oricare ar d
i

0, 1, . . . , c
i
1 (i 1, 2, . . . , n 1), m
0
0, 1, . . . , 2c
0
1 si m
n
N, mult imea
m
0
b
0
+ (2d
1
+e
1
)b
1
+ + (2d
n1
+e
n1
)b
n1
+ (2m
n
+e
n
)b
n
,
unde e
i
0, 1 pentru orice i 1, 2, . . . , n, cont ine cel mult un element din /. De
aici rezulta ca limsup
x
N(x)
x

1
2
n
. Aceasta relat ie arata ca, daca reusim sa construim
pentru ecare n N

cate un sistem b
0
= 1, b
1
, . . . , b
n
cu proprietat ile (p
n
) si (q
n
), atunci
0 limsup
x
N(x)
x

1
2
n
pentru orice n N

, de unde rezulta armat ia problemei.


Vom construi inductiv sisteme b
0
= 1, b
1
, . . . , b
n
cu proprietat ile (p
n
) si (q
n
). Sistemul
format din b
0
= 1 are n mod trivial proprietat ile (p
0
) si (q
0
). Fie acum sistemul b
0
=
1, b
1
, . . . , b
n
cu proprietat ile (p
n
) si (q
n
). Este imediat faptul ca b
0
+b
1
+ +b
n1
< b
n
.
Conform ipotezei problemei, putem gasi o mult ime S
n
N B formata din 6n numere
consecutive. Notam cu b
n+1
cel de-al doilea (n ordine crescatoare) multiplu de 2b
n
din
S
n
. Atunci, este evident ca pentru orice x 2b
n
, 2b
n
+ 1, . . . , 0 avem b
n+1
+ x
S
n
. Din denit ia lui S
n
obt inem si b
n+1
+ x S
n
pentru orice x 0, 1, . . . , 2b
n
. De
aici si din faptul ca sistemul b
0
, b
1
, . . . , b
n
are proprietatea (q
n
), rezulta ca sistemul b
0
=
1, b
1
, . . . , b
n
, b
n+1
are proprietatea (q
n+1
). Dar b
0
= 1, b
1
, . . . , b
n
, b
n+1
are proprietatea
(p
n+1
) prin construct ie, deci e un sistem de tipul cautat. Acest fapt ncheie pasul de
induct ie si solut ia.
Solut ia 2. Fie S mult imea valorilor pe care le poate lua limsup
x
N(x)
x
pentru diversele
mult imi /; cum S [0, 1], ea este marginita; punem L = sup S.
Presupunem ca L > 0. Exista atunci / si B ca n enunt astfel ncat limsup
x
N(x)
x
>
Aritmetica si teoria numerelor 249
3L
4
. Din condit ia de nemarginire din enunt rezulta ca exista m N

B. Atunci, / si
/ + m sunt disjuncte. Notam /

= / (/ + m) si N

(x) = [1, 2, . . . , x /

[. Atunci,
limsup
x
N

(x)
x
>
3L
2
> L, deci /

nu poate verica condit iile din enunt . Prin urmare, daca


notam B

= a

[ a

, a

, rezulta ca exista N N

astfel ncat din orice N numere


naturale consecutive sa se gaseasc a macar unul n B

. Dar
B

b +em[ b B, e 1, 0, 1,
de unde rezulta ca din orice n +2m numere naturale consecutive se va gasi n B cel put in
unul, contradict ie.
Ramane deci ca nu putem avea L > 0. Prin urmare, L = 0, de unde lim
x
N(x)
x
= 0.
Bibliograe
[1] L. Ahlfors, Complex Analysis, McGraw-Hill, 1979.
[2] T. Andreescu, R. Gelca, Putnam and Beyond, Springer, 2007.
[3] G. Berge, Graphs, North-Holland, 1985.
[4] Gh. Bucur, E. Campu, S. Gaina, Culegere de probleme de calcul diferent ial si integral,
III, Ed. Tehnica, Bucuresti, 1967.
[5] R. Diestel, Graph Theory, Springer, 1997.
[6] S. Francinou, H. Gianella, S. Nicolas, Exercices de mathematiques des oraux de
lEcole Polytechnique et des Ecoles Normale Superieures, Cassini, Paris, 2001.
[7] D. Flondor, N. Donciu, Algebra si analiza matematic a. Culgere de probleme, vol. I
si II, Ed. Didactica si Pedagogica, Bucuresti, 1979.
[8] L.C. Florescu, Analiza matematica, Ed. Universitat ii ,,Al.I. Cuza Iasi, 1999.
[9] A. Gibbons, Algorithmic Graph Theory, Cambridge Univ. Press, 1985.
[10] N.V. Ghircoiasu, C. Miron, Grafuri de uent a si aplicat ii n tehnica, Editura
Tehnica, Bucuresti, 1974.
[11] M. Ivan, Elemente de calcul integral, Ed. Mediamira, 2003.
[12] W. Kaczor, M. Nowak, Problems in Mathematical Analysis, vol. I, II, III, A.M.S.,
2003.
[13] G. Klambauer, Problems and Propositions in Analysis, Marcel Dekker, 1979.
[14] B. Makarov, M. Goluzina, A. Lodkin, A. Podkorytov, Selected Problems in Real
Analysis, A.M.S., 2000.
[15] R. Merris, Combinatorics, Willey-Interscience, 2003.
[16] T. Needham, Visual Complex Analysis, Clarendon Press, 1997.
[17] L. Panaitopol, A. Gica, O introducere n aritmetic a si teoria numerelor, Ed. Univer-
sitat ii din Bucuresti, 2001
[18] L. Panaitopol, A. Gica, Probleme de aritmetica si teoria numerelor, Ed. Universitat ii
din Bucuresti, 2006.
[19] G. Pavel, F.I. Tomut a, I. Gavrea, Matematici speciale, Ed. Dacia, Cluj-Napoca, 1981.
250
BIBLIOGRAFIE 251
[20] V. Pop, Geometrie combinatorica, Editura Mediamira, Cluj-Napoca, 2010.
[21] D. Popa, Calcul integral, Ed. Mediamira, 2005.
[22] E. Popa, Introducere n teoria funct iilor de o variabila complexa, Ed. Universitat ii
,,Al. I. Cuza Iasi, 2001.
[23] E. Popa, Analiza matematica, Ed. GIL, 2005.
[24] A. Precupanu, Bazele analizei matematice, Ed. Canova, Iasi, 1995.
[25] E. Ramis, C. Deschamps, J. Odoux, Analyse, vol. 1 si 2, Ed. Masson, Paris, 1993.
[26] V. Rudner, C. Nicolescu, Probleme de matematici speciale, Ed. Didactica si Peda-
gogica, Bucuresti, 1982.
[27] P.N. de Souza, J.-N. Silva, Berkeley Problems in Mathematics, Springer, 2004.
[28] G. Szekely (ed.), Contests in Higher Mathematics, Miklos Schweitzer Competitions
1962-1991, Springer, 1996.
[29] T. Trif, Probleme de calcul diferent ial si integral n R
n
, Univ. Babes-Bolyai, 2003.
[30] I. Tomescu, Introducere n combinatorica, Ed. Tehnica, 1972.
[31] I. Tomescu, Probleme de combinatorica si teoria grafurilor, Ed. Didactica si Peda-
gogica, Bucuresti, 1981.
[32] N. Vornicescu, M. Ivan, V. Popa, V. Pop, Calcul diferent ial, Ed. Mediamira, 2004.
Concursuri
[33] Ariel: Internet Mathematical Olympiad for Students, 2008-2011.
[34] IMC: International Mathematics Competition for University Students, 1994-2011.
[35] Iran: Iranian University Students Mathematics Competitions, 1973-2011.
[36] Vojtech Jarnik: Vojtech Jarnik International Mathematical Competition, 1991-2011.
[37] Putnam: William Lowell Putnam Mathematical Competition, 1938-2010.
[38] SEEMOUS: South Eastern European Mathematical Olympiad for University Stu-
dents, 2007-2011.

S-ar putea să vă placă și